101 Primary Care Case Studies: A Workbook for Clinical and Bedside Skills [1 ed.] 9780826182722, 0826182720, 9780826182739, 0826182739

Real-life primary care case studies* from more than 50 primary care providers, including physician assistants, nurse pra

4,461 419 9MB

English Pages [845] Year 2020

Report DMCA / Copyright

DOWNLOAD FILE

Polecaj historie

101 Primary Care Case Studies: A Workbook for Clinical and Bedside Skills [1 ed.]
 9780826182722, 0826182720, 9780826182739, 0826182739

Table of contents :
Cover
About the pagination of this eBook
Title
Copyright
Contents
Contributors
Reviewers
Foreword
Foreword
Foreword
Preface
Acknowledgments
How to Use this Book
Case 1: Ankle and Calf Pain, Adult Male
Chief Complaint
History of Present Illness
Review of Systems
Relevant History
Allergies
Medications
Physical Examination
Clinical Discussion Questions
Bedside Manner Question
Answer Key: Ankle and Calf Pain, Adult Male
Outcome
Insight from the PCP
References
Case 2: Fatigue and Weakness, Adult Female
Chief Complaint
History of Present Illness
Review of Systems
Relevant History
Allergies
Medications
Physical Examination
Clinical Discussion Questions
Bedside Manner Question
Answer Key: Left Hip Pain, Adult Female
Outcome
Insight from the PCP
References
Case 3: Abdominal Pain, Adolescent Male
Chief Complaint
History of Present Illness
Review of Systems
Relevant History
Allergies
Medications
Physical Examination
Clinical Discussion Questions
Bedside Manner Question
Answer Key: Abdominal Pain, Adolescent Male
Outcome
Insight from the PCP
References
Case 4: Left Hip Pain, Adult Female
Chief Complaint
History of Present Illness
Review of Systems
Relevant History
Allergies
Medications
Physical Examination
Clinical Discussion Questions
Bedside Manner Question
Answer Key: Left Hip Pain, Adult Female
Outcome
Insight from the PCP
References
Case 5: Vaginal Bleeding, Adult Female
Chief Complaint
History of Present Illness
Review of Systems
Relevant History
Allergies
Medications
Physical Examination
Clinical Discussion Questions
Bedside Manner Questions
Answer Key: Vaginal Bleeding, Adult Female
Outcome
Insight from the PCP
References
Case 6: Numbness in Hands and Legs, Geriatric Male
Chief Complaint
History of Present Illness
Review of Systems
Relevant History
Allergies
Medications
Physical Examination
Clinical Discussion Questions
Bedside Manner Questions
Answer Key: Numbness in Hands and Legs, Geriatric Male
Outcome
Insight from the PCP
References
Case 7: Painful Rash, Geriatric Male
Chief Complaint
History of Present Illness
Review of Systems
Relevant History
Allergies
Medications
Physical Examination
Clinical Discussion Questions
Bedside Manner Question
Answer Key: Painful Rash, Geriatric Male
Outcome
Insight from the PCP
References
Case 8: Fever and Body Aches, Adult Male
Chief Complaint
History of Present Illness
Review of Systems
Relevant History
Allergies
Medications
Physical Examination
Clinical Discussion Questions
Bedside Manner Questions
Answer Key: Fever and Body Aches, Adult Male
Outcome
Insight from the PCP
References
Case 9: Fever and Rash, Pediatric Male
Chief Complaint
History of Present Illness
Review of Systems
Relevant History
Allergies
Medications
Physical Examination
Clinical Discussion Questions
Bedside Manner Questions
Answer Key: Fever and Rash, Pediatric Male
Outcome
Insight from the PCP
References
Case 10: Blurry Vision, Adult Female
Chief Complaint
History of Present Illness
Review of Systems
Relevant History
Allergies
Medications
Physical Examination
Clinical Discussion Questions
Bedside Manner Questions
Answer Key: Blurry Vision, Adult Female
Outcome
Insight from the PCP
References
Case 11: Does Not Follow Directions, Pediatric Male
Chief Complaint
History of Present Illness
Review of Systems
Relevant History
Allergies
Medications
Physical Examination
Clinical Discussion Questions
Bedside Manner Questions
Answer Key: Does Not Follow Directions, Pediatric Male
Outcome
Insight from the PCP
References
Case 12: Crusty, Irritated Left Eye, Pediatric Male
Chief Complaint
History of Present Illness
Review of Systems
Relevant History
Allergies
Medications
Physical Examination
Clinical Discussion Questions
Bedside Manner Questions
Answer Key: Crusty, Irritated Left Eye, Pediatric Male
Outcome
Insight from the PCP
References
Case 13: Frequent Urination, Adult Male
Chief Complaint
History of Present Illness
Review of Systems
Relevant History
Allergies
Medications
Physical Examination
Clinical Discussion Questions
Bedside Manner Questions
Answer Key: Frequent Urination, Adult Male
Outcome
Insight from the PCP
References
Case 14: Vomiting, Pediatric Male
Chief Complaint
History of Present Illness
Review of Systems
Relevant History
Allergies
Medications
Physical Examination
Clinical Discussion Questions
Bedside Manner Question
Answer Key: Vomiting, Pediatric Male
Outcome
Insight from the PCP
References
Case 15: Difficulty Relating to Others, Pediatric Male
Chief Complaint
History of Present Illness
Review of Systems
Relevant History
Allergies
Medications
Physical Examination
Clinical Discussion Questions
Bedside Manner Questions
Answer Key: Difficulty Relating to Others, Pediatric Male
Outcome
Insight from the PCP
References
Case 16: Facial Numbness, Adult Male
Chief Complaint
History of Present Illness
Review of Systems
Relevant History
Allergies
Medications
Physical Examination
Clinical Discussion Questions
Bedside Manner Questions
Answer Key: Facial Numbness, Adult Male
Outcome
Insight from the PCP
References
Case 17: Elbow Injury, Pediatric Male
Chief Complaint
History of Present Illness
Review of Systems
Relevant History
Allergies
Medications
Physical Examination
Clinical Discussion Questions
Bedside Manner Question
Answer Key: Elbow Injury, Pediatric Male
Outcome
Insight from the PCP
References
Case 18: Thick Yellow Toenails, Adult Female
Chief Complaint
History of Present Illness
Review of Systems
Relevant History
Allergies
Medications
Physical Examination
Clinical Discussion Questions
Bedside Manner Question
Answer Key: Thick Yellow Toenails, Adult Female
Outcome
Insight from the PCP
References
Case 19: Speech Difficulties, Geriatric Male
Chief Complaint
History of Present Illness
Review of Systems
Medical History
Allergies
Medications
Physical Examination
Clinical Discussion Questions
Bedside Manner Question
Answer Key: Speech Difficulties, Geriatric Male
Outcome
Insight from the PCP
References
Case 20: Mood Changes, Adult Male
Chief Complaint
History of Present Illness
Review of Systems
Relevant History
Allergies
Medications
Physical Examination
Clinical Discussion Questions
Bedside Manner Question
Answer Key: Mood Changes, Adult Male
Outcome
Insight from the PCP
References
Case 21: Fever and Lump on Neck, Adolescent Male
Chief Complaint
History of Present Illness
Review of Systems
Relevant History
Allergies
Medications
Physical Examination
Clinical Discussion Questions
Bedside Manner Question
Answer Key: Fever and Lump on Neck, Adolescent Male
Outcome
Insight from the PCP
References
Case 22: Hand Pain at Night, Adult Female
Chief Complaint
History of Present Illness
Review of Systems
Relevant History
Allergies
Medications
Physical Examination
Clinical Discussion Questions
Bedside Manner Questions
Answer Key: Hand Pain at Night, Adult Female
Outcome
Insight from the PCP
References
Case 23: Burning With Urination, Adult Male
Chief Complaint
History of Present Illness
Review of Systems
Relevant History
Allergies
Medications
Physical Examination
Clinical Discussion Questions
Bedside Manner Questions
Answer Key: Burning With Urination, Adult Male
Outcome
Insight from the PCP
References
Case 24: Constipation, Adult Female
Chief Complaint
History of Present Illness
Review of Systems
Relevant History
Allergies
Medications
Physical Examination
Clinical Discussion Questions
Bedside Manner Questions
Answer Key: Constipation, Adult Female
Outcome
Insight from the PCP
References
Case 25: Recurrent Vomiting, Pediatric Female
Chief Complaint
History of Present Illness
Review of Systems
Relevant History
Allergies
Medications
Physical Examination
Clinical Discussion Questions
Bedside Manner Questions
Answer Key: Recurrent Vomiting, Pediatric Female
Outcome
Insight from the PCP
References
Case 26: Fatigue and General Malaise, Adult Male
Chief Complaint
History of Present Illness
Review of Systems
Relevant History
Allergies
Medications
Physical Examination
Clinical Discussion Questions
Bedside Manner Questions
Answer Key: Fatigue and General Malaise, Adult Male
Outcome
Insight from the PCP
References
Case 27: Nausea, Vomiting, and Headache, Adolescent Male
Chief Complaint
History of Present Illness
Review of Systems
Relevant History
Allergies
Medications
Physical Examination
Clinical Discussion Questions
Bedside Manner Questions
Answer Key: Nausea, Vomiting, and Headache, Adolescent Male
Outcome
Insight from the PCP
References
Case 28: Dizziness and Difficulty Hearing, Adult Male
Chief Complaint
History of Present Illness
Review of Systems
Relevant History
Allergies
Medications
Physical Examination
Clinical Discussion Questions
Bedside Manner Question
Answer Key: Dizziness and Difficulty Hearing, Adult Male
Outcome
Insight from the PCP
References
Case 29: Bad Headache, Adult Female
Chief Complaint
History of Present Illness
Review of Systems
Relevant History
Allergies
Medications
Physical Examination
Clinical Discussion Questions
Bedside Manner Question
Answer Key: Bad Headache, Adult Female
Outcome
Insight from the PCP
References
Case 30: Abdominal Pain, Adult Female
Chief Complaint
History of Present Illness
Review of Systems
Relevant History
Allergies
Medications
Physical Examination
Clinical Discussion Questions
Bedside Manner Question
Answer Key: Abdominal Pain, Adult Female
Outcome
Insight from the PCP
References
Case 31: Painful Urination, Adult Male
Chief Complaint
History of Present Illness
Review of Systems
Relevant History
Allergies
Medications
Physical Examination
Clinical Discussion Questions
Bedside Manner Question
Answer Key: Painful Urination, Adult Male
Outcome
Insight from the PCP
References
Case 32: Frequent Diarrhea, Adolescent Female
Chief Complaint
History of Present Illness
Review of Systems
Relevant History
Allergies
Medications
Physical Examination
Clinical Discussion Questions
Bedside Manner Question
Answer Key: Frequent Diarrhea, Adolescent Female
Outcome
Insight from the PCP
References
Case 33: Repetitive Bruising on Legs, Pediatric Female
Chief Complaint
History of Present Illness
Review of Systems
Relevant History
Allergies
Medications
Physical Examination
Clinical Discussion Questions
Bedside Manner Questions
Answer Key: Repetitive Bruising on Legs, Pediatric Female
Outcome
Insight from the PCP
References
Case 34: Swollen Knee, Pediatric Female
Chief Complaint
History of Present Illness
Review of Systems
Relevant History
Allergies
Medications
Physical Examination
Clinical Discussion Questions
Bedside Manner Question
Answer Key: Swollen Knee, Pediatric Female
Outcome
Insight from the PCP
References
Case 35: Widespread Painful Rash, Pediatric Female
Chief Complaint
History of Present Illness
Review of Systems
Relevant History
Allergies
Medications
Physical Examination
Clinical Discussion Questions
Bedside Manner Questions
Answer Key: Widespread Painful Rash, Pediatric Female
Outcome
Insight from the PCP
References
Case 36: Blurred Vision in the Left Eye, Adult Female
Chief Complaint
History of Present Illness
Review of Systems
Relevant History
Allergies
Medications
Physical Examination
Clinical Discussion Questions
Bedside Manner Questions
Answer Key: Blurred Vision in the Left Eye, Adult Female
Outcome
Insight from the PCP
References
Case 37: Fever and Back Pain, Geriatric Female
Chief Complaint
History of Present Illness
Review of Systems
Relevant History
Allergies
Medications
Physical Examination
Clinical Discussion Questions
Bedside Manner Questions
Answer Key: Fever and Back Pain, Geriatric Female
Outcome
Insight from the PCP
References
Case 38: Sore Throat, Fever, and Chills, Adult Male
Chief Complaint
History of Present Illness
Review of Systems
Relevant History
Allergies
Medications
Physical Examination
Clinical Discussion Questions
Bedside Manner Questions
Answer Key: Sore Throat, Fever, and Chills, Adult Male
Outcome
Insight from the PCP
References
Case 39: Bad Cough and Fever, Adolescent Female
Chief Complaint
History of Present Illness
Review of Systems
Relevant History
Allergies
Medications
Physical Examination
Clinical Discussion Questions
Bedside Manner Questions
Answer Key: Bad Cough and Fever, Adolescent Female
Outcome
Insight from the PCP
References
Case 40: Headache and Fever, Pediatric Male
Chief Complaint
History of Present Illness
Review of Systems
Relevant History
Allergies
Medications
Physical Examination
Clinical Discussion Questions
Bedside Manner Questions
Answer Key: Headache and Fever, Pediatric Male
Outcome
Insight from the PCP
References
Case 41: Heart Palpitations, Adult Female
Chief Complaint
History of Present Illness
Review of Systems
Relevant History
Allergies
Medications
Physical Examination
Clinical Discussion Questions
Bedside Manner Question
Answer Key: Heart Palpitations, Adult Female
Outcome
Insight from the PCP
References
Case 42: Double and Blurred Vision, Adolescent Male
Chief Complaint
History of Present Illness
Review of Systems
Relevant History
Allergies
Medications
Physical Examination
Clinical Discussion Questions
Bedside Manner Question
Answer Key: Double and Blurred Vision, Adolescent Male
Outcome
Insight from the PCP
References
Case 43: Nipple Pain, Adult Female
Chief Complaint
History of Present Illness
Review of Systems
Relevant History
Allergies
Medications
Physical Examination
Clinical Discussion Questions
Bedside Manner Question
Answer Key: Nipple Pain, Adult Female
Outcome
Insight from the PCP
References
Case 44: Irregular Menstrual Cycle, Adolescent Female
Chief Complaint
History of Present Illness
Review of Systems
Relevant History
Allergies
Medications
Physical Examination
Clinical Discussion Questions
Bedside Manner Questions
Answer Key: Irregular Menstrual Cycle, Adolescent Female
Outcome
Insight from the PCP
References
Case 45: Right Knee Pain, Pediatric Female
Chief Complaint
History of Present Illness
Review of Systems
Relevant History
Allergies
Medications
Physical Examination
Clinical Discussion Questions
Bedside Manner Questions
Answer Key: Right Knee Pain, Pediatric Female
Outcome
Insight from the PCP
References
Case 46: Chest Pain, Adolescent Male
Chief Complaint
History of Present Illness
Review of Systems
Relevant History
Allergies
Medications
Physical Examination
Clinical Discussion Questions
Bedside Manner Questions
Answer Key: Chest Pain, Adolescent Male
Outcome
Insight from the PCP
References
Case 47: Recurrent Pain and Redness in Left Foot, Adult Female
Chief Complaint
History of Present Illness
Review of Systems
Relevant History
Allergies
Medications
Physical Examination
Clinical Discussion Questions
Bedside Manner Question
Answer Key : Recurrent Pain and Redness in Left Foot, Adult Female
Outcome
Insight from the PCP
References
Case 48: Muscle and Joint Pain, Adult Female
Chief Complaint
History of Present Illness
Review of Systems
Relevant History
Allergies
Medications
Physical Examination
Clinical Discussion Questions
Bedside Manner Question
Answer Key: Muscle and Joint Pain, Adult Female
Outcome
Insight from the PCP
References
Case 49: Constipation, Adolescent Female
Chief Complaint
History of Present Illness
Review of Systems
Relevant History
Allergies
Medications
Physical Examination
Clinical Discussion Questions
Bedside Manner Question
Answer Key: Constipation, Adolescent Female
Outcome
Insight from the PCP
References
Case 50: Coughing and Wheezing, Pediatric Male
Chief Complaint
History of Present Illness
Review of Systems
Relevant History
Allergies
Medications
Physical Examination
Clinical Discussion Questions
Bedside Manner Questions
Answer Key : Coughing and Wheezing, Pediatric Male
Outcome
Insight from the PCP
References
Case 51: Shoulder Pain, Adult Male
Chief Complaint
History of Present Illness
Review of Systems
Relevant History
Allergies
Medications
Physical Examination
Clinical Discussion Questions
Bedside Manner Question
Answer Key: Shoulder Pain, Adult Male
Outcome
Insight from the PCP
References
Case 52: Painless Anal Bumps, Adult Transgender Male
Chief Complaint
History of Present Illness
Review of Systems
Relevant History
Medical Allergies
Medications
Physical Examination
Clinical Discussion Questions
Bedside Manner Questions
Answer Key: Painless Anal Bumps, Adult Transgender Male
Outcome
Insight from the PCP
References
Case 53: Fatigue and Decreased Libido, Adult Male
Chief Complaint
History of Present Illness
Review of Systems
Relevant History
Allergies
Medications
Physical Examination
Clinical Discussion Questions
Bedside Manner Questions
Answer Key: Fatigue and Decreased Libido, Adult Male
Outcome
Insight from the PCP
References
Case 54: Unsteady Gait and Nausea, Pediatric Female
Chief Complaint
History of Present Illness
Review of Systems
Relevant History
Allergies
Medications
Physical Examination
Clinical Discussion Questions
Bedside Manner Questions
Answer Key: Unsteady Gait and Nausea, Pediatric Female
Outcome
Insight from the PCP
References
Case 55: Excess Facial Hair, Adolescent Female
Chief Complaint
History of Present Illness
Review of Systems
Relevant History
Allergies
Medications
Physical Examination
Clinical Discussion Questions
Bedside Manner Question
Answer Key: Excess Facial Hair, Adolescent Female
Outcome
Insight from the PCP
References
Case 56: Fever and Body Aches, Adult Male
Chief Complaint
History of Present Illness
Review of Systems
Relevant History
Allergies
Medications
Physical Examination
Clinical Discussion Questions
Bedside Manner Questions
Answer Key: Fever and Body Aches, Adult Male
Outcome
Insight from the PCP
References
Case 57: Large Pimple on Leg, Adult Female
Chief Complaint
History of Present Illness
Review of Systems
Relevant History
Allergies
Medications
Physical Examination
Clinical Discussion Questions
Bedside Manner Question
Answer Key: Large Pimple on Leg, Adult Female
Outcome
Insight from the PCP
References
Case 58: Chest Pain, Adult Female
Chief Complaint
History of Present Illness
Review of Systems
Relevant History
Allergies
Medications
Physical Examination
Clinical Discussion Questions
Bedside Manner Question
Answer Key: Chest Pain, Adult Female
Outcome
Insight from the PCP
References
Case 59: Right Knee Pain, Geriatric Male
Chief Complaint
History of Present Illness
Review of Systems
Relevant History
Allergies
Medications
Physical Examination
Clinical Discussion Questions
Bedside Manner Question
Answer Key: Right Knee Pain, Geriatric Male
Outcome
Insight from the PCP
References
Case 60: Painful Urination, Adolescent Male
Chief Complaint
History of Present Illness
Review of Systems
Relevant History
Allergies
Medications
Physical Examination
Clinical Discussion Questions
Bedside Manner Question
Answer Key : Painful Urination, Adolescent Male
Outcome
Insight from the PCP
References
Case 61: Persistent Bad Cough, Adult Female
Chief Complaint
History of Present Illness
Review of Systems
Relevant History
Allergies
Medications
Physical Examination
Clinical Discussion Questions
Bedside Manner Question
Answer Key: Persistent Bad Cough, Adult Female
Outcome
Insight from the PCP
References
Case 62: Adult Well Visit, Geriatric Male
Chief Complaint
History of Present Illness
Review of Systems
Relevant History
Allergies
Medications
Physical Examination
Clinical Discussion Questions
Bedside Manner Questions
Answer Key: Adult Well Visit, Geriatric Male
Outcome
Insight from the PCP
References
Case 63: Low Platelets, Adult Male
Chief Complaint
History of Present Illness
Review of Systems
Relevant History
Allergies
Medications
Physical Examination
Clinical Discussion Questions
Bedside Manner Questions
Answer Key: Low Platelets, Adult Male
Outcome
Insight from the PCP
References
Case 64: Skin Lesions After Sun Exposure, Adult Female
Chief Complaint
History of Present Illness
Review of Systems
Relevant History
Allergies
Medications
Physical Examination
Clinical Discussion Questions
Bedside Manner Question
Answer Key: Skin Lesions After Sun Exposure, Adult Female
Outcome
Insight from the PCP
References
Case 65: Daytime Fatigue, Adult Male
Chief Complaint
History of Present Illness
Review of Systems
Relevant History
Allergies
Medications
Physical Examination
Clinical Discussion Questions
Bedside Manner Question
Answer Key: Daytime Fatigue, Adult Male
Outcome
Insight from the PCP
References
Case 66: Shortness of Breath, Geriatric Male
Chief Complaint
History of Present Illness
Review of Systems
Relevant History
Allergies
Medications
Physical Examination
Clinical Discussion Questions
Bedside Manner Question
Answer Key : Shortness of Breath, Geriatric Male
Outcome
Insight from the PCP
References
Case 67: Fatigue and Joint Aches, Adult Male
Chief Complaint
History of Present Illness
Review of Systems
Relevant History
Allergies
Medications
Physical Examination
Clinical Discussion Questions
Bedside Manner Question
Answer Key: Fatigue and Joint Aches, Adult Male
Outcome
Insight from the PCP
References
Case 68: Shoulder Pain, Adult Male
Chief Complaint
History of Present Illness
Review of Systems
Relevant History
Allergies
Medications
Physical Examination
Clinical Discussion Questions
Bedside Manner Questions
Answer Key: Shoulder Pain, Adult Male
Outcome
Insight from the PCP
References
Case 69: Unexplained Weight Loss, Adult Female
Chief Complaint
History of Present Illness
Review of Systems
Relevant History
Allergies
Medications
Physical Examination
Clinical Discussion Questions
Bedside Manner Questions
Answer Key: Unexplained Weight Loss, Adult Female
Outcome
Insight from the PCP
References
Case 70: Lightheadedness, Geriatric Female
Chief Complaint
History of Present Illness
Review of Systems
Relevant History
Allergies
Medications
Physical Examination
Clinical Discussion Questions
Bedside Manner Question
Answer Key: Lightheadedness, Geriatric Female
Outcome
Insight from the PCP
References
Case 71: Painful Lump in Armpit, Adult Female
Chief Complaint
History of Present Illness
Review of Systems
Relevant History
Allergies
Medications
Physical Examination
Clinical Discussion Questions
Bedside Manner Question
Answer Key: Painful Lump in Armpit, Adult Female
Outcome
Insight from the PCP
References
Case 72: Fever, Cough, and Runny Nose, Pediatric Female
Chief Complaint
History of Present Illness
Review of Systems
Relevant History
Allergies
Medications
Physical Examination
Clinical Discussion Questions
Bedside Manner Question
Answer Key: Fever, Cough, and Runny Nose, Pediatric Female
Outcome
Insight from the PCP
References
Case 73: Diabetes Check, Adult Male
Chief Complaint
History of Present Illness
Review of Systems
Relevant History
Allergies
Medication
Physical Examination
Clinical Discussion Questions
Bedside Manner Questions
Answer Key: Diabetes Check, Adult Male
Outcome
Insight from the PCP
References
Case 74: Buttocks Pain, Geriatric Male
Chief Complaint
History of Present Illness
Review of Systems
Relevant History
Allergies
Medication
Physical Examination
Clinical Discussion Questions
Bedside Manner Questions
Answer Key: Buttocks Pain, Geriatric Male
Outcome
Insight from the PCP
References
Case 75: Abdominal Pain, Adolescent Male
Chief Complaint
History of Present Illness
Review of Systems
Relevant History
Allergies
Medication
Physical Examination
Clinical Discussion Questions
Bedside Manner Questions
Answer Key: Abdominal Pain, Adolescent Male
Outcome
Insight from the PCP
References
Case 76: Irritable and Crying Toddler, Pediatric Male
Chief Complaint
History of Present Illness
Review of Systems
Relevant History
Allergies
Medication
Physical Examination
Clinical Discussion Questions
Bedside Manner Questions
Answer Key: Irritable and Crying Toddler, Pediatric Male
Outcome
Insight from the PCP
References
Case 77: Swelling Around Eyes and Ankles, Adolescent Male
Chief Complaint
History of Present Illness
Review of Systems
Relevant History
Allergies
Medication
Physical Examination
Clinical Discussion Question
Bedside Manner Questions
Answer Key: Swelling Around Eyes and Ankles, Adolescent Male
Outcome
Insight from the PCP
References
Case 78: Bilateral Heel Pain, Adult Female
Chief Complaint
History of Present Illness
Review of Systems
Relevant History
Medication
Allergies
Physical Examination
Clinical Discussion Questions
Bedside Manner Question
Answer Key: Bilateral Heel Pain, Adult Female
Outcome
Insight from the PCP
References
Case 79: Dry, Cracked Skin, Adult Female
Chief Complaint
History of Present Illness
Review of Systems
Relevant History
Allergies
Medication
Physical Examination
Clinical Discussion Questions
Bedside Manner Question
Answer Key: Dry, Cracked Skin, Adult Female
Outcome
Insight from the PCP
References
Case 80: Urine Leakage, Geriatric Female
Chief Complaint
History of Present Illness
Review of Systems
Relevant History
Allergies
Medications
Physical Examination
Clinical Discussion Questions
Bedside Manner Question
Answer Key: Urine Leakage, Geriatric Female
Outcome
Insight from the PCP
References
Case 81: Coughing and Gagging, Pediatric Male
Chief Complaint
History of Present Illness
Review of Systems
Relevant History
Allergies
Medications
Physical Examination
Clinical Discussion Questions
Bedside Manner Questions
Answer Key: Coughing and Gagging, Pediatric Male
Outcome
Insight from the PCP
References
Case 82: Eye Injury, Adult Male
Chief Complaint
History of Present Illness
Review of Systems
Relevant History
Allergies
Medications
Physical Examination
Clinical Discussion Questions
Bedside Manner Questions
Answer Key: Eye Injury, Adult Male
Outcome
Insight from the PCP
References
Case 83: Pain in Great Toe, Adult Female
Chief Complaint
History of Present Illness
Review of Systems
Relevant History
Allergies
Medications
Physical Examination
Clinical Discussion Questions
Bedside Manner Questions
Answer Key: Pain in Great Toe, Adult Female
Outcome
Insight from the PCP
References
Case 84: Bilateral Leg Pain, Pediatric Male
Chief Complaint
History of Present Illness
Review of Systems
Relevant History
Allergies
Medications
Physical Examination
Clinical Discussion Questions
Bedside Manner Questions
Answer Key: Bilateral Leg Pain, Pediatric Male
Outcome
Insight from the PCP
References
Case 85: Confusion, Geriatric Female
Chief Complaint
History of Present Illness
Review of Systems
Relevant History
Allergies
Medications
Physical Examination
Clinical Discussion Questions
Bedside Manner Question
Answer Key: Confusion, Geriatric Female
Outcome
Insight from the PCP
References
Case 86: Lump in Throat, Adult Male
Chief Complaint
History of Present Illness
Review of Systems
Relevant History
Allergies
Medications
Physical Examination
Clinical Discussion Questions
Bedside Manner Question
Answer Key: Lump in Throat, Adult Male
Outcome
Insight from the PCP
References
Case 87: Fever and Rash, Adult Male
Chief Complaint
History of Present Illness
Review of Systems
Relevant History
Allergies
Medications
Physical Examination
Clinical Discussion Questions
Bedside Manner Question
Answer Key: Fever and Rash, Adult Male
Outcome
Insight from the PCP
References
Case 88: Extreme Weakness, Adult Male
Chief Complaint
History of Present Illness
Review of Systems
Relevant History
Allergies
Medications
Physical Examination
Clinical Discussion Questions
Bedside Manner Question
Answer Key: Extreme Weakness, Adult Male
Outcome
Insight from the PCP
References
Case 89: Leg Pain, Geriatric Male
Chief Complaint
History of Present Illness
Review of Systems
Relevant History
Allergies
Medications
Physical Examination
Clinical Discussion Questions
Bedside Manner Question
Answer Key: Leg Pain, Geriatric Male
Outcome
Insight from the PCP
References
Case 90: Pain and Redness in Breast, Adult Female
Chief Complaint
History of Present Illness
Review of Systems
Relevant History
Allergies
Medications
Physical Examination
Clinical Discussion Questions
Bedside Manner Questions
Answer Key: Pain and Redness in Breast, Adult Female
Outcome
Insight from the PCP
References
Case 91: Stomach Pain and Bloating, Adolescent Female
Chief Complaint
History of Present Illness
Review of Systems
Relevant History
Allergies
Medications
Physical Examination
Clinical Discussion Questions
Bedside Manner Questions
Answer Key: Stomach Pain and Bloating, Adolescent Female
Outcome
Insight from the PCP
References
Case 92: Redness on Lower Right Leg, Adult Male
Chief Complaint
History of Present Illness
Review of Systems
Relevant History
Allergies
Medications
Physical Examination
Clinical Discussion Questions
Bedside Manner Question
Answer Key: Redness on Lower Right Leg, Adult Male
Outcome
Insight from the PCP
References
Case 93: Blood in Stool, Adult Female
Chief Complaint
History of Present Illness
Review of Systems
Relevant History
Allergies
Medications
Physical Examination
Clinical Discussion Questions
Bedside Manner Question
Answer Key: Blood in Stool, Adult Female
Outcome
Insight from the PCP
References
Case 94: Erection Difficulties, Adult Male
Chief Complaint
History of Present Illness
Review of Systems
Relevant History
Allergies
Medications
Physical Examination
Clinical Discussion Questions
Bedside Manner Question
Answer Key: Erection Difficulties, Adult Male
Outcome
Insight from the PCP
References
Case 95: Refusal to Eat or Drink, Pediatric Male
Chief Complaint
History of Present Illness
Review of Systems
Relevant History
Allergies
Medications
Physical Examination
Clinical Discussion Questions
Bedside Manner Question
Answer Key: Refusal to Eat or Drink, Pediatric Male
Outcome
Insight from the PCP
References
Case 96: Unexplained Weight Gain, Adult Female
Chief Complaint
History of Present Illness
Review of Systems
Relevant History
Allergies
Medications
Physical Examination
Clinical Discussion Questions
Bedside Manner Questions
Answer Key: Unexplained Weight Gain, Adult Female
Outcome
Insight from the PCP
References
Case 97: Worsening, Sharp Abdominal Pain, Adult Female
Chief Complaint
History of Present Illness
Review of Systems
Relevant History
Allergies
Medications
Physical Examination
Clinical Discussion Questions
Bedside Manner Question
Answer Key: Worsening, Sharp Abdominal Pain, Adult Female
Outcome
Insight from the PCP
References
Case 98: Lump in Breast, Adult Male
Chief Complaint
History of Present Illness
Review of Systems
Relevant History
Allergies
Medications
Physical Examination
Clinical Discussion Questions
Bedside Manner Question
Answer Key: Lump in Breast, Adult Male
Outcome
Insight from the PCP
References
Case 99: Back Pain and Poor Posture, Pediatric Female
Chief Complaint
History of Present Illness
Review of Systems
Relevant History
Allergies
Medications
Physical Examination
Clinical Discussion Questions
Bedside Manner Questions
Answer Key: Back Pain and Poor Posture, Pediatric Female
Outcome
Insight from the PCP
References
Case 100: Itchy Bumps, Pediatric Female
Chief Complaint
History of Present Illness
Review of Systems
Relevant History
Allergies
Medications
Physical Examination
Clinical Discussion Questions
Bedside Manner Question
Answer Key: Itchy Bumps, Pediatric Female
Outcome
Insight from the PCP
References
Case 101: Frequent Illness and Fatigue, Adult Female
Chief Complaint
History of Present Illness
Review of Systems
Relevant History
Allergies
Medications
Physical Examination
Clinical Discussion Questions
Bedside Manner Question
Answer Key: Illness and Fatigue, Adult Female
Outcome
Insight from the PCP
References
List of Abbreviations
Index

Citation preview

Sampath Wijesinghe

PRIMARY 101 CARE CASE STUDIES A Workbook for Clinical and Bedside Skills

About the pagination of this eBook Due to the unique page numbering scheme of this book, the electronic pagination of the eBook does not match the pagination of the printed version. To navigate the text, please use the electronic Table of Contents that appears alongside the eBook or the Search function. For citation purposes, use the page numbers that appear in the text.

Praise

for

101 Primary Care Case Studies

Dr. Wijesinghe’s book is not only a great multidisciplinary learning tool, but really highlights the best of evidence-based practice and interprofessional approaches to patient care. Dr. Beth Smolko, DMSc, PA-C President, American Academy of Physician Assistants

Dr. Sam Wijesinghe has assembled a group of primary care providers to craft an outstanding workbook of 101 common problems in practice. Each one reads like well-presented grand rounds that are informative, current, and concise. A perfect resource for the student who is encountering a new problem for the first time. A superior way to learn. Thomas E. Piemme, MD Emeritus Professor, George Washington University A multidisciplinary group of primary care providers led by Dr. Wijesinghe has put together an exceptional collection. Each scenario is well presented in a succinct format with evidence-based evaluation and treatment. Students learn well from case study presentations making this a perfect resource to build their knowledge base. Dr. Cindy Cooke, DNP, FNP-C, FAANP Adjunct Faculty at the University of Mary in Bismarck, North Dakota, and AANP past president The case study method is a time-honored approach to clinical teaching. Dr. Wijesinghe has assembled a seasoned group of expert clinicians and educators to produce a useful volume for students and practicing clinicians that provides a welcome dose of caring along with the curing. Dr. James F. Cawley, MPH, PA-C, DHL (Hon) Professor, School of PA Practice, Florida State University; Visiting Professor, Scholar-in-Residence PA Leadership and Learning Academy University of Maryland, Baltimore These primary care–based case studies are wonderfully composed to elicit the required critical thinking skills essential for patient care. I especially enjoyed the clinician insights at the conclusion of each case study that revealed their thought processes. Include this book by Dr. Wijesinghe in your resources to learn compassionate and competent primary care. Dr. Stephen A. Ferrara, DNP, FNP, FAANP, FNAP Associate Professor/Executive Director, The Nurse Practitioner Association, New York State This compilation of cases commonly encountered by primary care providers outlines a systematic and a practical approach to evaluation and management. I have no doubt that this book will be an invaluable source of learning and reference, not only for students in medical training, but also for all primary care providers. Leonard Ranasinghe, PhD, MD, DABEM Professor of Medical Education and Emergency Medicine, California Northstate University College of Medicine, Elk Grove

101 PRIMARY

CARE CASE STUDIES

Sampath ("Sam") Wijesinghe, DHSc, PA-C, AAHIVS, is a principal faculty member and clinical site director for the Central Valley California region at Stanford University School of Medicine’s master’s in science physician assistant program. He completed his PA education at Union College, where he received a master’s degree in PA studies. He also has a master’s degree in management information systems from the University of Nebraska and a doctor of health science degree with an emphasis in global health from A.T. Still University. Dr. Wijesinghe completed an HIV and AIDS clinical fellowship at University of California San Francisco, Fresno. He has been an HIV/AIDS specialist since 2014 and has worked in primary care medicine since 2010, primarily in underserved communities. He has been involved in medical education since 2013. His clinical interests include primary care medicine, HIV medicine, and global health. Prior to joining Stanford University School of Medicine, Dr. Wijesinghe was a clinical assistant professor at University of California, Davis. Dr. Wijesinghe practices primary care medicine and HIV medicine at Adventist Health Medical Center, Fowler, a small town in California, and Madera Community Hospital in Madera, California. Dr. Wijesinghe is passionate about teaching the next generation of clinicians. He has practiced in areas of need, including primary care and HIV medicine in underserved areas. He is passionate about medical education because it is an opportunity to improve patient outcomes and give back to the profession. A highly sought-after speaker and lecturer, Dr. Wijesinghe has presented at several national conferences and events and educates the next generation of PAs as a clinical preceptor. Dr. Wijesinghe lives with his wife, Nuwan, and has two children, Rynee and Ryler. He loves to travel with his family, listen to music, watch sports, and play sports whenever possible.

101 PRIMARY

CARE CASE STUDIES A Workbook for Clinical and Bedside Skills Sampath Wijesinghe, DHSc, MS, MPAS, AAHIVS, PA-C Editor/Author

Copyright © 2021 Springer Publishing Company, LLC All rights reserved. No part of this publication may be reproduced, stored in a retrieval system, or transmitted in any form or by any means, electronic, mechanical, photocopying, recording, or otherwise, without the prior permission of Springer Publishing Company, LLC, or authorization through payment of the appropriate fees to the Copyright Clearance Center, Inc., 222 Rosewood Drive, Danvers, MA 01923, 978-750-8400, fax 978-646-8600, [email protected] or on the Web at www.copyright.com. Springer Publishing Company, LLC 11 West 42nd Street, New York, NY 10036 www.springerpub.com connect.springerpub.com/ Acquisitions Editor: Suzanne Toppy Compositor: diacriTech ISBN: 9780826182722 ebook ISBN: 9780826182739 DOI: 10.1891/9780826182739 21 22 23 24 / 5 4 3 2 1 The author and the publisher of this Work have made every effort to use sources believed to be reliable to provide information that is accurate and compatible with the standards generally accepted at the time of publication. Because medical science is continually advancing, our knowledge base continues to expand. Therefore, as new information becomes available, changes in procedures become necessary. We recommend that the reader always consult current research and specific institutional policies before performing any clinical procedure or delivering any medication. The author and publisher shall not be liable for any special, consequential, or exemplary damages resulting, in whole or in part, from the readers’ use of, or reliance on, the information contained in this book. The publisher has no responsibility for the persistence or accuracy of URLs for external or third-party Internet websites referred to in this publication and does not guarantee that any content on such websites is, or will remain, accurate or appropriate. Library of Congress Control Number: 2020914270 Contact [email protected] to receive discount rates on bulk purchases. Publisher’s Note: New and used products purchased from third-party sellers are not guaranteed for quality, authenticity, or access to any included digital components. Printed in the United States of America.

This book is dedicated to My mentor, Alex Moir (1962–2015), who taught me true compassion and was a mentor and role model beyond compare. It was always my dream that we’d coauthor this book, but that dream could not come true. The world lost a fine man, who could have taught compassion and competent care to generations of future clinicians. My parents, David (1944–2015) and Rita Wijesinghe. When I was growing up, our family did not have much, but my parents’ unconditional love and fight to provide a better future for me and my loving sister Mangali laid a solid foundation for our journeys. Importantly, my parents always believed in me. I love you more! My wife, Nuwan. Meeting you in high school was the best thing that ever happened to me. Marrying you was the best decision I ever made. Everything I do personally or professionally is possible because you are in my life. I love you more than you can imagine. My children, Rynee and Ryler. Throughout the preparation of this book, you both selflessly gave me the space and time I needed. I know I have taken too much time away from you because of this publication. Your love and support from the beginning to end has meant the world to me. You two are the reason I want to contribute and make the world a better place! PAs, NPs, physicians, and future clinicians. We know practicing medicine and lifelong learning are inseparable. Let’s continue to support each other and learn from one another. I applaud your decision to practice medicine and make a difference in many people’s lives.

CONTENTS Contributors xi

16 Facial Numbness, Adult Male  61

Reviewers xv

17 Elbow Injury, Pediatric Male  65

Foreword by Alfred M. Sadler Jr, MD FACP ScD (Hon)  xvii

18 Thick Yellow Toenails, Adult Female  69

Foreword by Lynn H. McComas, DNP, ANP, PHN-C  xix

19 Speech Difficulties, Geriatric Male  73 20 Mood Changes, Adult Male  77

Foreword by Michael J. Huckabee, PhD, MPAS, PA-C  xxi

21 Fever and Lump on Neck, Adolescent Male 81

Preface xxiii

22 Hand Pain at Night, Adult Female  85

Acknowledgments xxvii

23 Burning With Urination, Adult Male  89

How to Use This Book  xxix

24 Constipation, Adult Female  93

1 Ankle and Calf Pain, Adult Male  1

25 Recurrent Vomiting, Pediatric Female  97

2 Fatigue and Weakness, Adult Female  5

26 Fatigue and General Malaise, Adult Male  101

3 Abdominal Pain, Adolescent Male  9

27 Nausea, Vomiting, and Headache, Adolescent Male 105

4 Left Hip Pain, Adult Female  13 5 Vaginal Bleeding, Adult Female  17 6 Numbness in Hands and Legs, Geriatric Male 21 7 Painful Rash, Geriatric Male  25 8 Fever and Body Aches, Adult Male  29 9 Fever and Rash, Pediatric Male  33

28 Dizziness and Difficulty Hearing, Adult Male 109 29 Bad Headache, Adult Female  113 30 Abdominal Pain, Adult Female  117 31 Painful Urination, Adult Male  121 32 Frequent Diarrhea, Adolescent Female  125

10 Blurry Vision, Adult Female  37

33 Repetitive Bruising on Legs, Pediatric Female 129

11 Doesn’t Follow Directions, Pediatric Male  41

34 Swollen Knee, Pediatric Female  133

12 Crusty, Irritated Left Eye, Pediatric Male  45

35 Widespread Painful Rash, Pediatric Female 137

13 Frequent Urination, Adult Male  49 14 Vomiting, Pediatric Male  53 15 Difficulty Relating to Others, Pediatric Male 57

36 Blurred Vision in Left Eye, Adult Female  141 37 Fever and Back Pain, Geriatric Female  145 38 Sore Throat, Fever, and Chills, Adult Male  149

viii  Contents

39 Bad Cough and Fever, Adolescent Female  153

67 Fatigue and Joint Aches, Adult Male  267

40 Headache and Fever, Pediatric Male  157

68 Shoulder Pain, Adult Male  271

41 Heart Palpitations, Adult Female  161

69 Unexplained Weight Loss, Adult Female  275

42 Double and Blurred Vision, Adolescent Male 165

70 Lightheadedness, Geriatric Female  279

43 Nipple Pain, Adult Female  169 44 Irregular Menstrual Cycle, Adolescent Female 173 45 Right Knee Pain, Pediatric Female  177 46 Chest Pain, Adolescent Male  181 47 Recurrent Pain and Redness in Left Foot, Adult Female 185 48 Muscle and Joint Pain, Adult Female  189

71 Painful Lump in Armpit, Adult Female  283 72 Fever, Cough, and Runny Nose, Pediatric Female 287 73 Diabetes Check, Adult Male  291 74 Buttocks Pain, Geriatric Male  295 75 Abdominal Pain, Adolescent Male  299 76 Irritable and Crying Toddler, Pediatric Male 303

49 Constipation, Adolescent Female  193

77 Swelling Around Eyes and Ankles, Adolescent Male 307

50 Coughing and Wheezing, Pediatric Male  197

78 Bilateral Heel Pain, Adult Female  311

51 Shoulder Pain, Adult Male  201

79 Dry, Cracked Skin, Adult Female  315

52 Painless Anal Bumps, Adult Transgender Male 205

80 Urine Leakage, Geriatric Female  319

53 Fatigue and Decreased Libido, Adult Male  209 54 Unsteady Gait and Nausea, Pediatric Female 213

81 Coughing and Gagging, Pediatric Male  323 82 Eye Injury, Adult Male  327 83 Pain in Great Toe, Adult Female  331

55 Excess Facial Hair, Adolescent Female  217

84 Bilateral Leg Pain, Pediatric Male  335

56 Fever and Body Aches, Adult Male  221

85 Confusion, Geriatric Female  339

57 Large Pimple on Leg, Adult Female  225

86 Lump in Throat, Adult Male  345

58 Chest Pain, Adult Female  229

87 Fever and Rash, Adult Male  349

59 Right Knee Pain, Geriatric Male  233

88 Extreme Weakness, Adult Male  353

60 Painful Urination, Adolescent Male  237

89 Leg Pain, Geriatric Male  357

61 Persistent Bad Cough, Adult Female  241

90 Pain and Redness in Breast, Adult Female  361

62 Well Visit, Geriatric Male  245

91 Stomach Pain and Bloating, Adolescent Female 365

63 Low Platelets, Adult Male  249 64 Skin Lesions After Sun Exposure, Adult Female 255

92 Redness on Lower Right Leg, Adult Male  369 93 Blood in Stool, Adult Female  373

65 Daytime Fatigue, Adult Male  259

94 Erection Difficulties, Adult Male  377

66 Shortness of Breath, Geriatric Male  263

95 Refusal to Eat or Drink, Pediatric Male  381

Contents ix

96 Unexplained Weight Gain, Adult Female  385 97 Worsening, Sharp Abdominal Pain, Adult Female 389

101 Frequent Illness and Fatigue, Adult Female 405

List of Abbreviations  409

98 Lump in Breast, Adult Male  393

Index 417

99 Back Pain and Poor Posture, Pediatric Female 397

Answer Key Available at https://connect .springerpub.com/content/book/ 978-0-8261-8273-9

100 Itchy Bumps, Pediatric Female  401

CONTRIBUTORS Valerie Berry, MD

Assistant Professor Master of Science Physician Assistant Program College of Health Sciences and Human Services California State University, Monterey Bay Salinas, California Michael S. Burney, EdD, MS, PA-C

Program Director/Chair, Clinical Associate Professor Chapman University PA Program Crean College of Health and Behavioral Sciences Irvine, California Michael Castillo, MD

Department of Family Medicine Kaiser Permanente Fresno, California Rachel Chappell, MHSc, PA-C

Program Director, Clinical Assistant Professor Interdisciplinary Human Studies, Physician Assistant Program School of Allied Health Professions Louisiana State University Health Science Center New Orleans, Louisiana Mark P. Christiansen, PhD, PA-C

Program Director and Department Chair Physician Assistant Program Department of Physician Assistant Education University of the Pacific Sacramento, California Dolores Davis, RN, FNP APRN-C

Madera Community Hospital, Family Health Services VA Central California Health Care System Fresno, California David Duensing, DO

Pediatrics, PC Lincoln, NE University of Nebraska Medical Center Adjunct Associate Professor Department of Pediatrics Omaha, Nebraska

Volunteer Clinical Preceptor, Advanced Practice Nursing Program University of Nebraska Medical Center Omaha, Nebraska Volunteer Clinical Preceptor, Physician Assistant Program Union College Lincoln, Nebraska Christopher P. Forest, DHSc, DFAAPA, PA-C

Founding PA Program Director Master of Science Physician Assistant Program College of Health Sciences and Human Services California State University, Monterey Bay Salinas, California Tosi Gilford, MD, PA-C

Program Director, Assistant Professor Physician Assistant Studies Program, Clinical and Diagnostic Sciences Department University of Alabama at Birmingham Birmingham, Alabama Nancy Hamler, DMSc, MPA, RDN, PA-C

Assistant Professor Physician Assistant Program Department of Physician Assistant Education University of the Pacific Sacramento, California Surani Hayre-Kwan, MBA, DNP, FNP-BC, FACHE, FAANP

Director, Professional Practice & Nursing Excellence Sutter Health Sacramento, California Nurse Practitioner, Russian River Health Center Guerneville, California Cherilyn M. Hendrix, DHEd, MSBME, PA-C, DFAAPA

Program Director/Associate Professor University of Maryland Baltimore/Anne Arundel Community College Collaborative Physician Assistant Program University of Maryland Baltimore Graduate School Baltimore, Maryland

xii  Contributors Lisa Hood, DNP, MSN, RN, FNP-C

Susan LeLacheur, DrPH, PA-C

Michael J. Huckabee, PhD, MPAS, PA-C

Kristy Luciano, MS, PA-C

Amanda J. Ingalls, MS, PA-C

Erin N. Lunn, PA-C, MHS

Family Nurse Practitioner Family Health Services Madera Community Hospital Madera, California

Director, Physician Assistant Program Associate Professor, Senior Associate Consultant-II College of Medicine and Sciences Mayo Clinic Rochester, Minnesota Clinical Instructor of Family Medicine Physician Assistant Program Keck School of Medicine University of Southern California Alhambra, California Johnny Jimenez, MSN, APRN, FNP-C

Nurse Practitioner, Family Medicine Sutter Health Memorial Hospital Los Banos Los Banos, California Paramjit Kaur, MSN, FNP-C

Past Family Practice Nurse Practitioner Adventist Medical Clinic Sanger, California Gerald Kayingo, PhD, MMSc, PA-C

Executive Director, Assistant Dean and Professor Physician Assistant Leadership and Learning Academy University of Maryland Graduate School Baltimore, Maryland Jason R. Kessler, MD, FAAP

Medical Director/Pediatrician Primary Health Care, Inc. Des Moines, Iowa Vasco Deon Kidd, DHSc, MPH, MS, PA-C

Office of Advanced Practice and UCI Health Department of Orthopaedic Surgery Orange, California Timothy Kuntz, MPAS, PA-C

Assistant Professor Physician Assistant Program Union College Lincoln, Nebraska

Aimee Larson, MSPA, PA-C

Program Director, Clinical Assistant Professor School of Education and Human Services Canisius College Physician Assistant Studies Buffalo, New York

Professor of Physician Assistant Studies The George Washington University School of Medicine and Health Sciences Washington, DC Assistant Professor and Director of Didactic Education Physician Assistant Program College of Health Sciences Midwestern University Downers Grove, Illinois Assistant Professor, Director of Clinical Education Department of Physician Assistant Studies University of South Alabama Mobile, Alabama Hema Majeno, PA-C

Internal Medicine/Hospitalist PA-C Mercy Hospital Merced, California David Malebranche, MD, MPH

Associate Professor of Medicine Department of Medicine Morehouse School of Medicine Atlanta, Georgia April H. Martin, MSPAS, PA-C

Program Director and Assistant Professor School of Health Sciences Francis Marion University Florence, South Carolina Mario Martinez, MD

Program Director Kaweah Delta Family Medicine Residency Program Visalia, California Medical Director Adventist Health Community Care Clinic, Central Valley Network Hospitalist, Vituity-Adventist Community Hospital Reedley, California Lynn H. McComas, DNP, ANP-C, PHN

President/CEO, PreceptorLink Encinitas, California

Kameko Hazley McGuire, DNP, PMHNP-BC, NP-C, RN

Visiting Professor, Chamberlain University MSN-FNP Program Downers Grove, Illinois Adjunct Faculty, Regis College PMHNP Program Weston, Massachusetts Adjunct Faculty, South University FNP Program Savannah, Georgia Graduate Adjunct PMHNP Professor Liberty University Lynchburg, Virginia

Contributors xiii

Jennifer Momen, MD, MPH, FAAP

Assistant Professor, Program Director Department of Human Performance Division of Physician Assistant Studies West Virginia University School of Medicine Morgantown, West Virginia Michael N. Moya, MD

Academic Hospitalist Associate Program Director Family Medicine Residency Program Department Chair, Family Medicine St. Agnes Medical Center Fresno, California Andrew Nevins, MD

Clinical Associate Professor Stanford University School of Medicine Stanford, California Karen Paolinelli, MSN, APRN, FNP-C, PA-C, DFAAPA

CEO/President Madera Community Hospital Madera, California

Shantha Parameswaran, MD, FAAP

Pediatrician One Community Health Sacramento, California Tushar M. Patel, MD

Pediatrician Children’s Medical Centers of Fresno Fresno, California Jason Radke, MMS, PA-C

Alfred M. Sadler Jr, MD, FACP, ScD (Hon)

Master of Science Physician Assistant Program College of Health Sciences and Human Services California State University, Monterey Bay Salinas, California Anabel Segovia, RN, MSN, FNP-C

Nurse Practitioner Sierra Kings Reedley, California

Tana Summers, MS, PA-C

Assistant Program Director Physician Assistant Program Samuel Merritt University Oakland, California Laura L. Van Auker, DNP, APRN, FNP-BC, MSN, SN-C

Assistant Clinical Professor Betty Irene Moore School of Nursing University of California, Davis Sacramento, California James Van Rhee, MS, PA-C

Program Director/Associate Professor Yale School of Medicine, Physician Assistant Online Program Yale University New Haven, Connecticut Veronica Vo, MPAC, PA-C

Family Medicine Physician Assistant Family Healthcare Network Hanford, California

Department Chair & Program Director Assistant Professor College of Health Professions Rosalind Franklin University of Medicine and Science North Chicago, Illinois

Anne Walsh, PA-C, MMSc, DFAAPA

Jennifer Ramos, MPAP, PA-C

Nurse Practitioner, Sexual Health Specialist College of Health Nurse Practitioner Tufts University Health Services Medford, Massachusetts

Clinical Instructor of Family Medicine Physician Assistant Program Keck School of Medicine University of Southern California Alhambra, California Karla Reinhart DNP, FNP-C, ARNP

Murphy Creek Wellness, Nurse Practitioner/Owner Medford, Oregon Oregon Health & Science University Ashland, Oregon Sara Rygol, MPAS, PA-C

Primary Care Physician Assistant CHI Health Clinic Council Bluffs, Iowa

Clinical Associate Professor Chapman University PA Program Crean College of Health and Behavioral Sciences Irvine, California Ariel L. Watriss, MSN, NP-C

Jennifer C. Weeks, DNP, FNP-C

Family Nurse Practitioner MedNow Urgent and Primary Care Augusta, Georgia Sampath Wijesinghe, DHSc, PA-C, AAHIVS

Principal Faculty, Master of Science in Physician Assistant Studies Stanford School of Medicine Primary Care PA-C and HIV Specialist at Adventist Health Central Valley Network, California Primary Care PA-C and HIV Specialist at Madera Community Hospital Madera, California

REVIEWERS Krishan Ariyarathna, MD

Internal/Hospital Medicine Nebraska Hospitalist, LLC Omaha, Nebraska Alan Brokenicky, MPAS, PA-C

Susan M. Fernandes, LPD, PA-C

Associate Dean for PA Education Clinical Professor of Pediatrics and Medicine Stanford School of Medicine Stanford, California

Academic Co-Director Physician Assistant Studies Program College of Medicine and Sciences Mayo Clinic Rochester, Minnesota

Nichole A. Flores, BA, MA. MAOB, PsyD

Joan Caruso, MPAS, PA-C

Associate Dean School of Allied Health Professions Program Director Department of PA Medicine Greer, South Carolina

Physician Assistant Program Director/Chair, Clinical Assistant Professor Department of Physician Assistant Studies Clarkson University Potsdam, New York Rachel Chappell, MHSc, PA-C

Program Director, Clinical Assistant Professor Interdisciplinary Human Studies, Physician Assistant Program School of Allied Health Professions Louisiana State University Health Science Center New Orleans, Louisiana Simerjit Singh Dhaliwal, PA-C, MS, RCP

Family Medicine, United Health Centers of San Joaquin Valley Selma, California Christy Eskes, DHSc, MPA, PA-C

Instructor, Former Assistant Professor and Past Program Director Department of Physician Assistant Sciences Loma Linda University Loma Linda, California Lead Advanced Practice Provider for Correctional Health Emergent Medical Associates Los Angeles, California Michael Estrada, DHSc, MS, PA-C

Founding Director and Chair Physician Assistant Program College of Arts and Sciences University of La Verne La Verne, California

Director of Behavioral Health Behavioral Health, Valley Health Team, Inc. Kerman, California Jordan Hairr, EdD, MSPAS, PA-C

Virginia McCoy Hass, RN, DNP, FNP-C, PA-C

Associate Clinical Professor (retired) Betty Irene Moore School of Nursing University of California, Davis Sacramento, California Megan Heidtbrink, MPAS, PA-C

Program Director, Associate Professor Union College Physician Assistant Program Union College Lincoln, Nebraska Cherilyn M. Hendrix, DHEd, MSBME, PA-C, DFAAPA

Program Director/Associate Professor University of Maryland Baltimore/Anne Arundel Community College Collaborative Physician Assistant Program University of Maryland Baltimore Graduate School Baltimore, Maryland Trenton Honda, PhD, MMS, PA-C

Associate Professor and Division Chief Division of PA Studies University of Utah Salt Lake City, Utah Amanda J. Ingalls, MS, PA-C

Clinical Instructor of Family Medicine Department of Family Medicine Division of Physician Assistant Studies University of Southern California Alhambra, California

xvi  Reviewers Jason R. Kessler, MD, FAAP

Medical Director/Pediatrician Primary Health Care, Inc. Des Moines, Iowa Rhonda Larsen, MHS, PA-C

Associate Program Director Clinical Assistant Professor Master of Science in Physician Assistant Studies Stanford School of Medicine Stanford, California Seth Lauterbach, MPAS, PA-C, CAQ-EM

Nebraska Emergency Medicine, PC, Bryan Trauma Bryan West Medical Center Lincoln, Nebraska Kevin Lohenry, PhD, PA-C

Clinical Professor of Family Medicine Director of USC Primary Care Physician Assistant Program Vice Chair for Educational Affairs University of Southern California Los Angeles, California Stephanie McGilvray, PA-C, MMS

Physician Assistant Program Department Chair and Program Director University of South Alabama Mobile, Alabama Jennifer Momen, MD, MPH, FAAP

Assistant Professor, Program Director Department of Human Performance Division of Physician Assistant Studies West Virginia University School of Medicine Morgantown, West Virginia Nina Multak, PhD, MPAS, PA-C, DFAAPA

Associate Dean and Randolph B. Mahoney Director School of Physician Assistant Studies University of Florida Gainesville, Florida Ila Naeni, DO

Associate Program Director & HS Associate Clinical Professor Family and Community Medicine University of California San Francisco-Fresno Fresno, California Ian Nelligan, MD, MPH

Clinical Assistant Professor and Family Medicine Physician Stanford School of Medicine Stanford, California Courtney Nelson, MMS, PA-C

Clinical Assistant Professor of Medicine Master of Science in Physician Assistant Studies Stanford School of Medicine Stanford, California

Jennifer Ramos, MPAP, PA-C

Clinical Instructor of Family Medicine Physician Assistant Program Keck School of Medicine University of Southern California Alhambra, California John Ramos, MMS, PA-C, EM-CAQ

Assistant Professor Physician Assistant Studies School of Health and Natural Sciences Dominican University of California San Rafael, California Michele Toussaint, MS, PA-C

Director of Student Scholarship and Clinical Assistant Professor of Medicine Master of Science in Physician Assistant Studies Stanford School of Medicine Stanford, California Judy Truscott, MPAS, PA-C

Program Director, Associate Professor Physician Assistant Studies Chatham University Pittsburgh, Pennsylvania Joseph Weber, DHSc, MPAS, MBA, PA-C

Physician Assistant Studies Department Chair and Program Director Presbyterian College Clinton, South Carolina Dee White, DNP, FNP

Doctor of Nurse Practitioner, Family Medicine Adventist Health Medical Clinic Kingsburg, California Sampath Wijesinghe, DHSc, PA-C, AAHIVS

Principal Faculty, Master of Science in Physician Assistant Studies Stanford School of Medicine Primary Care PA-C and HIV Specialist at Adventist Health Central Valley Network, California Primary Care PA-C and HIV Specialist at Madera Community Hospital Madera, CA Gordon H. Worley, MSN, RN, FNP-C, ENP-C

Assistant Clinical Professor Betty Irene Moore School of Nursing University of California, Davis Sacramento, California

FOREWORD This is a remarkable collection of primary care case studies, presented in a unique and comprehensive manner. Included is a broad spectrum of cases that emphasize the scientific as well as the psychologic and emotional dimensions of care. We can all learn from each other as physicians, physician assistants, and nurse practitioners. Anyone interested in primary care will benefit from this book. The book is arranged so you can pick and choose what interests you or relates to a specific case that you have. It is an ideal volume for those engaged in the primary care education of physicians, physician assistants, and nurse practitioners. It will also be a valuable resource for the primary care practitioner. Besides the standard presentation of a patient problem, each case includes evidence-based information, references for additional reading, ICD-10 and CPT codes (very practical), topics for education, legal concerns, and specific items to be addressed by the broader healthcare team from receptionist to specialist. Bedside manner questions allow the educator or practitioner to expand the scope of inquiry, as time and interest permit. Finally, the narrative by the primary care provider brings the discussion from the didactic/scientific to the personal. I am not aware of any other volume that does this. I congratulate Sampath Wijesinghe, who is a sterling example of a primary care educator and practitioner, for putting together this remarkable collection of cases and the stories that go with them. I know that the reader will find this multiauthored book invaluable and inspiring. Alfred M. Sadler Jr MD, FACP, ScD (Hon) Senior Advisor, California State University Master of Science Physician Assistant Program, Salinas, California

FOREWORD Primary care medicine is the heart of medicine. It is a field where you never know what patient is going to walk through your door next. Frequently, what is listed on the schedule is not what you discover as you sit across from your patient and ask, “So what brings you in today?” It’s also not what you discover when you dig a little deeper into a patient’s story. More than once, I have called out to my puzzled medical assistant, “Can you grab me an anoscope?” for a patient who came in for a sore throat! Variety is part of what makes primary care so wonderful. As a clinician, you also get to build wonderful, long-term relationships, and this is my favorite part of primary care medicine. When you have seen patients year upon year, possibly even from birth, you really get to know them. In time, you recognize when there is a change in the way Mr. Jones is dressed or note that his smile is a little less bright or his steps a little more shuffled. There will likely be a day when your patient says “You saved my life” and rewards you with their homemade chocolates. There is never a dull day in primary care medicine, and as clinicians, we really make a difference in the population in which we serve. For these reasons, I love primary care medicine and there is no place I would rather work as a nurse practitioner. As a new clinician, you will really find it is true when that medicine is both art and science. You will develop your individual approach to the art of medicine. As you learn and grow, you will learn how to master your “medical paintbrush.” You will see 10 clinicians might treat the same patient in 10 different ways, and all of them may be correct in their approaches. Listening carefully, really listening to a patient, a family, caregivers, will improve your diagnostic skills. You will also learn how to most effectively and thoughtfully deliver education or bad news. Your clinical skills are absolutely vital, but your “soft skills” are invaluable. However, traditional classroom is not the most effective place to learn them. Soft skills come with time and experience and include communication skills, listening skills, and an ability to demonstrate empathy. It is your ability to “read” others. It is a vital part of our skill set as clinicians. It is our job to earn our patients’ trust and to give them the very best care. Even when our schedules are packed with back-to-back appointments and our inbox is stacked with labs and reports to be reviewed, we must remember that “the patient in Room 1” is more than just a CPT code. If primary care medicine is the heart of medicine, then our patients are its soul. We have to work collaboratively as a team—from the front office person, to the clinician, to the biller. We have to treat patients the way we would want to be treated, because some day you may be the patient. Clinical rotations can be an overwhelming experience for any student. Despite all of your didactic preparation, you may feel completely underprepared. The first year as a new clinician is a formidable learning trajectory. Suddenly, you are making life and death decisions about your patient. Apps and reference manuals are helpful, but what you really want is real-world advice from those in the trenches, and that is exactly what this book provides. It is a real-world, practical handbook of genuine scenarios. All the cases in the book are real-life case studies shared by the authors. Through a stepwise process, you will be challenged to critically think through the steps of clinical decision-making—from the chief complaint to billing and coding, they are all discussed. In addition to clinical decisions, you will hear each author’s approach and some of the soft skills used. This book will be an invaluable asset, not only to a student or new clinician working in primary care but also to the seasoned professional who wants to gain insight by looking at other approaches in the art and science of medicine. Lynn H. McComas, DNP, ANP-C, PHN President/CEO PreceptorLink, Encinitas, California

FOREWORD Some might say there are plenty of primary care textbooks today. Why one more? I’m here to offer three reasons for 101 Primary Care Case Studies: the art, the appeal, and the author. This text reminds us of the art of medicine, beyond the requirement to practice by evidence-based medicine. Yes, each case study provides the science and relevant practice guidelines behind a particular and vital diagnosis. To practice medicine solely by evidence alone leads to a risk of recipe-card decisions rather than personalized patient care. Heuristics, when conjoined with evidence, leads to best practice. 101 Primary Care Case Studies enhances this foundational evidence-based content, integrating the artistic touch of medicine by personalizing each case to a real patient. Each chapter includes the unique section, titled Insight from the PCP, which offers a clinician’s pearls of wisdom about bedside manner and achieving optimal patient outcomes based on science and first-hand experience. The book’s collection of pearls alone offers tremendous value to the reader seeking to guide patients toward the best health outcomes. As it is written for primary care providers, this text emphasizes the appeal our career offers. In primary care, we have the privilege to develop patient relationships that extend across the lifespan. Representing these cradle-to-grave lives, the case studies in this book read as stories from the family. Only primary care expects a consistent patient relationship that celebrates births, school events, sporting achievements, graduations, marriages, promotions, relocations, retirements, and bereavements, with all the successes and disappointments along the journey. Clinical decisions wrestled in these pages are made with consideration of how it affects the individual life of a patient at a particular time. By reading only a handful of case studies herein, one comes away with how each patient deserves an individualized approach via team-based care with full consideration of the person’s current life circumstances. That approach epitomizes why we in primary care love what we do. Finally, to those who might say we have enough primary care textbooks, you don’t know the author, Dr. Wijesinghe. He cares for his patients with both exceptional clinical skills and compassionate bedside manners—and emphasizes the importance of both to be a good clinician. He respects all members of the healthcare team as critical to optimal patient care, including receptionists, the cleaning crew, and the ever-important medical assistant. Dr. Wijesinghe seeks to be a part of the solution whenever possible (why he chose primary care medicine in rural clinics including HIV medicine, where healthcare professional shortages are the greatest). So, when using a textbook by an author with this indomitable spirit of integrity, the reader can expect an excellent opportunity to learn from a master among the most caring and professional of clinicians. For learners entering the primary care professions of health delivery, or for learners with years of medical experience, your library is incomplete without this book. Nothing else combines the art of medicine with the appeal of primary care than what this author has created. We will serve our patients with better care by integrating the pearls of this text into our practices. Michael J. Huckabee, PhD, MPAS, PA-C Director, Physician Assistant Program, Associate Professor, Senior Associate Consultant-II, Mayo Clinic, Rochester, Minnesota

PREFACE Introduction

Primary care medicine is the largest medical specialty in the United States. While physician assistants, nurse practitioners, and physicians branch into specialties, a majority practices in primary care. When practicing primary care medicine, clinical skills and soft, or bedside manner, skills are important. Although there are many clinical medicine textbooks for students to learn all the important data about the hundreds of conditions a primary care provider will treat, there are few that use case studies as a primary means to help students work through a real patient scenario to strengthen clinical skills, let alone soft skills, which are often overlooked in a packed clinical curriculum. Current case study products typically focus solely on diagnostic skill. None focus on the combination of both clinical and bedside skills. This book is intended to bridge the gap between the objective clinical content presented in the didactic phase of a PA, NP, or medical curriculum and the clinical application/rotation phase. It can be used during both phases of a program. Students will use book knowledge to work through a real-life chief complaint presented by a patient, and through a series of questions determine the appropriate focused exam, workup, diagnosis, and treatment for the patient. Real-life medicine is not organized by organ system or diagnosis. Most often, patients will present with vague descriptions of symptoms, and a PCP will have to surmise the best approach to the problem. For example, a patient who presents with chest pain may have a diagnosis related to the heart, but the problem could also be related to anxiety or reflux.

The Origin

of

This Book

When I was a PA student, I enjoyed learning from clinicians who shared patient cases. I knew I would see similar cases when I started practice. When I began teaching at the University of California Davis PA/NP program, I taught students based on real cases. I recognized immediately that students are keen to learn from real cases. My experience first as a PA student and then as a medical educator led me to propose a book based on real cases. My initial idea was to write a book to help PA, NP, and medical students acquire competent skills.

Meeting Alex Moir

In August 2010, I started practicing primary care medicine in Sanger, a small town in California. I practiced medicine with Dr. Alex Moir, my collaborative physician. During my first week, Dr. Moir invited me to have lunch with him so we could get to know each other. I was looking forward to our lunch and was nervous at the same time. During our lunch, I said, “Dr. Moir, I am happy to work for you.” He immediately corrected me, “Please don’t think you work for me. I want you to know that you work with me. We are one team.” On that day, I realized I had the opportunity to work with a great human being. Dr. Moir was my mentor, who practiced competent and compassionate care. I wanted to practice medicine like Dr. Moir practiced medicine. I always thought I was pretty compassionate, but, as I worked with Dr. Moir, I realized my compassionate care skills were developing even more. Dr. Moir was teaching me to deliver compassionate care. I was committed to sharing this powerful message with other clinicians, particularly with future clinicians, by keeping compassionate care central to the book’s focus. So, I compiled this book based on real patient cases. I wanted Dr. Alex Moir to coauthor this book, but he died in a tragic accident. My grief put the book on hold for a time.

xxiv  Preface

When I began to think about the book again, I decided I would dedicate it to Dr. Moir, whose mentorship influenced my life as a husband, father, and clinician. With that in mind, I reached out to some PCPs who provide competent and compassionate care to contribute cases. Some have taught me during my didactic training in PA school, some have trained me during my clinical clerkships, some are friends and colleagues, and some are students I had trained. I intentionally included a wide range of experience in this book, from very experienced clinicians to a few new clinicians. I wanted to present a variety of cases from clinicians with short-term and long-term experience. I hope these patient cases help future and practicing clinicians to learn and provide the best care to their patients. This book was a collaborative effort of many people with a common goal: taking care of patients and doing something meaningful for medical education. The goal of this book is to improve competent skills and bedside skills. There is no disagreement that we can teach competence skills to the next generation and improve clinical skills. However, there is disagreement on whether compassion can be learned. I believe it can be, and there is evidence in support of my belief.

Are Bedside Skills

and

Compassion Necessary?

Are Bedside Skills

and

Compassion Practical

Compassion is fundamental to the delivery of healthcare,1 something that patients, families, clinicians, and policy makers all agree.2 The first principle of the American Medical Education cites compassion as being necessary in the delivery of care.2 According to the Code of Ethics, “A physician shall be dedicated to providing competent medical care, with compassion and respect for human dignity and rights.”2 PAs and NPs are expected to practice the same. Consequently, compassion is required of all clinicians. to

Teach?

There has been a long-term debate whether compassion can be taught. Some argue that compassion is an innate quality of character.2 Current evidence suggests that compassion can be developed and sustained over time.2 At the beginning of training, students demonstrate different levels of inherent compassion. Subsequently, capacity varies from student to student depending on their character at the baseline.2 Recently, a randomized controlled trial on empathy training suggested that inherent qualities can be developed and sustained.3 Eight observational studies focused on educational interventions aimed at improving compassionate care provided by clinicians and students in a clinical setting.4–12 The researchers used a variety methods (journals, simulations, reflection, etc.) and found that students demonstrate improved self-awareness, clinical communication skills, job satisfaction, caregiving competence, satisfaction with provision, and caregiver and workplace wellness.4–12 Considering these results, it is safe to suggest that compassion is teachable. Another group of authors found that common exemplary characteristics can improve humanistic behavior. Some of these were nonverbal communication, overt demonstrations of respect, building a personal connection, and eliciting and addressing patients’ affective response to illness.13 The authors concluded that there are many ways clinical teachers demonstrate humanistic behavior at the bedside.13 Collectively, these findings suggest that as medical educators we have a role to play in teaching by example: Our humanistic behavior will build the same in students.14 I am optimistic that the Insight from the PCP section of this book will reveal the art of medicine and be a valuable resource to promote a love of humanity.

Conclusion

Humanism and competent care are the primary focus of this book. If you are a PA, NP, or medical student, this will be a helpful and practical workbook during your education. These are real cases, so you know exactly what took place and how the cases were managed. To protect patients’ privacy, names and other identifying information have been omitted, and some details about the cases were changed. Additionally, the authors who treated these patients do not have a byline on the case to further protect patient privacy. My goal is to share the practical aspect from each case and provide you an excellent learning opportunity while protecting every patient’s privacy. Working through this book prior to clinical clerkships will provide a good foundation. Also, for practicing PCPs or clinicians about to enter practice, this book will be a helpful and practical resource.

Preface xxv

When treating a patient, a PCP has an opportunity to provide comprehensive care that is both evidenced based and compassionate. All of us practicing clinicians and/or medical educators and future clinicians should be deliberate in our efforts to provide comprehensive care. It is my sincere hope that 101 Primary Care Case Studies will contribute to your ability to be a compassionate and competent PCP. Be well. Sampath ("Sam") Wijesinghe

References

1. Sinclair S, Torres MB, Raffin-Bouchal S, et al. Compassion training in healthcare: what are patients’ perspectives on

training healthcare providers? BMC Med Educ. 2016;16:169. doi:10.1186/s12909-016-0695-0

2. American Medical Association. Code of medical ethics. Principle 1. 2001. https://www.ama-assn.org/sites/ama-

assn.org/files/corp/media-browser/principles-of-medical-ethics.pdf

3. Sinclair S, Norris JM, McConnell SJ, et al. Compassion: a scoping review of the healthcare literature. BMC Palliat

Care. 2016;15:6. doi:10.1186/s12904-016-0080-0

4. Riess H, Kelley JM, Bailey RW, et al. Empathy training for resident physicians: a randomized controlled trial of a

neuroscience-informed curriculum. J Gen Intern Med. 2012;27(10):1280–1286. doi:10.1007/s11606-012-2063-z

5. Adamson E, Dewar B. Compassionate care: student nurses’ learning through reflection and the use of story. Nurse

Educ Pract. 2014;15(3):155–161. doi:10.1016/j.nepr.2014.08.002

6. Betcher DK. Elephant in the room project: improving caring efficacy through effective and compassionate

communication with palliative care patients. Medsurg Nurs. 2010;19(2):101–105.

7. Blanco MA, Maderer A, Price LL, et al. Efficiency is not enough; you have to prove that you care: role modelling 8. 9. 10. 11. 12. 13. 14.

of compassionate care in an innovative resident-as-teacher initiative. Educ Health (Abingdon). 2013;26(1):60–65. doi:10.4103/1357-6283.112805 Deloney LA, Graham CJ. Wit: using drama to teach first-year medical students about empathy and compassion. Teach Learn Med. 2003;15(4):247–251. doi:10.1207/s15328015tlm1504_06 Dewar B, Cook F. Developing compassion through a relationship centred appreciative leadership programme. Nurse Educ Today. 2014;34(9):1258–1264. doi:10.1016/j.nedt.2013.12.012 Fortney L, Luchterhand C, Zakletskaia L, et al. Abbreviated mindfulness intervention for job satisfaction, quality of life, and compassion in primary care clinicians: a pilot study. Ann Fam Med. 2013;11(5):412–420. doi:10.1370/ afm.1511 Kalish R, Dawiskiba M, Sung YC, Blanco M. Raising medical student awareness of compassionate care through reflection of annotated videotapes of clinical encounters. Educ Health (Abingdon). 2011;24(3):490. Shih CY, Hu WY, Lee LT, et al. Effect of a compassion-focused training program in palliative care education for medical students. Am J Hosp Palliat Care. 2013;30(2):114–120. doi:10.1177/1049909112445463. Weissmann PF, Branch WT, Gracey CF, et al. Role modeling humanistic behavior: learning bedside manner from the experts. Acad Med J Assoc Am Med Coll. 2006;81(7):661–667. doi:10.1097/01.ACM.0000232423.81299.fe Institute of Medicine. Improving Medical Education: Enhancing the Behavioral and Social Science Content of Medical School Curricula. National Academies Press; 2004.

ACKNOWLEDGMENTS I want to thank the Springer Publishing Company for the opportunity to publish this book. Particularly, I would like to thank my editor, Suzanne Toppy, for her patience and sound guidance. A total of 26 physician assistants, 12 nurse practitioners, and 13 physicians have contributed cases for this publication. Also, 25 PAs, 3 NPs, 5 physicians, and 1 psychologist have served as peer reviewers. Thank you for your contributions and reviews. A special thank you to the following individuals: L

L L

L L L

Dr. Dan Fernando and Dr. Thomas Jayawardane for inspiring me to pursue my advanced degree and reinforcing that the United States was the best place to reach my goals Mr. D. Ratnayake and Mrs. Celine Ratnayake for standing by me when I really needed them Dr. Derrick Gruen for suggesting to me that primary care medicine is the best fit for my personality Dr. Sue Fernandes and Dr. Gerald Kayingo for believing in me as a PA educator Dr. Virginia Hass for her expertise and editorial support Dr. Ade Anast for her editorial support from beginning to end. I wouldn’t have completed this book without your help.

HOW TO USE THIS BOOK L

L

Cases are organized randomly in the table of contents by a patient’s chief complaint, which is how a patient realistically presents in clinic. You will work through real-life patient cases* and answer a series of questions to determine the workup, diagnosis, and treatment for a patient. Except for question 6, the questions in each case are the same, to help guide you on how to methodically work through a patient’s complaint. The questions are as follows: 1. What is the differential diagnosis? Pay careful attention to the history of present illness,

review of systems, relevant history, and physical exam to determine a list of differential diagnoses. Provide a rationale for each diagnosis on your list. Briefly explain why or why not it is the most likely diagnosis.

2. What is the most likely diagnosis? Why? Based on the information available, determine

the most likely diagnosis. Some cases are straightforward; others are not. That is the reality in primary care. Keep in mind you are making a most likely diagnosis, not a definitive diagnosis.

3. Demonstrate your understanding about the pathophysiology in regard to the most

likely diagnosis. While this is not a focus of the book, it is practical to have a basic understanding of a disease.

4. Should tests/imaging studies be ordered? Which ones? Why? Think about tests/

imaging beyond the primary care setting as well. Determine what diagnostic testing and/or imaging is needed to evaluate the patient. What tests will be performed in the clinic? What tests or images might you want to order but are not typically available in a primary care setting? The online supplement provides details about the tests/imaging ordered by the primary care provider (PCP) and other specialists involved in the case and also provides the results.

5. What are the next appropriate steps in management? Think about what your next steps

would be for this patient. Are you going to treat and manage the patient? Would you transfer the patient to the ED? Would you refer to a specialist? If you are unclear how to move forward, check the online supplement to determine your progress and then move forward as appropriate.

6. Review research on the diagnosis. Provide reference(s). This question will vary from

case to case, asking about risk factors, diagnostic criteria, prevalence, treatment options, and other information about the diagnosis under consideration. The use of evidencebased medicine is essential to stay abreast of recent developments, and no one has a greater need for an incredibly broad and deep knowledge base than a PCP. Read and rely on the best available research and refer to evidence whenever it is available. This, coupled with growing clinical expertise, is essential for practicing medicine. We are all lifelong learners.

7. What are the pertinent ICD-10 and CPT (E/M) codes for this visit? Provide a short

rationale. ICD-10 has approximately 68,000 codes. Medical practitioners are expected to

xxx  How to Use This Book

be specific when choosing these codes. This is an opportunity for future clinicians to learn how to do this. In addition, CPT (E/M) coding can be challenging for new clinicians. If there is an established patient with a medical diagnosis, clinicians may find it challenging to determine whether the visit is coded as level 3 or level 4. This exercise builds this skill. Remember to use the diagnostic codes for the initial visit. 8. What is the appropriate patient education for this case? PCPs have a responsibility

to educate patients on disease, diagnosis, treatment, and good health habits. This is mandatory and part of every patient visit.

9. If not managed appropriately, what is/are the medical/legal concern(s) that may arise?

Despite training and experience, there are rare occasions when a negative patient outcome or experience leads to legal action or medical consequence. It is important that young clinicians identify areas of legal exposure and the consequences of incorrect diagnoses and negligence.

10. Think about interprofessional collaboration for this case. Provide a list of specialties

or other disciplines and indicate what contribution these professionals might make to managing the patient. Medicine is a team sport. PCPs cannot do their jobs alone. It is essential that clinicians value each team member and recognize their contributions. This question will help you become aware of how many individuals actually contribute to a patient’s care.

L

L

L

L

L

L

L

The cases conclude with a few questions (11–13, but not all three are always included) about your personal bedside manner approach, including communication, handling a distressed patient or parent, and dealing with negative outcomes. These are subjective questions for which answers are not directly provided, but are intended to be thought-provoking and to encourage discussion. Pay careful attention to the history of present illness, review of systems, relevant history, and physical exam to determine the best answers to these questions. Answers to all the questions are available on Springer Publishing Connect. Instructions for how to access Connect appears on the Connect access card that accompanies this book. Try to answer the questions yourself before reviewing the answers and outcomes online. Note that the answers for the bedside manner questions are somewhat subjective. The Outcome and Insight from the PCP sections in the online supplement demonstrate the author’s bedside manner and compassion and how the case developed and concluded. The insights from the PCPs provide valuable information about effective communication skills and ways to build trust and underscore the importance of a PCP–patient relationship. Keep in mind that medicine is subjective, and clinicians have different approaches to managing patients. They may come up with the same diagnosis at the end; however, their approaches will likely differ. For instructors who are using this workbook in a classroom environment, a filterable table of contents is available to you to sort through cases by diagnosis, patient population, gender, and organ system to aid in assignments. Contact your Springer Publishing sales representative for access at springerpub.com/instructors. A breakdown of the number of cases by systems appears below: ❍

❍ ❍ ❍ ❍ ❍ ❍ ❍

Behavioral Medicine

5

Cardiovascular System Dermatologic System Endocrine System Eyes, Ears, Nose, and Throat Gastrointestinal System Genitourinary System Hematology/Oncology

5 5 7 8 11 4 5



❍ ❍ ❍ ❍ ❍ ❍

Infectious Disease/ Reproductive System

4

Infectious Disease Musculoskeletal System Neurologic System Pulmonary System Renal System Reproductive System

10 12 7 8 5 5

*Demographics and patient data have been altered to protect privacy.

ANKLE AND CALF PAIN, ADULT MALE

CASE

1

Chief Complaint “Ankle and calf pain.”

History

of

Present Illness

A 51-year-old man presents with a 2-hour-old right ankle and calf injury. He is a firefighter, married, father of teenaged boys, ages 15, 13, and 10 years. He reports he was trying to push his stalled car from out in the street when he heard a pop near his right ankle. He immediately fell to the ground in pain. When he tried to stand up and walk, he was unable to push off the ground with his right ankle and foot. He is unable to walk normally. He noticed immediate swelling at the back of his right heel bone and in the calf muscle. He denies right foot or ankle pain and also has no tingling, numbness, or weakness in the right leg, calf, or foot. He describes the pain as 8/10 when the injury happened, but now the pain has subsided to 2/10. The pain is relieved by rest and elevation of the right ankle and leg. It is made worse by putting weight on the right leg. He denies any prior injury to the right lower extremity, denies recent long-distance travel or sedentary lifestyle, denies any recent or remote steroid use, and denies recent antibiotic use.

Review

of

Systems

The patient’s ROS is positive for pain and swelling in the back of his right heel. His review of systems is negative for chest pain, SOB, fatigue, fever, chills, joint pain, numbness or tingling to the right leg/calf, or muscle weakness.

Relevant History

The patient’s history is significant for degenerative disc disease of the cervical spine. He drinks three beers per week but denies any smoking or illicit drug use. He is sexually active. He lives with his wife and children and describes his marriage as happy. Both his parents are alive; his father is alive, aged 73, with hypertension. His mother is 70 years old, with bilateral knee osteoarthritis.

Allergies

No known drug allergies; no known food allergies.

Medications

Ibuprofen 200 mg 2 tablets every 6 hours PO since the injury occurred.

Physical Examination Vitals: T 37°C (98.6°F), P 68, R 16, BP 132/78, HT 185.4 cm (73 in.), WT 86 kg (191 lbs), BMI 25.2. General: Appears conversant and uncomfortable. Psychiatric: Exhibiting anxiousness.

2  Case 1  •  Ankle and Calf Pain, Adult Male

Skin, Hair, and Nails: No obvious lesions, warm, no signs of infection. Lungs: Clear to auscultation bilaterally. Heart: RRR, without murmur or gallop. Musculoskeletal: His right patella appears atraumatic. The patellar tendon is not tender. His knee exhibits full active range of motion 0 to 130 degrees. No knee edema or effusion noted. No crepitus palpated, nor any joint line tenderness elicited medially or laterally. Negative varus, valgus, anterior and posterior drawer testing. Negative Lachman testing. McMurray test is negative. Edema noted posteriorly mid-calf with obvious bunching of the muscles. There is a palpable defect and tenderness 5 cm proximal to the insertion of the Achilles tendon on the calcaneus. The right ankle appears atraumatic, no edema or deformity, non-tender to medial and lateral malleoli. The medial ligaments are non-tender to palpation. The lateral ligaments are non-tender to palpation. There is full active dorsiflexion of the right ankle. There is no active plantarflexion of the right ankle. A Thompson test is performed and is positive. An ankle anterior drawer test is negative. His right foot appears atraumatic, with no edema or deformity. The right tarsals, metatarsals, and phalanges are palpated and are non-tender. The patient can invert and evert the foot without deficit, along with full plantar and dorsiflexion of the toes. Circulation of the dorsalis pedis artery is 2+. Neurologic: A&O×3. The sensory examination of the right lower extremity reveals intact light touch to all of the right lower extremity, and sharp/dull testing to the plantar and dorsal aspects of the right foot is fully intact.

Clinical Discussion Questions 1. What is the differential diagnosis?

2. What is the most likely diagnosis? Why?

3. Demonstrate your understanding about the pathophysiology in regard to the most likely diagnosis.

4. Should tests/imaging studies be ordered? Which ones? Why? Think about tests/imaging beyond the

primary care setting as well.

Case 1  •  Ankle and Calf Pain, Adult Male 3 5. What are the next appropriate steps in management?

6. Review a credible research article on the role of ultrasound in the diagnosis. Demonstrate your understanding

of the role of ultrasound versus MRI in the diagnosis. Provide references for your response.

7. What are the pertinent ICD-10 and CPT (E/M) codes for this visit? Provide a short rationale.

8. What is the appropriate patient education for this case?

9. If not managed appropriately, what is/are the medical/legal concern(s) that may arise?

10. Think about interprofessional collaboration for this case. Provide a list of specialties or other disciplines

and indicate what contribution these professionals might make to managing the patient.

4  Case 1  •  Ankle and Calf Pain, Adult Male

Bedside Manner Question

11. How would you communicate your likely diagnosis to the patient?

ANSWER KEY: ANKLE AND CALF PAIN, ADULT MALE

CASE

1

1. Differential Diagnosis L

L

L

L

L

Achilles tendon rupture is the most likely diagnosis here. Onset was sudden and precipitated by trying to push a vehicle. All clinical signs (including Thompson test) and indications are positive. Achilles tendonitis could be possible but unlikely given the patient’s history and presentation. Deep vein thrombosis is unlikely as the calf is not warm; there is no warmth of edema or erythema. There were no risk factors for DVT such as recent surgery, long distance travel, or sedentary lifestyle. Anterior cruciate ligament tear produces the popping sound that can be heard with an anterior cruciate ligament tear. However, on exam, the knee is normal atraumatic, without edema or pain, with full active range of motion. Negative knee anterior drawer and Lachman’s tests, which are specific examinations to test for anterior cruciate ligament rupture. Patellar tendon rupture is ruled out by an atraumatic knee exam, with full active knee extension.

2. Most Likely Diagnosis Achilles tendon rupture: All clinical signs (including Thompson test) and indications are positive. Positive clinical exam findings of bruising up of the calf muscles, positive palpable defect of the torn Achilles tendon on palpation during exam, and an absent plantar flexion at the ankle when the examiner squeezes the calf, when patient is prone with knee and ankle flexed at 90 degrees (positive Thompson test for Achilles tendon rupture), rule the diagnosis as Achilles tendon rupture.1 3. Pathophysiology The general mechanism of injury is sports participation, especially activities that involve lifting or pushing involving the gastrocnemius and soleus muscles. The sudden rupture creates the popping sound that patients hear. Achilles tendon ruptures are less commonly from systemic disease. Other factors that increase the risk of ruptures are anabolic steroid use, fluoroquinolone usage, and local corticosteroid injections.2 4. Diagnostic Tests/Images L L L

This is a clinical diagnosis. X-rays and MRI may help. X-rays of the distal tibia/fibula and ankle were ordered to rule out a fracture of the calcaneus. MRI ordered because it is sensitive for diagnosing an Achilles tendon rupture and in determining whether surgery is necessary.

Results X-rays of the tibia and fibula and ankle were obtained and were negative for fracture or dislocation. L MRI confirmed Achilles tendon rupture. L

5. Next Steps L L L

Immobilize the right calf and ankle with a short leg posterior splint. Provide crutches and instruction regarding non-weight-bearing status. Arrange for an MRI of the distal calf and ankle and set up referral and appointment to an orthopedic surgeon, who will decide if surgical repair is needed or if just closed treatment by casting can be done.3

6. Research Articles on the Role of Ultrasound When evaluating individuals with suspected Achilles tendon rupture, ultrasound is a recommended imaging modality. It’s a fast, inexpensive, and noninvasive imaging modality with 100% and 89.9% sensitivity and specificity, respectively.4 Also, the ultrasound can assess if it’s a complete or partial Achilles tendon rupture.5–7 However, MRI is still the gold standard and modality of choice for assessing patients with Achilles tendon rupture.8 7. ICD-10 and CPT (E/M) Codes ICD-10 Code: S 86.019A Achilles tendon rupture is the initial diagnosis at the primary care visit. The definitive diagnosis will be confirmed with an MRI study of the calf muscles and ankle, along with an evaluation by an orthopedic surgeon. CPT (E/M) Code: Level 4—99214 (Established Patient) Higher-level medical decision-making was needed to arrive at a likely diagnosis. Time spent with patient also exceeded 30 minutes, with history, physical exam, splinting, patient education, and documentation. 8. Patient Education Topics L L L L

Achilles tendon rupture Calf pain Ankle sprain Achilles tendon rupture treatment and recovery time

9. Medical or Legal Concerns It is imperative that clinicians be able to diagnose this injury, because timely diagnosis and referral to an orthopedist is the only way to achieve a satisfactory outcome. Missed diagnosis can lead to delayed care and referral and can make the surgical repair and healing more difficult, with potential for a poor outcome. 10. Interprofessional Collaboration L L L L L L L

Outcome

Receptionist registers the patient. MA takes brief history and vital signs. PCP formulates most-likely diagnosis. Orthopedic surgeon confirms diagnosis and makes treatment decision. Anesthesiologist assists in surgery. Postoperative care nurse attends to the patient in recovery. Physical therapist provides physical therapy treatment and rehabilitative exercises.

The patient was diagnosed with a right Achilles tendon tear. X-rays of the tibia, fibula, and ankle were negative for fracture or dislocation. He was immobilized in a short leg posterior fiberglass splint, placed on crutches, and made non-weight-bearing right lower extremity. An MRI was ordered confirming the diagnosis. He was referred to an orthopedic surgeon, who surgically repaired the tendon rupture 3 days after the injury. He was discharged home the same day and had outpatient physical therapy. He was out of work for 4 months while the tendon healed. The patient returned to his firefighting duties after the injury healed, and he was able to bear full weight on the affected extremity.

Insight

from the

PCP

It is important to communicate directly and effectively about the injury and discuss the plan in straightforward terms. Lay out the plan, get an MRI, and refer the patient to an orthopedist. The surgeon will discuss possible surgical repair of the Achilles tendon rupture. Let patients know up front that healing time is 8 to 12 weeks, and they will likely be non-weight-bearing during this time. It is vitally important as PCPs that we make a connection with a patient, showing empathy for pain, compassion for the aspects of what this injury means to a patient, which may include time away from work and activities, lost wages, and being more socially isolated from friends and family. Encouragement and kindness by the PCP will assist a patient’s recovery.

References

1. Armstrong A, Hubbard M. Essentials of Musculoskeletal Care. 5th ed. American Academy of Orthopedic Surgeons

and American Academy of Pediatrics; 2016.

2. Skinner H, McMahon P. Current Diagnosis and Treatment Orthopedics. 5th ed. McGraw-Hill; 2014. ISBN:

978-0-07-159075-4.

3. McPhee S, Papadakis M. Current Medical Diagnosis and Treatment. 58th ed. McGraw-Hill; 2019. ISBN:

978-1-26-011-7431.

4. Rockett MS, Waitches G, Sudakoff G, et al. Use of ultrasonography versus magnetic resonance 5. 6. 7. 8.

imaging for tendon abnormalities around the ankle. Foot Ankle Int. 1998;19:604–612. PMID: 9763166. doi:10.1177/107110079801900907 Kayser R, Mahlfeld K, Heyde CE. Partial rupture of the proximal Achilles tendon: a differential diagnostic problem in ultrasound imaging. Br J Sports Med. 2005;39:838–842; PMID: 16244194. doi:10.1136/bjsm.2005.018416 Margetić P, Miklić D, Rakić-Ersek V, et al. Comparison of ultrasonographic and intraoperative findings in Achilles tendon rupture. Coll Antropol. 2007;31:279–284. PMID: 17598414. Kotnis R, David S, Handley R, et al. Dynamic ultrasound as a selection tool for reducing Achilles tendon reruptures. Am J Sports Med. 2006;34:1395–1400. PMID: 16801690. doi:10.1177/0363546506288678 Garras DN, Raikin SM, Bhat SB, et al. MRI is unnecessary for diagnosing acute Achilles tendon ruptures: clinical diagnostic criteria. Clin Orthop Relat Res. 2012;470:2268–2273. PMID: 22538958. doi:10.1007/s11999-012-2355-y

FATIGUE AND WEAKNESS, ADULT FEMALE

CASE

2

chiEf comPlaint

“Fatigue and weakness.”

history

of

PrEsEnt illnEss

A 52-year-old Caucasian woman presents to her PCP with complaints of increasing fatigue and weakness over the past 3 to 4 months. She is an avid runner and finds it difficult to keep her usual distance or pace. She tried adding a multivitamin to her daily regimen and has been tracking her sleep on an exercise-tracking watch for the past month without significant findings. She does report having some irregularity with her menses over this past year, but her routine gynecologic exam showed no significant concerns and she denies excessive or heavy bleeding. Her gynecologist reportedly told her she is likely in perimenopause after confirming she is not pregnant with an in-office urine hCG, and she has a follow-up appointment scheduled in about 6 weeks. The patient denies any anxiety or depression and reports good social activity. Her diet is vegetarian and reported as well balanced.

rEViEW

of

systEms

The patient’s ROS is positive for asthenia without anorexia or weight loss and positive for angular cheilosis and increased soreness of tongue and mouth at times, and she does note some mild hair loss and brittle nails. Her ROS is also positive for some dyspnea on exertion and exercise intolerance, and she has some constipation. She denies chest pain or palpitations. The ROS is also negative for abdominal pain, nausea, vomiting, or diarrhea. She denies blood in stools, hematuria, or dysuria. The patient also denies dizziness, blurred vision, headache, anxiety, and depression. She denies fever, chills, and night sweats and has had no significant change in weight or unusual bruising reported.

rElEVant history

The patient’s history is unremarkable. Her menses are irregular, less frequent for the past 9 months, with her last menstrual period about 8 weeks ago without excessive or heavy bleeding. Her surgical history is significant for one cesarean section at age 28 and appendectomy at age 35. The patient is married and lives at home with her spouse and youngest child. She is employed as an accountant full-time with a graduate school education. She denies smoking or illicit drug use. She drinks socially, reporting about four to five glasses of wine per week. Her family history includes her mother, aged 76 with hypertension, and father, aged 77 with hyperlipidemia, hypertension, and gout.

allErgiEs

Fluoroquinolones-rash; no known food allergies.

mEDications L L

Women’s over-50 multivitamin. Ooccasional ibuprofen 400 mg PRN for pain.

6  Case 2  •  Fatigue and Weakness, Adult Female

Physical Examination

Vitals: T 36°C (96.8°F), P 62, R 14, BP 112/62; HT 170 cm (67 in.), WT 57 kg (126 lbs), BMI 19.4. General: A&O×3, well groomed. Psychiatric: Normal affect, pleasant and cooperative. Skin, Hair, and Nails: Skin dry but without rash or lesions, normal hair distribution, brittle nails. Eye: Mild pallor of conjunctiva. ENT/Mouth: Evidence of healing angular cheilosis bilaterally with some smoothness and redness of tongue. Mucous membranes with some mild pallor. Neck: Supple without thyromegaly. Head and neck lymph nodes within normal limits. Lungs: Clear to auscultation with normal respiratory effort. Heart: S1S2, RRR; no murmur noted. Abdomen: Soft, nontender, no distention. No organomegaly. Positive and normal bowel sounds in all quadrants. Genital/Rectal: No external hemorrhoids, negative for occult blood. Genital exam deferred. Neurologic: CN II–XII grossly intact with normal patellar deep tendon reflexes.

Clinical Discussion Questions 1. What is the differential diagnosis?

2. What is the most likely diagnosis? Why?

3. Demonstrate your understanding about the pathophysiology in regard to the most likely diagnosis.

4. Should tests/imaging studies be ordered? Which ones? Why? Think about tests/imaging beyond the

primary care setting as well.

Case 2  •  Fatigue and Weakness, Adult Female 7 5. What are the next appropriate steps in management?

6. What are the causes, risk factors, and treatment options for this diagnosis? Provide references for your response.

7. What are the pertinent ICD-10 and CPT (E/M) codes for this visit? Provide a short rationale.

8. What is the appropriate patient education for this case?

9. If not managed appropriately, what is/are the medical/legal concern(s) that may arise?

10. Think about interprofessional collaboration for this case. Provide a list of specialties or other disciplines

and indicate what contribution these professionals might make to managing the patient.

8  Case 2  •  Fatigue and Weakness, Adult Female

Bedside Manner Questions

11. How would you communicate your likely diagnosis to the patient?

12. If the patient shows distress at what you communicate, how would you provide support?

ANSWER KEY: LEFT HIP PAIN, ADULT FEMALE

CASE

2

1. Differential Diagnosis L

L

L

L

L

Macrocytic anemia should be considered in a patient presenting with complaints of fatigue and weakness, social history of specialized diet, and some moderate or routine alcohol intake. Macrocytic anemia presents similarly to other anemias, with common signs and symptoms of fatigue, weakness, pale or yellowish skin, irregular heartbeats, SOB, dizziness or lightheadedness, chest pain, cold hands and feet, and headaches. In early anemia, patients often do not experience symptoms at all, so reporting of more than one of these symptoms usually indicates the presence of an anemia for some time. Differentiation comes first from history and more significantly with laboratory studies. Microcytic anemia is high on the differential list. While there is no evidence of acute or insidious blood loss, the patient's physical exam findings are consistent with a microcytic anemia secondary to iron deficiency. Hypothyroidism is a reasonable diagnosis to consider given the patient’s complaint of progressive fatigue, weakness, and exercise intolerance as well as prevalence. The findings of some dryness of hair, skin, and nails may lend support to this differential diagnosis as well. The absence of weight gain commonly seen in hypothyroidism may be explained by her healthy diet and regular exercise. Hypothyroidism should be considered and tests ordered. Sleep apnea is also a reasonable diagnosis to consider given the patient’s complaints of fatigue, weakness, and exercise intolerance. The use of an exercise tracker for sleep is not always accurate for findings of mild to moderate sleep apnea. In the absence of any other evidence on early testing, this is a reasonable thing to consider testing with an overnight polysomnography. Nutritional deficiency is a less likely diagnosis but important to consider in a patient presenting with vague reports of fatigue, particularly in a patient with a specialized diet. For this patient, the concern was for vitamin D deficiency in a perimenopausal Caucasian female. Investigating further if she is protein deficient is also reasonable.

2. Most Likely Diagnosis Microcytic anemia: This patient is a vegetarian who only recently added a multivitamin to her regimen after increasing fatigue was experienced. Her physical exam findings of angular cheilosis, brittle nails, and pallor of mucus membranes seem to support this initial suspicion. Additional questions to ask on the history might include food cravings, which could suggest pica. 3. Pathophysiology The metallic element iron is essential for the growth and survival of all organisms. Its outer shell of electrons is ideally poised for complex coordination chemistry, enabling the binding of ligands such as oxygen as well as participation in critical oxidation–reduction reactions. Iron is required for the biological activity of heme proteins such as hemoglobin, myoglobin, and cytochrome and is a key cofactor in a number of enzymes spanning a wide range of metabolic activities. However, because of its high degree of reactivity, iron can catalyze the generation of oxygen free radicals and other toxic species, leading to cellular and tissue injury by way of protein crosslinks, lipid peroxidation, and damage to DNA. Therefore, for iron to fulfill its biological functions safely, an exquisite degree of control is required.1 Iron deficiency is the most common cause of anemia. Approximately 2% of women and 1% of men develop anemia due to iron deficiency. The average adult has 2 to 4 g of stored iron. About

65% of this reserve is located in the RBCs, with the remainder in the bone marrow, liver, spleen, and other body tissues. Iron deficiency occurs when there is a net imbalance resulting from either excessive loss or poor intake.2 4. Diagnostic Tests/Images CBC (screen anemia), CMP (screen electrolytes, nutritional deficiencies, kidney functions, and liver functions), TSH (screen thyroid disorders), UA (screen urinary and/or renal abnormalities), vitamin D (screen vitamin D deficiency), vitamin B12 (screen B12 deficiency), and serum iron/ ferritin/TIBC (to further evaluate iron deficiency). Results L CBC: WBCs 8.6 x 109, Hb 9.7 g/dL, Hct 35.2%, MCV 72 µm3, MCH 25 pg, MCHC 27 g/dL, RDW 15.7% L CMP: Within normal limits L TSH: 2.13 L UA: Within normal limits L Vitamin D: 32 L Vitamin B12: 680 L Serum iron: 15 µg/L L Serum ferritin: 14 µg/L L TIBC: 434 µg/L 5. Next Steps L L

Fasting laboratory testing with a follow-up appointment scheduled in 2 weeks. Patient asked to keep a diary of symptoms, foods and exercise consumed, sleep patterns, and bowel habits.

6. Causes, Risk Factors, and Treatment Options L

L

L

L

Iron deficiency is the most prevalent cause of anemia worldwide. In the majority of cases, it is due to blood loss. Heavy menstrual blood flow (menorrhagia) and pregnancy frequently lead to iron deficiency in women of childbearing age. In the developed world, this disease is easily identified and treated but frequently overlooked by providers. In contrast, it is a health problem that affects major portions of the population in underdeveloped countries. Overall, the prevention and successful treatment for iron deficiency anemia remains woefully insufficient worldwide, especially in underprivileged women and children.3 Women develop iron deficiency more readily than men because of increased potential for iron loss. On average, women lose an additional 1 mg of iron each day during menstruation. Metabolic and potential blood losses associated with pregnancy, lactation, and delivery required an additional 1,000 mg iron intake each.4 The importance of iron replacement therapy is matched by the need to establish the cause of the deficiency. All patients should be evaluated for gastrointestinal blood loss. Unless there is a convincing explanation for iron deficiency such as menorrhagia, frequent blood donations, or upper intestinal malabsorption, the diagnostic inquiry should extend beyond testing the stool for occult blood to radiologic/endoscopic evaluation of the gastrointestinal tract, particularly in postmenopausal women and males of any age.1 Ferrous sulfate, 325 mg once daily or every other day on an empty stomach, is a standard approach to replenish iron stores. As oral iron stimulates hepcidin (regulator of iron metabolism) production, once-daily or every-other-day dosing maximizes iron absorption compared to multiple doses per day, and with fewer side effects. Nausea and constipation limit compliance with ferrous sulfate. Extended-release ferrous sulfate with mucoproteose is a welltolerated oral preparation. Taking ferrous sulfate with food reduces side effects but also its absorption. An appropriate response is a return of the hematocrit level halfway toward normal within 3 weeks with full return to baseline after 2 months. Iron therapy should continue for 3 to 6 months after restoration of normal hematologic values to replenish iron stores.2

7. ICD-10 and CPT (E/M) Codes ICD-10 Code: R53.83 Fatigue, other. This is the initial diagnosis at the primary care visit. The diagnosis will be confirmed with laboratory testing. Even though the most likely diagnosis is iron deficiency anemia, it is not the definitive diagnosis yet. Therefore, provide the ICD-10 code accordingly. CPT (E/M) Code: Level 3—99213 (Established Patient) This was a 15-minute face-to-face encounter with the patient. The visit consisted of expanded problem focused history, expanded problem focused exam, and low complexity medical decision-making. 8. Patient Education Topics L L L

Iron deficiency anemia Good food sources of iron Preventing nutritional deficiency

9. Medical or Legal Concerns Microcytic anemia secondary to blood loss, particularly related to an insidious bleed in the GI or urinary tracts, could result in life-threatening anemia if missed. It is important to rule this out through evaluation of urine and occult blood in stool. Both tests are relatively inexpensive and noninvasive and should be performed anytime there is evidence of low hemoglobin. Furthermore, it is important to be sure to document the status and results of all appropriate preventive health screenings. In this case, the patient was up to date with her colonoscopy, which helped to reduce the risk of a missed gastrointestinal cause of her anemia. She was also up to date on her women’s health screenings. 10. Interprofessional Collaboration L L L L

L L

Outcome

Receptionist registers patient. MA rooms the patient and takes vital signs. Nurse obtains urine pregnancy test and initial medical history/review of medications. PCP takes history, physical exam; orders diagnostics; formulates differential diagnosis; and schedules follow-up. Phlebotomist performs venipuncture and registers test results for patient. Dietitian counsels the patient after diagnosis to review patient education handouts on nutritional deficiency and discuss how to increase iron intake in a vegetarian diet.

At her follow-up visit, the patient presented with a lifestyle diary that reflected a healthy vegetarian diet in general, with a notable absence of iron-rich foods. Activity levels and sleep patterns were normal, and she did report some slight improvement of constipation with use of polyethylene glycol 3350 and increased hydration. She does not plan to use this too often, and she was counseled on overuse of laxatives. Lab testing reviewed with the patient did indicate a microcytic anemia secondary to iron deficiency. While her remaining labs were essentially normal, she was also counseled to increase her vitamin D intake given the low-normal value and her risks for osteoporosis. The patient was asked to meet a clinical dietitian following our discussion to assist in finding dietary sources of iron in her vegetarian diet, and we reassured her of the ease of management with use of a supplement and improved dietary intake. She was prescribed ferrous sulfate 325 mg daily in the morning (before meals) for best absorption and was also encouraged to take with vitamin C or orange juice. She may continue with her multivitamin but should be sure she is getting at least 1,000 IU of vitamin D daily. We reviewed possible side effects of ferrous sulfate use, most notably with constipation and GI upset, and the patient was encouraged to keep exercising and hydrating to mitigate these effects. She was recommended to use NSAIDs to a minimum and was provided some suggestions on topical pain relievers for her occasional musculoskeletal pain.

We also discussed risks associated with poor compliance and persistence of her anemia. Although it was less likely that her anemia was related to her menses, I did ask that she please keep her scheduled follow-up with the gynecologist to discuss this further, and her recent laboratory studies were forwarded to the gynecologist for review. Repeat CBC and serum iron studies were ordered to repeat in 2 to 4 weeks and will also be readily available for her gynecologist to review and consider. She received this information well, verbalized understanding of our plan, and seemed comfortable with this. If she had shown any signs of fear or concern, I would have provided her with some easy-to-read research about how common and treatable iron-deficiency anemia is with evidence that her care plan follows common standards of practice. I would also ensure she has her appointment with the dietitian in the next week to be sure she is supported and reassured and can begin her modifications quickly. At her 8-week follow-up appointment, her CBC and iron studies returned to normal limits, and her gynecology appointment was reported as otherwise unremarkable. She is confirmed to be in perimenopause on laboratory studies done by her gynecologist. She is tolerating her ferrous sulfate well if taken with food and reports improved exercise tolerance and energy levels. Laboratory studies will be repeated again in 3 months.

Insight

from the

PCP

This particular case was very straightforward, with expected outcomes occurring throughout the evaluation process. However, anemia can be tricky to evaluate, especially in the early or asymptomatic phases. I have had patients (both women and men) who presented without symptoms who were found to have critical hemoglobin levels on routine annual lab testing, a most memorable patient having a hemoglobin of 6.7 g/dL on her annual physical labs without a single complaint. In any case, it is important to find the cause of anemia and consider repeating the CBC if results are not consistent with the history or physical exam. Inappropriate tourniquet technique (too tight) on blood draw or inappropriate result entry can cause unnecessary interventions due to inaccurate results. Prior to requesting blood transfusion for my outpatients, I always repeat the CBC with differential in addition to blood typing. In my career so far, I have had two patients for whom I was about to order a blood transfusion, but repeated the CBC to confirm the deficiency first. Both times the repeat CBC study results were normal. One of the most important aspects to care of my patients is a thorough, culturally sensitive, and non-judgmental history taking. I take the majority of my patient visit time to talk with them about their lifestyle; even the little nuances you may not think are important may be very relevant to your case. In this case, she is a healthy woman who takes pride in her vegetarian lifestyle. It is my job to respect that and find ways to help her with lifestyle modifications that suit her wants and needs, not to judge her for her dietary restrictions or recommend changes she may not be comfortable with. I also want to be sure I am not noting any eating habits that might reflect unhealthy choices or a psychological disorder. Referring her to the dietitian provides her with a well-trained source of information and support while showing her I am committed to her long-term health.

References

1. Bunn H, Heeney MM, Bunn H, et al. Iron homeostasis: deficiency and overload. In: Aster JC, Bunn H, Aster JC,

et al. eds. Pathophysiology of Blood Disorders, 2nd ed.. McGraw-Hill; 2020. http://accessmedicine.mhmedical.com.

2. Damon LE, Andreadis C, Damon LE, et al. Iron deficiency anemia. In: Papadakis MA, McPhee SJ, Rabow MW, et al.

eds. Current Medical Diagnosis and Treatment 2020. McGraw-Hill; 2020. http://accessmedicine.mhmedical.com.

3. Miller JL. Iron deficiency anemia: a common and curable disease. Cold Spring Harb Perspect Med.

2013;3(7):a011866. doi:10.1101/cshperspect.a011866

4. Primack BA, Mahaniah KJ, Primack BA, et al. Anemia. In: South-Paul JE, Matheny SC, Lewis EL, et al. eds. Current

Diagnosis & Treatment: Family Medicine, 4th ed.. McGraw-Hill; 2015. http://accessmedicine.mhmedical.com.

ABDOMINAL PAIN, ADOLESCENT MALE

CASE

3

chiEf comPlaint

“Abdominal pain.”

history

of

PrEsEnt illnEss

A 17-year-old boy presents to his PCP with abdominal pain, nausea, vomiting, poor appetite, headache, and diarrhea. He has been sick since last night. He describes his abdominal pain as “really bad” and states his pain level as 5/10. He states, “I don’t know how to describe the pain, but I know it’s really bad.” He points to the entire abdomen as the location of the pain. When asked where the pain started, he states, “The entire stomach.” He vomited once this afternoon and is now nauseous. He states he had loose stool this morning. He indicates that “most likely it was diarrhea this morning; but I am not 100% sure.” He further states, “I do have loose stools a lot.” He denies fever, chills, cough, runny nose, sore throat, abdominal bloating, acid reflux, neck stiffness, urinary urgency, dysuria, hematochezia, back pain, wheezing, SOB, recent travel, new medication, or known sick contacts, including home and school. He does not recall eating or drinking anything out of ordinary. He has been eating and drinking almost the same food and drinks as the entire family. He has been taking acetaminophen 500 mg every 6 hours to manage his pain, and it has helped his pain slightly. He is here today with his older sister, who drove him to the appointment.

rEViEW

of

systEms

His ROS is positive for abdominal pain, nausea, and vomiting. He reports headache and lack of appetite. The ROS is questionable for diarrhea. The ROS is negative for fever, chills, constipation, cough, runny nose, sore throat, abdominal bloating, acid reflux, neck stiffness, urinary urgency, dysuria, hematochezia, back pain, wheezing, SOB, or chest pain.

rElEVant history

The patient’s medical history is significant for acne (onset age 14), obesity, hepatic steatosis, and mild intermittent asthma (onset age 8). He has no surgical history. His social history includes playing football and traveling with his family. He has two sisters and two brothers. He lives with his father and does not have much interaction with his mom. He has been sexually active since age 16 with one female partner. He uses condoms. His family history includes diabetes (paternal grandparents), hypertension (father), and prostate cancer (paternal grandfather). He does not know his maternal family history.

allErgiEs

No known drug allergies; no known food allergies.

mEDications L L

Benzoyl peroxide 4% topical QD. Albuterol inhaler 2 puffs every 4–6 hours PRN.

10  Case 3  •  Abdominal Pain, Adolescent Male

Physical Examination Vitals: T 37.5°C (99.5°F), P 93, R 19, BP 130/87, WT 95.25 kg (210 lbs), HT 177.8 cm (70 in.), BMI 30.1. General: Mild to moderate distress, obese. Psychiatric: Cooperative, appropriate mood and affect. Eyes: Conjunctiva and sclera clear. ENT/Mouth: Within normal limit. Neck: Supple, FROM. Lungs: Clear to auscultation bilaterally. Heart: Normal rate, regular rhythm, no heart murmur. Abdomen: Tender abdomen throughout; greatest tenderness in right lower quadrant—guarding even with mild palpation; McBurney point tenderness noted; psoas sign +; Rovsing sign questionable. Neurologic: A&O´3.

Clinical Discussion Questions 1. What is the differential diagnosis?

2. What is the most likely diagnosis? Why?

3. Demonstrate your understanding about the pathophysiology in regard to the most likely diagnosis.

4. Should tests/imaging studies be ordered? Which ones? Why? Think about tests/imaging beyond the

primary care setting as well.

Case 3  •  Abdominal Pain, Adolescent Male 11 5. What are the next appropriate steps in management?

6. Demonstrate your understanding of the prevalence, diagnostic criteria, and treatment options. Include a

list of your reference(s).

7. What are the pertinent ICD-10 and CPT (E/M) codes for this visit? Provide a short rationale.

8. What is the appropriate patient education for this case?

9. If not managed appropriately, what is/are the medical/legal concern(s) that may arise?

10. Think about interprofessional collaboration for this case. Provide a list of specialties or other disciplines

and indicate what contribution these professionals might make to managing the patient.

12  Case 3  •  Abdominal Pain, Adolescent Male

Bedside Manner Questions

11. What would your communication style/approach be with this patient and his sister?

12. If a patient and his sister are distressed by the diagnosis, what might offer support?

ANSWER KEY: ABDOMINAL PAIN, ADOLESCENT MALE

CASE

3

1. Differential Diagnosis L

L

L

L

L

L

Bacterial gastroenteritis is a possibility; however, the patient’s abdominal pain is much more severe than for someone with gastroenteritis. In addition, there is guarding and severe tenderness in the RLQ. Signs as noted on the physical exam all suggest possible appendicitis. In the presented case, the patient denies constipation but admits he “probably has diarrhea.” However, it was not convincing if he really has diarrhea or not, which was why that symptom was listed as “questionable.” Without constipation or clear evidence of diarrhea, it is unlikely that the patient has bacterial gastroenteritis. Viral gastroenteritis is a possibility. The patient’s abdominal pain is much more severe than for someone with gastroenteritis. And there is not a significant history of diarrhea. In addition, there is the RLQ guarding and severe tenderness. Food poisoning would be likely with frequent diarrhea and vomiting; however, these symptoms are not present in this patient. His abdominal pain is more severe compared to a patient with food poisoning. In addition, frequent diarrhea and/or vomiting are not present. Acute appendicitis is high on the differential diagnosis list in this case. The patient’s presenting signs and symptoms fit appropriately with the classic presentation for acute appendicitis. While he is not febrile, he has extreme pain and tenderness in the RLQ and is nauseous. Acute gastritis is a possible diagnosis here. The patient’s abdominal pain included his epigastric region, consistent with gastritis. In addition, he presented with nausea and vomiting without fever. Therefore, clinicians should consider acute gastritis; however, severe tenderness with guarding on RLQ indicates something else (more serious) is going on. Small bowel obstruction should be on the differential list until it is ruled out with appropriate imaging studies. With that stated, there were no relevant risk factors associated with this patient to have a small bowel obstruction (i.e., no prior abdominal surgery). In addition, proximal small bowel obstruction leads to severe vomiting, and distal small bowel obstruction leads to abdominal distension—none of which was true in this patient’s case. Finally, the patient did not mention any problem with bowel movements.

2. Most Likely Diagnosis Acute appendicitis. All the classic symptoms of appendicitis are observed in this patient: RLQ severe pain with guarding, anorexia, and nausea and vomiting. He was not febrile (technically, it should be 100.4°F; his was 99.5°F). The physical exam was the most helpful component in this case to determine the diagnosis. Clinicians should know how to perform all the appropriate exams when suspecting appendicitis and comprehensively review the patient’s physical exam findings during an abdominal exam. 3. Pathophysiology The pathophysiology of appendicitis is not yet definitively established. Based on the current evidence, it appears that luminal obstruction leads to acute appendicitis. However, this has not been proven. Current literature indicates perforated and non-perforated appendicitis are separate entities that have different pathogeneses.1 With perforated appendicitis, stool within the appendix leaks into the abdomen and causes an inflammatory reaction.

4. Diagnostic Tests/Images ED/Hospital Setting L Abdominal CT (to evaluate abdominal pain) L CBC (to screen infection) L CRP (to screen inflammatory process) Results Appendicitis was confirmed with abdominal CT. L CBC with WBC elevation. L CRP is elevated. L

Because the presentation was consistent with acute appendicitis and bowel obstruction, the PCP wanted to order an ultrasound or CT of the abdomen, the preferred tests for evaluating suspected appendicitis. Because ultrasonography or CT was not available at the clinic, the PCP recommended the patient go to the nearest ED. 5. Next Steps Acute appendicitis is a medical emergency, and it is vital to evaluate the patient with an ultrasound or CT of the abdomen and CBC to evaluate WBC count. The PCP recommended that the patient and his sister go immediately to the nearest ED. 6. Prevalence, Diagnostic Criteria, and Treatment Options L

L

L

L

The prevalence of appendicitis is highest in males aged 10 to 19. The male-to-female ratio is 1.4:1, and males have a lifetime incidence of 8.6% compared to 6.7% for females. The incidence is approximately 233/100,000 population.2 If clinicians suspect a patient has appendicitis, they should follow the modified Alvarado score for diagnosis.3 See Table 3.1 linked with this specific case. If a patient’s total is 3 or less, it is unlikely that appendicitis is the correct diagnosis. If a patient’s total is 4 or higher, further evaluation should be done to rule out appendicitis. In this case, the patient had a score of 6, warranting further evaluation. An appendectomy is the standard treatment for appendicitis. Surgery may be performed through an abdominal incision or laparoscopically.4 Laparoscopic surgery is the preferred procedure. Prophylaxis antibiotics are effective in the prevention of postoperative complications, and a patient may be prescribed an antibiotic pre-, peri- or post-operatively.5 Table 3.1. Modified Alvarado Scoring System and Interpretation CLINICAL FINDING

POINT

Migratory right lower quadrant pain



Anorexia



Nausea or vomiting



Tenderness in the right lower quadrant



Rebound tenderness in the right lower quadrant



Fever >37.5°C (>99.5°F)

No

Leukocytosis of white blood cell count >10 × 109/L Total possible score for this patient

Unknown during the initial visit 6

Note: × indicates it was present with the case. Source: Data from Alvarado A. A practical score for the early diagnosis of acute appendicitis. Ann Emerg Med. 1986;15:557.

7. ICD-10 and CPT (E/M) Codes ICD-10 Code: R10.31 Right lower quadrant pain ICD-10 Code: R11.2 Nausea with vomiting, unspecified ICD-10 Code: R63.0 Anorexia These were the initial ICD-10 codes at the primary care visit. The patient has classic symptoms of acute appendicitis and the physical exam is confirmatory. However, there is not enough evidence to confirm this diagnosis. Consequently, the previously mentioned ICD-10 coding would be included. It is good medical practice to document that acute appendicitis is suspected. The patient was referred to the ED for additional diagnostic testing to confirm the diagnosis. CPT (E/M) Code: Level 4—99214 (Established Patient) This is a primary care visit for the evaluation and management of an established patient. According to the findings, the patient has a new complaint with a possibility for major health complications if untreated or misdiagnosed. This criterion meets 99214, the appropriate level for this clinical scenario. 8. Patient Education Topics L L L

Acute appendicitis Abdominal pain in male Nonspecific abdominal pain

9. Medical or Legal Concerns Appendicitis may be associated with morbidity and mortality in any age group. High index of suspicion and early surgery are important to avoid perforation and subsequent morbidity. Every clinician should have an understanding of how to evaluate appendicitis. The Modified Alvarado score is a good way to assess a patient with suspected appendicitis. 10. Interprofessional Collaboration L L L L L L L L L L

Receptionist registers the patient. MA takes brief history and vital signs. PCP formulates most likely diagnosis. ED triage nurse triages the patient. ED nurse takes brief history and vital signs. ED provider diagnoses appendicitis and contacts general surgeon on call. General surgeon and surgery team perform the surgery. Anesthesiologist assists in surgery. Postoperative care nurse takes care of the patient in recovery. Pharmacist consults regarding antimicrobial therapy.

Outcome

The patient was driven by his sister to the nearest ED for evaluation and management. During the ED evaluation, a CT of the abdomen, CBC, and CRP were done. Because of the patient’s young age, ultrasound would have been appropriate to reduce the radiation exposure. It is unclear why ED clinicians chose a CT over an ultrasound. The CT of the abdomen clearly indicated appendicitis. The patient’s WBC and CRP were elevated. He underwent the surgery the same day. Because appendicitis was strongly suspected, the PCP observed his progress through the integrated outpatient and ED electronic medical record and was informed that the patient had surgery. The PCP contacted the general surgeon a few hours later and was advised that surgery was performed without any complications.

Insight

from the

PCP

There are two insights I would like to share with the case: the importance of a physical exam and knowing the family background.

Until the physical exam, I thought this patient’s most likely diagnosis was gastroenteritis. He reported symptoms of nausea, vomiting, diarrhea, and abdominal pain. He mentioned his pain was 5 of 10, a subjective assessment. In general, I do not consider 5 of 10 as severe pain and would not have thought about appendicitis. It was difficult to understand the specifics about his abdominal pain as his answers were vague. I wondered if the abdominal pain started in the epigastric region first and it radiated to the RLQ. Yet, he kept saying his entire abdomen hurt. When I started the abdominal exam, it was evident his problem was more serious than I initially suspected. His RLQ tenderness was so severe, he started guarding during the exam. He had severe tenderness even with very light palpation. This level of pain was closer to 9 or 10 of 10. The pain scale is highly subjective. When his abdominal exam was positive for rebound tenderness and most severe in RLQ, I immediately suspected appendicitis. I performed the other physical exam tests, most of which supported a diagnosis of appendicitis. Performing a thorough physical exam using proper techniques is imperative. With the advanced medical tests and diagnostic tools, clinicians may overlook a thorough physical exam: if I had not performed a thorough abdominal exam, I might have missed his diagnosis. Because appendicitis was suspected, I referred the patient to ED for further evaluation. His sister wanted to let their father know about the ED visit. The father was on the speaker phone but unaware I was listening. He wondered if the son really needed to go to ED. He was concerned about a $125 copay. I knew the family and knew firsthand how much this man loves his children. I also knew they have had financial hardships for a long time. I was troubled by their financial situation and greatly concerned his son had appendicitis—a medical emergency. For that reason, I repeated the abdominal exam on the patient and found the same result. I told the family I understood their situation but that it was critical the son be evaluated and have a further evaluation to rule out appendicitis. Both the family and I were pleased that I trusted my medical judgment.

References

1. Petroianu A, Barroso, TVV. Pathophysiology of acute appendicitis. JSM Gastroenterol Hepatol. 2016;3:1062–1066.

Disponível em: https://www.jscimedcentral.com/Gastroenterology/gastroenterology-4-1062.pdf

2. Addiss DG, Shaffer N, Fowler BS, Tauxe RV. The epidemiology of appendicitis and appendectomy in the United

States. Am J Epidemiol. 1990;132(5):910–925. doi:10.1093/oxfordjournals.aje.a115734

3. Alvarado A. A practical score for the early diagnosis of acute appendicitis. Ann Emerg Med. 1986;15:557.

doi:10.1016/s0196-0644(86)80993-3

4. Baird DLH, Simillis C, Kontovounisios C, Rasheed S, Tekkis PP. Acute appendicitis. BMJ. 2017;357:j1703.

doi:10.1136/bmj.j1703

5. Andersen BR, Kallehave FLFLK, Andersen HK. Antibiotics versus placebo for prevention of postoperative

infection after appendicectomy. Cochrane Database of Systematic Reviews 2003, issue 2. Art. No.: CD001439. doi:10.1002/14651858.CD001439

LEFT HIP PAIN, ADULT FEMALE

CASE

4

chiEf comPlaint

“Left hip pain.”

history

of

PrEsEnt illnEss

A 55-year-old woman presents with an ongoing complaint of left hip pain. She had been doing physical therapy as ordered and is back to the office to learn her x-ray results. Her initial injury was approximately 2 months prior when she had come in with concern about an inguinal/groin strain. She was working full-time as a medical assistant for a local home health agency. On the day of the injury, she heard a pop in her left hip while transferring an older adult patient. Her groin felt tight, “like a vise around my pelvis,” but she was able to continue working for another 6 weeks. She came to the clinic initially with a request for physical therapy. She was also started on ibuprofen 600 mg by mouth, three times a day. She returned to the clinic after 3 weeks and saw a different provider. Her physical therapist shifted emphasis from her groin to her left hip, but this did not help, and she reported that “the pain was worse” following exercise. The provider prescribed low-dose narcotics and a muscle relaxant. A left hip x-ray was ordered, as well as a pelvic ultrasound and x-ray of the lumbar spine. The patient then sent the PCP an electronic message through her electronic health record requesting short-term disability because she did not feel safe lifting or transferring bed-bound patients. Prior to her injury, she had been going to the gym three times per week to lift weights but denies any activity that may have hurt her hip in the past. She was a long-distance runner 5 years ago but stopped because of time constraints. She is single, has two adult children, and is normally healthy. Her last laboratory tests, performed 3 months before injury, were in the normal range.

rEViEW

of

systEms

The patient’s ROS is positive for left hip pain with active range of motion and constipation when using narcotics. The ROS is negative for limping, numbness, or tingling in the left foot, and she has no back pain.

rElEVant history

Her medical history is significant for childhood asthma (unknown treatment), trigger finger and arthritis in left thumb (age 48), heartburn (age 50), varicose veins in the bilateral lower extremities (age 52), and pain in both feet (age 54). Her surgical history includes a tubal ligation and a right thumb trigger release. The patient was a previous smoker (23 pack years) but quit 12 years ago. She rarely drinks alcohol and uses no recreational drugs. She has not been sexually active for “a couple of years.” She lives alone, exercises regularly, and has two grown children. Her family history is positive for mesothelioma (deceased father), obesity, type 2 diabetes (deceased mother), and lung cancer (deceased grandfather).

allErgiEs

Erythromycin—stomach upset.

mEDications

Ibuprofen 600 mg PO TID PRN pain.

14  Case 4  •  Left Hip Pain, Adult Female

Physical Examination Vitals: T 36.4°C (97.6°F), P 78, R 16, BP 116/72, HT 172.5 cm (68 in.), WT 72.6 kg (160 lbs), SpO2 99% (room air), BMI 24.14. General: Well-appearing female, mild discomfort with movement in and out of sitting position. Appears stated age. Psychiatric: Mildly anxious. Skin, Hair, and Nails: Skin normal in appearance, no rashes or lesions. Normal hair distribution. Nail beds pink with no cyanosis/clubbing. Head: Normal shape and appearance. Eyes: Conjunctivae are clear without exudates or hemorrhage. Sclera is non-icteric. Bilateral. Extraocular muscle intact; pupils equal, round, and reactive to light and accommodation. Eyelids are normal in appearance without swelling or lesions. ENT/Mouth: The external ears are non-tender and without swelling; hearing grossly intact. Nose has no obstruction or discharge. Oropharynx has no inflammation, swelling, exudate or lesion. Mouth with no lesions; dentition good. Neck: Supple with no adenopathy. Lungs: Lung sounds clear in all lobes. Heart: Rate and rhythm are normal. Abdomen: Soft, non-tender, no masses or guarding. Musculoskeletal: Leg length equal; sacroiliac stress tests positive for three of five tests: distraction, no pain; thigh thrust, no pain; FABER, positive for pain. Compression, positive for pain. Gaenslen’s maneuver, positive for pain. Gait without limp. Neurologic: Alert/oriented to person/place/time, color/sensation/movement within normal limits on bilateral legs and feet.

Clinical Discussion Questions 1. What is the differential diagnosis?

2. What is the most likely diagnosis? Why?

3. Demonstrate your understanding about the pathophysiology in regard to the most likely diagnosis.

Case 4  •  Left Hip Pain, Adult Female 15 4. Should tests/imaging studies be ordered? Which ones? Why? Think about tests/imaging beyond the

primary care setting as well.

5. What are the next appropriate steps in management?

6. What are the risk factors, causes, and most common site for this diagnosis? Provide references for your response.

7. What are the pertinent ICD-10 and CPT (E/M) codes for this visit? Provide a short rationale.

8. What is the appropriate patient education for this case?

9. If not managed appropriately, what is/are the medical/legal concern(s) that may arise?

16  Case 4  •  Left Hip Pain, Adult Female 10. Think about interprofessional collaboration for this case. Provide a list of specialties or other disciplines

and indicate what contribution these professionals might make to managing the patient.

Bedside Manner Questions

11. How would you communicate your likely diagnosis to the patient?

12. If the patient shows distress at what you communicate, how would you provide support?

13. If diagnostic evidence points to a more complicated case that could potentially result in a negative

outcome, how would you communicate this possibility?

ANSWER KEY: LEFT HIP PAIN, ADULT FEMALE

CASE

4

1. Differential Diagnosis L

L

L L

L

L

Soft tissue trauma (e.g., muscle strain or labrale tear) is a somewhat likely diagnosis. The patient’s work as a home health aide can cause injury to her back and core if proper lifting techniques are not used. Also, she was lifting weights. This is a common injury if someone is lifting heavy weights. The patient has stopped these activities for 2 months, and her pain has not resolved. Hip fracture is also somewhat likely because it is associated with traumatic fall or injury. This can also be associated with older patients. Osteoarthritis is a less likely diagnosis given the patient’s age and acute onset of symptoms. Myelodysplastic syndrome is unlikely because the patient has not had any notable lab abnormalities (note routine labs checked 3 months prior to injury and were normal). Developmental dysplasia of hip is unlikely because patient has not had any previous pain or problems with hip. Avascular necrosis is unlikely given the patient has no known risk factors, but her symptoms and history keep this on the differential list.

2. Most Likely Diagnosis Avascular necrosis: The patient initially reported nontraumatic inguinal/groin pain. This is the most common symptom with hip pain appearing in the later stages of the disease. A thorough musculoskeletal exam of the hip and groin is especially important to rule out other causes of pain; ask about onset and what causes or relieves pain. Early AVN of bone can be difficult to diagnose if a patient has no symptoms. 3. Pathophysiology L

L

AVN is caused by trauma to the bone, called osteonecrosis. The bone is damaged by cell death, becomes ischemic, and then the bone begins to resorb.1,2 Possible triggers for AVN could be microemboli in blood vessels, sickle cells, or other clotting abnormalities.1,2 AVN may also be caused by abnormalities in the bone marrow or genetic cytotoxicity.1,2 AVN can be found up to 6 months after a fracture or glucocorticoid therapy. The necrosis eventually leads to bone collapse. Other nontraumatic causes of AVN are wide ranging, from excessive alcohol intake to overexposure to corticosteroids, genetic diseases, inflammatory diseases, and smoking.1–4 The progression of AVN can be staged in plain film radiology. Stage 1 may remain normal for months after pain begins.2 Stage 2 may show mild bone density changes, followed by presence of sclerosis and cysts.2 Stage 3 will show various levels of flattening on the femoral head until Stage 4, that is, absence of joint space because of total collapse.2

4. Diagnostic Tests/Images L

L L

Plain film imaging of the hip (to evaluate hip pain), pelvic ultrasound (to rule out GU/GI disorder), and x-ray of the lumbar spine (to assess lumbar spine because of the recent injury). MRI of the hip and pelvis (to screen any condition that is not detected by plain films). Considered but deferred CBC (to rule out myelodysplastic syndrome), CRP, and Sed Rate (to evaluate any inflammatory condition).

Results Pelvic ultrasound and x-ray of the lumbar spine were negative. L The positive finding on the left hip x-ray was reviewed with the patient. There was more than 50% subchondral collapse. L She was sent for an MRI of the left hip (result was Stage 4 AVN) and urgently referred to an orthopedic surgeon. L

5. Next Steps Urgent referral to orthopedics for surgical repair. 6. Risk Factors, Causes, and Most Common Site L

L

L

AVN can occur with or without trauma, in the presence of chronic disease, inflammatory conditions, genetic conditions, and exposure to smoking and alcohol.2,3 AVN can develop in up to 40% of patients receiving long-term glucocorticoid therapy.2–4 Short-term therapy with doses less than 20 mg/day is lower risk. AVN can occur in any bone in the body or in more than one bone at different times, but it is most common in the hip joint.2–5 If diagnosed in any other bone, the hip should also be evaluated using imaging.5

7. ICD-10 and CPT (E/M) Codes ICD-10 Code: S39.013A Strain of muscle, fascia, and tendon of pelvis, at the initial encounter. Initially, the patient was treated using this code. ICD-10 Code: M87.052 Idiopathic aseptic necrosis of left hip. This was the final diagnosis after multiple visits. CPT (E/M) Code: Level 3—99213 (Established Patient) This is a primary care visit for the evaluation and management of an established patient. According to the findings, the patient has an ongoing condition with need for surgical repair. The condition is serious but does not require emergency intervention. The patient does require urgent referral to surgical specialty. This criterion meets 99213 the appropriate level for this clinical scenario. 8. Patient Education Topics L L L

Reduce risk factors for AVN, as these can lead to long-term effects on bone health. Avoid long-term corticosteroid exposures. Post-operative recovery/risks.

9. Medical or Legal Concerns A clinician has an obligation to do a thorough workup when a patient has complaints of pain. This includes review of a medical history, medication use, injuries, and family history. Ruling out pertinent positives help make treatment decisions, and discussing the reasons behind different tests will allow a patient to understand the importance of follow-through. Maintaining a clinical relationship with a patient is important so timely follow-up can be done. This will allow for progressive workup if symptoms fail to resolve, or progress, and referral to specialists is needed. A patient should demonstrate understanding of treatment outcomes and risks and benefits of each. 10. Interprofessional Collaboration L L L L L L

Receptionist registers the patient. MA takes brief history and vital signs. PCP formulates most likely diagnosis. Orthopedic physician and surgery team perform the surgery. Anesthesiologist assists surgery. Postoperative care nurse takes care of the patient in recovery.

L L

RN cares for patient post-operatively. PT provides post-operative therapy and home care instructions.

Outcome

The patient was seen by an orthopedic surgeon the day after her MRI. She was offered conservative treatment, essentially limited ambulation, and weight bearing, until she healed or underwent a total hip replacement. She chose to have surgery, and her hip was replaced 6 weeks later. She spent three uneventful days in the hospital, was discharged home, and completed 8 weeks of physical therapy. Her pain resolved, and she was able to return to work as a medical assistant in home health.

Insight

from the

PCP

I had been the designated PCP for this patient months before her first visit with me. We had an opportunity to establish a clinical relationship when she came in for her annual physical and I learned she was considering going back to school. I was able to talk with her about options in the healthcare field, and she decided to apply for a local medical assisting program, completing her externship in our clinic. It was stressful for her to be injured so early in her new career, and I spent quite a bit of time supporting her through her disability and reassuring her through recovery. She has returned to her annual clinic appointments and is proactive about maintaining her health. Following her AVN diagnosis, I was determined to uncover the cause. She had no pertinent family history; she had given up cigarettes more than a decade previously and rarely drank alcohol. Shortly after her positive hip x-ray result, I met with her and took her back through her medical history again even though we had done this several times. It was during this discussion I learned of her corticosteroid use about 2 years ago. She had been exposed to poison oak and developed a rash on her face, arms, trunk, and lower legs. She had been prescribed prednisone 40 mg/day for 5 days. Two days later she returned to the clinic and saw a different clinician. She was prescribed a higher prednisone burst with a taper (60 mg/day for 7 days, 40 gm/4 days, 20 mg/3 days) plus a topical steroid cream. She reported she stopped the 60-mg dosage because it made her “jittery” but it was not clear how much prednisone she had actually used. Two months after this episode, she presented with a rash and was prescribed hydrocortisone. Two months later, she presented with left thumb pain that progressed to a trigger finger. She received an injection with cortisone, and 2 days later she sustained a fall. This may have been the injury that led to the AVN; the corticosteroid prior to this may have caused bone density damage. We will never know for sure the actual cause of her AVN diagnosis, but if I had known about the prednisone and cortisone use earlier, I would have suspected AVN earlier; this case is a reminder for a thorough check of medication history, including current medications and prior use of any medications that may have a bearing on the working differential. This experience is a good reminder of the caution we must use when prescribing medications—even seemly benign ones; they can have unexpected effects on the body many years later.

References

1. Shah KN, Racine J, Jones LC, Aaron RK. Pathophysiology and risk factors for osteonecrosis. Curr Rev Musculoskelet

Med. 2015;8(3):201–209. doi:10.1007/s12178-015-9277-8

2. Lespasio MJ, Sodhi N, Mont MA. Osteonecrosis of the hip: a primer. Perm J. 2019;23:18–100. doi:10.7812/

TPP/18-100

3. Lafforgue P. Pathophysiology and natural history of avascular necrosis of bone. Joint Bone Spine. 2006;73(5):500–

507. doi:10.1016/j.jbspin.2006.01.025

4. April MD, Watts RE, Hunter CJ. Avascular necrosis of the hip. J Emerg Med. 2015;49(4):e115–e116. doi:10.1016/j.

jemermed.2015.03.042

5. Hölmich P, Thorborg K. Groin pain in athletes: assessment and nonsurgical treatment. Hip Joint Res. 2016:315–322.

doi:10.1007/978-1-4614-0694-5_29

VAGINAL BLEEDING, ADULT FEMALE

CASE

5

Chief CoMPlaint

“Vaginal bleeding.”

historY

of

Present illness

A 28-year-old woman presents to her PCP with a 3-day history of dizziness, fatigue, and headache. She is mother of two girls, ages 5 and 2. She and her husband want another child, hopefully a boy, and have planned accordingly. She missed her last period and suspects she is pregnant; a home pregnancy test last week was positive. Until 3 days ago, she felt fine, other than some fatigue and mild nausea. Over the last 2 days, she began experiencing abdominal cramping and irregular vaginal bleeding. She points to both lower and upper quadrants as the main location of the pain. She describes her abdominal pain as 7 of 10. She has not used over-the-counter pain medication because she wants to avoid medication if pregnant. Additionally, she states, “When changing positions, I get lightheaded.” When the PCP asks about her bleeding, she states she has had similar bleeding during her other pregnancies. “All my pregnancies are like that, it’s normal.”

reVieW

of

sYsteMs

A ROS is positive for fatigue, nausea, abdominal pain, and vaginal bleeding. The patient reports headache and dizziness. The ROS is negative for fever, chills, vomiting, diarrhea, constipation, shortness of breath, or chest pain.

releVant historY

The patient’s medical history is significant for cholecystectomy (age 27) and chlamydia infection (age 17). Her social history includes drinking one glass of wine per weekend since age 18. She had a few male sex partners prior to her marriage. She lives with her husband and children and describes her marriage as happy. Both children were full-term babies and the pregnancies were normal. Her family history is unknown as she was adopted.

allergies

No known drug allergies; no known food allergies.

MeDiCations None.

PhYsiCal eXaMination Vitals: T 37°C (98.6°F), P 70, R 19, BP 130/80, WT 78.7 kg (173.5 lbs), HT 167.6 cm (66 in.), BMI 28. General: Appears anxious and fatigued, mild acute distress. Skin, Hair, and Nails: No rash, skin warm and dry. No abnormal findings with hair or nails. Lungs: Clear to auscultation bilaterally, good air movement throughout. Heart: RRR, without murmur or gallop.

18  Case 5  •  Vaginal Bleeding, Adult Female

Abdomen: Abdomen was soft, non-distended, and moderately tender; generalized. Neurologic: A&O×3, cranial nerves II to XII grossly intact.

Clinical Discussion Questions 1. What is the differential diagnosis?

2. What is the most likely diagnosis? Why?

3. Demonstrate your understanding about the pathophysiology in regard to the most likely diagnosis.

4. Should tests/imaging studies be ordered? Which ones? Why? Think about tests/imaging beyond the

primary care setting as well.

5. What are the next appropriate steps in management?

6. Demonstrate your understanding about the most common location of the diagnosis, risk factors, and

treatment options. Provide reference for your responses.

Case 5  •  Vaginal Bleeding, Adult Female 19 7. What are the pertinent ICD-10 and CPT (E/M) codes for this visit? Provide a short rationale.

8. What is the appropriate patient education for this case?

9. If not managed appropriately, what is/are the medical/legal concern(s) that may arise?

10. Think about interprofessional collaboration for this case. Provide a list of specialties or other disciplines

and indicate what contribution these professionals might make to managing the patient.

Bedside Manner Questions

11. What would your communication style/approach be with this patient?

12. If a patient is distressed by the diagnosis, what might offer support?

ANSWER KEY: VAGINAL BLEEDING, ADULT FEMALE

CASE

5

1. Differential Diagnosis L

L

L

L

L

Ectopic pregnancy should be an immediate suspicion for a woman of childbearing age with acute abdominal cramping and vaginal bleeding. The confirmation of pregnancy increases the likelihood of ectopic pregnancy as the cause of her symptoms. Spontaneous abortion is a likely diagnosis. However, ectopic pregnancy is a life-threatening complication of pregnancy. Therefore, ectopic pregnancy should be considered as the most likely diagnosis until proven otherwise. Gestational trophoblastic disease is a group of rare diseases where abnormal trophoblast cells grow inside the uterus after conception. It is not as common as spontaneous abortion or ectopic pregnancy. Cervical, vaginal, or uterine pathology is less likely in this case because of the positive pregnancy test. Subchorionic hematoma is unlikely because the patient’s symptoms appear to be more than subchorionic hematoma. Case presentation is more consistent (i.e., abdominal pain) with ectopic pregnancy or spontaneous abortion.

2. Most Likely Diagnosis Ectopic pregnancy is suspected when a pregnant patient presents with acute abdominal cramping and vaginal bleeding. Clinical judgement is critical as this condition can lead to a medical emergency. Another risk factor for ectopic pregnancy in this case is the medical history of chlamydia at age 17. 3. Pathophysiology The most common site of ectopic implantation is a fallopian tube; another common site is the uterine cornua. Rare ectopic pregnancy sites are the cervix, a cesarean delivery scar, an ovary, the abdomen, or fallopian tube interstitium. The structure containing the fetus usually ruptures after 6 to 16 weeks. Rupture may lead to gradual bleeding or a rapid bleed that may cause hemorrhagic shock. A delayed rupture with rapid blood loss increases the risk of maternal death.1 4. Diagnostic Tests/Images Primary Care Clinic L A urine pregnancy test (UA hCG) was ordered to confirm the pregnancy. L Because of the acute abdominal pain and vaginal bleeding, the PCP wanted an ultrasound to rule out ectopic pregnancy, a medical emergency. Because ultrasonography was not available at the clinic, the PCP recommended the patient go to the nearest ED. L Note: A point-of-care fingerstick hemoglobin test would have provided additional information) about the patient’s condition (i.e., anemia and degree of blood loss). Results UA hCG—positive. L Point-of-care fingerstick hemoglobin test—deferred because results would not have made a difference in the decision to send the patient to an ED. L

ED Setting Transvaginal ultrasound (to confirm the ectopic pregnancy). L UA hCG (to screen pregnancy). L CBC (to assess blood loss). L

Results UA hCG—positive. L Endovaginal ultrasound confirmed the ectopic pregnancy. L CBC indicated that she was anemic. L

5. Next Steps L

L

The PCP made sure the patient did not drive to the hospital alone. Accompanied by a family member, she was transported to the nearest ED. The PCP called the ED and informed the charge nurse that the patient was on her way for further evaluation. The PCP evaluated the decision between transferring the patient with her family’s own vehicle versus ambulance. Because the patient was hemodynamically stable, a family member was able to drive, and the ED is nearby, the PCP recommended that the family take the patient immediately to the ED.

6. Common Location, Risk Factors, and Treatment Options L

L

L

An ectopic pregnancy occurs outside of the uterine cavity. The majority of ectopic pregnancies occur in a fallopian tube (96%).2 A major cause of ectopic pregnancy is disruption of normal tubal anatomy from factors such as infection, surgery, congenital anomalies, or tumors. Anatomic distortion can be accompanied by functional impairment from damaged ciliary activity. The highest risk is associated with a history of prior ectopic pregnancy or tubal surgery.3,4 Historically, ectopic pregnancies are resolved surgically, but in current practice, treatment with methotrexate is preferred for qualified patients (hemodynamically stable, no kidney/liver or hematologic disorders, ability to comply with posttreatment monitoring, pretreatment serum hCG concentration less than 5,000 mIU/mL, tubal size of less than 3 to 4 cm, and no fetal cardiac activity).5 Some women undergo surgical therapy by choice or by necessity; others are candidates for expectant management.5 Expectant management criteria include asymptomatic patients, small adnexal mass, and low hCG levels.

7. ICD-10 and CPT (E/M) Codes ICD-10 Code: O20.9 Hemorrhage in early pregnancy: unspecified. This is the initial diagnosis at the primary care visit. The patient is pregnant and experiencing vaginal bleeding. You may suspect ectopic pregnancy at this time; however, you do not have enough evidence to confirm this diagnosis. Consequently, you include "unspecified" with the ICD-10 coding. CPT (E/M) Code: Level 4—99214 (Established Patient) This is a primary care visit for the evaluation and management of an established patient. According to the case, the patient has a new complaint with a possibility for serious complications if untreated or misdiagnosed. This criterion meets 99214 the appropriate level for this clinical scenario. 8. Patient Education Topics L L L

Ectopic pregnancy Vaginal bleeding during pregnancy Abdominal pain/cramping during pregnancy

9. Medical or Legal Concerns It is crucial that clinicians suspect ectopic pregnancy promptly and take the appropriate next step. Ectopic pregnancy may lead to rapid blood loss and, at times, death. If a clinician misses the diagnosis and the patient has complications or dies, there is a high risk of litigation. 10. Interprofessional Collaboration L L L L L L L L L

Receptionist registers the patient. MA takes brief history and vital signs. PCP formulates most likely diagnosis. ED triage nurse properly triages the patient. ED nurse takes brief history and vital signs. ED provider diagnoses ectopic pregnancy and calls OBGYN provider on call. OBGYN provider and surgery team perform the surgery. Anesthesiologist assists with the surgery. Postoperative care nurse takes care of the patient after the procedure.

Outcome

During her ED evaluation, the patient became hemodynamically unstable. She was evaluated with transvaginal ultrasound, UA hCG, and a CBC. Because of her unstable condition and potential for tubal rupture, the gynecologist on call was paged immediately and her tubal ectopic pregnancy was treated surgically.

Insight

from the

PCP

I have cared for this patient and her family for several years. This was an important opportunity to reassure her and build trust. When I suspected her pregnancy was at risk, I had a lengthy discussion with her and pointed out losing the pregnancy was a possibility. I mentioned I did not have the evidence to confirm this at the moment, but I wanted her to know it was potential risk. At the time, she seemed to be in denial and was crying, which is expected. She believed she was carrying a baby boy, and knowing she might lose the pregnancy was extremely difficult. When we discussed the possibilities including ectopic pregnancy, spontaneous abortion, or other vaginal or uterine etiologies of her symptoms, she continued crying. Again, I explained this was not a definitive diagnosis until further testing was performed. Because she was so distraught, I asked if I could give her a hug. She said yes. I hugged and told her we would have a follow-up appointment after the ED visit. About 2 weeks after the ED visit and surgery to resolve her ectopic pregnancy, she came back to our clinic with baked cookies. She told us that day had been one of the most difficult in her life. Then she stated how I was comforting and consoling during a difficult time. Consequently, she brought cookies to thank me. She said, “This is the least I can do to thank you.” Reassurance, appropriate communication, compassion, and, sometimes a hug, are important to a patient in need.

References

1. Shao R, Feng Y, Zou S, et al. The role of estrogen in the pathophysiology of tubal ectopic pregnancy. Am J Transl

Res. 2012;4(3):269–278.

2. Bouyer J, Coste J, Fernandez H, et al. Sites of ectopic pregnancy: a 10-year population-based study of 1800 cases.

Hum Reprod. 2002;17:3224. doi:10.1093/humrep/17.12.3224

3. Murray H, Baakdah H, Bardell T, Tulandi T. Diagnosis and treatment of ectopic pregnancy. CMAJ. 2005;173:905.

doi:10.1503/cmaj.050222

4. Bouyer J, Coste J, Shojaei T, et al. Risk factors for ectopic pregnancy: a comprehensive analysis based on a large

case-control, population-based study in France. Am J Epidemiol. 2003;157:185. doi:10.1093/aje/kwf190

5. Capmas P, Bouyer J, Fernandez H. Treatment of ectopic pregnancies in 2014: new answers to some old questions.

Fertil Steril. 2014;101:615. doi:10.1016/j.fertnstert.2014.01.029

NUMBNESS IN HANDS AND LEGS, GERIATRIC MALE

CASE

6

chief coMPlaint

“Numbness in hands and legs.”

history

of

Present illness

A 65-year-old man presents to his PCP with an initial complaint of insidious onset of hand and leg numbness and weakness. He describes tingling and weakness over the last 3 days. He has numbness in a glove-like pattern to both hands and distal forearms, as well as numbness and weakness to his lower extremities. He admits to dropping items now and cannot shake off the tingling and sense of imbalance because of the numbness in his feet. The patient admits to having difficulty getting out of his recliner. He describes progressive and worsening numbness and weakness just over the last day. He also admits to precedent pain to extremities prior to onset of numbness. He has pain in his upper and lower limbs that he describes as a dull, non-radiating ache with severity 5 to 6 of 10, at its worst. The patient states the discomfort has interfered with his sleep, contributing to his fatigue. He has a history of motor vehicle accident, 15 years ago, when his car was rear-ended, and he sustained a lower back and whiplash injury. He states after a year of physical therapy, his painful condition resolved and he was able to resume work. The patient states, “My neck and back pain had been fine for years until 5 days ago.” He has no prior surgeries to back or upper extremities. He admits to recent illness, about 3 weeks ago, which he describes as bad “food poisoning” and experienced diarrhea for 5 days with malaise, fatigue, and weight loss of 10 lbs. He was seen by urgent care, closer to home, and was reassured that he had an occurrence of viral gastroenteritis. Since then, he has been fatigued with loss of appetite and been unable to regain his 10-lb weight loss. He states, “It hit me hard and sapped the energy out of me.” The patient is here today with his wife.

reVieW

of

systeMs

A ROS is positive for decreased appetite, fatigue, nausea, insomnia, occasional dizziness, and some mild blurring of vision and imbalance problems related to numbness, weakness, and tingling of feet. He denies fever, chills, nausea, vomiting, worsening of chronic headaches, tinnitus, diplopia, dysphagia, SOB, dyspnea on exertion, orthopnea, chest pain, palpitations, paroxysmal nocturnal dyspnea, abdominal pain, current diarrhea, or constipation. The patient has good bowel and bladder control.

releVant history

The patient’s medical history is significant for well-controlled hypertension and hyperlipidemia. He had musculoskeletal neck and back pain for a year after a car accident and was treated conservatively to resolution. The patient is married with two children and works for the postal service. Active prior to the recent illness, he enjoyed hiking and biking at least once a week with his wife. He does not smoke and has one glass of wine with dinner each evening. He has no history of recreational drug use. He had the usual childhood illnesses. His family history is noncontributory except for a mother with lupus.

allergies

No known drug allergies; no known food allergies.

22  Case 6  •  Numbness in Hands and Legs, Geriatric Male

Medications ● ● ●

Lisinopril 20 mg PO QD. Atorvastatin 20 mg PO QHS. Daily multivitamin.

Physical Examination Vitals: T 37°C (98.6°F), P 98, R 14, BP 140/90, WT 82.5 kg (182 lbs), HT 175 cm (69 in.), BMI 27. General: The patient is in no apparent distress and is ambulatory without an assistive device. He has a slightly ataxic gait and holds onto hallway rails. Psychiatric: Alert, oriented, and conversant individual. Good historian. Good judgment and insight. Skin, Hair, and Nails: No lesions, rashes. Hair and nails unremarkable. Hair present to lower extremities and dorsum feet, with even distribution bilaterally. Head: Normocephalic. Atraumatic. Eyes: PERRLA. EOMI. ENT/Mouth: Oral mucosa with good dentition. Gross hearing intact. Bilateral tympanic membrane intact and not inflamed. Neck: FROM. Trachea midline. No adenopathy. Chest: Symmetrical. No axillary adenopathy. Lungs: Bilateral clear lung fields with good air movement. Heart: RRR, without murmur or gallop. Abdomen: Abdomen soft, non-tender, bowel sounds intact in all quadrants. No discernible organomegaly. Genital/Rectal: No anal lesions. Good sphincter tone. Normal male genitalia. Musculoskeletal: Mild bilateral paraspinous tenderness with deep palpation to base of neck extending to the mid interscapular areas and laterally to posterior shoulders. No nuchal rigidity. No scapular winging. Full active range of motion to neck and shoulder with end range discomfort. Bilateral hands warm and dry. Allen’s test negative. Grasp equal but weak symmetrically. No intrinsic muscle wasting noted. Tinel’s noted with pain elicited to forearm. Phalen’s negative. Hypoesthesia both dorsal and volar aspects of hand extending to wrist in glove-like pattern. Lower lumbosacral spine without tenderness. Straight leg raises negative bilaterally. Lhermitte’s negative. Lower extremities with active FROM. Motor 4/5 to knees and 4/5 ankles. Hypoesthesia to bilateral feet and ankles extending to distal calf in glove-like pattern with poor discrimination testing to sharp and dull. Neurologic: Cranial nerves II to XII grossly intact. No facial paresis noted. Deep tendon reflexes: triceps: 1+; brachioradialis/biceps: absent; radial: absent; patellar: 1+; ankle: absent with reinforcement. Vascular: Peripheral pulses (dorsalis pedis, posterior tibial, radial and ulnar) all 2+ bilaterally. Normal hair growth in lower and upper extremities with quick capillary refill bilateral toes and fingers.

Clinical Discussion Questions 1. What is the differential diagnosis?

Case 6  •  Numbness in Hands and Legs, Geriatric Male 23

2. What is the most likely diagnosis? Why?

3. Demonstrate your understanding about the pathophysiology in regard to the most likely diagnosis.

4. Should tests/imaging studies be ordered? Which ones? Why? Think about tests/imaging beyond the primary care setting as well.

5. What are the next appropriate steps in management?

6. What are the triggers, associated microorganisms, critical predictors, and complications of the disease? Provide references for your response.

7. What are the pertinent ICD-10 and CPT (E/M) codes for this visit? Provide a short rationale.

24  Case 6  •  Numbness in Hands and Legs, Geriatric Male

8. What are the appropriate patient education topics for this case?

9. If not managed appropriately, what is/are the medical/legal concern(s) that may arise?

10. Think about interprofessional collaboration for this case. Provide a list of specialties or other disciplines and indicate what contribution these professionals might make to managing the patient.

Bedside Manner Questions

11. What would your communication style/approach be with this patient and his wife?

12. If a patient and his wife are distressed by the diagnosis, what might offer support?

ANSWER KEY: NUMBNESS IN HANDS AND LEGS, GERIATRIC MALE

CASE

6

1. Differential Diagnosis ●









Peripheral nerve impingements must be considered based on the patient’s initial presentation. Patients may present with nerve impingements, affecting the cervical nerve root, median nerve, or brachial or ulnar cubital tunnel in varied combinations and concomitant with other spinal impingements. This diagnosis is unlikely given the symmetrical presentation and absence of radiculopathy. Likewise, the patient had no bowel or bladder concerns that would likely be suspected given the extent of neuromuscular involvement. Cervical stenosis must also be considered on an initial differential for patients with history of prior neck and back trauma associated with sensory and motor deprivation to periphery. Degenerative changes to the vertebral body, discs, or the posterior longitudinal ligament, related to prior trauma, may contribute to bony overgrowth and impingement of spinal cord. This diagnosis is unlikely because the patient’s pain was more acute onset; cervical stenosis is more insidious. Given the degree of symptoms and preservation of bowel and bladder control, this diagnosis is ruled out. Guillain Barré syndrome is a likely diagnosis for this patient because of the sudden onset, progressive ascending symmetrical sensory, and motor loss in a glove-like pattern on upper and lower extremities. Idiopathic/malignant plexoneuropathies are an unlikely diagnosis because the pain is typically an insidious onset. Dependent on the etiology, whether related to tumor or unknown denervation neuropathies, the presentation may be similar. Signs of distal neuropathies in a glove-like pattern may be noted, as well as correlation with tumor involvement, if related to carcinogenic etiology as noted in multiple myeloma, lymphomas, and so on. Most typically, the progression is less intense. Myasthenia gravis is an unlikely diagnosis because the patient presents with a progressive and constant weakness of the limb muscles. In myasthenia gravis, weakness fluctuates and is associated with complaints of muscle fatigue. Symptoms are usually worse after exercise or at the end of day.

2. Most Likely Diagnosis Guillain-Barré syndrome. The patient experienced acute onset in less than 7 days and experienced a lack of deep tendon reflexes to periphery, blurred vision, fatigue, with quick progression of symptoms, all factors associated with this disease. Likewise, the patient had a possible precedent event with diarrhea 3 weeks prior to presenting to the clinic. Most important, given the presenting symptoms, a high index of suspicion should be considered by a clinician, because of the potential for rapid acceleration of the disease, which may be fatal with respiratory compromise. 3. Pathophysiology Guillain-Barré syndrome is a disease that falls under the umbrella of polyneuropathies. Specifically, it falls in the category of immune-mediated polyneuropathies. It is an autoimmune response typically resulting after an antecedent infection 2 to 4 weeks prior to the onset of symptoms. Campylobacter and HIV are more often implicated antecedent events. In some cases, hepatitis, Zika, surgery, and trauma have been associated. Flu vaccines had been previously implicated as a potential contributing factor; however, studies lack definitive correlation. 1 Similarly, the exact trigger mechanism is not well known. After the initial insult to the body, there is a trigger of a hypervigilant immune-mediated response producing antibodies. These antibodies are capable of

attacking nerve and myelin sheaths, through molecular mimicry, allowing invasion and destruction of the nerve membrane. Demyelination ensues, resulting in sensory and motor nerve dysfunction. Disease progression has a potential to escalate to axonal involvement in a cephalad and proximal direction that can affect diaphragmatic innervation resulting in respiratory arrest.2 4. Diagnostic Tests/Images The diagnosis for Guillain-Barré syndrome is heavily weighted on the clinical presentation because testing during the first 2 weeks of symptoms may not provide definitive diagnostic results. In consideration of the clinical presentation, the PCP sought immediate neurological consultation and facilitated transition of care for this patient. Further diagnostic studies were deferred as patient was referred immediately to the ED. Emergency Department Testing L Labs: CBC, CMP, UA, and heavy metal screening performed to rule out possible metabolic disorders, infective processes, or heavy metal intoxication accounting for some neurotoxicitys. Results CBC: WBC 5.8, Hgb 14.9, Hct. 44.1, platelets 199, MCV 88, MCH 30, MCHC 34 34, neutrophils 42.7, lymphocytes 32.4. L CMP: Glucose 102, sodium 138, potassium 3.9, chloride 102, carbon dioxide 29, urea nitrogen 16, creatinine 1.1, calcium 9.1, Anion Gap 9, eGFR >60. L UA: pH 5.0, specific gravity 1.02, urobilinogen negative, blood negative, leukocytes negative, nitrites negative, leukocyte esterase negative. L Heavy metal screen: Negative for lead, mercury, arsenic, or cadmium. L

Imaging Plain spinal and CT imaging to rule out mechanical spinal instability, cervical impingement, or infective processes accounting for complex neurologic symptoms. Results Plain spinal imaging: Cervical, thoracic, and lumbar spines showed unremarkable alignment. Mild DJD changes cervical and lumbar spine with disc narrowing at C5–C6, C6–C7 levels consistent with mod discogenic disease. No foraminal or cord impingement noted. L CT scan: Cervical discogenic DJD; otherwise no focal spinal lesions. L

Inpatient Acute Care Testing EMG/NCS: Nerve studies were obtained to identify patterns consistent for Guillain-Barré reflected by absence of hallmark “H” wave reflex and “F” wave latency. Median, ulnar, and sural nerves tested. L Lumbar puncture: CSF fluid may be used to support evidence for Guillain-Barré syndrome if elevated protein is noted with a typical 99% of individuals exposed to the organism do not develop the disease. The population risk of Guillain-Barré syndrome is less than 1 per 1,000 individuals.2

L

L

L

Campylobacter jejuni is found to be the most common preceding infection in Guillain-Barré syndrome, involving up to 50% of patients. Other organisms that have been implicated are cytomegalovirus, Epstein-Barr virus, influenza A virus, Mycoplasma pneumoniae, and Haemophilus influenzae.3,4 As high as 30% of neuromuscular respiratory failure requiring ventilation is associated with Guillain-Barré syndrome. Critical predictors include rapid progression, onset 60 years are more susceptible to postherpetic neuralgia, requiring sometimes years of chronic pain management. In one study of 385 patients >65 years of age, the mean duration of postherpetic neuralgia was 3.3 years.2,5,6 After an initial episode of herpes zoster, recurrence rates are typically 40 years. Physical examination of the prostate was negative for prostate enlargement.

2. Most Likely Diagnosis

Type 2 diabetes mellitus. The patient presents with frequent urination (polyuria), excessive thirst (polydipsia), fatigue, blurry vision, cuts/bruises slow to heal, weight changes, and tingling (paresthesia), all of which are common signs and symptoms of type 2 DM. There are significant risk factors such as family history of diabetes, sedentary lifestyle, obesity, and middle age. The physical examination findings are consistent with symptoms for diabetes: intraretinal hemorrhages, exudates and cotton wool spots, dry skin, discoloration in body folds (acanthosis nigricans), and the reduced peripheral pulses. The patient satisfied the diagnostic criteria: L A1C ≥ 6.5% (48 mmol/mol). L A random plasma glucose ≥ 200 mg/dL (11.1 mmol/L) in a setting of classic symptoms of hyperglycemia. L Absence of ketones suggests a diagnosis of type 2 diabetes rather than type 1. Further testing will be done to differentiate between type 1 and type 2 DM. 3. Pathophysiology

DM is a chronic metabolic disease caused by either absolute or relative insulin deficiency or varying degrees of insulin resistance resulting in high blood glucose levels (hyperglycemia). If pancreatic beta cells do not produce enough insulin or the body does not respond to the insulin, glucose builds up in the blood instead of being used by the cells, leading to prediabetes or diabetes. Over time, high blood glucose damages nerves and blood vessels leading to complications. Diabetes is classified into the following general categories. L Type 1 DM (due to beta-cell destruction leading to insulin deficiency). L Type 2 DM (progressive loss of insulin secretion on the background of insulin resistance).

L L

L

L

Gestational DM (diagnosed in the second or third trimester of pregnancy). MODY—Maturity-onset diabetes of youth caused by mutations of different types: Most common forms are glucokinase and HNF alpha mutations. LADA (type 1.5 DM)—Latent autoimmune diabetes of adults is a form of type 1 diabetes that develops during the adult years. It is sometimes misdiagnosed as type 2 diabetes given the adult onset and similarities with type 2 diabetes. Long-term complications of diabetes are due to microvascular and macrovascular disease. The pathophysiology of diabetes-related complications involves multiple metabolic pathways including the polyol pathway, AGEs accumulation, defective stimulation of PI 3-kinase activity, the PKC pathway, and the hexosamine pathway.1,2

4. Diagnostic Tests/Images Initial Visit Testing L L L

L

RBS to check for hyperglycemia. HbA1C to check for the average level of blood glucose of the past 3 months. UA to detect or rule out a wide range of disorders, such as UTI, kidney disease, and diabetes. UA can detect glucose in the urine. An EKG was ordered to ascertain a baseline cardiac function.

Results L L L L

RBS: 340 mg/dL UA: 3+ glucose, no ketones HbA1C: 8.8% EKG: Normal sinus rhythm with mild left ventricular hypertrophy

Follow-up Visit Testing L L

L L L L

L L

Fasting plasma glucose. Fasting lipids to assess for hyperlipidemia and metabolic syndrome and estimate the patient’s 10-year ASCVD risk (tools.acc.org/ASCVD-Risk-Estimator-Plus/#!/calculate/estimate/). C-peptide levels and autoantibodies to rule out type 1 diabetes. CBC to rule out infections or anemia. CMP to rule out any electrolyte abnormalities, kidney, or hepatic dysfunction. Serum creatinine and urinary albumin-to-creatinine ratio to estimate daily albumin excretion and assess renal damage and macrovascular disease. Thyroid tests (TSH) to rule out thyroid etiologies. PSA to rule out BPH.

Results L L

L L

L L L L L L L

Fasting plasma glucose: 145 mg/dL. Fasting lipid profile: Total lipids: 246 mg/dL; LDL: 164 mg/dL, HDL: 38 mg/dL, triglycerides: 202 mg/dL. 10-year ASCVD risk: 16.5% (intermediate; according to American College of Cardiology). Pancreatic assays revealed a mean C-peptide level of 5.0 ng/mL, and islet cell antibody testing was negative. CBC values all were in normal range. CMP results were also within normal limits except for glucose. Serum creatinine: 1.06 mg/dL. eGFR: 55 mL/min/1.73 m2. Urinary albumin-to-creatinine ratio was 32. TSH was 0.7 µ/L. PSA was 2.5 ng/mL.

5. Next Steps L

While in the office, the patient was given 10 units of regular insulin to lower his blood sugar. A long-term treatment plan was developed after confirming a diagnosis of type 2 DM, ruling out other possible causes of hyperglycemia, assessing renal function, and estimating the ASCVD risk. We started with an effective communication plan to inform the patient about his diagnosis, risk factors, and possible complications. This was followed by assessing his understanding of the disease and readiness to change. The patient had family members with

L

L

diabetes and had a good understanding of the disease. Social workers and case managers were engaged to help with insurance and psychosocial concerns. Behavioral health professionals had already engaged him in managing his substance use disorder and other mental health conditions. A certified diabetic educator was brought on the team to discuss lifestyle modifications (diet and exercise) and self-management and educate the patient on how to use glucose meters. Initial pharmacological treatment options included the following: metformin (850 mg BID) for DM, hydrochlorothiazide and lisinopril (12.5/10 mg) for HTN, a moderate dose statin (atorvastatin 40 mg daily) for hyperlipidemia, and low-dose aspirin (81 mg daily). These medication orders were faxed directly to the sober house, which has a policy of directly observed therapy. A LVN dispensed medications, and residents (clients) were expected to take medications as directed. The LVN also checked blood sugar and kept a log at the sober house. The LVN communicated with pharmacists and the prescribing providers as needed. The patient was given a new glucose meter, test strips, lancets, and glucose tablets in case of hypoglycemia emergencies. Follow-up visits were scheduled every week for the first month to titrate medications, assess side effects and compliance, and offer ongoing support. A BP target of less than 130/80 mmHg was recommended. The patient was encouraged to quit tobacco. A heart-healthy diet emphasizing intake of vegetables, fruits, legumes, nuts, whole grains, and fish was recommended. Glucose logs, BP measurements, and weight loss goals were reviewed on every subsequent visit. His HbA1C was checked every 3 months with a goal of R. Breasts: Firm, slightly pendulous, symmetric bilaterally, without nipple discharge or retraction. Tender left tail of Spence, extending proximally to erythematous left axillary mass. Lungs: Clear to auscultation bilaterally, without wheezes. Heart: RRR without murmurs, rubs, or gallops. Peripheral Vascular: Extremities warm throughout, BUE and BLE pulses 2+ and symmetric. capillary refill brisk, less than 2 seconds throughout. Upper and lower distal extremities without edema. No varicosities. Musculoskeletal: FROM of right upper extremity. LROM left shoulder, eliciting severe pain with left upper extremity extension overhead with external rotation. FROM left elbow, wrist, hand, and fingers. Neurologic: Cranial nerves II to XII grossly intact. Upper and lower extremity sensation intact to light and sharp touch. Muscle bulk, tone, 5/5 strength symmetric throughout. Bilateral brachioradialis and patellar reflexes 2+/symmetric.

Clinical Discussion Questions 1. What is the differential diagnosis?

2. What is the most likely diagnosis? Why?

3. Demonstrate your understanding about the pathophysiology in regard to the most likely diagnosis.

Case 71  •  Painful Lump in Armpit, Adult Female 285 4. Should tests/imaging studies be ordered? Which ones?Why? Think about tests/imaging beyond the

primary care setting as well.

5. What are the next appropriate steps in management?

6. What are the exacerbation factors, diagnosis, and treatment of the diagnosis? Provide references for your

responses.

7. What are the pertinent ICD-10 and CPT (E/M) codes for this visit? Provide a short rationale.

8. What is the appropriate patient education for this case?

9. If not managed appropriately, what is/are the medical/legal concern(s) that may arise?

286  Case 71  •  Painful Lump in Armpit, Adult Female 10. Think about interprofessional collaboration for this case. Provide a list of specialties or other disciplines

and indicate what contribution these professionals might make to managing the patient.

Bedside Manner Question

11. What would your communication style/approach be with this patient?

ANSWER KEY: PAINFUL LUMP IN ARMPIT, ADULT FEMALE

CASE

71

1. Differential Diagnosis ●









Left axillary folliculitis is caused by micro-abrasions allowing staphylococcus to enter the skin, the presentation of folliculitis is more papular and pustular, with each papule or pustule found pierced by a single hair shaft, making this a less likely diagnosis for this patient. Folliculitis is pruritic, tender, and may lead to desquamation.1 Left axillary hidradenitis suppurativa is highly suspicious here. The patient’s clinical presentation of the elongated, cord-like mass that was painful to the touch, along with the mechanism of injury (using her boyfriend’s razor), points to this. That this has occurred before, but not as pronounced is suggestive of this diagnosis.2 Contact dermatitis, a hypersensitivity reaction to new colognes, perfumes, deodorants, soaps, detergents, clothing, and jewelry is unlikely as the condition is restricted to the patient’s left axilla and she denies introduction of any of these. Hodgkin lymphoma is a lymphoid tumor that may arise from infection such as Epstein–Barr virus, chronic inflammation, and dyes. Low-grade lymphomas are slowly progressive and painless, noticed as a skin lesion and lymphadenopathy. Incidence is higher in males than in females, but the typical peak age is 15 to 34 years.3 The patient’s acute onset and pain make this an unlikely diagnosis. Non-Hodgkin lymphoma is the fifth most common cancer in the United States, typical age of diagnosis is 65 to 74 years of age, although 16% are under the age of 35 years.4 It presents as palpable, painless, bulky lymphadenopathy in the axilla, cervical or inguinal areas. This is an unlikely diagnosis given the patient’s presentation and pain.

2. Most Likely Diagnosis Left axillary hidradenitis suppurativa. Because of the multiple comedo-like follicular occlusions in the setting of inflamed nodules that coalesced into a painful, rope-like mass, this is the most likely diagnosis for this patient. 3. Pathophysiology Largely considered a disorder of apocrine-bearing skin at the epithelial level of the terminal follicle, this condition tends to be progressive, chronic, and disabling. Although its exact etiology is unknown, it produces a range of systemic symptoms, including mucopurulent drainage from abscesses, fistulas, sinus tracts, and scars. It is a recurring condition with insidious onset, occurring typically in otherwise healthy adolescents and adults. Comparisons are often made with acne vulgaris due to the inflamed occluded comedones; however, suppurative hidradenitis has not been found to demonstrate an increase in sebaceous activity. The axillae and groin are the two most commonly affected areas.1 The most commonly isolated bacteria are Staphylococcus aureus, coagulase-negative staphylococci, and anaerobic streptococci.1,2 4. Diagnostic Tests/Images ●



Urine pregnancy test (it is unlikely she is pregnant as she is on birth control pills; however, it is important to rule out pregnancy to choose an appropriate antibiotic) Wound culture, left axilla abscess

Results Pregnancy test negative ● Culture was positive for staphylococci ●

5. Next Steps ●







● ●

The PCP performed local incision and drainage of the abscess, taking great care to break open loculations and fistulas that trap additional purulent debris. When these occluded comedones were deroofed, copious thick, and mucopurulent, malodorous discharge was expressed. Loculations were broken up to further release deeply seated abscess pockets in need of drainage. This axillary mass communicated via multiple tracts. Once drained, the abscess cavity was irrigated with normal saline and packed with gauze to allow healing via secondary intention. A tetanus immunization was given prior to patient discharge from the clinic, as was a prescription for an antibiotic (doxycycline) to cover suspected staphylococcus infection, and short course of opioid pain medication. A wound culture was taken and sent to the lab for the identification of the causative agent. Results pending for 48 hours. Follow-up visit to wound check and repacking in 2 days. Note given to stay home from work for the next 2 days.

6. Exacerbation Factors, Diagnosis, and Treatment ●





There are some possible exacerbation factors (comorbidities and lifestyle risk factors) associated with HS. Those include obesity, smoking, DM, and hyperlipidemia.5,6 HS is a clinical diagnosis. No laboratory and/or imaging tests are necessary for the diagnosis. Pathological examination of HS specimens can demonstrate follicular plugging accompanied by fibroplasia and a mixed inflammatory cell infiltrate.5 When appropriate, as part of the initial treatment, recommendations for weight loss, smoking cessation, and glycemic control can be helpful as conservative measures.7 Topical clindamycin 1% or oral tetracycline is the first-line medical treatment if the problem is mild to moderate.5,7 An incision and drainage can be helpful for the relief of painful abscess cavities related to HS. Almost all patients who experience an incision and drainage for HS have a recurrence.7 In patients who undergo surgical excision in axillary region shows a low recurrence rate.7

7. ICD-10 and CPT (E/M) Codes ICD-10 Code: L73.2 Hidradenitis suppurativa. This patient’s HPI and physical exam findings are consistent with HS. It is a clinical diagnosis. Therefore, this is the ICD-10 code during the initial visit. CPT (E/M) Code: Level 5—99215 (Established Patient) This was a comprehensive visit of established patient, 40 minutes of face-to-face time to elicit history, perform examination, incision and abscess and pack wound, give patient education and wound care, and need for follow up visit in 48 hours. CPT (E/M) Code: 10180 This code is for incision and drainage of complex wound, necrotic tissue excised, wound packed open for continuous drainage. 8. Patient Education Topics ● ● ● ● ● ●

The need for antibiotics and instructions to take them for the full course. Instructions and cautions on use of opioid pain medication. Use of heating pad. Changing the dressing. Hygiene while healing and after. Smoking cessation.

9. Medical or Legal Concerns Ignoring the need to incise, drain, and pack the wound would have serious consequences for both patient (extension of infection, sepsis) and PCP (liability); in this case, the patient did not need to visit the ED, neither did she require hospital admission. Technically, she was not febrile (37.7°C/99.8°F); therefore, no additional labs were indicated at this time. Patient education is paramount for a good outcome. The patient needs to be told about follow-up appointments, how to care for her wound, and the need to go immediately to the ED if she experiences a fever of 38.7°C (101.5°F), chills, or pain she cannot control. 10. Interprofessional Collaboration ● ● ● ●

Receptionist registers patient and makes calls reminding of follow-up appointments. MA takes brief history and vitals, and assists with I&D. PCP examines patient, makes diagnosis, and treats patient. Laboratory technician examines culture.

Outcome

The patient tolerated her procedure as well as could be expected, as this is a very painful condition. Incision and drainage is a painful procedure for any patient to endure, not to mention the repeated wound packings that must be performed daily to every other day. She was diligent in her personal hygiene and in her follow-up visits. She returned to the clinic without fail for the first follow-up appointment and asked that her boyfriend be taught how to pack the wound daily so she did not have to miss any more work. Because the patient and her boyfriend had been very attentive to her care, he was given supplies to aid with taking care of the wound, including gloves, gauze, saline wash, and forceps. He was instructed to wash his hands before and after wound care, and to use gloves each time he took care of the wound. He was shown how to remove the contaminated packing and how to dispose of the packing materials by placing them in a separate bag that could be placed in the trash can. He was shown how to irrigate the wound cavity with normal saline, how to absorb the saline wash with gauze, and how to repack the wound with the proper amount of gauze using forceps provided. He understood any materials that came into contact with the wound or the drainage from the wound, including the saline wash, would be considered contaminated and would need to be disposed of properly. He must also minimize his risk of contracting the same infection by exercising proper hygiene and handwashing. The patient had appointments with the PCP to reassess the healing process once a week for the next few weeks. With such attentive care, her wound took just over 5 weeks to heal completely with minimum scarring. Both the patient and her boyfriend were grateful for the genuine concern the provider and staff had for them both, demonstrating compassion, empathy, and a non-rushed demeanor from the very first visit.

Insight

from the

PCP

Not every patient is as diligent in her personal hygiene and care as this patient. Having a caretaker present and willing to help with dressing changes can make a big difference in a patient’s outcome. She was shocked to learn that she and her boyfriend should not be sharing their razors. Taking the time to explain every step of her management, demonstrating the procedure and wound packing process with her caretaker and commenting on her positive progression on follow-up visits encouraged her. It is also important to talk in terms a patient understands rather than using medical jargon. Explaining things to the patient all about allowing the patient to draw their conclusions by being well informed by me. It is my job to give her all the information I can so she can make the best decisions for herself. I was honored to take care of her; she later told me she would like to one day become a PA too!

References

1. Jovanovic M, Kihiczak G, Schwartz RA. Hidradenitis suppurativa. Medscape. August 30, 2019. https://emedicine.

medscape.com/article/1073117-overview

2. Papadakis MA, McPhee SJ, Rabow MW. Furunculosis and carbuncles. In: Current Medical Diagnosis and Treatment.

75th ed. McGraw-Hill Education. 2018;158–159. ISBN 978-1-25-986148-2

3. Lash BW, Wolfe Z, Argiris A. Hodgkin lymphoma. Medscape. September 12, 2018. https://emedicine.medscape.

com/article/201886-overview

4. Vinjamaram S, Estrada-Garcia DA. Non-Hodgkin lymphoma. Medscape. June 12, 2019. https://emedicine.

medscape.com/article/203399-overview

5. Alikhan A, Lynch PJ, Eisen DB. Hidradenitis suppurativa: a comprehensive review. J Am Acad Dermatol.

2009;60(4):539–561; quiz 562. doi:10.1016/j.jaad.2008.11.911

6. Wiltz O, Schoetz DJ Jr, Murray JJ, et al. Perianal hidradenitis suppurativa. The Lahey clinic experience. Dis Colon

Rectum. 1990;33:731–734. doi:10.1007/BF02052316

7. Michel C, DiBianco JM, Sabarwal V, et al. The treatment of genitoperineal hidradenitis suppurativa: a review of the

literature. Urology. 2019;124:1–5. doi:10.1016/j.urology.2018.10.013

FEVER, COUGH, AND RUNNY NOSE, PEDIATRIC FEMALE

CASE

72

Chief CoMPlaint

“Fever, cough, and runny nose.”

historY

of

Present illness

During summer season, a mother brings her 9-month-old daughter to her PCP for evaluation of fever, cough, and runny nose for the past week. The mother states her child’s fever started off at 37.8°C (100°F), but for the past 2 days it has been fluctuating between 38°C to 39°C (102°F to 103°F). She has been alternating between acetaminophen and ibuprofen q4-6h. This brings the fever to 38.3°C (101°F). A dry intermittent cough has been present for the past 4 days and is negative for post-tussive emesis. Her runny nose has progressively been getting worse over the past 7 days and the discharge is green. She has a poor appetite but is drinking okay. The mother has not noticed the child pulling her ears or showing any discomfort with urination. There is no history of vomiting or diarrhea.

reVieW

of

sYsteMs

The child’s ROS is positive for fever, cough, runny nose, and a decrease in appetite. The ROS is negative for vomiting and diarrhea.

releVant historY

The child was born by a normal spontaneous vaginal delivery, without any complications. She is up to date with immunizations including influenza vaccines. The child lives with her parents, has two siblings, and attends daycare. Her father is a smoker, but smokes away from the house and the car. Family history includes father is prediabetic and mother is obese.

allergies

No known drug allergies; no known food allergies.

MeDiCations l l

Acetaminophen 160/5 mL 1 tsp q4h PRN. Ibuprofen 100/5 mL 1 tsp q6h PRN.

PhYsiCal eXaMination

Vitals: T 39.3°C (102.7°F), P 130, R 28, BP 98/62, WT 10 kg (22 lbs) 75th percentile, HT 68.6 cm (27 in.) 50th percentile, BMI 21.2. General: Slightly uncomfortable, but aware of her surroundings. Not toxic. Skin, Hair, and Nails: Warm and pink. No abnormal findings with hair or nails.

288  Case 72  •  Fever, Cough, and Runny Nose, Pediatric Female

ENT/Mouth: Right TM: bulging, dull red, landmarks not visible. Left TM: slightly pink with a positive light reflex. Nose: purulent drainage. OP: patent and moist. Lungs: Coarse breath sounds, mostly transmitted from upper airway congestion. Negative for wheezing or rhonchi. Heart: RRR. No murmur was appreciated. Abdomen: Soft, non-tender, non-distended. Neurologic: Alert and interactive.

Clinical Discussion Questions 1. What is the differential diagnosis?

2. What is the most likely diagnosis? Why?

3. Demonstrate your understanding about the pathophysiology in regard to the most likely diagnosis.

4. Should tests/imaging studies be ordered? Which ones?Why? Think about tests/imaging beyond the

primary care setting as well.

5. What are the next appropriate steps in management?

Case 72  •  Fever, Cough, and Runny Nose, Pediatric Female 289 6. What are the prevalence, prevention, risk factors, and treatments of the diagnosis? Provide the references

for your responses.

7. What are the pertinent ICD-10 and CPT (E/M) codes for this visit? Provide a short rationale.

8. What is the appropriate parent/patient education topics for this case?

9. If not managed appropriately, what is/are the medical/legal concern(s) that may arise?

10. Think about interprofessional collaboration for this case. Provide a list of specialties or other disciplines

and indicate what contribution these professionals might make to managing the patient.

Bedside Manner Question

11. What would your communication style/approach be with this parent/patient?

ANSWER KEY: FEVER, COUGH, AND RUNNY NOSE, PEDIATRIC FEMALE

CASE

72

1. Differential Diagnosis l

l

l

l

l

Acute otitis media is high on the differential. The hallmark sign of AOM is a bulging TM. Marked redness of the TM is also a diagnostic factor but is rarely seen without bulging present as well. Fever and pain are often present. Acute bacterial rhinosinusitis is a possible diagnosis. The following are three different criteria for making this diagnosis: (a) the child has persistent URI symptoms more than 10 days; or (b) recurrence of symptoms after initial improvement; or (c) severe onset of symptoms like fever or purulent nasal discharge lasting more than three consecutive days associated with facial tenderness or headache. This child does not meet first two criteria—sick only for 7 days or did not have recurrence after initial improvement. However, severe onset of symptoms like fever or purulent nasal discharge lasting more than three consecutive days associated with facial tenderness or headache (third criteria) may be true in this case. With a 9-month-old it is not possible to find out if she has facial tenderness or headache. While this is a possibility, her physical exam suggests a different diagnosis. Influenza is a possible diagnosis, though GI symptoms tend to be more prominent in children with influenza. This child’s symptoms of nasal congestions, fever, and cough are consistent with influenza. Additionally, AOM may be a complication of flu. Likelihood of underlying influenza will depend upon the time of year (this child’s sickness occurred during the summer and not during the flu season). Her immunization is up to date (note: annual influenza vaccine is recommended, and influenza vaccination for children age 6 months to 8 years who have never received it before requires two vaccinations 4 weeks apart). Considering all of the above, it is unlikely influenza. Otitis media with effusion, also known as “serous OM,” presents with a retracted TM that is amber or blue, and fluid or bubbles may be seen on exam. It is typically associated with viral illness or allergies and is not associated with fever. This patient’s symptoms are more suggestive of AOM than OME. Pertussis is a likely diagnosis here. However, a child with fevers, runny nose, cough, and bulging TM on physical exam highly suggest AOM. In addition, this child is up to date with the immunization. Therefore, it is unlikely this is the diagnosis.

2. Most Likely Diagnosis Acute otitis media. A child with fever, runny nose, cough, and possible ear pain is suspicious for having AOM. It is important to examine the ear to check the TM for color, presence of swelling/ bulging, or other signs of acute inflammation, which were present in this case. Also, it should be noted this child attends daycare, a risk factor for OM. In addition, her father smokes, another risk factor for OM. Although father smokes outside the home and car, this may still play a role in this child’s current OM. 3. Pathophysiology The eustachian tube epithelium prevents otopathogens from colonizing; however, in young children, the eustachian tube anatomy is immature. In children and infants, the immature eustachian tube’s size and shape cannot act as the body’s defense against otopathogens and this results in transmission to the middle ear making children susceptible to OM.1 The supine position, common for infants, is also a risk factor for OM. As a child’s epithelium matures, transmission or colonization of otopathogens is reduced.1

4. Diagnostic Tests/Images In children diagnosis is typically made by examination of the ear with an otoscope following the diagnostic criteria for AOM. Complete and accurate diagnosis is made by following the COMPLETES mnemonic2: l l l l l l l l l

Color Other conditions (fluid level, presence of bubbles, perforation, bullae, retraction pockets) Mobility Position (retracted, bulging, etc.) Lighting Entire surface (all quadrants of TM need to be examined) Translucency External auditory canal and auricle (is it displaced, deformed, inflamed, etc.) Seal

While this mnemonic can ensure a complete otoscopic examination, it is important to differentiate between AOM and OME based on individual components. There are no labs or imaging used to diagnose OM. 5. Next Steps l l l l

l

PCP prescribed antibiotic and ibuprofen. Prescription sent to pharmacy electronically. Supportive care discussed with the mother. Reassurance provided to mother. Recommended that mother bring child back to the clinic if her symptoms are not improving or are getting worse. ED precautions given.

6. Prevalence, Prevention, Risk Factors, and Treatments l

l

l

l l

l

AOM is diagnosed frequently in children ages 6 to 18 months. AOM is the most frequent reason infants and toddlers are prescribed antibiotics.3 To help prevent AOM, the CDC encourages parents to ensure children are up to date on all vaccinations. Breastfeeding for 6 months and longer is also a recommended prevention.3 In one study, the risk factors for OM were found to be male sex (until age 4), exposure to tobacco smoke, siblings, and the presence of pet birds in the home.4 For infants younger than 6 months, immediate antibiotic treatment is recommended.5 The AAP and the AAFP guidelines recommend the use of amoxicillin when antibiotics are necessary. In some nonsevere cases, antibiotics can be held in place of close follow-up, unless symptoms worsen in 2 to 3 days.5 Tympanocentesis performed by a qualified clinician is indicated when OM presents in patients with severe otalgia, patients who do not respond to antibiotic treatment, or OM onset happens while on antibiotics, newborns, and immunocompromised patients.3

7. ICD-10 and CPT (E/M) Codes ICD-10 Code: H65.01 Acute serous otitis media, right ear. This code was assigned based on the clinical findings. ICD-10 Code: R50.9 Fever, unspecified. This code was assigned based on the history of fever. ICD-10 Code: J06.9 Acute upper respiratory infection, unspecified. This code was assigned based on the history of runny nose. ICD-10 Code: R05 Cough. This code was assigned based on the history of cough.

CPT (E/M) Code: Level 3—99213 (Established Patient) This is a primary care visit for the evaluation and management of an established patient. How this code was determined can be explained through history, exam, and medical decision-making. History and physical exam can be categorized as expanded problem-focused. The medical decisionmaking was low complexity. Considering these three components, it is reasonable to code this clinical scenario as a Level 3. 8. Patient Education Topics l l l

What is AOM? Risk factors for OM Treatment for OM

9. Medical or Legal Concerns The over-prescription of antibiotics is a serious problem, resulting in increasing antimicrobial drug resistance and avoidable drug side effects. It is therefore crucial clinicians differentiate between OME and AOM to avoid prescribing antibiotics when unnecessary. Uncomplicated AOM has a relatively low risk of complications; however, children should be followed up to assess hearing and development. Patients who are immunocompromised and who do not respond as expected to antimicrobial therapy are at higher risk, requiring close monitoring, modification of therapy, and specialist consultation (ENT, infectious disease, neurology) as indicated. 10. Interprofessional Collaboration l l l

Receptionist registers the patient. MA takes a brief history and vital signs. PCP formulates most likely diagnosis and treatment plan.

Outcome

The patient was started on amoxicillin 90 mg/kg/day and improvement was seen in 48 hours. She was also prescribed ibuprofen 10 mg/kg/dose q6h for her fever. A follow-up was scheduled for 1 week. At her follow-up the patient’s TM was no longer bulging and was gray in color with landmarks visible. Her cough, fever, and runny nose were no longer present.

Insight

from the

PCP

In cases as straightforward as this, diagnosing OM was simple. If a PCP pays close attention to child’s related history, history alone can be very helpful. In this case, the child goes to daycare and father is a smoker: both are risk factors for OM. Most of the time, patients or parents do not volunteer information unless we ask the right questions. Asking the right question based on the initial presentation helps reach an accurate medical diagnosis. While we need to be open-minded during medical evaluations, it is important to ask pertinent questions based on the clinical scenario. In this age group, I often diagnose children with OM, so at the beginning it is in my differential diagnosis. Then I asked the questions to rule out or rule in OM. By the time, I started the physical examination, OM was a likely diagnosis. After I performed the ENT exam, it was very clear this child had OM. In general, asking your patients or their caregivers the right questions will help you reach the most likely diagnosis. Then clinical skills further determine the most likely diagnosis. Remember: listening to your patients or their families, asking the right questions, and performing a problem-focused physical exam will strengthen your clinical skills.

References

1. Massa HM, Lim DJ, Cripps AW. Mucosal immunology. In: Russel MW, Lambrecht B, eds. This Is a Comprehensive

Review of the Innate and Acquired Immunology of the Middle Ear and the Eustachian Tube. 4th ed. 2015:1423–1942.

2. Kaleida PH. The COMPLETES exam for otitis. Contemp Pediatr. 1997;14:93. 3. Lieberthal AS, Carroll AE, Chonmaitree T, et al. The diagnosis and management of acute otitis media.

Pediatrics.2013;131(3):e964–e999. doi:10.1542/peds.2012-3488

4. van Ingen G, le Clercq CMP, Touw CE, et al. Environmental determinants associated with acute otitis media in

children: a longitudinal study. Pediatr Res. 2020;87:163–168. doi:10.1038/s41390-019-0540-3

5. Stephen P. Acute otitis media in children: treatment. https://www.uptodate.com/contents/acute-otitis-media-in-

children-treatment

DIABETES CHECK, ADULT MALE

CASE

73

Chief Complaint

“Diabetes check.”

History

Present Illness

of

A 58-year-old Hispanic man presents to a new PCP to check on his diabetes status. His last annual visit was almost 3 years ago, and it has been 6 months since his last diabetes visit. The patient also states he used to see a “specialist for his kidney” but stopped going and wants to initiate care here due to his insurance. He was diagnosed with diabetes almost 5 years ago (cannot recall exact years), and he has been taking metformin 500 mg BID for his diabetes most of the days (he forgets some days). For his high cholesterol, he has been taking atorvastatin 40 mg daily. He was also told he has high BP. He was started on a medication for his BP but stopped taking because he felt OK. The patient admits he does not keep appointments well and he used to be on insulin prior; however, he was not adherent. He states cost as the barrier that kept him from adhering to his medication regimen. He now has active insurance. His diet consists of red meats, fast food, rice, beans, and potatoes. He also complains of chronic fatigue and headaches that are intermittent but occur more frequently now and are worse in severity. The headache located in the occipital region and radiates to the temporal regions. The patient describes the pain as a throbbing and pulsating sensation. He notes some eye blurriness and nausea at times and thinks he drinks more than what he voids but has no difficulty voiding.

Review

of

Systems

The patient’s ROS is positive for fatigue, nausea, urine retention, and occipital headache. Pertinent negatives are aura, vomiting, sudden vision changes, or loss or halos. He denies cognitive problems, angina, SOB, diaphoresis, palpitations, or extremity swelling. He further denies polydipsia, polyphagia, polyuria dysuria, hematuria, discharge, hesitancy, frequency, or dribbling.

Relevant History

The patient’s history is significant for hypertension, type 2 DM, and hyperlipidemia. He has a family history of HTN, DM (type unknown), and hyperlipidemia. He is a field worker, is married, and has two children.

Allergies

No known drug allergies; no known food allergies.

Medication ● ●

Metformin 500 mg BID Atorvastatin 40 mg QD

292  Case 73  •  Diabetes Check, Adult Male

Physical Examination

Vitals: T 36.8°C (98.2°F), P 92, R 16, BP 178/106, HT 170.18 cm (67 in.),WT 83 kg (183 lbs), BMI 28.66. General: Alert, well hydrated, in no distress, well developed, well nourished. Psychiatric: Cognitive function intact, cooperative with exam, good eye contact. Eyes: EOMI, PERRLA. Neck: Supple, FROM, no cervical lymphadenopathy, no thyromegaly. Chest: Normal shape and expansion. Lungs: Clear to auscultation bilaterally, good air movement. Heart: RRR, no jugular vein distention or murmurs. Abdomen: Active bowel sounds ×4, no distention, nontender with no rebound, soft on palpation. Musculoskeletal: Mild pitting edema bilaterally 1+, pedal pulses 2+ bilaterally, no clubbing, cyanosis. Neurologic: Nonfocal, A&O×3, normal gait. Cranial nerves II to XII grossly intact.

Clinical Discussion Questions 1. What is the differential diagnosis?

2. What is the most likely diagnosis? Why?

3. Demonstrate your understanding about the pathophysiology in regard to the most likely diagnosis.

4. Should tests/imaging studies be ordered? Which ones? Why? Think about tests/imaging beyond the

primary care setting as well.

Case 73  •  Diabetes Check, Adult Male 293 5. What are the next appropriate steps in management?

6. Using credible literature, explain whether Hispanic patients are at a greater or lesser risk for this

diagnosis and why. Provide references for your responses.

7. What are the pertinent ICD-10 and CPT (E/M) codes for this visit? Provide a short rationale.

8. What is the appropriate patient education for this case?

9. If not managed appropriately, what is/are the medical/legal concern(s) that may arise?

10. Think about interprofessional collaboration for this case. Provide a list of specialties or other disciplines

and indicate what contribution these professionals might make to managing the patient.

294  Case 73  •  Diabetes Check, Adult Male

Bedside Manner Questions

11. What would your communication style/approach be with this patient?

12. If a patient is distressed by the diagnosis, what might offer support?

ANSWER KEY: DIABETES CHECK, ADULT MALE

CASE

73

1. Differential Diagnosis ●





● ●

Acute kidney injury is unlikely here. Although patients with AKI can clinically present with fatigue and nausea, an acute cause needs to be present. There is no underlying cause evident. No prerenal, intrarenal, or postrenal causes have been yet identified; however, further testing is needed. Congestive heart failure is also a possibility given the patient’s presenting symptoms. But his cardiac exam was negative for rales, palpitations, and so forth. CHF can be ruled out with diagnostic testing. Chronic kidney disease is a likely diagnosis given this patient’s symptoms and history. His fatigue, unstable hypertension, possibly uncontrolled diabetes (because of noncompliance), loss of appetite, and headache suggest CKD. Renal function labs will rule this in or out. Urinary obstruction is unlikely. The patient denies pain or a weak urine stream. Polycystic kidney disease is also unlikely given the absence of pain during exam.

2. Most Likely Diagnosis Chronic kidney disease. The patient’s presenting symptoms and his history of poorly managed diabetes and hypertension strongly suggest this as his primary diagnosis. However, additional labs are required to confirm the diagnosis. 3. Pathophysiology CKD interferes with the ability of renal function such as BP control, maintaining fluid and electrolyte balance, and waste control. This is chronic in nature and is essentially because of an underlying disease. Specifically, CKD is the result of persistent damage to the kidneys accompanied by a reduction in the GFR and the presence of albuminuria. If untreated, CKD can lead to end-stage renal disease. In the early stage, patients are usually asymptomatic, but as the disease progresses patients eventually demonstrate signs and symptoms. Damage to the glomeruli can lead to increase in protein filtration.1 Again, it is irreversible loss of nephrons in the kidneys that results in a toxic state known as uremia as there is no room to filter. Anemia can result from decrease erythropoietin as well as iron loss due to increase hepcidin (suppresses iron). Specific to this case: hypertension can lead to renal artery stenosis, thickening of the artery, and decreased blood flow within the nephron (functional unit within the kidney that filters the urine). Then the kidneys release renin and trigger a cascade of other hormones resulting in blood vessel constriction, which further increases HR and BP. Glomerulosclerosis, in which blood filtration is severely compromised and loss of nephrons is the end result. Additional contribution is loss of oxygen and nutrients via slowing blood flow.2 Glomerulosclerosis can lead to proteinuria, hematuria, HTN, and nephrosis.3 4. Diagnostic Tests/Images Point of Care Testing ● UA to evaluate glucosuria ● RBS to evaluate blood sugar ● HbA1C to assess diabetes

Results UA: pH: 5.8; Spec Gav: 1.010; Protein: 1+; Glu: Neg; Nitrate: Neg; WBC: Neg; RBC: Sml ● RBS 250 ● HbA1C: 9.7%

l

Ordered Labs Results will be available during the follow-up visit. l ● ● ●

CBC to assess Hgb and rule out infection CMP to evaluate electrolytes, kidney functions, and liver functions Urine albumin-to-creatinine ratio to assess kidney function Lipid panel to evaluate cholesterol

Results CBC ❍ Hgb: 8.6 g/dL  ● CMP ❍ Na: 134 ❍ K+: 5.9 ❍ BUN: 72 ❍ Creat: 3.4 ❍ Phos: 10.5 ❍ GFR: 47 ● UA albumin-to-creatinine ratio: 356 ● Lipids ❍ Total cholesterol: 178 mg/dL ❍ Triglyceride: 150 mg/dL ❍ HDL: 50 mg/dL ❍ LDL: 98 mg/dL ●

Nephrology Lab ● PTH Results PTH: 245 pg/mL (normal values are 10-55 pg/mL)



5. Next Steps Initial Visit ● Initiate ARB for renal protection: losartan 50 mg daily ● Prescribe atorvastatin 40 mg daily. ● Send urine for culture. ● Refer patient to diabetic educator and/or nutritionist for diabetes and hypertension counseling. ● Make follow-up appointment 1 week or sooner as needed Follow-up Visit Additional lab results available. ● Discontinue metformin due to low GFR and elevated creatinine. ● Initiate semaglutide 0.25 mg SC every week for 4 weeks and empagliflozin 10 mg every morning for diabetes management. ● The patient was re-referred to a nephrologist for workup, evaluation, and treatment. ● Education on nephrotoxic agents, smoking cessation, and necessary vaccines. ●

6. Race and Ethnicity This case provides an excellent opportunity to examine risk factors in an underserved population. Hispanics as a group have a greater risk for both hypertension and diabetes.4 The reasons for this are multifactorial and include physiologic characteristics, cultural beliefs, language, and poor access

to healthcare. A national study of Hispanic health5 found this population has the lowest awareness and control of hypertension; 40% of Hispanics who met the diagnostic criteria for diabetes were unaware of their disease. The long-term solution to better serving this population lies in population-based interventions (e.g., the CDC’s Million Hearts campaign), community-based interventions tailored to the social determinants of health in a specific community (e.g., rural, Hispanic), and system-based approaches (e.g., EMR decision-making tools).4 7. ICD-10 and CPT (E/M) Codes ICD-10 Code: R53.83 Fatigue. ICD-10 Code: R11.0 Nausea. ICD-10 Code: I10 Essential hypertension. ICD-10 Code: E11.8 Type 2 diabetes mellitus with unspecified complications. Note that these diagnosis codes were used for the first visit. The ICD-10 code for CKD, N18.9, should not be used until the diagnosis is confirmed by labs. CPT (E/M) Code: Level 3—99203 (New Patient) This visit for a new patient required a detailed history, detailed exam, and moderate complexity decision-making. 8. Patient Education Topics ● ● ●

Diabetes management BP management Necessary lifestyle modifications to help diabetes management

9. Medical or Legal Concerns This patient has a chronic, irreversible condition and a history of non-compliance. This should be documented in his chart. Patient education on his disease and the need to comply with a treatment plan should be clearly explained and the patient should be asked to verbalize his understanding. All this should also be carefully documented. If language is a barrier to patient understanding, a translator should be made available. 10. Interprofessional Collaboration ● ● ● ● ●

Outcome

Receptionist registers patient. MA takes brief history and vital signs. PCP conducts exam, formulates most likely diagnosis, and makes referral to nephrologist. Nephrologist confirms diagnosis and initiates treatment plan. Referral clerk processes the nephrology referral.

The nephrologist repeated labs and additionally ordered a 24-hour urine, renal US, renal panel, iron panel, hepatitis panel, PTH, uric acid, ANA, and anti-GBM Ab. The diagnoses were stage 3 CKD, anemia, and tertiary hyperparathyroidism. The nephrologist believes the etiology to be from the patient’s underlying chronic hypertension with some diabetes involvement. Both the nephrologist and the PCP are carefully following the patient. Labs are drawn every 6 months. In addition to his diabetes, cholesterol, and BP medications, he was prescribed epoetin alfa and calcitriol. The patient is more compliant and understanding of the disease process.

Insight

from the

PCP

This case involves a member of the agricultural community, a population that is typically underserved and uninsured. Unfortunately, this patient is a victim of disparities in the healthcare system. It is important PCPs advocate to the fullest extent possible for patients and make sure all available resources are made available to a patient; this includes support groups, counseling, and ongoing careful monitoring. To the extent we can, we must help patients overcome the barriers that prevent from being healthy.

References

1. Buffet L, Ricchetti C. Chronic kidney disease and hypertension: adestructive 2. 3. 4. 5.

combination. US Pharmacists. 2012;37(6):26–29. https://uspharmacist.com/article/ chronic-kidney-disease-and-hypertension-a-destructive-combination-35118 American Heart Association. How high blood pressure can lead to kidney damage or failure. October 2016. https://www.heart.org/en/health-topics/high-blood-pressure/health-threats-from-high-blood-pressure/ how-high-blood-pressure-can-lead-to-kidney-damage-or-failure Cash JC, Glass CA. Family Practice Guidelines. Springer Publishing Company; 2017. Sorlie PD, Allison MA, Avilés-Santa ML, et al. Prevalence of hypertension, awareness, treatment, and control in the Hispanic community health study/study of Latinos. Am J Hypertens. 2014;27(6):793–800. doi:10.1093/ajh/hpu003 Rodriquez F, Ferdinand KC. Hypertension in minority populations: new guidelines and emerging concepts. Adv Chronic Kidney Dis. 2015;22(2):145–153. doi:10.1053/j.ackd.2014.08.004

BUTTOCKS PAIN, GERIATRIC MALE

CASE

74

chiEf comPlaint

“Buttocks pain.”

history

of

PrEsEnt illnEss

A 67-year-old man presents to establish care as a new patient. He has hypertension, type 2 DM, hyperlipidemia, and mental retardation for which he has been off medication for 6 months. He complains of buttocks pain for a few days. He denies any falls. He is living with his disabled sister and niece who have taken care of him for the last few years after they removed him from a residential facility due to cost. He was in a program that provided sheltered work experience and socialization. His niece is the patient’s payee and manages his financial and medical needs; however, she has not taken him to a doctor for over 6 months and is not keeping him adherent to his medical regimen. The patient’s last HbA1C was 8.6% over 6 months ago, and he does not have a glucometer for BG monitoring. The dates of his last eye exam or dental exam are unknown. His sister has limited ability to help the patient meet his medical and physical needs due to her medical condition. The niece is not interested in home health services. She uses the patient’s and her mother’s disability checks to pay for bills and does not have money left for medications. The patient states he washes dishes and watches TV during the day for his activities. He is able to bathe and toilet himself with prompting and assistance. He uses a walker for stability with ambulation. He is missing his glasses and dentures due to frequent moves over the last few years.

rEViEW

of

systEms

The patient’s ROS is positive for weight loss of 20 lbs, missing teeth, poor visual acuity, right shoulder pain with movement, right foot drop, and weakness. His ROS is negative for fever, chills, fatigue, vomiting, diarrhea, polyuria, polydipsia, and polyphagia. All other ROS are negative.

rElEVant history

The patient’s history is significant for intellectual disability, type 2 diabetes, hypertension, hyperlipidemia, right foot drop, and nearsightedness. He had a full dental extraction in 2013. He lives with his sister, niece, and his niece’s three children. He does not drink alcohol or use tobacco products or illicit drugs.

allErgiEs

No known drug allergies; no known food allergies.

mEDications ● ● ● ● ●

Enalapril 20 mg PO BID. Glipizide 5 mg PO BID. Metformin 1000 mg PO BID. Metoprolol tartrate 50 mg PO BID. Pravastatin 40 mg PO QD.

296  Case 74  •  Buttocks Pain, Geriatric Male

Physical Examination

Vitals: T 37°C (98.6°F), P 88, R 16, BP 180/100, HT 185.4 cm (6’1”), WT 126.6 kg (279 lbs), BMI 36.81. General: Well-developed, malnourished appearing 67-year-old man in acute distress. Poor hygiene with dirty clothes and foul body odor. Able to answer questions. Psychiatric: Normal mood. Skin, Hair, and Nails: Dry appearing, macerated, erythematous skin on buttocks and gluteal fold with dry fecal material. Partial thickness, loss of dermis with shallow ulceration on left buttocks. Dirty, long fingernails. Toenails are thickened and yellow with extensive growth needing trimming to normal length. ENT/Mouth: Dry oral mucosa, teeth are absent, no dentures. Lungs: Clear to auscultation bilaterally. Heart: RRR, without murmur, gallop, or rub. Abdomen: Soft, non-distended. Normoactive bowel sounds are heard in all four quadrants. Musculoskeletal: No joint swelling or pain. Wide gait with dragging of right foot. Neurologic: Coordination abnormal with drop foot pattern of right foot. Sensory exam of the foot is abnormal with monofilaments in all locations. Weak pulses, no lesions, or ulcers. Mini-cog was normal.

Clinical Discussion Questions 1. What is the differential diagnosis?

2. What is the most likely diagnosis? Why?

3. Demonstrate your understanding about the pathophysiology in regard to the most likely diagnosis.

Case 74  •  Buttocks Pain, Geriatric Male 297 4. Should tests/imaging studies be ordered? Which ones? Why? Think about tests/imaging beyond the

primary care setting as well.

5. What are the next appropriate steps in management?

6. What are the risk factors, clinical presentation, and PCP responsibility associated with the diagnosis?

Provide references for your response.

7. What are the pertinent ICD-10 and CPT (E/M) codes for this visit? Provide a short rationale.

8. What are appropriate patient/caregiver education topics for this case?

9. If not managed appropriately, what is/are the medical/legal concern(s) that may arise?

298  Case 74  •  Buttocks Pain, Geriatric Male 10. Think about interprofessional collaboration for this case. Provide a list of specialties or other disciplines

and indicate what contribution these professionals might make to managing the patient and family.

Bedside Manner Question

11. What would your communication style/approach be with this family?

ANSWER KEY: BUTTOCKS PAIN, GERIATRIC MALE

CASE

74

1. Differential Diagnosis ●











Alzheimer disease is a progressive form of dementia starting with mild memory loss gradually leading to severe decline in thinking, behavior, and social skill that impair a person’s daily living. Early signs can be forgetting recent events, conversations, or appointments. As the disease progresses, a person may lose ability to function leading to malnutrition, infection, and later death. This is an unlikely diagnosis in this patient as he does not exhibit cognitive impairment with short-term memory. Elder abuse encompasses physical, emotional, or sexual harm; financial exploitation; or neglect of an older adult usually by a caregiver or person that directly is involved in the elder’s care. Abusers tend to live with the elder and are usually adult children or other family members. BASE helps practitioners assess the likelihood of elder abuse. Non-adherence refers to a patient inability to comply to medical treatment such as medications, appointments or treatment plans due to cost, lack of education, mental illness, or intelligence. This diagnosis is not likely because the patient cannot be responsible for adherence given his disability. Major depressive disorder is a mood disorder with persistent mood changes such as sadness, lack of motivation, insomnia or oversleeping, hopelessness, guilty feelings, anxiety, fatigue, appetite changes, or irritability. These mood changes usually affect daily activities such as social isolation, withdrawal from hobbies or other normal activities, poor hygiene, or somatic complaints such as backaches or headaches. This patient did not exhibit these symptoms or complaints. Including PHQ to further assess this diagnosis is helpful. Falling injury. Because patient has buttock pain, falling injury should be considered. During the history, the patient denied any falling injury; however, due to his history of mental retardation, it is reasonable to consider this as a differential diagnosis (he may have completely forgotten). He is able to answer questions (though he was having mental challenges) satisfactorily. Therefore, it is reasonable to assume he likely did not suffer a fall. Buttocks pain, not otherwise specified is on the differential because the patient complains of buttock pain. Physical exam indicates partial thickness, loss of dermis with shallow ulceration on left buttocks. While this can be a diagnosis, it is important to pay attention to the entire picture and try to look for the underlying condition for the pain.

2. Most Likely Diagnosis Elder abuse is the most likely diagnosis based upon physical abuse, financial abuse, and neglect by his caregiver. This patient’s caregiver was abusing his finances by misusing his disability checks for personal use. His family was neglecting his chronic medical conditions by not attending to his health care based on medication non-adherence, no access to medical care, and neglect of hygiene and physical care that led to pressure ulceration. His physical examination revealed poor hygiene, pressure ulceration on buttocks, no dentures, glasses, and a sudden weight loss. 3. Pathophysiology While there is no pathophysiology per se for elder abuse, we can glean some understanding from the epidemiology. Elder abuse by definition is perpetrated upon persons age 60 and over. It occurs worldwide and is generally believed to be underreported. Recent meta-analyses estimate the prevalence in the community dwelling population to be 15.7%1 and as high as 64.2% among institutionalized populations.2 Elder abuse occurs across all racial, socioeconomic and religious groups, although there is a higher prevalence in white, non-Hispanic populations than other groups.3

4. Diagnostic Tests/Images The patient needed to be assessed for his medical conditions: Type 2 DM, HTN, and hyperlipidemia, and therefore the following tests were ordered: ● CBC to assess for anemia or signs of infection with pressure ulcer ● CMP to assess glucose, renal and liver function, and electrolytes ● Urine microalbumin to assess microalbuminuria ● Fasting lipid panel to assess total cholesterol, triglycerides, HDL, and LDL ● HbA1C to monitor glucose for diabetes ● PHQ for depression screening ● BASE elder abuse survey Results CBC: Within normal limit l CMP: Fasting blood sugar was 158 mg/dL ● Urine microalbumin: 60 mg ● Fasting lipid panel: LDL was 160 mg/dL and total cholesterol 224 mg/dL ● HbA1C: 10% ● PHQ for depression normal score of 0 ● BASE elder abuse survey positive. l

5. Next Steps ● ● ●



● ● ● ●

● ●

Mandatory reporting to adult protective services Social work consultation Home health services: PT and OT assessments, medication management, and assistance with ADLs Resume all the medications as listed above: no change (his BP and diabetes are not at the goal; however, because he was noncompliant with medications, no dose adjustments or new medication added at this time) Neurocognitive testing for competency Dental examination for denture replacement Diabetic eye exam and replacement of glasses Diabetic education with certified diabetic educator for glucose monitoring and further education of disease to caregiver Podiatry referral for diabetic foot care Wound care for pressure ulcer on buttocks

6. Risk Factors, Clinical Presentation, and PCP Responsibility ●





Risk factors for physical and financial elder abuse are increased with living with a larger number of household members other than a spouse. Lower income increases the likelihood of financial abuse, emotional, or physical abuse.4 Clues to the diagnosis of elder abuse/self-neglect may include a patient’s ill-kempt appearance, recurrent acute visits, missed appointments, suspicious physical exam findings, and unexplainable injuries.5 Adult Protective Services should be notified of suspected elder abuse as required by state law in most states. Mental health professionals may assess an elder patient for decision-making capacity or for concerns of untreated mental illness contributing to the problem.6

7. ICD-10 and CPT (E/M)Codes ICD-10 Code: T74.91XA Unspecified adult maltreatment, confirmed, initial encounter.

ICD-10 Code: E11.9 Type 2 diabetes mellitus without complication. ICD-10 Code: I10 Essential hypertension. ICD-10 Code: E78.2 Mixed hyperlipidemia. ICD-10 Code: M21.371 Foot drop, right foot. ICD-10 Code: L89.322 Pressure ulcer left buttocks, Stage 2. CPT (E/M) Code: Level 3—99203 (New Patient) This visit was for a new patient to establish care with evaluation and treatment of multiple chronic medical conditions plus a new complaint requiring a thorough history and physical examination and expected time with patient of about 30 minutes. 8. Caregiver Education Topics ● ● ● ● ● ●

In this case, it is the caregiver who needs to be educated Understanding her uncle’s medical condition The need for medication adherence, proper diet, and hygiene BG monitoring The need to replace glasses and dentures The need to schedule and keep routine appointments

9. Medical or Legal Concerns A PCP is a mandated reporter of any suspicion of abuse, neglect, or financial exploitation of a vulnerable patient. Failure to report could result in serious ramifications with an employer and medical board. Elders subjected to abuse have higher rates of morbidity and mortality. Early intervention by clinicians is essential. 10. Interprofessional Collaboration ● ● ●









● ● ●

Receptionist checks patient into clinic. MA takes vital signs, brief medical history, and collects list of medications. PCP takes medical history, reviews prior medications, examines patients, formulates diagnoses, and completes medical documentation for health care services and social work. Social worker reports situation to adult protective services and works with family for home health services or placement into PACE, and concerns with financial barriers. Clinical care coordinator assesses patient’s medication costs and educates caregiver (niece) on importance of future medical appointment for medical management and concerns with financial barriers. Certified diabetic educator educates caregiver (niece) on how to manage and measure BG, foot care, and diabetic medications. Neuropsychologist assesses patient’s cognitive status and competency to make medical and financial decision. OT/PT assesses patient’s ADLs and lower extremity weakness with right foot drop. Podiatry provides diabetic foot care of overgrown toenails. Wound care specialist provides treatment of pressure ulcer on buttocks.

Outcome

After the office visit, the clinical care coordinator (RN) and LSW in our clinic were consulted about how to further handle this case. Because the patient had limited cognitive abilities and understanding, the clinical care coordinator worked with his niece to fill all the patient’s medications at our pharmacy and filled his medication dispenser. Detailed written and verbal instructions were given to the niece regarding his medical conditions, medications, and treatment plan. Diabetic education including demonstrating home BG monitoring was shown to the niece and patient. Referrals for dental and eye care were given for patients to reestablish denture and eyeglasses use. Physical therapy and occupational therapy were consulted for right foot drop as well as increasing lower extremity strength and improved mobility with an ankle-foot orthosis brace for right foot drop. Podiatry consultation was made for diabetic foot care of overgrown toenails. His lab results were very consistent with medication non-compliance. His diabetes has gotten worse (A1C 10%), LDL was not at the goal (160 mg/dL), and microalbumin indicated early kidney disease (60 mg). We started monitoring all these conditions and his hypertension closely during subsequent visits.

Insight

from the

PCP

I was the PCP for this new patient at our clinic. This case was complex and difficult requiring true interprofessional collaboration with many providers and professionals who worked hard in the best interest of this patient. We tried to keep the patient in the home with his disabled elderly sister and adult niece after we provided detailed diabetic education, medications set up for the patient, and home health to assess patient’s at-home needs. However, his medical and daily needs were still not met evidenced by missed appointments, non-adherence to glucose monitoring and medication refills, and consultations. The niece became disengaged and overwhelmed with his care and dealing with her children, so it was decided to apply for the PACE through Medicare. The patient was able to attend a day program and have his care transferred to a specific PACE physician to manage his overall care. I have not seen the patient since he was accepted into the program. He was a very pleasant man, and I am very grateful for the wonderful staff who helped get him the care he needed. This case was particularly difficult because the patient was new to us. There was no relationship to fall back on, as we knew nothing about the family. While much was obvious upon history and exam, there was still a larger family story we needed to understand. When faced with a situation like this a PCP needs to communicate in a way that will not be taken as accusatory or shaming. Real empathy is needed for all the parties here. Simple, clear, and repetitive messages are best and ongoing reassurance that you will do all you can to help is essential.

References

1. Yon Y, Mikton CR, Gassoumis ZD, Wilber KH. Elder abuse prevalence in community settings: a systematic review

and meta-analysis. Lancet Glob Health. 2017;5(2):e147–e156. doi:10.1016/S2214-109X(17)30006-2

2. Yon Y, Ramiro-Gonzalez M, Mikton CR, et al. The prevalence of elder abuse in institutional settings: a systematic

review and meta-analysis. Eur J Public Health. 2019;29(1):58–67. doi:10.1093/eurpub/cky093

3. Hernandez-Tejada M, Amstadter A, Muzzy W, Acierno R. The national elder mistreatment study: race and

ethnicity findings. JEld Abus Negl. 2013;25(4):281–293. doi:10.1080/08946566.2013.770305

4. Lachs MS, Pillemer KA. Elder abuse. N Engl J Med. 2015;373(20):1947–1956. doi:10.1056/NEJMra1404688 5. Jin J. Screening for intimate partner violence, elder abuse, and abuse of vulnerable adults. JAMA. October 23,

2018;320(16):1718. doi:10.1001/jama.2018.14851

6. Harper GM, Johnston CB, Landefeld CS. Management of common geriatric problems. In: Papadakis MA, McPhee

SJ, Rabow MW, eds. Current Medical Diagnosis and Treatment. McGraw-Hill; 2020. https://accessmedicine. mhmedical.com/content.aspx?bookid=2683§ionid=225032453

ABDOMINAL PAIN, ADOLESCENT MALE

CASE

75

Chief CoMPlaint

“Abdominal pain.”

historY

Present illness

of

A 14-year-old boy presents to his PCP with abdominal pain that has been ongoing for 1 week. His foster mother is unable to pinpoint any particular symptom that stands out. He has no complaints of nausea, vomiting, diarrhea, or fever. He denies any aggravating or relieving factors. He does not eat spicy food, eats chocolate only occasionally, and doesn’t skip meals. He drinks soda and orange juice occasionally. He does not drink coffee. He is unable to localize the pain. The consistency of his stool is soft. His appetite has decreased. He adds that he unable to sleep at night and is not sure why. His body language and the tone of his voice at this juncture see, as though he is scared and anxious about something. To put him at ease, the remainder of the history is taken with a chaperone and the foster mother is requested to step from the room. When asked if there is anything else that he would like to share, he opens up about what happened 2 months ago at a party. “It was a wild party, and everyone was hooking up.” He really didn’t know anyone there but ended up “hooking up” with a girl he just met. He wanted to fit in at the party and is now worried that he may have caught something. He denies any penile discharge or dysuria.

reVieW

of

sYsteMs

The patient’s ROS is positive for anxiety, anorexia, insomnia, and abdominal pain. His ROS was negative for nausea, vomiting, diarrhea, constipation, fever, rash, and dysuria.

releVant

historY

The patient has no chronic conditions and hospitalizations. His immunizations are up to date. He is in middle school and lives with a foster mother and three other foster children at home. His family history is unknown.

allergies

No known drug allergies; no known food allergies.

MeDiCations None.

PhYsiCal eXaMination

Vitals: T 37.0°C (98.7°F), P 100, R 20, BP 110/68, WT 50.8 kg (112 lbs), HT 162.6 cm (64 in.), BMI 19.2. General: Alert and not in any distress. Psychiatric: Anxious. Skin, Hair, and Nails: Jaundice +. No signs of nail-biting or cuts on the wrist. No rash. No abnormal findings with hair or nails.

300  Case 75  •  Abdominal Pain, Adolescent Male

Eyes: Red reflex intact. PERRL. No icterus. Abdomen: Non-tender and soft on palpation. Not distended. Bowel sounds are normal. No organomegaly or masses. Genital/Rectal: Normal penile and scrotal exam. No skin lesions noted. Rectal exam was deferred. Neurologic: A&O×3.

Clinical Discussion Questions 1. What is the differential diagnosis?

2. What is the most likely diagnosis? Why?

3. Demonstrate your understanding about the pathophysiology in regard to the most likely diagnosis.

4. Should tests/imaging studies be ordered? Which ones? Why? Think about tests/imaging beyond the

primary care setting as well.

5. What are the next appropriate steps in management?

Case 75  •  Abdominal Pain, Adolescent Male 301 6. What are the prevalence, transmission, and screening of the diagnosis? Provide references for your

response.

7. What are the pertinent ICD-10 and CPT (E/M) codes for this visit? Provide a short rationale.

8. What is the appropriate patient education for this case?

9. If not managed appropriately, what is/are the medical/legal concern(s) that may arise?

10. Think about interprofessional collaboration for this case. Provide a list of specialties or other disciplines

and indicate what contribution these professionals might make to managing the patient.

Bedside Manner Questions

11. What would your communication style/approach be with this patient?

302  Case 75  •  Abdominal Pain, Adolescent Male 12. If a patient or his guardian is distressed by the diagnosis, what might offer support?

ANSWER KEY: ABDOMINAL PAIN, ADOLESCENT MALE

CASE

75

1. Differential Diagnosis l

l

l

l

l

Psychosocial stress presenting as abdominal pain is a possibility here. Children and adolescents may express psychiatric symptoms via somatic complaints. Stress causes abdominal pain or makes it worse. The sensation of having “butterflies in your stomach.” In this case the combination of the history and physical exam for abdominal pain are benign, laying the foundation for a psychosocial stressor. Stress could be related to home, school, social interactions, eating, sexuality, alcohol or drugs, mental health, and/or safety.1 Constipation is a common symptom throughout all ages in the pediatric population, both sexes, and all educational and socioeconomic levels. It is estimated that 12% to 30% of the general population suffers from functional constipation. Based on the patient’s history, this diagnosis is unlikely. Sexually transmitted infection should be high on our list in context to this story. It is imperative STIs be ruled out secondary to his high-risk behavior. His physical exam shows jaundice suggesting that hepatitis C should be ruled out. Gastritis should be in the differential; however, there is no supporting history to suggest this is the most likely diagnosis. Moreover, there is no abdominal tenderness during the exam. In general, patients with gastritis have epigastric tenderness. Gastroenteritis would be a likely diagnosis in someone with abdominal pain, soft stool, and decreased appetite may have gastroenteritis. While it is reasonable to list this as a differential, his history and clinical picture suggest something else might be the problem here.

2. Most Likely Diagnosis Psychosocial stress and possible sexually transmitted infection. Apparently, this patient’s psychosocial stress presenting as abdominal pain. A cause of his stress is his fear that may have an STI. Therefore, STI screening should be performed part of the evaluation. Among STIs, hepatitis C is highly suspected because of the jaundice found on the physical exam. 3.

Pathophysiology There are seven known genotypes of hepatitis C virus, 1a, 1b, 2, 3, 4, 5, and 6.2 HCV infection, while not fully understood, may be a result of cell injury and some immune-mediated response that causes chronic inflammation.3,4 Acute HCV infection progresses to chronic infection a majority of cases and clears spontaneously in 15% to 25% of all patients.5 Of persons infected, 60% to 70% will develop chronic liver disease, 5% to 20% will develop cirrhosis over a period of years, 1% to 5% will die from a complication of the infection and 1% to 3% will develop hepatocellular carcinoma.5

4. Diagnostic Tests/Images The following tests are ordered to rule out STIs: l HIV 4th generation test l Syphilis RPR l Gonorrhea PCR l Chlamydia PCR l Acute hepatitis panel l HSV screening considered but not ordered

Results Positive for acute hepatitis C. All the other tests were negative. 5. Next Steps l l l

l

l l l

l l

Provide reassurance an education to patient. Guarantee confidentiality. Get permission from the teen to discuss this matter with his foster mother. Discuss the delicate nature of this matter with his foster mother, once the patient grants permission. Bring both of them together so that we can work as a team, regardless of the outcome of the lab results. Be a resource for the family for any questions. Offer an excuse note to take time off from school until the follow-up appointment. Management and treatment will depend on lab results as the clinical exam is non-contributary. Follow up within 72 hours to review the lab results. If test results are confirmed positive for any STI, the clinic is required by law to report the case to the public health department.

6. Prevalence, Transmission, and Screening l

l

l

About 71 million people suffer from chronic HCV infection worldwide, with an estimated 2.7 to 3.9 million in the United States.5 Between 2010 and 2015 there was nearly a threefold increase in the number of infections in the United States. The common forms of transmission are sharing needles between drug abusers, needle sticks in a healthcare setting, or at birth from an infected mother to child. HCV can also be contracted through sexual activity with a person who is infected.5 The anti-hepatitis C Ab is a highly specific screening test capable of identifying antibodies 4 to 10 weeks after exposure. This screen should be followed by a hepatitis C RNA viral load by PCR to confirm diagnosis.5

7. ICD-10 and CPT (E/M) Codes ICD-10 Code: R10.9 Unspecified abdominal pain. This teen did not have a specific location for his abdominal pain and his pain was random in onset and relief. A physical exam did not elicit any discomfort. ICD-10 Code: Z11.3 Encounter for screening for infections with a predominantly sexual mode of transmission. This code was used for the initial screening labs. ICD-10 Code: B17.1 Acute hepatitis C. This code was used as the final diagnosis for the follow up visit based on lab results. CPT (E/M) Code: Level 4—99214 (Established Patient) This was an established patient office visit with a detailed history, a detailed exam, and decision making of moderate complexity. 8. Patient Education Topics l l l l l

Dealing with stressors Nonspecific abdominal pain Symptoms of STIs Contracting STIs: risk factors Safe sex

9. Medical or Legal Concerns Not every PCP includes hepatitis C screening with every STI screening. It is important to screen for hepatitis C whenever necessary. In this case, testing for STI was required; however, hepatitis C was included mainly because of the jaundice found on physical exam. Consequences of missed/delayed diagnosis of hepatitis C may lead to medico-legal issues and hepatitis C should be included with STI screening whenever necessary. STIs must be reported to the health department. There is also concern here for maintaining patient confidentiality. 10. Interprofessional Collaboration l l l l l l l

Receptionist registers the patient. MA takes brief history and vital signs and files report with the public health department. PCP formulates most likely diagnosis. Phlebotomist draws blood. Referral clerk arranges referral patient to gastroenterologist. Gastroenterologist manages hepatitis C infection. Behavioral therapist provides counseling.

Outcome

The patient was diagnosed with hepatitis C. He was referred to a gastroenterologist for further management and offered psychological support. After 12 weeks of treatment, he was cured of his infection. The patient continues to come to our clinic and now avoids the kind of social situations that resulted in his infection.

Insight

from the

PCP

The outcome of this case was heartbreaking and humbling. Listening to the patient and empathizing with him guided me to do the right thing. I did not imagine when I walked into the room, I would order a STI panel; particularly to include hepatitis C. I had to keep an open mind and listen to this young man’s cry for help. I’m glad I was able to sense his anxiety and look further into his concerns. This is one of the cases where I could not exactly determine the most likely diagnosis. I wanted to rule out STIs because of the history he shared. At the same time, I thought his presenting complaint was probably linked to anxiety. I remember having a young teenager a few years ago insisting on a HIV screen because he had an incident similar to this patient. That particular patient thought he definitely contracted HIV from the one-time risky encounter and came to the clinic at least five times requesting HIV screening test. In his case every HIV screening was negative. At the end, it became clear to me that he was very anxious, which was the main reason that he kept requesting HIV screening. In this case (hepatitis C), the patient suspected he might have contracted an STI, which at the end happened to be true. The main thing to remember is that it is vital to keep an open mind and look and listen carefully to all your patients.

References

1. Jasik CB, Ozer EM. Preventive health care for adolescents and young adults. In: Neinstein LS, Katzman DK,

Callahan TS, et al. eds. Neinstein’s Adolescent Health Care: A Practical Guide. 6th ed. Wolters Kluwer;2016:50.

2. Ferraro M DO, Painter MS DO. Hepatitis C—screening, diagnosis, management & treatment. Osteopath Fam Phys.

January/February 2019;11(1):12–19.

3. Pawlotsky JM. Pathophysiology of hepatitis C virus infection and related liver disease. Trends Microbiol.

2004;12(2):96–102. doi:10.1016/j.tim.2003.12.005

4. Bostan N, Mahmood T. An overview about hepatitis C: a devastating virus. Crit Rev Microbiol. 2010;36(2):91–133.

doi:10.3109/10408410903357455

5. Centers for Disease Control and Prevention. Hepatitis C FAQ for Healthcare Professionals. 2017.

IRRITABLE AND CRYING TODDLER, PEDIATRIC MALE

CASE

76

Chief CoMPlaint

“Irritable and crying toddler.”

historY

Present illness

of

A mother presents with her 16-month-old son to his PCP for an evaluation of irritability and crying for the past 5 hours. According to his mother, the child was fine, when suddenly became fussy and appeared to be in pain. He has no nausea, vomiting, diarrhea, fever, cough, or cold symptoms. His mother was alarmed by the sudden off-on episodes of crying. She explains that when he has short bursts of excruciating pain he doubles over or if he is being held his legs have pull up to his chest. The child seems to be fine in between these episodes. Typically, the child is calm.

reVieW

of

sYsteMs

The patient’s ROS is positive for irritability and possible abdominal pain. His ROS is negative for nausea, vomiting, bloody stool, constipation, fever, chills, and cough.

releVant

historY

The child was a normal spontaneous vaginal delivery at term without any complications. He has continued to thrive hitting all developmental milestones. He has not been hospitalized neither is he on any chronic medications.

allergies

No known drug allergies; no known food allergies.

MeDiCations None.

PhYsiCal eXaMination

Vitals: T 37.0°C (98.6°F), P 120, R 28, BP 100/65, WT 9.9 kg (22 lbs) 10th percentile, HT 76.2 cm (30 in.) 5th percentile, BMI 16.5. General: Alert and engaging when not having an episode of pain. He is non-toxic looking. Psychiatric: Irritable. Skin, Hair, and Nails: Pink with a 1–2-second capillary refill. No abnormal findings with hair or nails. Head: No signs of trauma. Eyes: PERRL. ENT/Mouth: Moist oral mucosa with no signs of dehydration. Neck: FROM with no nuchal rigidity.

304  Case 76  •  Irritable and Crying Toddler, Pediatric Male

Lungs: Clear to auscultation bilaterally, equal breath sounds. Heart: RRR, no murmurs. Abdomen: Soft and non-distended. Mild tenderness with fullness in the right upper abdomen. The right lower abdomen is scaphoid and feels “empty” on palpation (Dance sign). Genital/Rectal: Genital exam without abnormal finding; stool is hemoccult negative. Neurologic: Alert and not lethargic.

Clinical Discussion Questions 1. What is the differential diagnosis?

2. What is the most likely diagnosis? Why?

3. Demonstrate your understanding about the pathophysiology in regard to the most likely diagnosis.

4. Should tests/imaging studies be ordered? Which ones? Why? Think about tests/imaging beyond the

primary care setting as well.

5. What are the next appropriate steps in management?

Case 76  •  Irritable and Crying Toddler, Pediatric Male 305 6. What is a cost-effective and highly specific diagnostic method for this diagnosis? What are the treatment

options of this diagnosis? Provide references for your responses.

7. What are the pertinent ICD-10 and CPT (E/M) codes for this visit? Provide a short rationale.

8. What is the appropriate parent education for this case?

9. If not managed appropriately, what is/are the medical/legal concern(s) that may arise?

10. Think about interprofessional collaboration for this case. Provide a list of specialties or other disciplines

and indicate what contribution these professionals might make to managing the patient.

Bedside Manner Questions

11. What would your communication style/approach be with this parent/patient?

306  Case 76  •  Irritable and Crying Toddler, Pediatric Male 12. If a patient’s parent is distressed by the diagnosis, what might offer support?

ANSWER KEY: IRRITABLE AND CRYING TODDLER, PEDIATRIC MALE

CASE

76

1. Differential Diagnosis l

l

l

l

l

Gastroenteritis may initially present with abdominal cramping with eventual symptoms of vomiting and diarrhea. Typically, these children are non-toxic with a mild degree of discomfort. These children typically do not writhe in pain. Intussusception is acute in onset with the classic presentation of intermittent abdominal pain in an otherwise well appearing child, later evolving to vomiting, currant-jelly stool, and lethargy. This is high on the differential. Malrotation with volvulus presents with intermittent pain and vomiting. These children are ill-appearing, pale, in distress with bilious vomiting. Depending on the location of the obstruction, abdominal distension is seen. Incarcerated hernia may also present with intermittent abdominal and vomiting that eventually becomes bilious in nature. However, the onset is over several days to weeks and not sudden. Testicular torsion requires a high index of suspicion as is a urologic emergency. The typical presentation is sudden onset of intermittent pain that gradually localizes to the affected testicle.

2. Most Likely Diagnosis Intussusception. In a child who presents with sudden onset of intermittent pain with or without currant-jelly stool, there should be a high index of suspicion for an intussusception. The intermittent nature of the pain with a brief honeymoon periods in between the bouts is a significant clue toward diagnosis. 3. Pathophysiology While the pathophysiology is not completely understood, it is thought intussusception is a mass in or on the bowel wall that disrupt normal peristalsis. A mass and the disruption of bowel peristalsis causes the bowel to intussuscept. As this progresses, more and more bowel telescopes into intussuscipiens. This is a medical emergency requiring immediate attention to avoid perforation or necrosis.1 In children under age 5, this condition may be related to Peyer patches—the lymphoid tissue in the bowel. In older children, the pathology may be Meckel diverticulum, intestinal polyps, or cancer.1 4. Diagnostic Tests/Images No point-of-care tests apply. Diagnostic studies would strictly be conducted in an ED. 5. Next Steps l

l

Explain to the mother suspected diagnosis and the need for the child to seek emergent care for diagnostic testing and treatment. Arrange ambulance transfer of child to ED.

6. Cost-Effective and Highly Specific Diagnostic Method and Treatment Options l l

l

A POCUS has been found to have high predictive accuracy to diagnose intussusception.2 In pediatric patients, reduction is typically done without having to resort to surgery. Fluoroscopy-guided enemas, using a water-soluble contrast or air, can exert sufficient pressure to reduce the intussusception can be achieved without recourse to surgery in most cases.3 One research study found children with intussusception in rural areas tend to undergo surgery more often than children with intussusception in urban areas.4

7. ICD-10 and CPT (E/M) Codes ICD-10 Code: R45.83 Excessive crying of child. ICD-10 Code: R10.9 Abdominal pain; non specific. This is the initial diagnosis at the primary care visit. The patient is having abdominal pain and fussiness; the physical exam suggests intussusception. At the initial visit there is not enough evidence for a definitive diagnosis. CPT (E/M) Code: Level 4—99214 (Established Patient) This is a primary care visit for the evaluation and management of an established patient. According to the case, the patient has a new complaint with a possibility for major health complications if untreated or misdiagnosed. This criterion meets 99214, the appropriate level for this clinical scenario. 8. Patient Education Topics l l l

Abdominal pain in pediatric patients Fussy child: differential diagnosis Suspected intussusception

9. Medical or Legal Concerns It is crucial clinicians suspect intussusception promptly and take the appropriate next step. Intussusception is an emergency that requires prompt resolution. If a clinician misses or delays diagnosis the likelihood for severe complications places a patient and a provider at risk. 10. Interprofessional Collaboration l l l l l l l l

Outcome

Receptionist registers the patient. MA takes initial history and vitals. PCP takes history, exam, and makes preliminary diagnosis. EMTs provide transport to ED. ED provider manages child’s care during visit. A nurse inserts an IV line into a vein of child’s arm to give fluids intravenously. Radiologist/physician performs the ultrasound and pneumatic enema. Radiologic technologist assists with the procedure.

The child and his mother went to the ED. There, a series of tests including plain abdominal films and an ultrasound were performed to confirm the diagnosis. As a result of the classic presentation for an intussusception, the patient had a pneumatic enema study done that successfully reduced the intussusception.

Insight

from the

PCP

As with any case, being familiar with typical and atypical presentations in conjunction with a differential diagnosis that is age appropriate is vital in pediatrics. This minimizes missing a diagnosis that could cause significant patient morbidity or mortality. In this instance, recognizing the classic presentation expedites the parent and child to ED. Communicating with the child’s mother was essential here. Explaining to her what I believed the diagnosis to be and why we need to take the child directly to ED had to be done with the right amount of concern using language that would not provoke her to panic. It’s difficult to communicate with a distraught mother holding a squirming, crying child who is clearly in pain. It’s at times like this that a PCP has to rely on being calming, decisive, and compassionate.

References

1. Smith CA, Maloney E. Diagnosis and management of pediatric patients with intussusception. Dig Dis Interv.

2019;3(4):326–332. doi:10.1055/s-0039-1700514

2. Trigylidas TE, Hegenbarth MA, Patel L, et al. Pediatric emergency medicine point-of-care ultrasound for the

diagnosis of intussusception. J Emerg Med. September 2019;57(3):367–374. doi:10.1016/j.jemermed.2019.06.007. https://www.jem-journal.com/article/S0736-4679(19)30464-0/fulltext 3. Radiopaedia. Intussusception. https://radiopaedia.org/articles/intussusception?lang=us - last visited Oct 02, 2020 4. Otero HJ, White AM, Khwaja AB, et al. Imaging intussusception in children’s hospitals in the United States: trends, outcomes, and costs. J Am Coll Radiol. 2019;16(12):1636–1644. doi:10.1016/j.jacr.2019.04.011.

SWELLING AROUND EYES AND ANKLES, ADOLESCENT MALE

CASE

77

chiEf comPlaint

“Swelling around eyes and ankles.”

history

of

PrEsEnt illnEss

A 16-year-old boy of African-American and Filipino heritage arrives at his PCP office with his adoptive father for worsening swelling around his eyes and swollen ankles over the past week. He reports his eyes get puffy with seasonal allergies every fall, but they have never been this puffy. This morning, his feet were so swollen he could not put on shoes. He thought he had the stomach flu with loss of appetite, nausea, a couple of episodes of loose stool, and mild abdominal discomfort over the past week. He also states he has urinated only twice in the past 2 days. He has tried drinking more sports drink, but this has not helped. He works at a fast-food restaurant 3 evenings a week and admits French fries are a routine part of his diet. He plays on the school football team and has practice each day but wonders why he is so “out of shape” and exhausted. Today, he reports he is very tired, does not think he can participate in football practice, and is concerned his feet are too swollen for his cleats to fit.

rEViEW

of

systEms

The patient’s ROS is positive for loss of appetite, mild gastrointestinal upset, nausea, loose stool, reduced urinary output, and significant swelling of eyelids and lips, torso, legs, and feet over the past couple of days. He also reports fatigue, poor exercise tolerance, and mild SOB over the past week. The ROS is negative for fever, chills, rashes, vomiting, constipation, dysuria, hematuria, sore throat, cough, chest pain, backache, injuries, or trauma.

rElEVant

history

The patient’s history is limited due to adoption at age 3. His adoptive father reports typical childhood illnesses since his adoption, with eczema as a toddler and seasonal allergies since age 9. He has asthma with URIs and exercise-induced asthma, onset 1 year ago. No known illness or injuries are reported. He was seen for a sports clearance exam with this provider 9 weeks ago. He measured 157.48 cm (62 in.) and weighed 51.25 kg (113 lbs). At that visit, urine dip was normal with only trace protein. Before he was adopted, he was in foster care with an older sister for 2 years due to parental substance abuse. Perinatal drug exposure was suspected. His older biologic sister was adopted into another family and has a history of kidney transplant for unknown cause. He has no contact with his biologic parents or relatives. Adoptive parents have contact with the adoption agency and will seek more information about family and sibling medical history.

allErgiEs

No known drug allergies; no known food allergies.

mEDications L L

Albuterol HFA inhaler, 2 puffs 15 min pre-exercise and q4h PRN for asthma. Naproxen sodium 220 mg PO, occasional use for post-sports-activity muscle aches.

308  Case 77  •  Swelling Around Eyes and Ankles, Adolescent Male

Physical Examination

Vitals: T 37°C (98.6°F), P 110, R 14, BP 122/62, SpO2 98%, HT 157.48 cm (62 in.), WT 55.79 kg (123 lbs), BMI 22.5. Note: 10 lbs weight gain from exam 9 weeks prior. General: Mildly ill-appearing child alert and cooperative but with low energy. Psychiatric: Fatigued, mildly anxious, but nondepressed appearing. Skin, Hair, and Nails: Warm, dry, intact without rash; mild generalized swelling of trunk; mild edema of arms, moderate 3+ pitting edema lower extremities, especially ankles and feet, with moderate swelling extending into the scrotum. Eyes: PERRLA; eyes are narrowed to slits due to periorbital edema. ENT/Mouth: No lesions, posterior pharynx without erythema or exudate, uvula midline. Neck: No cervical lymphadenopathy, no jugular venous distention, mild generalized edema of face and neck. Lungs: Clear to auscultation bilaterally. Heart: RRR; S1, S2 without murmur, gallop, or bruit. Abdomen: Mild distention with generalized edema, non-tender, no hepatosplenomegaly, no costovertebral angle tenderness. Genital/Rectal: Generalized swelling into scrotum; no testicular lesions, warmth, redness, tenderness, or masses; no penile discharge. Rectal exam was deferred. Neurologic: A&O×3. Cranial nerves II to XII grossly intact, appropriate verbal responses. Office-Based Lab: Urine dip SG 1.035, 3+ protein; negative glucose, blood, leukocytes, or nitrites. Note: urine dip on prior visit normal with only trace protein.

Clinical Discussion Question 1. What is the differential diagnosis?

2. What is the most likely diagnosis? Why?

3. Demonstrate your understanding about the pathophysiology in regard to the most likely diagnosis.

Case 77  •  Swelling Around Eyes and Ankles, Adolescent Male 309 4. Should tests/imaging studies be ordered? Which ones? Why? Think about tests/imaging beyond the

primary care setting as well.

5. What are the next appropriate steps in management?

6. Review a reliable, recent reference and demonstrate an understanding of the initial and long-term

management of this diagnosis. Provide references for your response.

7. What are the pertinent ICD-10 and CPT (E/M) codes for this visit? Provide a short rationale.

8. What are appropriate patient and parent education topics for this case?

9. If not managed appropriately, what is/are the medical/legal concern(s) that may arise?

310  Case 77  •  Swelling Around Eyes and Ankles, Adolescent Male 10. Think about interprofessional collaboration for this case. Provide a list of specialties or other disciplines

and indicate what contribution these professionals might make to managing the patient.

Bedside Manner Questions

11. What would your communication style/approach be with this patient and adoptive father?

12. If a patient and his adoptive father are distressed by the diagnosis, what might offer support?

ANSWER KEY: SWELLING AROUND EYES AND ANKLES, ADOLESCENT MALE

CASE

77

1. Differential Diagnosis L

L

L

L

L

Acute nephritic syndrome is a possibility. This disease is commonly the result of group A streptococcal infection that inflames and swells the renal glomeruli, resulting in poststreptococcal glomerulonephritis leading to impaired kidney function. Primary symptoms are elevated BP, edema, and foamy urine due to high protein levels and dusty colored urine due to hematuria. This diagnosis is less likely with a normal BP and in the absence of symptoms or exam findings of strep infection or hematuria. It would be useful to obtain a strep Ab titer (ASO titer) to rule out recent subclinical strep infection.1 Acute kidney injury is unlikely the diagnosis here. The patient and his father report no recent trauma or nephrotoxic medication exposure. This diagnosis is common in hospitalized pediatric patients with exposure to highly nephrotoxic medications. It is less likely in the absence of recent exposure to nephrotoxic medications including naproxen sodium, which he uses occasionally for post-sports-activity muscle aches.2 Pediatric nephrotic syndrome is a likely diagnosis due to the chief complaints of anorexia; fatigue; subcutaneous edema of face, neck, scrotum, and extremities; and absence of exposure to nephrotoxic medications or infections.3 Angioedema is a condition of allergic reaction resulting in submucosal edema of the oral cavity, larynx, or gut or subcutaneous tissues of the lips, face, neck, and extremities. It may be induced by certain foods or medications including nonsteroidal antiinflammatory agents. Angioedema typically is a self-limiting condition, resolving within 2 days. This is a less likely diagnosis as the patient had no recent medication or new food exposures and the edema condition persisted beyond 2 days.4,5 Sickle cell nephropathy is the result of an inherited sickle cell gene from each parent and is common in African Americans. This is a less likely diagnosis as SCD typically becomes evident during the first year of life. Sickle cell trait, a single inherited gene from one parent, rarely causes disease. Sickle cells may be identified by their abnormal red cell morphology on a blood smear differential.6

2. Most Likely Diagnosis Pediatric nephrotic syndrome. About 95% of children with pediatric nephrotic syndrome will present initially with pitting edema, and about 30% will have a history of allergies. 3 Acute onset and timing of this condition reduces the likelihood of chronic congenital conditions such as sickle cell anemia without evidence of prior episodes in infancy. A peripheral blood smear for sickle cell morphology or genetic testing would eliminate the likelihood of sickle cell, supporting nephrotic syndrome. Proteinuria and absence of gross hematuria on an office urine dipstick would support the likely diagnosis. A confirmatory diagnosis of nephrotic syndrome and identification of nephrotic syndrome subset will be determined by a pediatric nephrologist through further lab evidence of associated nephrotic range proteinuria on 24-hour urine, elevated urine protein/creatinine ratio, and presence of hyperlipidemia and hypoalbuminemia. Renal biopsy for diagnosis may be required if resistance to treatment therapies occurs.3

3. Pathophysiology PNS in children, also called INS, is the direct result of renal-focused damage to the glomeruli of the kidneys resulting in deficient filtration and excessive protein clearance.7 In the United States, nephrotic syndrome incidence is approximately 20 cases per 1 million children.7 There are four major subsets of PNS, each with unique processes leading to glomerular destruction, listed according to incidence: (1) MCN, (2) focal glomerulosclerosis, (3) membranous nephropathy, and (4) hereditary nephropathies. MCN is the most common form of primary pediatric nephrosis but has the lowest risk for chronic renal disease with adequate management, despite common recurrences throughout childhood. Secondary forms of nephrotic syndrome are the result of systemic disorders such as diabetes, systemic lupus erythematosus, viral infections such as HIV, hepatitis B and C, malignancies, toxic exposures to certain drugs, chemicals and heavy metals, and congenital or genetic conditions causing secondary glomerular damage such as sickle cell.7 In both primary and secondary nephrotic syndrome, the pediatric forms manifest as (1) proteinuria, (2) hypoalbuminemia, (3) hyperlipidemia, and (4) edema. Hypercoagulability creates a further potential risk for thrombus in all major organs and vessels.8 The initiating event leading to proteinuria is not well understood for INS, but researchers suspect a strong autoimmune correlation given the successful response to treatment using glucocorticosteroids in most primary nephrosis cases.3 Additionally, up to 40% of children have some form of allergic or autoimmune condition prior to onset of nephrotic disease. Current research focuses upon circulating inflammatory and immune-modulating cytokines and interleukin factors.7 Once the kidneys have been stimulated to massive proteinuria, a decrease in circulating albumin leads to intravascular osmotic fluid shift, fluid retention, and the resultant edema. In addition to decreased oncotic pressure, it is believed there is increased activity of aldosterone and vasopressin, which further influences sodium and fluid retention. Oncotic pressure changes are thought to stimulate the liver to produce more albumin in response to the hypoalbuminemia, resulting in increased hepatic lipoprotein synthesis and the resultant multiform hyperlipidemia associated with nephrotic syndrome. In more severe cases, respiratory compromise may occur due to respiratory constriction with ascites and edema. There is increased risk for cardiac events due to added workload and the hypercoagulable state of nephrosis, producing thrombus formation potentially leading to life-threatening embolus. Response to steroid therapy, the severity of the presenting form of INS, and the recurrence frequency all impact the potential long-term prognosis for primary INS. While relapse is a common characteristic of MCN, in the most common form of pediatric nephrotic syndrome, only 5% of children will have persistent steroid-responsive disease beyond age 18.9 Outcomes for secondary nephrosis will be directly influenced by the source and extent of renal injury and the ability to manage the primary source of systemic or extrarenal impacts. There are no current identified measures to prevent idiopathic and congenital pediatric nephrotic syndrome.10 4. Diagnostic Tests/Images Urine dip: SG 1.035, 3+ protein; negative glucose, blood, leukocytes, or nitrites. Note: urine dip on prior visit normal with only trace protein. 5. Next Steps L

L

The proteinuria on office urine dip and significant generalized edema with documented 10-lb weight gain strongly suggested an acute renal syndrome. A call was placed to the closest pediatric nephrologist, located in a large urban center 60 miles away from the patient’s home. Due to the risk of fluid overload and renal injury with oliguria, further local testing was deferred to transport the patient for immediate admission to a pediatric ICU in a large urban teaching hospital. The pediatric nephrologist met the patient at the hospital where the diagnosis of pediatric nephrotic syndrome was confirmed.

6. Initial and Long-Term Management The treatment for pediatric nephrotic syndrome is targeted at the cause. The most common form of pediatric nephrotic syndrome, MCN, also called lipoid nephrosis or nil disease, is the cause of 85% to 95% of nephrosis in children and 50% in adolescents.9 In episodes of severe edema, hospitalization

will provide initial therapy with diuretics to reduce fluid overload, correction of hypoalbuminemia, and close monitoring of kidney function.11 Additional goals of initial and long-term management include early identification and risk management for infection, blood clots, and hyperlipidemia. Pediatric MCN has a high response to glucocorticosteroids such as prednisone and low risk for long-term sequela, but typically multiple episodes recur within 5 years from initial case.7 Steroidresistant nephrotic syndrome occurs in 10% to 20% of pediatric cases and increases risk for renal failure requiring kidney transplant.12 Management by pediatric specialists may be required in secondary nephrosis where conditions such as systemic lupus or sickle cell require focused rheumatology or hematology therapy. Long-term monitoring requires management of fluid retention and proteinuria through use of diuretics and angiotensin antagonists.11 Routine use of corticosteroids requires monitoring every 3 months for steroid toxicity and growth retardation. Monitoring of calcium and vitamin D levels will identify early risk for osteoporosis and annual ophthalmology exams monitor for steroid-induced cataracts. Adequate immunization requires special attention to timing related to immunosuppressive therapy and exposure to live virus vaccines. The patient and family may benefit from psychosocial counseling to monitor the effect of chronic disease on emotional wellness.9 7. ICD-10 and CPT (E/M) Codes ICD-10 Code: R60.9 Edema, unspecified. This is the initial diagnosis as the most significant acute exam finding. Although nephrotic syndrome is strongly suspected, it requires additional specialist evaluation for a final diagnosis. It is appropriate to list additional ICD-10 codes supporting the need to address suspected diagnosis of acute nephrotic syndrome: ICD-10 Code: R63.0 Loss of appetite (anorexia) ICD-10 Code: H02.849 Edema of unspecified eye, unspecified eyelid ICD-10 Code: R80.9 Proteinuria, unspecified ICD-10 Code: R53.83 Fatigue, other CPT (E/M) Code: Level 4—99214 (Established Patient) This is a primary care visit for an established patient that requires a detailed history and examination and is of moderate complexity and decision-making. 8. Patient Education Topics L

L

L

L

L

L

Review of the diagnosis of nephrotic syndrome, symptoms, basic pathophysiology, and positive long-term prognosis with diligent management and monitoring. Sport and physical education following the acute episode and completion of diagnostics is not restricted as activity will reduce formation of blood clots. Diet should focus upon adequate caloric and protein intake and low sodium diet. Fluid restriction is not routinely required. Daily morning monitoring of weight, fluid balance, and urine for protein using dipstick urines can track renal status and identify recurrence in early stages. Maintaining wellness, addressing infection quickly by temperature monitoring and routine follow-up for health maintenance, and vaccination lower the risk for chronic renal disease and management potential side effects of medications. Promoting self-care and adolescent independence will enhance routine health monitoring and reduce risk for emotional impacts of chronic disease on adolescent development.

9. Medical or Legal Concerns It is crucial that children with suspected nephrotic syndrome receive proper early identification; appropriate referral; long-term management of fluid retention, hyperlipidemia, and hypertension risk; and monitoring for potential infection or blood clots. The effect of a child’s nephrotic disease upon the family for medical visits, therapies, and hospitalizations may lead to significant loss of work or school time. Nephrotic syndrome is a qualifying disability and warrants accommodations in work and school environments including eligibility for the FMLA for caregivers. The child is eligible for special accommodations in school for fatigue, protection from infection, and impact upon school attendance due to disease management. Communication from medical providers must document appropriate accommodations and activity restrictions to support safety and academic needs. 10. Interprofessional Collaboration L L

L

L

L

L

Receptionist registers the patient. MA takes brief history and vital signs and alerts clinician to significant edema with 10-lb weight gain from prior visit. PCP formulates most likely diagnosis; contacts specialist for phone consult regarding acute patient management and ongoing patient and family education regarding the condition; and communicates with school as needed. Pediatric nephrologist consults with PCP and provides hospital-based assessment for definitive diagnosis, renal management, and post-discharge follow-up. Pediatric hospitalist admits to pediatric intensive care, manages general care, and coordinates with pediatric nephrologist. School nurse is given the patient’s medical diagnosis and explanation of accommodations required in school. The school nurse initiates 504 plan for qualifying medical condition in school, coordinating with school administration and faculty to maintain academic progression and success during illness.

Outcome

A consultation phone call was placed to the nearest available pediatric nephrologist located at a large urban teaching hospital. The patient was driven by his father to the urban hospital ED and directly admitted to the pediatric ICU. His diagnosis of pediatric nephrotic syndrome was confirmed, and he had positive response to steroids and diuretics. He received dietary counseling and education for strict fluid and urine monitoring. He was discharged home with activity restriction from sports and physical education until full renal assessment was completed and full convalescence achieved. Studies showed no residual nephropathy, but close monitoring by pediatric nephrology continued throughout his young adulthood.

Insight

from the

PCP

This was a particularly memorable patient experience. The profound edema extending into the scrotum was remarkable and associated with proteinuria and oliguria, and new exercise intolerance prompted the concern for an acute renal condition. My initial concern for glomerulonephritis was negated by the absence of hematuria, rash, and exposure to or history of sore throat associated with strep pharyngitis. The assessment required that I quickly review differentials online prior to calling a pediatric nephrologist. Practice in a rural community limits resources in many specialties, especially in pediatrics. It requires an understanding of the limitations of the local community hospital and a general knowledge of advanced care available at the closest tertiary hospital. Because this hospital was over 60 miles from the patients’ home, the father needed confirmation that these services were necessary. Post-discharge, the patient and family were required to make this commute to the nephrologist to be monitored for many years. I was able to provide local support for the patient, coordinating his return to school. It was especially important as his PCP that I maintain local follow-up for this teenager and family experiencing a serious chronic health condition. It was disappointing and discouraging for him when he was unable to return to playing contact sports. He found the daily monitoring and initial diet restrictions burdensome as well. Together, we brainstormed positive social involvement, and he chose to accept a trainer position with the football team, which allowed

him continuity with his social group. He also increased his commitment to his show dogs, where he proved to be highly successful. I remained his PCP until he was 20, identifying a second episode of nephrotic syndrome early with rapid resolution. I reinforced the fact that diligence to his morning urine monitoring for protein had protected his health, allowing for this early diagnosis and treatment. I developed a strong relationship of trust and effective communication with this patient and family that promoted optimal health where severe renal disease could have occurred. Seeing an adolescent reach adulthood physically and mentally healthy is an incredibly rewarding experience in primary care, especially in small communities, where complex care can be a challenge to locate.

References

1. Fathallah-Shaykh S. Pediatric nephritis. Medscape. April 19, 2018. 2. Ciccia E, Devarajan P. Pediatric acute kidney injury: prevalence, impact, and management challenges. Int J Nephrol

Renovasc Dis. 2017;10:77–84. doi:10.2147/IJNRD.S103785

3. Lane JC, Langman CB. Pediatric nephrotic syndrome. Medscape. Updated. October 24, 2018. 4. Syue Y, Li C, Chen W, et al. Significant predictive factors of the severity and outcomes of the first attack of acute

angioedema in children. BMC Pediatr. 2019;19:423. doi:10.1186/s12887-019-1809-8

5. Depetri F, Tedeschi A, Cugno M. Angioedema and emergency medicine: from pathophysiology to diagnosis and

treatment. Eur J Intern Med. 2019;59:8–13. doi:10.1016/j.ejim.2018.09.004

6. Evidence-based management of sickle cell disease: NIH expert panel report, 2014. Pediatrics. December

2014;134(6):e1775. doi:10.1542/peds.2014-2986

7. Cohen EP, Batuman V. Nephrotic syndrome. Medscape. July 9, 2019. 8. Al-Azzawi HF, Obi OC, Safi J, Song M. Nephrotic syndrome-induced thromboembolism in adults. Int J Crit Illn Inj

Sci. 2016;6(2):85-88. doi:10.4103/2229-5151.183019

9. Mansur A. Minimal-change disease. Medscape. 2018. https://emedicine.medscape.com/article/243348-overview. 10. National Institute of Diabetes, and Digestive, and Kidney Diseases. Childhood Nephrotic Syndrome. NIH. https://

www.niddk.nih.gov/health-information/kidney-disease/children/childhood-nephrotic-syndrome.

11. Sinnakirouchenan R. Nephrotic syndrome treatment & management. Medscape. 2020. https://emedicine.medscape.

com/article/244631-treatment#d1.

12. Imig JD, Ryan MJ. Immune and inflammatory role in renal diseases. Compr Physiol. April 2013;3(2):957–976.

doi:10.1002/cphy.c120028

BILATERAL HEEL PAIN, ADULT FEMALE

CASE

78

Chief CoMPlaint

“Bilateral heel pain.”

historY

of

Present illness

A 28-year-old obese woman presents as a new patient with a several-month history of bilateral heel pain. She denies any precipitating event, trauma, or specific incident that triggered her pain. The patient describes the heel pain as a sharp and stabbing sensation, exacerbated by prolonged standing and walking. The patient reports that the heel pain begins after taking the first few steps after getting out of bed. She did go and see a local urgent care provider, who diagnosed the patient with overuse syndrome and prescribed ibuprofen, rest, and OTC inserts for her shoes. However, the patient reported minimal relief in her symptoms. She works as a grocery clerk and is on her feet 8 hours a day.

reVieW

of

sYsteMs

The patient’s ROS is positive for heel pain with prolonged standing and walking. Additionally, there is also pain at night. The patient’s ROS is negative for bilateral swelling of the ankle, foot, and digits. She denies any altered sensation or weakness in both extremities. She also denies any gait abnormalities or flat feet, any temperature changes, any recent illnesses, or hospitalizations.

releVant historY

The patient’s medical history is unremarkable other than obesity. She works in a local grocery as a grocery clerk. She is single and currently not sexually active. She doesn’t drink alcohol or use any recreational drugs, and her family history is noncontributory.

MeDiCations

Ibuprofen 200 mg 1 or 2 tablets q6h PRN for pain.

allergies

No known drug allergies; no known food allergies.

PhYsiCal eXaMination

Vitals: T 37°C (98.6°F), P 84, R 20, BP 134/88, HT 167.6 cm (66 in.), WT 112.5 kg. (248 lbs), BMI 40. General: No apparent distress. Musculoskeletal/Bilateral Foot and Ankle: There is no swelling, deformity, crepitus, or bogginess of the feet and ankle. There are no skin changes. There is no tenderness over the forefoot and midfoot foot. ROM is as follows: dorsiflexion: 15/15, plantar flexion: 40/40, eversion: 20/20, inversion: 35/35, all without pain or discomfort. An anterior drawer test is negative. There is no lateral ligamentous laxity. There is mild hallux valgus deformity to both the great toes without tenderness. There is a negative Tinel’s over the tibial and sural nerves. There is moderate pes cavus of both feet. There is no forefoot varus or “too many toes sign.” There is no hindfoot valgus. There is no tenderness over the subtalar joint. There is exquisite point tenderness over the medial aspect

312  Case 78  •  Bilateral Heel Pain, Adult Female

of the calcaneal tuberosity. Heels are symmetrical without any bony masses palpated. Dorsalis pedis and posterior tibialis are 2+. Capillary refill is less than 2 seconds. Neurologic: Sensation intact to the dorsal and plantar aspects of the foot, ankle, and leg. No signs of foot drop. No altered sensation compared to the contralateral extremity. No ankle clonus. Achilles reflex 2+.

Clinical Discussion Questions 1. What is the differential diagnosis?

2. What is the most likely diagnosis? Why?

3. Demonstrate your understanding about the pathophysiology in regard to the most likely diagnosis.

4. Should tests/imaging studies be ordered? Which ones? Why? Think about tests/imaging beyond the

primary care setting as well.

5. What are the next appropriate steps in management?

Case 78  •  Bilateral Heel Pain, Adult Female 313 6. Review a reliable, recent reference and demonstrate an understanding of the treatment recommendations

associated with this diagnosis. Include the name of the references.

7. What are the pertinent ICD-10 and CPT (E/M) codes for this visit? Provide a short rationale.

8. What is the appropriate patient education for this case?

9. If not managed appropriately, what is/are the medical/legal concern(s) that may arise?

10. Think about interprofessional collaboration for this case. Provide a list of specialties or other disciplines

and indicate what contribution these professionals might make to managing the patient.

314  Case 78  •  Bilateral Heel Pain, Adult Female

Bedside Manner Question

11. What would your communication style/approach be with this patient?

ANSWER KEY: BILATERAL HEEL PAIN, ADULT FEMALE

CASE

78

1. Differential Diagnosis L

L

L

L

L

Posterior tibial tendonitis is the most common cause of acquired pes planus (flat foot) in an adult. Acquired pes planus is a loss of the longitudinal arch of the foot, and a patient may present with medial foot pain. According to the patient’s physical examination, she has moderate pes cavus, characterized by a high longitudinal arch in the foot. This condition is unlikely the diagnosis. Plantar fasciitis is a possibility. Patients universally complain of heel pain early in the morning and at rest during the night. Obesity is a risk factor for developing plantar fasciitis. Additionally, patients will report heel pain with prolonged standing and walking. Heel spurs can lead to heel pain at times. Only 5% of the population with heel spurs exhibit heel pain.1–3 It may be difficult to differentiate a painful heel spur from plantar fasciitis.1–3 Heel spurs are bony projections and not commonly considered a risk factor for the development of plantar fasciitis. The patient’s physical examination did not reveal a bony prominence over the heel area, making this condition an unlikely diagnosis. Calcaneal stress fracture is a good consideration. However, most calcaneal fractures are the result of trauma, overuse, or repetitive stress. The patient did not report any traumatic event or recent illnesses. Stress fractures of the calcaneus are very rare. This diagnosis is unlikely. Tarsal tunnel syndrome is caused by compression of the tibial nerve following an ankle injury. Compression of the medical calcaneal nerve (proximally) can lead to heel pain.3 Tarsal tunnel syndrome can be very painful and often presents with paresthesia that radiates to various parts of the ankle and foot. The patient’s history and physical examination do not support the diagnosis of tarsal tunnel syndrome.

2. Most Likely Diagnosis Plantar fasciitis. The patient’s symptoms and physical examination are consistent with this condition. The risk of plantar fasciitis increases with weight gain and prolonged standing and walking. Patients with plantar fasciitis report heel pain immediately after taking their first couple of steps after getting out of bed.1–3 3. Pathophysiology This condition affects about 2 million Americans annually. Histologic changes associated with plantar fasciitis are related to collagen degeneration associated with receptive micro-trauma to the heel, leading to pain. Clinicians commonly refer to this condition as an overuse injury. Only 15% of cases are due to systemic factors.4 4. Diagnostic Tests/Images Bilateral standing views consisting of anteroposterior, lateral, and oblique radiographs showed no fractures, dislocations, or abnormalities. The Bohler angle is 35° (normal range is 20°–40°). Considered Test An ultrasound or MRI can confirm the diagnosis of plantar fasciitis; however, these diagnostic tests should not be the first option, unless there is concern of an injury to the plantar fascia or surrounding structures.

5. Next Steps L

L

L L

L

Refer the patient to physical therapy to start a series of exercises to stretch the plantar fascia, plantar flexor muscles, and Achilles tendon. Advise the patient about weight loss strategies to decrease the weight load pressure on the plantar fascia. Recommend night splint for the passive stretch of the plantar fascia. Recommend that the patient be placed on modified duty for several weeks, so that she is not standing on her feet for more than 8 hours. A controlled ankle motion walker (cam boot), short leg casting, ESWT, and cortisone injection are sometimes used to manage recalcitrant plantar fasciitis.5

6. Treatment Recommendations L

L

L

L

There is no indication for narcotics in common musculoskeletal conditions such as plantar fasciitis.6 PT may help plantar fasciitis in several ways, including reducing pain, guiding stretching exercises, supervising strengthening exercises, and providing an additional point of contact for the patient during the long recovery. Also, the therapist can use manual therapy including joint and soft tissue mobilization to improve lower extremity flexibility, to decrease pain, and to improve function.7 If plantar fasciitis lasts over 6 months, minimally invasive procedures may be indicated to relieve pain. Those procedures include corticosteroid injection, botulinum toxin injection, PRP injection, dry needling, ESWT, or intense therapeutic ultrasound.8 If these previously mentioned treatments failed, surgery may be the next step for patients with chronic plantar fasciitis. Operative treatments include partial plantar fasciotomy or gastrocnemius lengthening performed through a variety of approaches.9

7. ICD-10 and CPT (E/M) Codes ICD-10 Code M72.2 Plantar fasciitis. Plantar fasciitis is primarily a clinical diagnosis. Although plain x-rays are still pending to further evaluate the condition, it is appropriate to use this ICD-10 code during the initial visit. CPT (E/M) Code: Level 3—99203 (New Patient) This is the evaluation of a new patient with a new complaint. We used level 3 for this case as the physical exam was limited to the patient's chief complaint; it was not a full initial new patient visit. 8. Patient Education Topics L L L L

Rest Reduce heel impact by limiting high-impact activities Weight loss Supportive shoe and arch support

9. Medical or Legal Concerns While plantar fasciitis is fairly easy to diagnose based on history and symptoms, it can lead to significant morbidity if left untreated. Early detection and treatment are crucial to resolving the patient’s symptoms. It should be noted that when plantar fasciitis does not respond to conservative treatment, a patient should be referred to a podiatrist or orthopedist. Patients do not require FMLA but may benefit from modified duty restrictions, which are highly provider dependent. Recommendations include no lifting up to 15 lbs intermittently for 4 hours per day; standing for 2 hours per day cumulative; no climbing, twisting, or pulling for 4 to 6 weeks.

10. Interprofessional Collaboration L L L L L L

Receptionist registers the patient. MA takes history and vitals. PCP formulates the most likely diagnosis. PT provides evidence-based PT for the heel pain. Radiology technician performs plain radiography procedure. Radiologist reads the radiography and provides interpretation.

Outcome

The patient was prescribed a night splint at a 90-degree angle, PT, and weight loss strategies. PT modalities were to stretch the plantar fascia, plantar flexor muscles, and Achilles tendon. Iontophoresis and ultrasound modalities were included as well. The patient was given a modified work order for 6 weeks to reduce heel impact. After 3 months of treatment, her condition improved significantly.

Insight

from the

PCP

As the PCP, I was able to make the clinical diagnosis based on a complete history and physical examination. Plantar fasciitis is a common reason why patients present to a PCP with heel pain. Clinicians should have a high index of suspicion when patients complain of heel pain with the first initial steps after getting out of bed. Obesity along with high-impact activities can exacerbate heel pain. Talking to patients about weight loss can be challenging. As a provider, I always want to be respectful and nonjudgmental when discussing weight loss strategies. The patient was receptive to my suggestions and followed through with the recommendations. She lost 12 lbs within 3 months since our first encounter. Her symptoms resolved and she was clinically asymptomatic.

References

1. Koca T, Aydın A, Sezen D, Başaran H, Karaca S. Painful plantar heel spur treatment with Co-60 teletherapy: factors

influencing treatment outcome. Springerplus. 2014;3:1–4. doi:10.1186/2193-1801-3-21

2. Crawford F, Thomson CE. Interventions for treating plantar heel pain. Cochrane Database of Systematic Reviews

2010, Issue 1. Art. No.: CD000416. DOI: 10.1002/14651858.CD000416.pub2

3. Thomas MJ, Menz HB, Mallen CD. Plantar heel pain. BMJ. 2016;353:i2175. doi:10.1136/bmj.i2175 4. Buchanan BK, Kushner D, eds. Plantar Fasciitis. Stat Pearls Publishing;2018. 5. Gollwitzer H, Saxena A, Di Domenico LA, et al. Clinically relevant effectiveness of focused extracorporeal shock 6. 7. 8. 9.

wave therapy in the treatment of chronic plantar fasciitis: a randomized, controlled multicenter study. J Bone Joint Surg Am. May 6, 2015;97:701. doi:10.2106/JBJS.M.01331 Kidd VD. Opioid prescribing in orthopaedic surgery: an evolving paradigm. JBJS J Orthop Phys Assist. 2018;6(1):e4. doi:10.2106/JBJS.JOPA.17.00033 Martin RL, Davenport TE, Reischl SF, et al. Heel pain-plantar fasciitis: revision 2014. J Orthop Sports Phys Ther. 2014;44(11):A1–A33. doi:10.2519/jospt.2014.0303 Miller LE, Latt DL. Chronic plantar fasciitis is mediated by local hemodynamics: implications for emerging therapies. N Am J Med Sci. 2015;7:1–5. doi:10.4103/1947-2714.150080 Daniel Latt L, Jaffe DE, Tang Y. Evaluation and treatment of chronic plantar fasciitis; First published February 13, 2020 Review Article. https://doi.org/10.1177/2473011419896763

DRY, CRACKED SKIN, ADULT FEMALE

CASE

79

Chief CoMPlaint

“Dry, cracked skin.”

historY

of

Present illness

A 32-year-old woman presents to her PCP with complaints of a persistent “scaly dry rash” just beneath the left knee. She states the condition is chronic, and it seems a little larger now and it has become bothersome at times. The condition is intermittent, itchy, and red. This time, she attempted to scratch it off but it started to hurt and bled when she “peeled” a little off. The patient then attempted OTC hydrocortisone cream and warm compresses, with some relief. Regardless of what she does, the rash comes back on the same spot. The patient explains that she is simply wanting better relief and answers. She thinks the scaly patch was worse during the winter months but is unsure. She has no history of allergies and has not changed soaps or daily routine. She has not been around anyone with any dermatologic conditions and reports no concerns with STIs.

reVieW

of

sYsteMs

The patient’s ROS is positive for increased stress, dry rash that is itchy and scaly, and increasing severity during winter months. Her ROS is negative for fever, nausea/vomiting, diarrhea, sore throat, other rashes, or sudden weight changes. She denies any exudates or blistering. The ROS is negative for angina and SOB.

releVant HistorY

The patient has a history of elevated BP and obesity. She is a full-time student and mother and works part-time in retail. She states her life stressors have been particularly difficult over the past year, including stress in her marriage. She has two children. She denies tobacco or drug use; she drinks socially. There is no family history of dermatologic or autoimmune conditions. Her family history is noncontributory.

allergies

No known drug allergies; no known food allergies.

MeDiCations

Ibuprofen 400 mg PRN TID for pain/bleed associated with rash.

PhYsiCal eXaMination

Vitals: T 37.2°C (98.9°F), P 82, R 19, BP 132/88, HT 170.2 cm (67 in.), WT 87.1 kg (192 lbs), BMI 32. General: Alert; appears well hydrated and nourished and in no apparent distress. Psychiatric: A&O with cognitive function being intact, cooperative with exam, good eye contact, and good insight. Skin, Hair, and Nails: Patient has a scaly whitish-gray rash just inferior to the left patella, which appears as plaque. Some mild blood spots/pinpoints (possible Auspitz sign) noted. There is erythema noted throughout rash. Lichenification questionable. Hair and nails non-concerning.

316  Case 79  •  Dry, Cracked Skin, Adult Female

Neck: Supple, FROM, no lymphadenopathy. Chest: Normal chest rise/fall. Lungs: Clear to auscultation bilaterally; no wheezes, rales, or rhonchi noted. Heart: Normal sinus, no murmurs, normal rate, no jugular venous distention.

Clinical Discussion Questions 1. What is the differential diagnosis?

2. What is the most likely diagnosis? Why?

3. Demonstrate your understanding about the pathophysiology in regard to the most likely diagnosis.

4. Should tests/imaging studies be ordered? Which ones? Why? Think about tests/imaging beyond the

primary care setting as well.

5. What are the next appropriate steps in management?

Case 79  •  Dry, Cracked Skin, Adult Female 317 6. Review a reliable, recent reference and discuss treatment approaches and comorbidities associated with

this case. Provide the name of the references.

7. What are the pertinent ICD-10 and CPT (E/M) codes for this visit? Provide a short rationale.

8. What is the appropriate patient education for this case?

9. If not managed appropriately, what is/are the medical/legal concern(s) that may arise?

10. Think about interprofessional collaboration for this case. Provide a list of specialties or other disciplines

and indicate what contribution these professionals might make to managing the patient.

Bedside Manner Question

11. What would your communication style/approach be with this patient?

ANSWER KEY: DRY, CRACKED SKIN, ADULT FEMALE

CASE

79

1. Differential Diagnosis L

L

L

L

L

Eczema is chronic in nature, causing repeat exacerbations of pruritus and erythema and has a lichen-like appearance. This can be similar to a psoriatic appearance, causing confusion in a diagnosis. In this patient’s case, eczema would be less likely due to the scaliness. Acute cellulitis causes significant symptoms and is usually not chronic. Erythema is usually present, along with tenderness and potential infection-like symptoms such as exudates. Swelling and/or hardening of skin can also present depending on the severity and spread of infection. These significant symptoms can rule it out from a chronic skin condition as cellulitis is an acute finding. Contact dermatitis can cause a local reaction of erythema, with or without small papules. Hives can also be present. Recontact with the same agent can cause repeat skin rash of similar presentation. Location can be anywhere anatomically and is generally not a chronic condition. This is an unlikely diagnosis because this patient’s condition is chronic. Also, it is unlikely that something is directly contacting and irritating the skin just below the knee for a lengthy period without the patient’s knowledge. Psoriasis is a chronic condition with exacerbations. It is characterized by a rapid proliferation of skin cells with an inflammatory response to the epidermis. It can present with a dry, scaly, and itchy rash. It may have a positive Auspitz sign, though this finding is not sensitive or specific to psoriasis. This diagnosis is high on the differential list due the presenting history and physical appearance of the rash. Candidiasis is unlikely. Superficial mycosis may be repeated, however, only under certain risk factors, such as being an athlete, being overweight, chronic sweating, or heat conditions. This skin infection can be ruled out because it typically has multiple locations of origin and is an acute skin reaction.

2. Most Likely Diagnosis Psoriasis.This is a chronic idiopathic condition with no cure, causing repeat exacerbations of symptoms. It is an inflammatory disorder that typically causes erythema and silvery gray scales/plaques. Not all symptoms may be present but repeat exacerbations occur, especially in certain weather conditions. For this patient, her complaint and examination findings are consistent with this diagnosis. 3. Pathophysiology Psoriasis involves a rapid proliferation of squamous epithelial cells. These cells undergo a rapid mitotic division, thus making the maturation abnormal. The dermal immune response that occurs is believed to be a T-lymphocyte-mediated autoimmune response.1 The interleukin/T helper 17 cell axis plays an important role in the pathogenesis of psoriasis. The dermis can thicken, and the epidermis thickens through a hyper-proliferation of keratinocytes. The thickness can also be from the process of expanded dermal vasculature, infiltration of neutrophils and lymphocytes, and inflammation.2 4. Diagnostic Tests/Images No tests or imaging are indicated specifically for psoriasis.

Considered Tests If joint involvement is suspected, as with psoriatic arthritis or another inflammatory response, an RF, ESR, uric acid, ANA, and an anti-CCP Ab would be ordered to exclude other autoimmune inflammatory conditions. Psoriatic arthritis is usually seronegative. L Plain x-rays of the hands may identify psoriatic arthritis. L

5. Next Steps L

L L

L

The patient is treated with triamcinolone 0.1% ointment. This is a medium-potency topical as a mild potency would be ineffective for psoriasis. Start with a lower “medium” potency and then adjust upward as necessary. Follow-up visit recommended in 2 weeks. Discontinue ibuprofen use. Phototherapy and emollients (vitamin D) are also explained to the patient for this stage of presentation. Another useful treatment tool is bleach baths (per protocol), which have shown some effectiveness. If there is scalp involvement, there are shampoo options such as coal tar topical. Dermatology and/or rheumatology consult is indicated if skin lesions persist or psoriatic arthritis develops.

6. Treatment Approach and Comorbidities L

L

L

The current literature3 supports treating mild-to-moderate plaque psoriasis with topical therapy during the maintenance phase with twice-a-week or weekend applications of a topical steroid, vitamin D analog, or a fixed-dose combination of both. The goal is to control the condition and prevent recurrence. The fixed-dose combination is the preferred topical for both initial and maintenance phases.3 Corticosteroids are often the preferred topical treatment for psoriasis. However, corticosteroid phobia (concern regarding the use of topical corticosteroids) is common among patients with dermatology and often results in treatment non-adherence.4 Therefore, patient education and explanation regarding the appropriate use, safety, and potency of topical corticosteroids is important to improve compliance.5 Evidence suggests that patients with psoriasis experience greater chronic sleep impairment compared to healthy counterparts. Also, other comorbidities seen in patients with psoriasis include cardiovascular disease, obesity, diabetes, obstructive sleep apnea, and chronic obstructive pulmonary disease.6 Therefore, PCPs need to be mindful of these comorbidities associated with psoriasis, so that they may start early interventions (preventive strategies) with patients.

7. ICD-10 and CPT (E/M) Codes ICD-10 Code: L40.0 Psoriasis, unspecified. Plaque psoriasis, characteristic plaques in any part of the skin. Common areas: knees, elbows, and scalp. Erythema may or may not be present. CPT (E/M) Code: Level 3—99213 (Established Patient) Established patient, problem-focused patient encounter, and low–moderate decision-making. 8. Patient Education Topics L L L L

Phototherapy considerations, vitamin D/zinc Patient understanding of the chronic nature and appearance of the disease Psoriasis support groups Stress reduction

9. Medical or Legal Concerns At this point, the legal concerns are relatively low as the patient’s condition seems to be in the mild– moderate stages with no complications. This, however, can change in severity toward moderate to severe as well as could cause complications. In the case of severe dermatology conditions and/or

other organ system involvement, it is crucial to monitor/assess for such known complications. In the cases of severe skin conditions, a patient could suffer from detrimental psychologic stressors. In the case of severe psoriasis, patients could be placed on potent drugs (such as disease-modifying antirheumatic drugs) that can put them at risk for adverse reactions. 10. Interprofessional Collaboration L L L

L

Receptionist registers the patient. MA takes a brief history and vital signs and calls for referral. PCP assesses, examines, and formulates most likely diagnosis; prescribes treatment; refers to specialist; and provides ongoing care. Dermatologist confirms diagnosis and manages and treats patient.

Outcome

The patient responded well to treatment with a medium potency topical corticosteroid with no moderate or severe psoriasis noted and is now following up on a 3-month basis or as needed. Fortunately, no support groups or depression counseling were needed in this case. The patient is using home emollients/vitamin D creams as well as prescribed topical corticosteroid PRN. She has initiated some healthy lifestyle modifications (routine exercise) to reduce her stress. The patient understands potential complications and possible disease progression.

Insight

from the

PCP

Psoriasis can have a dramatic effect on a person’s life depending upon disease stage or progression. If affected areas are visible or if plaques comprise a great deal of skin surface area, therapeutic communication by a PCP is a must. Education plays a big part here, and patients will be in desperate need of your help. It is difficult, but a PCP must gently and compassionately explain that this is a manageable chronic autoimmune condition but there is risk of progression. Be aware of psoriasis resources and support groups for patients as well as consider SSRIs if indicated. Many younger patients will self-isolate from dating and social activities as a result of psoriasis, especially when present on the face and hands, leading to symptoms of depression. Being empathetic and supportive means the world to these patients.

References

1. Cash JC, Glass CA. Family Practice Guidelines. Springer Publishing Company; 2017. 2. McCance KL, Huether SE. Pathophysiology: The Biologic Basis for Disease in Adults and Children. Mosby; 2015. 3. Imafuku S, Zheng M, Tada Y, et al. Asian consensus on assessment and management of mild to moderate plaque

psoriasis with topical therapy. J Dermatol. 2018;45(7):805–811. doi:10.1111/1346-8138.14338

4. Mueller SM, Itin P, Vogt DR, et al. Assessment of “corticophobia” as an indicator of non-adherence to topical

corticosteroids: a pilot study. J Dermatol Treat. 2017;28(2):104–111. doi:10.1080/09546634.2016.1201189

5. Segaert S, Calzavara-Pinton P, de la Cueva P, Jalili A, Lons Danic D, Pink AE, Thaçi D & Gooderham M. Long-term

topical management of psoriasis: the road ahead, J Dermatol Treat. 2020. doi:10.1080/09546634.2020.1729335

6. Reddy V, Myers B, Brownstone N, et al. Update on sleep and pulmonary comorbidities in Psoriasis. Curr Dermatol

Rep. 2020;9:30–35. doi:10.1007/s13671-020-00293-3

URINE LEAKAGE, GERIATRIC FEMALE

CASE

80

Chief Complaint

“Urine leakage.”

History

of

Present Illness

A 66-year-old Caucasian woman presents with complaints of “leaking urine” and urgency of urination for 6 to 12 months. These symptoms have persisted to the point that she knows every public bathroom between her home and work and developed clear preferences for bathrooms in certain gas stations and fast food establishments. When she enters a new restaurant, she instinctively looks for the bathroom before being seated. Her concern is that when she is struck with the urge to urinate, she begins to leak, and she may not have more than a minute to get to the bathroom before she has an “accident.” She experiences urinary frequency 8 to 10 times daily, often with episodes of leaking and occasionally with nocturia. She rates the symptoms as 7 of 10 on a bother scale. Pelvic floor–strengthening exercises (Kegel exercises) and OTC herbal remedies have not helped. She experiences limited benefit from fluid restriction and is careful to urinate at home before driving. She wears pads to avoid the embarrassment of leaking and soiling her clothes in public.

Review

of

Systems

A ROS is positive for fatigue, nausea, abdominal pain, and hematuria. The patient reported headache and dizziness. The ROS is negative for fever, chills, glaucoma, vomiting, diarrhea, constipation, SOB, or chest pain.

Relevant History

Medical history is significant for Cesarean section (age 34) and a total hysterectomy (age 51) for menorrhagia. Her social history includes drinking 1 glass of wine per week since age 20, and she does not use recreational drugs. She admits to smoking a half pack of cigarettes per day for the last 10 years. Her two children are grown and have families of their own. She has suffered from intermittent depression since her husband died 8 years ago. Her family history is unremarkable.

Allergies

No known drug allergies; no known food allergies.

Medications L L L L L

Acetaminophen 500 mg PRN for arthralgia. Magnesium 250 mg nightly for sleep. Calcium 1,200 mg daily. Vitamin D 600 IU daily. Multivitamin daily.

320  Case 80  •  Urine Leakage, Geriatric Female

Physical Examination

Vitals: T 36.9°C (98.4°F), P 76, R 14, BP 122/78, HT 168 cm (66 in.), WT 86 kg (189.5 lbs), BMI 30.5. General: Well-developed, well-nourished, obese woman in no acute distress. Psychiatric: Judgment and insight intact; rate of thoughts normal and logical; pleasant, calm, and cooperative; patient appears to be happy and content without overt anxiety or depression. Abdomen: Active bowel sounds; abdomen soft, non-tender; no masses, no hernias, no suprapubic distension. Genital/Rectal: No bladder tenderness on palpation and no distention noted. Atrophic external genitalia without prolapse, pelvic masses, or gross lesions. Vagina mucosa exhibits thinning and pallor with loss of rugae. No urinary leakage on cough test with a full bladder. On the rectal exam, perineal sensation intact; sphincter tone intact; able to contract the anal sphincter. Neurologic: A&O×3; cranial nerves grossly intact; communication ability within normal limits; attention and concentration normal; sensation to light touch is intact; gait is within normal limits for age.

Clinical Discussion Questions 1. What is the differential diagnosis?

2. What is the most likely diagnosis? Why?

3. Demonstrate your understanding of the pathophysiology in regard to the most likely diagnosis.

4. Should tests/imaging studies be ordered? Which ones? Why? Think about tests/imaging beyond the

primary care setting as well.

Case 80  •  Urine Leakage, Geriatric Female 321 5. What are the next appropriate steps in management?

6. Review recent and credible research article(s) on this diagnosis. What are the helpful tools for making the

diagnosis and treatment options? List your reference(s).

7. What are the pertinent ICD-10 and CPT (E/M) codes for this visit? Provide a short rationale.

8. What is the appropriate patient education topic for this case?

9. If not managed appropriately, what is/are the medical/legal concern(s) that may arise?

10. Think about interprofessional collaboration for this case. Provide a list of specialties or other disciplines

and indicate what contribution these professionals might make to managing the patient.

322  Case 80  •  Urine Leakage, Geriatric Female

Bedside Manner Question

11. What would your communication style/approach be with this patient?

ANSWER KEY: URINE LEAKAGE, GERIATRIC FEMALE

CASE

80

1. Differential Diagnosis L

L

L

L

L

L

Stress incontinence is unlikely in this case because the urinary leakage is not provoked by sudden increases in abdominal pressure such as coughing, sneezing, laughing, or exertion. Urinary frequency and urgency are not characteristics of stress incontinence. Intrinsic sphincteric deficiency is unlikely in this case because ISD does not present with urinary frequency and urgency. ISD is a form of stress incontinence where neuromuscular damage from multiple pelvic surgeries results in loss of urethral muscular tone. Overactive bladder is a likely diagnosis given this patient’s history and presentation of frequent urgency of urination and leakage. Prolapsed bladder is an unlikely diagnosis based on the history, because there are no complaints of vaginal pressure, abdominal discomfort, or obstructive urinary symptoms. Physical examination verifies that there is no visible or palpable cystocele. Detrusor underactivity/overflow incontinence is also unlikely in this case because incomplete emptying of the bladder in overflow incontinence results in continuous urinary leakage instead of urgency. Interstitial cystitis/bladder pain syndrome is unlikely in this case due to the absence of pain. Although both conditions can include urinary frequency, urgency, and incontinence, interstitial cystitis classically presents with bladder or pelvic pain and dyspareunia.

2. Most Likely Diagnosis Overactive bladder. Urinary urgency with episodes of incontinence is characteristic of OAB. Accompanying symptoms often include urinary frequency, dysuria, and nocturia. In patients with OAB, symptoms are not exacerbated by sudden increases in abdominal pressure, such as while coughing or laughing. The absence of pain on urination, continuous incontinence, and cystocele further supports the diagnosis. 3. Pathophysiology Urinary continence is multisystemic. Musculoskeletal, neurologic, and psychologic factors work to simultaneously relax the detrusor muscles and contract the bladder neck and pelvic floor muscles. Among the central neurotransmitters, glutamate is involved in control of the lower urinary tract, serotonin contributes to storage of urine, dopamine D1 receptors are involved in suppressing bladder activity, and dopamine D2 receptors are thought to facilitate voiding. Sympathetic nerve fibers from spinal cord levels T11 to L2 cause contraction of smooth muscle at the bladder neck and proximal urethra, resulting in bladder filling.1 Sensory stretch receptors in the bladder wall trigger parasympathetic contraction of the detrusor and relaxation of the external urinary sphincter and pelvic floor muscles. The urge to urinate usually occurs when the bladder volume reaches 400 to 600 mL.2 In OAB, the detrusor is overactive and contracts regardless of the volume of fluid in the bladder. Normal cortical inhibition ceases to provide control over involuntary detrusor contractions, resulting in bladder emptying.1 OAB is most common in White women over 75 years of age with depression or insulin-dependent diabetes. Other risk factors include obesity, increased parity, use of hormonal replacement therapy, and arthritis. The prevalence of OAB is estimated at 38% in women over 65 years of age.1 When the patient experiences frequency, nocturia, and urgency-related incontinence, it is referred to as OAB-wet, in contrast to OAB-dry, which is without incontinence.1

4. Diagnostic Tests/Images Primary Care Setting L Urinalysis was done to rule out hematuria and infection. L Urine culture was done to rule out a subclinical infection that could be aggravating symptoms. L Urethral culture was done to screen for a STI. L POCUS of the bladder demonstrated no significant post-void residual. L Urodynamic studies were not indicated because there was no suspicion of underlying conditions or complications. L Diagnostic renal ultrasonography was not indicated because there was no suspicion of underlying renal pathology. Results All tests were within normal limits. The patient was asked to keep a bladder diary for 48 hours to track fluid intake, voiding patterns, and incontinence episodes. This diary would serve as a baseline to gauge future response to therapy. The OAB-q was administered to quantify the degree of bother and quality of life. Urology Cystoscopy was ordered to rule out a bladder tumor because she had a positive history of cigarette smoking. Results Normal cystoscopy. 5. Next Steps L

L

L

L

L

L

The patient is treated using the stepwise approach recommended in the AUA guidelines.2 This includes behavioral and pharmacologic management (as listed in the following). Patient is referred to a physical therapist for formal training on pelvic floor exercises, relaxation/distraction skills, and voiding schedules. With the finding of atrophic vaginitis, the patient is started on conjugated estrogens 0.625 mg/g vaginal cream as adjunctive therapy for the urgency incontinence, dosed as 0.5 g daily for 21 days and then off for 7 days. An option would be to dose the patient with 0.5 g twice a week for 21 days and then off for 7 days.3 Providers should always be cognizant of the potential adverse events associated with estrogen replacement therapy. Based on the severity of symptoms and the patient’s lack of confidence that additional behavioral training would be of benefit, the patient is started on oxybutynin 2.5 mg BID to reduce muscle spasm in the bladder, to be increased gradually to 5 mg BID or TID based on tolerance. The patient is referred to urology for a cystoscopy to rule out a bladder tumor. Because of her smoking history, this is appropriate step during the initial visit. The patient is referred for ophthalmology consultation due to potential ophthalmologic adverse effects from antimuscarinic agents.

6. Diagnostic Tools and Treatment Options The most psychometrically robust questionnaires used to aid diagnosis are the Overactive Bladder-­ Validated 8 and Bladder Control Self-Assessment Questionnaire; both achieve extremely high discriminatory values of validation and correlate strongly to gold standard means of testing. 4 Urodynamic studies, cystoscopy, and diagnostic renal and bladder ultrasonography should not be ordered as part of the initial workup of uncomplicated cases. They should be reserved for refractory and complex cases.2 Behavioral therapies and education should be offered to all patients with OAB as first-line therapy (Evidence Strength Grade B). Starting antimuscarinic agents at the same time may have some clinical benefit.2 Pelvic floor exercises, augmented by biofeedback, can reduce symptoms of frequency and urge incontinence by 54% to 85%.5

7. ICD-10 and CPT (E/M) Codes ICD-10 Code: N32.81 OAB; detrusor muscle hyperactivity ICD-10 Code: N39.41 Urge incontinence ICD-10 Code: N95.2 Postmenopausal atrophic vaginitis CPT (E/M) Code: Level 4—99214 (Established Patient) This code is designated for an established patient office visit for the evaluation and management of an established patient, which requires at least two of the following three key components: a detailed history; a detailed examination; and medical decision-making of moderate complexity. Typically, this requires 25 minutes face to face with a patient or family. 8. Patient Education Topics L

L L L L

Dietary factors that exacerbate symptoms, including caffeinated beverages, alcohol, and tobacco Behavioral modification to schedule urinary voiding Proper performance of pelvic floor exercises Relaxation and distraction techniques to effectively delay voiding Potential side effects of anticholinergic and antimuscarinic medications

9. Medical or Legal Concerns The most significant concerns arise from being unaware of an underlying pathology such as a neoplastic process or being inattentive to the contraindications to medications used to treat OAB. Antimuscarinics, such as oxybutynin, should not be used in patients with narrow-angle glaucoma unless approved by an ophthalmologist in advance. These agents should also be used with extreme caution in patients with impaired gastric emptying or who have a history of urinary retention. Mirabegron, a beta-3 adrenergic agonist, should be used with caution. Use of this medication can result in increased BP, urinary retention, and angioedema. Legal repercussions may result from using this drug in patients with severe or uncontrolled hypertension or bladder outlet obstruction. 10. Interprofessional Collaboration L L L

L

L

L L L

Outcome

Receptionist registers the patient. MA takes the chief complaint and vital signs and draws blood specimen. PCP takes a thorough history, completes physical examination, formulates the most likely diagnosis, orders appropriate tests, and refers the patient to specialists and physical therapy. PT takes thorough history, administers screening tools, and provides patient education, including pelvic floor muscle training. Ophthalmologist is consulted due to potential ophthalmologic adverse effects from antimuscarinic agents, such as exacerbation of angle closure glaucoma, mydriasis, photophobia, and blurred vision. Ultrasound technologist performs bladder ultrasound if POCUS is unavailable in the clinic. Radiologist reads results of bladder ultrasound if not performed in the clinic. Urologist performs cystoscopy and determines the results.

There was no abnormal finding with the cystoscopy. Oxybutynin was discontinued due to the patient’s poor toleration of side effects. She was then started on second-line therapy, extended-­ release oral tolterodine 2 mg daily. This an antimuscarinic agent that targets bladder receptors better

than oxybutynin, with fewer side effects and improved tolerance. Because antimuscarinic agents can cause adverse ocular effects, the patient was referred for ophthalmology consultation. Other treatment options considered were transdermal oxybutynin or fesoterodine, a bladder-selective muscarinic antagonist, 4 mg daily. If the patient fails this approach, third-line therapy would be discussed with the patient. This would include the possibility of treatment with sacral neuromodulation, PTNS, or an intradetrusor injection of onabotulinumtoxina. At the 1-month follow-up visit, the patient was pleased to report a 50% improvement in symptoms and almost full resolution of symptoms at 6 months. At that point, she was slowly weaned off of the medication and maintained on the habits she learned in behavioral therapy.

Insight

from the

PCP

The goals of treatment are not curative but to maximize control of symptoms and improve the quality of life, while minimizing side effects. Being a multifactorial complex, a thorough history and physical are necessary to determine the most appropriate treatment for each patient. Before initiating treatment, it is important to exclude an underlying infection or pathologic condition. In counseling a patient, it is critical to set realistic expectations for improvement. Patients should be told that symptom control may require trials of multiple different medications. A bladder diary is a valuable tool for a patient to track progress. Another simple method is to have a patient track the number of pads used each day. These simple tools reinforce a patient’s confidence and minimize some of the anxiety that contributes to the condition. Patients who experience anxiety about having an accident during sexual activity should be encouraged to try different positions until they find the one that is most comfortable. Patients who have learned Kegel exercises from watching videos online often do not realize that strengthening the pelvic floor muscles requires performing these exercises 40 to 80 times a day for 4 to 6 weeks to obtain full benefit. Sending a patient to a nurse specialist or physical therapist who is trained in teaching pelvic floor exercises is much more beneficial than patients realize at first. When prescribing anticholinergics and antimuscarinic agents, it is always wise to begin older patients on half of the normal starting dose and slowly titrate upward. If a patient finds the side effects of these medications intolerable, extended-release forms or transdermal patches should be tried. Clinicians should try to manage constipation and dry mouth before abandoning these medications. If symptoms are mild to moderate and the patient improves sufficiently with pelvic-floor-muscle exercises, it may be possible to discontinue the medication. An important component of management is to determine if and when to discontinue the medication once signs and symptoms resolve. It is important to not take these medications for granted and make the habit of following all patients carefully at each visit for adverse events.

References

1. Tarnay CM, Medendorp AR, Cohen SA, Mwesigwa PJ. Urinary incontinence & pelvic floor disorders. In: De 2. 3. 4. 5.

Cherney AH, Nathan L, Laufer N, Roman AS, eds. Current Diagnosis & Treatment: Obstetrics & Gynecology. 2019 12th ed. McGraw-Hill. Gormley EA, Lightner DJ, Burgio KL, et al. Diagnosis and treatment of overactive bladder (non-neurogenic) in adults: AUA/SUFU guideline. 2019. https://www.auanet.org/Documents/Guidelines/PDF/Overactive-Bladder.pdf Wyeth Pharmaceuticals, LLC. Premarin vaginal conjugated estrogen cream package insert. 2018. http://labeling. pfizer.com/showlabeling.aspx?id=132 Malde S, Kelly S, Saad S, Sahai A. Case-finding tools for the diagnosis of OAB in women: a narrative review. Neurourol Urodyn. 2019;10:1–12. doi:10.1002/nau.24171 Diokno AC, Sampselle CM, Herzog AR, et al. Prevention of urinary incontinence by behavioral modification program: a randomized, controlled trial among older women in the community. J Urol. 2004;171(3):1165–1171. doi:10.1097/01.ju.0000111503.73803.c2

COUGHING AND GAGGING, PEDIATRIC MALE

CASE

81

Chief CoMPlaint

“Coughing and gagging.”

historY

Present illness

of

A mother brings her 3-month-old previously healthy infant to his PCP for evaluation of an apparent apnea episode that occurred that morning. He had developed a cough and nasal congestion about 5 days prior to this visit, and on the morning of the visit had a prolonged coughing spell followed by a gagging episode. His mother heard the cough and gagging over the baby monitor and when the gagging began, she ran to his room to check on him. By the time she picked him up from his crib, he did not seem to be breathing and his face and lips were blue. She immediately brought the baby to her husband, by which time the child had begun to breathe, and his color quickly returned to normal. The episodes of cough have become more frequent and prolonged since the onset of the illness, but he has not had a fever. His mother reported she had been fighting off a “nagging cough” for about 3 weeks. The infant’s 2-year-old sister and 4-year-old brother had no respiratory symptoms. None of the children attended day care.

reVieW

of

sYsteMs

The infant’s ROS is positive for decreased feeding and a few episodes of vomiting following coughing spells. The ROS is negative for fever, conjunctivitis, diarrhea, rash, or seizures.

releVant historY

The infant was born by vaginal delivery at 39 weeks’ gestation following an uncomplicated pregnancy. His mother received regular prenatal care and had no history of untreated cervical infection. His birth weight was 6 lbs 14 oz, and he was discharged home at 48 hours of age after an uneventful stay in the newborn nursery. He had an appropriate weight gain and normal development and physical examination at his 2-week, 1-month, and 2-month well baby visits, and his routine newborn screening was normal. He had received all recommended immunizations, including those routinely administered at 2 months.

allergies

No known drug allergies; no known food allergies.

MeDiCations None.

PhYsiCal eXaMination

Vitals: T 37.1°C (98.8°F); P 140; R 40; BP 80/40; SpO2 96%; HT 61 cm (24 in.), 43rd percentile; WT 5.5 kg (12 lbs), 10th percentile; BMI 14.6. General: Alert and active, in no distress. Skin, Hair, and Nails: Acyanotic, no rash or lesions. No abnormal findings with hair or nails.

324  Case 81  •  Coughing and Gagging, Pediatric Male

Head: Anterior fontanelle soft and flat. Eyes: No discharge or conjunctival injection. ENT/Mouth: Nares congested, tympanic membranes clear, oral mucosa moist and without lesions. Lungs: Clear to auscultation bilaterally, breath sounds equal bilaterally; no grunting, retractions, or nasal flaring. Heart: RRR, S1 and S2 normal intensity, no murmur, pulses 2+ in all extremities. Abdomen: Soft, no masses or hepatosplenomegaly. Neurologic: Alert, moving all extremities equally, normal muscle bulk and tone.

Clinical Discussion Questions 1. What is the differential diagnosis?

2. What is the most likely diagnosis? Why?

3. Demonstrate your understanding about the pathophysiology in regard to the most likely diagnosis.

4. Should tests/imaging studies be ordered? Which ones? Why? Think about tests/imaging beyond the

primary care setting as well.

Case 81  •  Coughing and Gagging, Pediatric Male  325 5. What are the next appropriate steps in management?

6. What are the preventive plans and treatment approaches for the diagnosis? Provide references for your

response.

7. What are the pertinent ICD-10 and CPT (E/M) codes for this visit? Provide a short rationale.

8. What are appropriate parent education topics for this case?

9. If not managed appropriately, what is/are the medical/legal concern(s) that may arise?

326  Case 81  •  Coughing and Gagging, Pediatric Male 10. Think about interprofessional collaboration for this case. Provide a list of specialties or other disciplines

and indicate what contribution these professionals might make to managing the patient.

Bedside Manner Questions

11. What would your communication style/approach be with this patient’s mother?

12. If the patient’s mother is distressed by the diagnosis, what might offer support?

ANSWER KEY: COUGHING AND GAGGING, PEDIATRIC MALE

CASE

81

1. Differential Diagnosis L

L

L

L

L

Viral upper respiratory illness is possible. The infant had an episode of apnea with cyanosis and is afebrile. An infant with a viral upper respiratory infection is not likely to experience a BRUE such as apnea and is more likely to have a fever. Sepsis cannot be ruled out without appropriate cultures. Infants with sepsis are likely to appear ill on physical examination and often have a history of fever. Bacterial sepsis may cause symptoms such as apnea and cyanosis. When sepsis is suspected, appropriate broad-spectrum intravenous antibiotics must be started after obtaining cultures. Bacterial pneumonia is a less likely diagnosis. An infant with pneumonia is likely to have a history of fever and physical examination findings consistent with pneumonia (focal decreased breath sounds, crackles, tachypnea). Symptoms of pneumonia due to Chlamydia trachomatis are similar to those of the most likely diagnosis (listed next), but the absence of untreated cervical infection in his mother and the fact that she had regular prenatal care make this diagnosis less likely.1 Pertussis is a likely diagnosis. The infant has no abnormal lung findings. His cough, nasal congestion, apnea, and immunization history (up to date but had only 2 months’ immunization) raise suspicion for pertussis. In addition, the mother’s nagging cough for about 3 weeks increases further suspicion. Performing a CBC will be helpful to further assess the diagnosis. Bronchiolitis (respiratory syncytial virus) may cause apnea, and infants with bronchiolitis are likely to have findings on lung exam such as diffuse crackles, wheezing, tachypnea, and retractions and are also likely to have a fever. Infants with bronchiolitis often have a history of contact with a young child with bronchiolitis or an upper respiratory tract infection.

2. Most Likely Diagnosis Pertussis is the most likely diagnosis, although this cannot be confirmed without appropriate diagnostic studies. The infant has cough, nasal congestions, and apnea. His mother has been coughing about 3 weeks; thus, it is possible she passed pertussis to the infant. Even though he is up to date with immunizations, he is not completely protected from pertussis until the series is complete. (DTaP vaccine is administered at the ages of 2 months, 4 months, and 6 months, and then a booster around 15 months, and at the age of 4.) The differential diagnosis for an infant with an episode of apnea and cyanosis (a BRUE) is extensive and should include seizures and metabolic disorders. The rapid resolution of this patient’s apnea and its association with an episode of coughing and gagging led to a focus on respiratory infection as the cause. Therefore, seizures and metabolic disorders were excluded on the differential. 3. Pathophysiology Pertussis (also known as whooping cough) is a respiratory infection caused by the bacterium Bordetella pertussis. It is a highly contagious illness, with the potential for significant morbidity and mortality in infants and young children. Infection is spread by respiratory droplets, and the incubation period is usually between 1 week and 3 weeks. After the bacterium is inhaled, it adheres to ciliated respiratory epithelial cells. Following adherence, local tissue damage is brought about by the production of biologically active substances and virulence factors, leading to cough and systemic manifestations (lymphocytosis).2

4. Diagnostic Tests/Images Primary Care Setting A CBC was done as a stat order in the primary care office. Results Total white blood cell count of 21,000/mm3 with 72% lymphocytes. Hospital Setting L CBC to provide insight regarding the presence and type of infection L Chemistry profile to evaluate for dehydration or an electrolyte abnormality, which might have contributed to the patient’s clinical presentation L Urinalysis to provide additional information regarding hydration status (specific gravity) and the presence of indicators of infection (white blood cells, nitrite, leukocyte esterase) L Chest radiography to evaluate for signs of lung or heart disease, given the presence of respiratory symptoms and history of cyanosis and apnea L Nasopharyngeal aspirate for PCR for pertussis and pertussis culture to confirm the suspected diagnosis of pertussis L Nasopharyngeal swab for influenza A and B to rule out influenza infection L Nasopharyngeal aspirate to rule out respiratory syncytial virus infection L Blood and urine cultures to rule out bacterial infection, given the infant’s high white blood cell count and history of apnea and cyanosis Results CBC results from the hospital were consistent with the CBC results from the primary care clinic (a high percentage of lymphocytes along with a high overall number of white blood cells). L PCR for pertussis and pertussis culture were both positive. L Other results were unremarkable. L

5. Next Steps The urgency of the likely diagnosis was explained to the patient’s mother and an immediate referral to a hospital made. The family agreed with admission to the local hospital, given that it has a pediatric ICU, it is close by, and a transfer could be arranged expeditiously. 6. Preventive Plans and Treatment Approach L

L

L

Postexposure prophylaxis is recommended for all household contacts of the patient regardless of immunization status. This patient’s mother, father, sister, and brother were all candidates for postexposure antibiotic prophylaxis. This patient’s mother had a prolonged cough and was likely the source of infection. Prophylaxis is also recommended for other close contacts: persons with close face-to-face exposure to a patient (within 3 feet); persons with direct contact with a patient’s respiratory, oral, or nasal secretions; or persons who were in a confined space with a patient for an hour or more.3, 4 Antibiotic prophylaxis for household and other close contacts of a patient with pertussis should be initiated within 21 days of onset of cough in the index patient.3, 4 Recommendations for antibiotic agent and dosing regimen for prophylaxis are the same as those for treatment of pertussis. At the time of presentation, this patient had been ill for only 5 days, and antibiotic prophylaxis of contacts was therefore indicated to control the spread of infection. Macrolide antibiotics are the preferred treatment for pertussis. The specific recommended macrolide antibiotic varies by patient age. Clarithromycin is not recommended in infants younger than 1 month, and azithromycin is preferred over erythromycin in this age group. Both azithromycin and erythromycin are associated with an increased risk of pyloric stenosis (especially in infants younger than 2 weeks) while the risk with clarithromycin is not known. Any of the macrolide antibiotics may be used in infants older than 1 month.3

L

The PCP has a critical role to prevent the spread of pertussis infection by ensuring that patients of all ages receive the pertussis vaccine at the recommended intervals and through the identification of individuals who are candidates for postexposure prophylaxis. Additionally, PCPs can prevent the spread of pertussis to young infants by ensuring that pregnant women are immunized against pertussis with the Tdap vaccine during each pregnancy and by recommending and supporting breastfeeding.5 It is also important that other household contacts be protected by keeping up with pertussis vaccine as recommended by age.

7. ICD-10 and CPT (E/M) Codes ICD-10 Code: R68.13 Apparent life-threatening event in infant is the most appropriate ICD-10 code for the office visit as it applies to patients ages 0 through 17 years with a life-threatening BRUE (manifested as apnea and cyanosis in this patient). Although the diagnosis of pertussis was suspected, this could not be confirmed at the office visit, and therefore the code for pertussis should not be used for the visit. CPT (E/M) Code: Level 5—99215 (Established Patient) This is an established patient with a high complexity of medical decision-making (a new problem with additional workup planned and an illness with a threat to life). Documentation in support of this CPT code should include a comprehensive history and/or physical examination, both of which are critical in a case such as this. 8. Patient Education Topics L L L L

Importance of immunizations for the patient and family members (including parents) Basic facts about pertussis Identification of close contacts who would benefit from postexposure prophylaxis Expected duration of cough

9. Medical or Legal Concerns Even with appropriate management in a hospital, pertussis infection in young infants (even those who are otherwise healthy) is associated with significant morbidity and mortality. Complications include cyanosis, apnea, and seizures. It is the standard of care to hospitalize young infants with pertussis for close monitoring, specific treatment, and supportive care. Failure to admit such patients may lead to increased morbidity and mortality and significant anxiety for the family in the event that a complication or deterioration in condition is experienced at home. Given the potential for rapid deterioration, consideration should be given to transferring young infants with pertussis to a hospital with a pediatric ICU. The need for transfer should take into consideration the preferences of the family, distance to a tertiary care facility, and comfort level of a PCP. 10. Interprofessional Collaboration L L

L

L

L

Receptionist triages call from parent and identifies need for urgent appointment. MA gathers vital signs, height, and weight; obtains brief history; communicates concerns to the PCP; and calls for admission. PCP takes focused history, performs physical examination, develops differential diagnosis and plan of care, coordinates hospital admission, provides admitting nurse with admission orders, and coordinates care during hospitalization. Hospital nurse institutes appropriate infection control precautions; monitors patient’s respiratory, nutrition, and hydration status; and assesses the ability of caretakers to provide care following discharge. Respiratory therapist provides supplemental oxygen, assists in cardiorespiratory monitoring, and looks for signs of deteriorating respiratory status.

L

L

Pre-hospital care providers (paramedic/EMT) may be needed to transfer the patient with caregiver to a higher level of care. Public health department tracks contacts of confirmed pertussis cases.

Outcome

The infant was admitted to the pediatrics unit of a local hospital with a suspected diagnosis of pertussis. Extensive testing confirmed the suspected diagnosis and ruled out alternative diagnoses. Specific treatment for pertussis, including continuous cardiorespiratory monitoring, was initiated as soon as pertussis was suspected. After initial diagnostic testing, the patient was started on a broad-spectrum intravenous antibiotic (ceftriaxone) pending results of blood and urine cultures as well as oral erythromycin for the suspected diagnosis of pertussis. A chest radiography showed a normal cardiac silhouette and no evidence of pulmonary disease. Testing for respiratory syncytial virus and influenza was negative. PCR for pertussis and pertussis culture were both positive, and the case was reported to the local health department. Ceftriaxone was discontinued 72 hours after admission, at which time blood and urine cultures showed no growth. The infant remained hospitalized for 4 days, and oral erythromycin was continued for 14 days. He had no further episodes of apnea, bradycardia, or cyanosis during the hospitalization and was discharged home when the severity of the coughing episodes decreased and his oral intake returned to normal.

Insight

from the

PCP

Symptoms of pertussis in young infants overlap those of other respiratory infections, and the PCP must have a high index of suspicion to make the diagnosis and provide appropriate treatment. Young infants with pertussis should be admitted to the hospital even if well appearing.6 A PCP should not delay treatment for pertussis while awaiting results of confirmatory testing. This is especially important for younger infants, who are at greater risk for complications. Close follow-up of patients after discharge is important, and a PCP has a significant responsibility to ensure that a child’s caretakers are competent to provide care and have the necessary resources to seek further medical care if needed. Given that this infant most likely acquired the infection from his mother, whose persistent cough was a symptom of pertussis infection acquired as a result of a natural decline in immunity years after vaccination, it was important to approach the discussion of pertussis prevention and transmission in a sensitive manner. The mother expressed guilt that she was the likely source of his infection. As the PCP who had a long-standing relationship with the family, I was able to reassure her that the phenomenon of declining pertussis immunity is only now being appreciated and that no one in the medical community would consider her to be at fault. Following this reassurance, she was receptive to the recommendation that she receive a booster dose of Tdap.

References

1. Centers for Disease Control and Prevention. Chlamydial infections. June 4, 2015. https://www.cdc.gov/std/tg2015/

chlamydia.htm#neonates.

2. Mattoo S, Cherry JD. Molecular pathogenesis, epidemiology, and clinical manifestations of respiratory infections 3. 4. 5. 6.

due to Bordetella pertussis and other Bordetella subspecies. Clin Microbiol Rev. 2005;18(2):326–382. doi:10.1128/ CMR.18.2.326-382.2005 Altunaiji SM, Kukuruzovic RH, Curtis NC, Massie J. Antibiotics for whooping cough (pertussis). Cochrane Database of Systematic Reviews 2007, Issue 3. Art. No.: CD004404. doi:10.1002/14651858.CD004404.pub3 Centers for Disease Control and Prevention. Pertussis (whooping cough): postexposure antimicrobial prophylaxis. August 7, 2017. https://www.cdc.gov/pertussis/outbreaks/pep.html. Baxter R, Bartlett J, Fireman B, Lewis E, Klein NP. Effectiveness of vaccination during pregnancy to prevent infant pertussis. Pediatrics. 2017;139(5):1–8. doi:10.1542/peds.2016-4091 Cherry JD, Wendorf K, Bregman B, et al. An observational study of severe pertussis in 100 infants ≤ 120 days of age. Pediatr Infect Dis J. 2018;37(3):202–205. doi:10.1097/INF.0000000000001710

EYE INJURY, ADULT MALE

CASE

82

Chief CoMPlaint “Eye injury.”

historY

of

Present illness

A 32-year-old man presents to the clinic complaining of getting something in his left eye that he feels is still there. Just prior to presenting, he was stretching a bungee cord across the top of his kayak to secure it to the roof racks of his car, when the bungee cord snapped in half, ricocheting the rubber cord with the metal S hook toward his face. He reports something hitting his left eye as he could not get out of the way fast enough, has tried looking in a mirror to see if he could see what was in his eye, and has tried to flush out whatever might be stuck in his eye using all 16 oz of his bottled water. He reports the left eye pain as 10/10, stinging, sharp, throbbing, and very sensitive to light. He complains also of very blurry vision in the left eye, not being able to make much out, including shapes. He denies double vision, denies any eye injury in the past, does not wear corrective lenses of any kind, has not had eye surgery, and cannot recall his last tetanus immunization. He has not taken anything to alleviate the pain other than to hold an ice pack over his left eye. He denies any other injury to his face, mouth, or teeth. He is here with his wife.

reVieW

of

sYsteMs

The patient’s ROS is positive for foreign body sensation in the left eye with severe pain, photosensitivity, excessive tearing, redness and extreme blurred vision in left eye, and throbbing headache behind left eye. His ROS is negative for double vision, spots, specks, flashing lights, corrective lenses, facial lacerations, abrasions, or head injury.

releVant historY

The patient has no contributory medical or surgical history. He is happily married and father of two children. He is a social drinker. He denies tobacco or drug use. His family history is noncontributory.

allergies

Sulfonamides create rash and intense pruritis; no known food allergies.

MeDiCations

Multivitamin PO daily.

PhYsiCal eXaMination

Vitals: T 37.2°C (99.0°F), P 114, R 16, BP 148/94, HT 185.4 cm (73 in.), WT 86.2 kg (190 lbs), BMI 25.1. General: Frightened male of stated age, worried about left eye vision. A&O; answers all questions appropriately. Skin, Hair, and Nails: Warm throughout; no lesions, lacerations, ecchymosis. No abnormal findings with hair or nails. Head: Atraumatic, normocephalic, non-tender.

328  Case 82  •  Eye Injury, Adult Male

Eyes: No periorbital lacerations, lesions, ecchymosis, step offs, or tenderness bilaterally. Left eye: Swollen and tender upper and lower lids, good position, unable to close fully. Injected sclera and hyperemic conjunctiva, subconjunctival hemorrhage at the medial canthus extending to the iris, excessive tearing, tear-drop-shaped pupil not reactive to light or accommodation. Foreign body not visible to the naked eye. No fluid visualized as being emitted from the anterior or lateral portions of the globe. EOMs limited in movement. Difficult to assess retinal structures funduscopically due to patient’s pain level and difficulty with visualization. Right eye: No lid edema, good position and closure, injected sclera, pink conjunctive. Pupil 4 mm, round, reactive to light and accommodation. EOMIs intact, no disc edema or swelling, venous pulsations, or AV nicking. ENT/Mouth: Hearing acuity intact bilaterally to whispered voice. No deformities, lacerations, lesions, or tenderness externally, bilaterally. TMs pearly gray, positive cone of light; no discharge or bleeding bilaterally. No maxillary or frontal sinus tenderness. Nares patent, septum midline, dark pink mucosa with clear nasal drainage L>R. Teeth and gums in good repair; no bleeding. Oral mucosa and tongue without lesions or lacerations, oropharynx patent, tonsils 1+ and symmetric, uvula midline. Neck: FROM, supple, no adenopathy. Neurologic: Cranial nerve II grossly intact OD; unable to assess cranial nerve II OS due to extreme blurred vision. Cranial nerves III to XII grossly intact.

Clinical Discussion Questions 1. What is the differential diagnosis?

2. What is the most likely diagnosis? Why?

3. Demonstrate your understanding about the pathophysiology in regard to the most likely diagnosis.

Case 82  •  Eye Injury, Adult Male 329 4. Should tests/imaging studies be ordered? Which ones? Why? Think about tests/imaging beyond the

primary care setting as well.

5. What are the next appropriate steps in management?

6. What are the treatment approaches, prevalence, and physical exam techniques of this diagnosis? Provide

references for your responses.

7. What are the pertinent ICD-10 and CPT (E/M) codes for this visit? Provide a short rationale.

8. What are the appropriate patient education topics for this case?

330  Case 82  •  Eye Injury, Adult Male 9. If not managed appropriately, what is/are the medical/legal concern(s) that may arise?

10. Think about interprofessional collaboration for this case. Provide a list of specialties or other disciplines

and indicate what contribution these professionals might make to managing the patient.

Bedside Manner Questions

11. What would your communication style/approach be with this patient and his wife?

12. If a patient and his wife are distressed by the diagnosis, what might offer support?

ANSWER KEY: EYE INJURY, ADULT MALE

CASE

82

1. Differential Diagnosis L

L

L

L

L

L

Corneal abrasion or laceration may be possible given the patient’s injury. He complains of feeling as though a foreign body has been retained in his eye when there may not be. An abrasion may be present after inspection with fluorescein stain; however, globe rupture due to corneal laceration must first be ruled out as fluorescein stain should not be applied to the eye if this is the case. Corneal foreign body is also likely. The patient complains of the painful sensation of a foreign body in his eye. Before inspection of the cornea with fluorescein stain, globe rupture must first be ruled out as fluorescein stain should not be applied to the eye if this is the case. Also, foreign body was not visible to the naked eye. Globe rupture is suspected because of the patient’s misshapen left pupil. Caused by a corneal laceration, aqueous humor may be visualized as seeping from the globe. A misshapen pupil confirms the diagnosis of globe rupture, and orbital trauma must also be ruled out. Hyphema is blood pooling, or red blood cell layering, in the anterior chamber secondary to blunt force trauma to the globe. It is best seen with the patient in a sitting position, not laying supine, as gravity pulls blood inferiorly. Blood in the anterior chamber would more than likely not be seen if the patient were laying supine. Glaucoma as a diagnosis would be secondary to hyphema, as blood pooling in the anterior chamber would cause an elevated intraocular pressure, hence a secondary glaucoma. Tonometry would need to be performed but not if globe trauma is to be ruled out first. Orbital fracture from high-velocity blunt force trauma would heighten the concern for an orbital rim or blow out fracture. Intact extraocular muscle movements in addition to a noncontrast CT scan of the orbits help rule this out.

2. Most Likely Diagnosis Globe rupture. The patient has a probable a full-thickness injury penetrates completely through the cornea, causing a ruptured globe from the high-velocity, blunt force trauma. The misshapen pupil and the patient’s inability to perceive anything other than light with his left eye upon examination made this the most likely diagnosis. 3. Pathophysiology When a sharp object or small foreign body hits the eye at a high velocity, it may perforate the globe directly. Small objects may be retained in the globe without the patient’s knowledge. When a blunt object makes impact with the surrounding orbit, the globe becomes compressed, causing the intraocular pressure to immediately elevate to the point where the sclera tears. The rupture site is typically where the sclera is the thinnest and weakest, just posterior to rectus muscle insertion points.1 4. Diagnostic Tests/Images L

L

Snellen eye chart for visual acuity i. 20/30 OD ii. Light perception only OS CT scan of orbits upon arrival to ED

5. Next Steps Emergent consultation with an ophthalmologist with operating room privileges at the closest hospital center is needed. Patient will be transferred via EMS. The patient needs to be kept sitting upright. He is not to rub his left eye or place anything in or near it. A rigid shield will be placed over the patient’s left eye.2 He is to be made NPO, given a tetanus immunization, and placed in the ambulance for rapid transfer to the emergency room. 6. Treatment Approach, Prevalence, Physical Exam Techniques L

L

L

L

A globe rupture is an ophthalmologic emergency. Damage to the posterior aspect of the eye by penetrating or blunt force trauma has a high propensity to result in permanent vision loss. It is essential to provide a rapid assessment to make sure the patient is stable, promptly recognizing disruption of the outer layers of the eye, and to obtain emergent ophthalmologic intervention.1–3 There are more than 2 million cases of eye trauma in the United States annually, with over 40,000 of those cases resulting in varying degrees of permanent visual impairment. The prognosis depends on the mechanism of injury, extent of injury, and time from injury to appropriate surgical intervention.1 Eye injuries associated with the quick release of a bungee cord are becoming increasingly common.3 PCPs should always check visual acuity first before performing the physical examination. Try to assess which structures are injured. If globe rupture is suspected, do not apply a pressure patch, do not apply fluorescein stain to the eye, do not attempt tonometry, and do not press on the globe any more than necessary. Doing any of these increases the risk of extruding the inner contents of the eye outward.1–3

7. ICD-10 and CPT (E/M) Codes ICD-10 Code: S05-32XA Ocular laceration without prolapse or loss of intraocular tissue, left eye, initial encounter. This is based on the initial presentation of the patient, his visual acuity examination, and his left pupil findings on examination. CPT (E/M) Code: Level 5—99215 (Established Patient) Comprehensive assessment. This includes a comprehensive history and detailed examination. Medical decision-making is of high complexity, with about 40 minutes spent face to face with the patient. 8. Patient Education Topics L

L L

The need to be followed closely by an ophthalmologist for periodic dilated funduscopic examinations. Because of the trauma, there is an increased risk for developing angle-closure or chronic glaucoma, retinal tears, and cataracts.1,3 Using protective eyewear to decrease the risk of another eye injury. Wearing appropriate UV-protective sunglasses when outdoors.

9. Medical or Legal Concerns Had the patient been laying supine the entire visit, the tear-drop-shaped pupil might not have been evident on examination and the diagnosis may have been missed. Gravity pulls on the pupil and the contents of the injured globe, changing the pupil’s shape. The pupil may have appeared round if he were to be examined laying supine. Delaying time to get this patient to the OR would have had a much worse outcome. Missed/delayed diagnosis may lead to permanent loss of vision and resultant liability.

10. Interprofessional Collaboration L

L

L L

L L L L

L L L L

Receptionist registers the patient, notifies the appropriate staff that the patient has an emergency, and calls 911/EMS. The receptionist also calls a friend of the patient’s wife (after obtaining their consent) to drive her to the hospital. MA/technician provides rapid patient intake and vitals, provides the rigid eye shield, and administers the tetanus immunization to the patient prior to hospital transfer. Office manager comforts the patient’s wife in a private room until PCP could be with her. PCP exams the patient and formulates the likely diagnoses, explains the need for the family to take the patient to the ED, arranges transfer to the ED and consults with the on-call physician, provides reassurance to the family, and interfaces with the EMS staff to transfer the patient to the ED. EMS staff transfer patient to ED. ED triage nurse triages patient. ED nurse takes history and vitals. ED physician confirms diagnosis, admits patient, and arranges emergency consult with ophthalmologist. Ophthalmologist evaluates and treats with surgical repair. Anesthesiologist performs general anesthesia during the surgery. Surgical technician assists the surgical repair. OR nurse facilitates the surgery.

Outcome

The patient received expert ophthalmology evaluation and intervention as promptly as possible. Once in the ED, he had an IV placed for the antibiotic cefazolin 1 g IV and maintenance fluids and received hydromorphone 2 mg IV q1h PRN for pain relief and promethazine 25 mg IV PRN for nausea. A non-contrast CT scan of the orbits was ordered, and the OR prepared to receive the patient as soon as the ophthalmologist arrived. Although the vision in his left eye is far from perfect, he is able to engage in most of his typical activities, after an approximate 6-month recovery.

Insight

from the

PCP

When I asked the patient why he decided to present to the primary care office rather than to the ED with an injury as catastrophic as this, he replied that he “did not want to wait the typical four or five hours to be seen.” This was the perfect opportunity to give the patient some quick education while waiting for EMS to arrive. During the 3-minute window of time I had before EMS arrived to take the patient to the hospital, I calmly spoke to the patient about emergency medicine triage and the nature of his injury and told him that he would have been seen immediately. I was very sympathetic to his concerns and made sure his wife knew everything that was happening with him, including the need for emergent transfer to a facility that could best deal with his injury. She was in a bit of shock as the news was hard to take, so I asked our receptionist to call a friend of the patient’s wife to pick her up and take her to the hospital. EMS protocol would not allow the patient’s wife to accompany him to the hospital, so I had her wait with me in an examination room away from others in the waiting room. The patient’s wife was understandably distraught; she had no business driving a car at that time. Her friend arrived just as the ambulance was leaving. Once an eye injury has been sustained, the clock is ticking to save the patient’s vision. The time it takes for ophthalmologic intervention has severe consequences if not taken as an emergency, which was certainly the case for this patient. I was honored to have been able to rapidly assess his injury, ensure he was stable, provide the intervention/medical management that I could from a primary care standpoint, place the appropriate phone calls to a specialist and the ED charge nurse, and have the patient immediately transferred.

References

1. Acerra JR, Gilman SL, Golden DJ. Globe rupture. Medscape. September 3, 2019. https://emedicine.medscape.com/

article/798223-clinical.

2. Cydulka RK, Cline DM. Tintinalli’s Emergency Medicine Manual. 2017, 8th ed. McGraw-Hill; 820–822. ISBN-13:

978-0071837026

3. Kent C. Managing serious cases of ocular trauma. Review of Ophthalmology. November 18, 2008. https://www.

reviewofophthalmology.com/article/managing-serious-cases-of-ocular-trauma

PAIN IN GREAT TOE, ADULT FEMALE

CASE

83

chiEf comPlaint

“Pain in great toe.”

history

of

PrEsEnt illnEss

A 54-year-old woman presents with complaints of right great toe pain that has been bothering her for the past 4 to 6 weeks. She states the pain has been a constant 5/10 and is somewhat relieved with ibuprofen and changing positions frequently. She cannot recall what caused this pain, denying any trauma to the toe or foot. A few weeks ago, the pain became so annoying she sought medical attention in an ED. She reports she underwent x-rays of her right foot and great toe, both of which were negative for fracture. She states she was told that a fracture may not show up on film right away, so she would need to follow up with an orthopedic specialist within 2 weeks to obtain new x-rays. The ED provider buddy-taped her right great toe to the adjacent toe, placed her in a post-op shoe, and asked her to limit weight bearing and to continue to take ibuprofen for pain. She denies arthritis, gout, blood clots, and previous injury/trauma to her right foot or great toe. She denies radiation of the pain into her calf but does admit there are times when she feels her right great toe goes numb. The pain returns when she stands up from a sitting position, stands for prolonged periods of time, climbs stairs, sits on a hard chair, or drives a car. She admits to being in a car accident 3 months ago, where she was the driver who was rear-ended at a stop light. She refused to be taken to the ED at that time, complaining only of minimal neck pain, which has resolved. She states she has experienced low back spasms that come and go since then and denies any diagnostic studies of her lower spine.

rEViEW

of

systEms

The patient’s ROS is positive for pain in the right great toe and lower back spasms. Her ROS is negative for fever, chills, diaphoresis, insomnia, urinary problems, bowel movement changes, or recent unintentional weight loss or gain. The patient denies rashes, lumps, lesions, itchiness, dryness, color changes, or change in nails or hair. She also denies swollen glands, pain or stiffness, leg cramps, varicose veins, or history of DVT.

rElEVant history

The patient has a history of hypertension, hypercholesterolemia, hypertriglyceridemia, and fibrocystic breast disease. She has a 20-pack-year tobacco history and quit 8 years ago. She drinks two glasses of red wine on the weekends and denies illicit or recreational drug use. Her diet is rich in seafood, especially shrimp and fish. She does not eat organ meats or consume caffeine. Her family history is significant for hypertension, type 2 diabetes, hypercholesterolemia, angina, and glaucoma.

allErgiEs

No known drug allergies; no known food allergies.

mEDications l l l l

Amlodipine 10 mg PO QD Chlorthalidone 25 mg PO QD Rosuvastatin 40 mg PO QD Ibuprofen 400 mg PO q6h PRN for right great toe pain.

332  Case 83  •  Pain in Great Toe, Adult Female

Physical Examination Vitals: T 37°C (98.6°F), P 80, R 14, BP 154/92, HT 168 cm (66 in.), WT 66.2 kg (146 lbs), BMI 23.6. General: Pleasant female of stated age in no acute distress, sitting in chair adjacent to exam table. A&O. Skin, Hair, and Nails: No erythema, rashes, lesions, or masses. No abnormal findings with hair or nails. Neck: FROM, supple without tenderness. Peripheral Vascular: Warm throughout, all pulses 2+ symmetric bilaterally, capillary refill < 2 seconds throughout. Musculoskeletal: No curvatures of the spine, slight pelvic tilt with left iliac crest slightly higher than right. Tender L3 to L5 spinous processes without step-offs. Painful right-sided paravertebral muscle spasms initiated with deep palpation of L4 to L5 and right sciatic nerve within the right buttock with patient positioned in fetal position. Decreased ROM L-spine, right lateral bending. No deformities at BLEs; tender right first metatarsophalangeal joint without increased warmth, swelling, or erythema. FROM at left lower extremity. FROM at right lower extremity except decreased ROM at right great toe in flexion and extension. Neurologic: Cranial nerves II to XII grossly intact; speech appropriate; antalgic gait favoring right foot. L1, L2, L3, S1, S2: sharp/dull discrimination and vibratory sense intact at BLEs L4, L5: sharp/ dull discrimination and vibratory sense intact at left lower extremity. Unable to discriminate dull sensation, at lateral aspect of right calf, extending distally to right mid-foot and great toe.

Clinical Discussion Questions 1. What is the differential diagnosis?

2. What is the most likely diagnosis? Why?

3. Demonstrate your understanding about the pathophysiology in regard to the most likely diagnosis.

Case 83  •  Pain in Great Toe, Adult Female 333 4. Should tests/imaging studies be ordered? Which ones? Why? Think about tests/imaging beyond the

primary care setting as well.

5. What are the next appropriate steps in management?

6. What is the treatment approach for this diagnosis? Provide your references(s).

7. What are the pertinent ICD-10 and CPT (E/M) codes for this visit? Provide a short rationale.

8. What is the appropriate patient education for this case?

9. If not managed appropriately, what is/are the medical/legal concern(s) that may arise?

334  Case 83  •  Pain in Great Toe, Adult Female 10. Think about interprofessional collaboration for this case. Provide a list of specialties or other disciplines

and indicate what contribution these professionals might make to managing the patient.

Bedside Manner Question

11. What would your communication style/approach be with this patient?

ANSWER KEY: PAIN IN GREAT TOE, ADULT FEMALE

CASE

83

1. Differential Diagnosis l

l

l

l

l

Gout is a disorder of purine metabolism that commonly involves a single joint such as the great toe, with swelling, warmth, erythema, and tenderness.1 Uric acid or urate is an end-stage byproduct of purine metabolism and accumulates in blood and tissues. When tissues become supersaturated with urate, the urate salts precipitate and form monosodium urate monohydrate crystals.1 Plain radiographs of the great toe may demonstrate erosions with overhanging edges, pathognomonic for gout, but this patient’s plain films of her right great toe and foot failed to demonstrate this. Patients with comorbidities of hypertension, hypertriglyceridemia, and hypercholesterolemia tend to have a higher incidence of gout.1 This diagnosis can be ruled out by simple blood work. Pseudogout, as with gout, is a common crystal-induced arthropathy.1 Pseudogout commonly involves a single joint such as the knee and rarely affects the great toe, with acute swelling, warmth, erythema, and pain caused by calcium pyrophosphate crystal deposition. Synovial joint fluid analysis would be needed for microscopic examination of crystals; however, this patient’s symptoms did not occur suddenly, and there were no physical examination findings that demonstrated erythema or swelling of the right great toe.1 This rule outs pseudogout. Osteoarthritis, the most common joint disease, is a degenerative disorder due to the breakdown of hyaline cartilage in the synovial joints.2 Patients typically complain of deep achy joint pain exacerbated by extensive use; stiffness during rest, with morning stiffness commonly alleviated within 30 minutes; and decreased range of motion, typically with crepitus.2 The progression of osteoarthritis is slow, occurring over many years. This condition of chronicity can become debilitating as degenerative changes occur. This patient complains of an acute pain in her right great toe over the past 4 to 6 weeks, not a chronic pain of many years. Sciatica presents with lower extremity pain that can involve the foot and/or toes with or without neurologic deficit, localized to a distribution of one or more lumbosacral nerve roots.3 The most common distribution involves L4 to S2, with lower extremity pain aggravated by prolonged sitting or standing, as the patient describes. Her pain is alleviated with walking and constantly changing positions. She sustained a lower back injury in a car accident 3 months prior to presentation. Since then she has been suffering from intermittent back spasms. Considering her history and physical exam, this is probably the most likely diagnosis. Sacroiliitis is considered. The sacroiliac joints are diarthrodial synovial joints receiving innervation from S1 to S4.4 This patient had pain and spasms at the levels of L4 to L5, along with pain with palpation of the right sciatic nerve deep within the buttock. The radiation of pain was specific to the dermatomal distribution of L4 to L5, involving the right outer calf, mid-foot, and great toe.

2. Most Likely Diagnosis Sciatica. The patient’s presenting symptoms, her HPI, ROS, and physical exam findings all suggest sciatica. 3. Pathophysiology Sciatica is lower extremity pain with or without neurologic deficit, localized to a distribution of one or more lumbosacral nerve roots.3 The most common distribution involves L4 to S2, with lower extremity pain aggravated by prolonged sitting or standing and by bending forward or performing tasks with outstretched arms elevated and away from the body. Pain is typically alleviated with reclining, walking, or constantly changing positions, as these movements allow the lumbosacral

spine to become unloaded or to experience more balanced multidirectional forces, resulting in ­offloading of force applied to the nerve roots.3 When it is not clear which dermatomal distribution is involved, the terms radiculopathy or nonspecific radicular pattern are commonly used descriptions. Neck and back strains tend to affect many pain-sensitive structures of the spine, such as ligamentous support, spinal muscles, facet joints, and discs.3 Although disc herniation is a common cause of spinal and radicular pain, there is no correlation between the degree of a patient’s symptoms and the extent of the disc herniation. Degenerative changes and injuries to the spinal structures produce variable lower back and leg pain. The diagnosis of acute painful spinal conditions is commonly nonspecific, but when the lumbosacral nerve roots are involved, the leg pain is referred to as sciatica.3 4. Diagnostic Tests/Images l

l

Serum uric acid level to rule out hyperuricemia ❍ Patient to utilize the laboratory in the same office building to have this fasting blood lab drawn the following morning ❍ Results will be known in 48 hours. Normal values are 2.4 to 6.0 mg/dL in females. Lmbosacral-spine plain films ❍ Patient to utilize the radiology center in the same office building to have this study performed upon leaving the clinic ❍ Results will be known in 48 hours as this is not a STAT reading.

Results l Uric acid was within normal limit. l LS-spine plain films demonstrated degenerative changes. 5. Next Steps l

l

l l

l

l

l

Electronically prescribe (via EMR) the following medications during the initial visit: ❍ Naproxen 500 mg BID PRN for pain; discontinue ibuprofen (better compliance with naproxen because it is taken BID) ❍ Cyclobenzaprine 5 mg at night PRN for pain If these two medications do not help adequately, start prednisone (5 days each of 60 mg, 40 mg, and 20 mg; total cumulative dose =  600 mg). Discuss risks, benefits, and side effects of all medications including prednisone. Follow up by phone after 2 days with the patient once results of serum uric acid level and LS-spine plain film series are known. Refer to an orthopedist within the week and advise the patient to pick up the results of her studies to take with her to her appointment. Refer to a physical therapist, who will evaluate and treat the patient over the next 6 weeks with the approval of the orthopedic specialist. Remove purine-rich foods from her diet until gout is formally ruled out.

6. Evidence-Based Medicine/Treatment Approach l

l

l

l

Passively applied physical therapy for 6 weeks may allow inflammation to subside at the levels of L4 to L5. Spine care education and active exercise to include core stabilization along with physical therapy should be initiated with the goal of achieving pre-injury levels of physical functioning for the patient.3,5 Prolonged bed rest is not recommended as it may lead to hypomobility of joints, shortened soft tissues, atrophied musculature and decreased muscle strength, cardiopulmonary deconditioning, and loss of bone mineral density.3 Research supports the use of acetaminophen as a reasonable first step, although its efficacy may be equal to that of oral NSAIDs use.3 Muscle relaxants may be of value; however, sedation and dizziness are common side effects.3 It is important to educate patients on their utility in treating lower back pain and spasms by taking this medication at bedtime, when the risk for falls and other injuries is less likely. Oral prednisone may aid improvement in patients complaining of acute radiculopathy. The initial dose for oral prednisone is 1 mg/kg taken once daily, tapering the dose over 10 to 15 days.5

7. ICD-10 and CPT (E/M) Codes ICD-10 Code: M54.32 Sciatica, right side. This code is used based on the past medical history elicited during the HPI, the ROS, and physical examination findings in an age-appropriate female patient. CPT (E/M) Code: Level 4—99214 (Established Patient) This includes a detailed history and physical examination, medical decision-making of moderate complexity, and 25 minutes spent face to face with the patient. 8. Patient Education Topics l l

l l

Sciatica due to lumbar disc herniation typically resolves with conservative treatment.3 Prednisone should be taken to decrease inflammation in the lower back and surrounding right sciatic nerve. Importance of follow-up in 1 week with an orthopedist. Importance of physical therapy.

9. Medical or Legal Concerns It is possible to manage uncomplicated sciatica in primary care with physical therapist consultation. However, if the case is complicated, the patient needs to be evaluated and treated by an orthopedic specialist, who works with a PT. Risks associated with systemic prednisone use should be communicated with the patient and enough information should be provided to help the patient make an informed decision about taking this medication. Gout will also need to be ruled out, although it is not as high on the differential list as sciatica. Results from the patient’s diagnostic studies need to be relayed to the patient, the orthopedic specialist, and physical therapist so that nothing in this patient’s treatment is missed or dropped. In this case, cauda equina syndrome may be included in the differential diagnosis. Because the patient has no urinary or bowel dysfunction, it is not listed. However, PCPs need to keep cauda equina syndrome in mind. Missed or delayed diagnosis of cauda equina syndrome may end up with medical-legal liability cases. The patient needs to be given very strict directions that if her symptoms get worse, she needs to go to the nearest ED for prompt evaluation and treatment. 10. Interprofessional Collaboration l

l l

l l l

Outcome

Receptionist registers the patient, calls the radiology center to make same day appointment for LS-spine plain films, and also calls the laboratory in the office building to make next day appointment for the patient for her fasting serum uric acid level draw. MA performs patient intake and takes vital signs. PCP examines patient and makes diagnosis, calls in prescription for prednisone, and calls an orthopedic specialist’s office to make an appointment that week for the patient, with instructions for probable physical therapy referral. Office manager sends medical records to orthopedist. Orthopedist exams patients, makes definitive diagnosis, and initiates treatment. Physical therapist creates tailored plan for patient based on diagnosis.

Gout was ruled out in this patient, and she was allowed to return to her normal diet. The results of her LS-spine plain films demonstrated degenerative changes consistent with her age and loss of bone mineral density. She received an appointment for a DEXA scan that month, which demonstrated osteopenia, and she was placed on a weekly vitamin D and daily calcium supplementation regimen. She was seen and treated by the orthopedic specialist and physical therapist, who helped her regain her exercise tolerance and build her core strength to support her lower back. The prednisone worked almost immediately for this patient, subsiding her pain. She completed the 15-day tapering regimen as discussed. She used naproxen periodically after her physical therapy sessions, which she attended three times per week.

Insight

from the

PCP

The patient was very grateful and relieved to learn that her complaints were more than likely related to her lower back spasms experienced after the car accident. She stated that she should have sought medical attention for her back but did not think her injuries were significant. She stated that she thought her back spasms were from just getting older. The patient is well established in this practice and has a demonstrated record of following all instructions, treatments, and education.

References

1. Rothschild BM, Miller AV, Francis ML. Gout and pseudogout. Medscape. February 3, 2020. https://emedicine.

medscape.com/article/329958-overview.

2. Lozada CJ, Culpepper PSS. Osteoarthritis clinical presentation. Medscape. February 7, 2020. https://emedicine.

medscape.com/article/330487-overview.

3. Chawla J, Schneck MJ, Wheeler AH. Low back pain and sciatica. Medscape. August 22, 2018. https://emedicine.

medscape.com/article/1144130-overview#a1.

4. Chawla J, Schneck MJ, Wheeler AH. Characteristics of pain-sensitive structures: sacroiliac pain. Medscape. August

22, 2018. https://emedicine.medscape.com/article/1144130-overview#a1.

5. Papadakis MA, McPhee SJ, Rabow MW. Lumbar disk herniation. Anthony Luke, & C. Benjamin Ma, eds.

In: Current Medical Diagnosis and Treatment. 75th ed. McGraw-Hill Education; 2018:1709–1710. ISBN 978-1-25-986148-2

BILATERAL LEG PAIN, PEDIATRIC MALE

CASE

84

Chief CoMPlaint

“Bilateral leg pain.”

historY

Present illness

of

A woman brings in her 5-year-old African American son for an evaluation of worsening leg pain over the past 2 days. She states the boy started complaining of leg pain in both legs 2 days ago, after waking in the morning. She denies any known provoking trauma or falls and has not seen any evidence of injury to his legs. The boy points to both of his legs and states the pain is “all over inside.” The boy’s mother denies noticing any swelling or bruising of the legs and denies the boy having any similar symptoms in the past. The mother states her son is usually very active but has not been able to play or run due to the pain. The boy is crying due to pain, which prompted her to come in for evaluation. The mother tried applying warm and cold packs to his legs, without relief. She gave the boy a dose of acetaminophen yesterday morning without relief; however, the dose of ibuprofen she gave this morning seemed to calm him down a little.

reVieW

of

sYsteMs

The patient’s ROS is positive for bilateral leg pain L = R, with gait disturbance due to pain. His ROS is negative for fever, chills, weakness, vomiting, change in bowel, cough, SOB, and history of joint or bone pain.

releVant historY

The child’s history is significant for a full-term birth at 39 weeks, born in a small rural village in Martinique, at home, to his mother without pregnancy complications. His family immigrated to the United States when the patient was 3 years old. His immunizations are up to date. He has no history of neonatal infections or hospitalizations. He has met all developmental milestones, and growth trajectories are stable at 25th percentile for height and weight. He was diagnosed with strep throat last fall and treated with 7 days of antibiotics. He lives with his parents, attends pre-K full time, and eats a balanced diet. His mother is healthy, age 28 years. His father has an undiagnosed “blood condition” that caused him to be hospitalized for 3 days when he was 20 years old.

allergies

No known drug, food, or environmental allergies.

MeDiCations None.

PhYsiCal eXaMination Vitals: T 37°C (98.6°F); P 125; R 24; BP 98/54; HT 107 cm (42 in.), 25th percentile; WT 17 kg (37.4 lbs), 25th percentile; BMI 14.9. Growth on trajectory since initial visit at age 3 years. General: Slender boy, laying on exam table in mild distress, whimpering in pain at times, nontoxic appearing. Skin, Hair, Nails: No rashes, skin warm to touch with increased warmth of BLE compared to trunk/arms, full head of hair on scalp. No abnormal findings with nails.

336  Case 84  •  Bilateral Leg Pain, Pediatric Male

Head: Normocephalic, atraumatic. Eyes: Mild scleral icterus bilaterally, conjunctiva clear bilaterally. ENT/Mouth: Oropharynx clear with 2+ tonsillar hypertrophy bilat; no erythema or exudates noted. Nares patent bilaterally without drainage. Normal dentition, gums pink and moist. Moist mucous membranes. Neck: Supple, with shotty anterior bilateral cervical lymphadenopathy, non-tender. Chest: No deformity. Lungs: Clear to auscultation bilaterally without wheezes/ronchi/crackles. Heart: RRR without murmur, prominent point of maximal impulse. Abdomen: Soft, non-tender, non-distended, spleen tip palpable. Genital/Rectal: Uncircumcised penis, Tanner stage 1. Musculoskeletal: Normal bulk. Lower extremities without deformity or erythema. BLEs show increased warmth to palpation compared to trunk/upper extremities. Non-tender to palpation over bony prominences of lower extremities or knee/ankle joints. No edema or effusion noted bilaterally. Able to bear weight. Bilateral FROM lower extremities while supine. Strength 5/5 BLEs. Neurologic: Alert, refusal to stand due to pain but cranial nerves II to XII intact. BLE DTRs 2+ intact.

Clinical Discussion Questions 1. What is the differential diagnosis?

2. What is the most likely diagnosis? Why?

3. Demonstrate your understanding about the pathophysiology in regard to the most likely diagnosis.

Case 84  •  Bilateral Leg Pain, Pediatric Male 337 4. Should tests/imaging studies be ordered? Which ones? Why? Think about tests/imaging beyond the

primary care setting as well.

5. What are the next appropriate steps in management?

6. What are the risk factors, early symptoms, and treatment approach of the diagnosis? Provide references

for your responses.

7. What are the pertinent ICD-10 and CPT (E/M) codes for this visit? Provide a short rationale.

8. What are appropriate parent/patient education topics for this case?

338  Case 84  •  Bilateral Leg Pain, Pediatric Male 9. If not managed appropriately, what is/are the medical/legal concern(s) that may arise?

10. Think about interprofessional collaboration for this case. Provide a list of specialties or other disciplines

and indicate what contribution these professionals might make to managing the patient.

Bedside Manner Questions

11. What would your communication style/approach be with this parent and patient?

12. If a patient and his mother are distressed by the diagnosis, what might offer support?

ANSWER KEY: BILATERAL LEG PAIN, PEDIATRIC MALE

CASE

84

1. Differential Diagnosis L

L

L

L

L

L

Osteomyelitis commonly affects long bones in children and often includes systemic symptoms such fever and malaise, which are absent in this case. Absence of prior trauma and presence of generalized bilateral leg pain also make this diagnosis unlikely. Growing pains is a highly possible diagnosis given this patient’s age and the location of the pain, affecting BLEs. However, growing pains are more typical in the latter part of the day, improving overnight while sleeping. Leukemia is also a highly possible diagnosis, as it often presents with bone pain. Leukemia, specifically acute lymphoblastic leukemia in this age range, most commonly causes bone pain but is often associated with additional signs and symptoms of persistent fevers, bruising, and bleeding, with possibly organomegaly (which this patient has—enlarged spleen). Legg-Calve-Perthes disease and slipped capital femoral epiphysis are possible diagnoses, especially in young boys presenting with leg pain, and may or may not present with bilateral symptoms. However, the patient’s age does not align with the typical presenting age for these conditions, and the associated warmth found on physical exam are not typical of these conditions. Juvenile idiopathic arthritis is less likely given the physical exam findings and the location of pain in the lower extremities, as JIA is more likely to affect the hands/feet. Typically, JIA is associated with fevers and often presents with unilateral symptoms. Sickle cell disease/vaso-occlusive crisis is a likely diagnosis, given the patient’s description of symptoms and physical exam findings. The lack of trauma and the description of his pain “all over inside” depict a pathology that affects the bone or bone marrow, as is found in sickle cell. His ethnicity and the paternal history of a “blood condition” requiring hospitalization also increase the likelihood of this diagnosis.

2. Most Likely Diagnosis Sickle cell disease/vaso-occlusive crisis. Given the patient’s birth and early life in a country with limited medical resources, it is likely that his disease went undiagnosed. While most developed countries screen newborns for SCD, Martinique (and most of the smaller Caribbean areas) do not have access to such testing or to specialists familiar with the condition. This may also have contributed to the lack of formal diagnosis for his father’s anemia. As such, his father likely has sickle cell trait or homozygous disease, and his mother may be a carrier or is unaffected. In the United States, patients are diagnosed via newborn screen, and there is often more access to hematologists to make a formal diagnosis in patients presenting with his clinical picture. 3. Pathophysiology Sickle cell anemia results from a gene mutation in the beta globin hemoglobin chain that causes polymerization of the hemoglobin molecule, producing HgbS in addition to adult hemoglobin.1 This polymerization results in stiff erythrocytes that are predisposed to sickling and vaso-occlusion.1 The hereditary condition can result in sickle cell anemia (homozygous SS, most severe form) or sickle cell trait (heterozygous AS), depending on parents. Other forms include SB o-thalassemia, SB thalassemia, or hemoglobin SCD, which are less common and milder in nature.2 The pathogenesis of microvascular occlusions, a.k.a. vaso-occlusive episodes or crises, occurs because sickled (misshapen,

crescent-shaped erythrocytes) RBCs express higher levels of adhesion molecules, causing sticking.1 The RBCs become “stuck” in smaller blood vessels, producing pain, and result in ischemia. In addition, the occlusion leads to increased blood flow turbulence and higher velocity around the occlusion. Occlusions can occur at site but are more common in bone. More severe vaso-occlusive crises can occur in the brain (stroke), retina (retinopathy), spleen (splenic infarct, splenomegaly, splenic sequestration), and lungs (acute chest syndrome).2 Situations such as dehydration, cold, and hypoxia/high altitudes can predispose patients to more frequent sickling, so if this patient was more active and had increased work of breathing, especially in the summer months (leading to hydration) or in the winter months (due to the cold), this may have been the provoking factor. 4. Diagnostic Tests/Images L L

L

L

CBC w/differential ordered to assess for anemia Bilateral leg x-rays ordered to assess for fracture, leukemic changes, or deformities that may be causing pain ESR and CRP ordered to assess for inflammation and osteomyelitis. (ESR more likely to be elevated with JIA than other differentials; CRP should be negative with SCD.) Bilirubin also ordered to assess for hemolysis

Results CBC w/differential revealed a mild microcytic anemia (Hgb 8.4 g/dL), with mild leukocytosis. L Blood smear revealed a few sickled cells. L LE x-rays normal. L ESR mildly elevated. L CRP normal. L Unconjugated hyperbilirubinemia noted. L

5. Next Steps Initial Visit L Treat pain with NSAIDs and warm packs. If pain is not well controlled, low dose opiate may be prescribed for the short term or the patient may be referred to the ED for pain management if needed. L Parent and patient were encouraged to increase hydration via oral fluid intake. Follow-up Visit During follow-up visit, lab results will be discussed with parents, including the possibility of a sickle cell diagnosis. L Referral to hematology to confirm diagnosis and discuss management options. L An RBC transfusion may be needed if the patient’s Hgb falls below 7 g/dL with significant symptoms. L Follow-up scheduled in one week to review lab and x-ray results. L

6. Risk Factors, Early Symptoms, and Treatment Approach L

L

L L

L

A significant risk factor for sickle cell anemia is race. This disease disproportionately affects African Americans. Approximately 1 in 13 African American neonates are born with the sickle cell trait and 1 in 365 neonates are born with the disease. Couples planning to have a child can receive genetic screening.3 Early symptoms vary, and a child may present with jaundice, icterus, extremity pain, or irritability from the pain.3 The only cure for this disease is a blood and bone marrow transplant.3 Children with the disease should have regular appointments with a PCP and receive an annual flu and pneumonia vaccination.2 The transition to adult care is complicated for these patients, and parents and young adults need to be aware of common complications.3

7. ICD-10 and CPT (E/M) Codes ICD-10 Code: M79.606 Pain in leg, unspecified. Leg pain is presumed to be related to the presumed sickle cell, but there is no confirmed test for SCD/vaso-occlusive crisis at this time. Therefore, this is the most appropriate ICD-10 code to use during the initial visit and during the evaluation. CPT (E/M) Code: Level 4—99284 This was an emergency visit with a complicated history and complex medical decision-making. The complexity of the patient’s history and symptoms, leading to the possibility of a new chronic disease diagnosis, meet the criteria for a Level 4 visit. The complexity of labs ordered and interpretation, to make a possible diagnosis and determine treatment, including patient counseling and determination of treatment plan effectiveness, also help to meet the criteria for Level 4. 8. Patient Education Topics L L L L L

Sickle cell anemia Anemia symptoms Vaso-occlusive crisis Pain management Hemolysis

9. Medical or Legal Concerns Discussion of a possible diagnosis of a chronic condition, and the precautions to take plus the referral needed, can be perceived by patients negatively, and if the presumed diagnosis is not made or ruled out by a specialist, there exists a possibility for litigation for mental distress. Also, if treatment was initiated based on presumed diagnosis and a more serious condition was missed and not treated, complications can arise. In this case, it is preferred to discuss the possible conditions that the patient may have and why the provider feels it could be that but not to speak in actual diagnosis terms. Once the diagnosis is made, it is important to discuss the common complications of severe vaso-occlusive crises, such as AVN due to repeated crises, and risk of osteomyelitis. Failure to treat his anemia and manage the disease appropriately could lead to complications such as increased risk of infection and/or sepsis, splenic sequestration, stroke, and possible death. 10. Interprofessional Collaboration L L L L L

Outcome

Receptionist registers patient. MA takes chief complaint and vitals. PCP takes thorough history and physical exam and determines most likely diagnosis. Phlebotomist draws blood sample and prepares for testing. Pediatric hematologist confirms the diagnosis and manages the patient.

During the follow-up visit 5 days later, laboratory results and the likely diagnosis were discussed with the parents and patient. The parents admitted to hearing about SCD in their home country but had never been tested for it because they had never had symptoms. An initial hematology consult, done via phone when labs were available, confirmed that the patient was stable to be managed at home due to the lack of symptoms related to his anemia, with a plan for initial visit by a hematologist within 2 weeks. Return precautions were reviewed with parents regarding symptoms of anemia, new/worsened pain, new fever, and/or new neurologic changes. During this visit, the parents admitted that the patient was no longer complaining of pain and that they had been giving oral ibuprofen 200 mg around the clock every 6 hours and occasional warm baths and application of warm packs to legs for comfort.

Insight

from the

PCP

While these symptoms are classic of a young child with sickle cell anemia, it was a unique situation that this child had not been diagnosed sooner. In the United States, and many developed countries, the diagnosis is typically made via newborn screening. Having the opportunity to discuss the family’s access to healthcare in their home country and ascertain their knowledge of the disease allowed for a deeper connection between myself and the family. This case also provided an opportunity to explore the health disparities that exist for patients in lesser-developed countries, even with increasing use of newborn screenings in these countries. It also provided us an opportunity to get to know the culture of the country the family originated from and their adaptation to living in the United States, with access to healthcare and a new living environment. Sickle cell anemia is a complex disorder that involves many aspects of care, from basic pediatric management to specialty care and access to higher-level centers for acute treatment. I had the privilege of following this patient throughout his treatment, along with members of the hematology team. Due to the state health system’s plan for providing care coverage to any child with a life-threatening or chronic condition, this patient received all necessary care from the hematology team and his pediatrician. Although the family lived near the hospital and clinic, due to some changes in employment with the father, the family faced struggles with keeping appointments. However, when we collaborated with the hospital’s social worker, we were able to find solutions to rectify this, through the use of travel vouchers and scheduling appointments convenient with the mother’s schedule. The hematologist and nurse practitioner followed the patient’s maintenance management, and I as the PCP saw the patient during his red cell transfusion therapy and for any sick visits. Long-term care for this patient included chronic RBC transfusions every 3 weeks to keep Hgb S levels 100,000 CFU/mL. Susceptible to ciprofloxacin, levofloxacin, penicillin, tetracycline, and vancomycin; resistant to nitrofurantoin. L Vaginal wet mount confirmed the absence of infectious vaginitis as well as a lack of normal lactobacilli, consistent with low estrogen effect. M Paucity of lactobacilli; negative for yeast, clue cells, flagellates; few PMNs. Negative whiff test, pH 5.0. L Normal CBC ruled out systemic infection. M WBC 6.1 × 10³/mcL, Hgb 13.1 g/dL, Hct 40.1%, PLT 212 × 10³/mcL, with normal differential. L Normal chemistry panel ruled out electrolyte disturbance and blood sugar abnormalities. M Glucose 99 mg/dL, BUN 14 mg/dL, Creat 1.04 mg/dL(H), eGFR 57 mL/min/1.73(L), Na 141 mmol/L, K+ 4.3 mmol/L, Cl 102 mmol/L, ALT 11 IU/L, AST 14 IU/L. L Normal TSH ruled out thyroid disease as possible cause of the patient’s confusion. M TSH 0.474 mcIU/Ml. L

5. Next Steps L

L

L

L

L

The patient was started on a course of amoxicillin/clavulanate 875 mg/125 mg BID for 7 days, based on review of prior urine culture sensitivity testing, while awaiting current culture results. She was referred to a psychotherapist for her anxiety and refused. She also refused a trial of any low-dose SSRI, saying, “I know that’s an antidepressant and I’m not depressed.” With shared decision-making and full discussion of the risks and benefits, it was agreed she should take her diazepam every night at bedtime for now and avoid all alcohol. A check of the state’s controlled substance database revealed no problem use. She was counseled and reassured that there was no evidence of sepsis, was re-referred to her urologist, and was convinced to schedule four weekly visits with her primary physician at the practice to continue counseling for her uncontrolled anxiety. Recommendations were also made to use the talcum powder in skin folds rather than vulvovaginal mucosa and to start vaginal estrogen cream while slowly tapering off of oral estrogen therapy, in consultation with her urologist. The patient was advised to use hemorrhoidal ointment up to QID PRN for hemorrhoids.

6. Clinical Presentation of Older Adults and Initial Treatment The most common indication for ordering a urine culture in nursing homes is confusion, yet up to 50% of female nursing home residents (up to 40% of males) are positive for asymptomatic

bacteriuria. This results in the overdiagnosis and overtreatment of UTIs in a large subset of patients, along with the harms caused by unnecessary antibiotic use. A systematic review3 of 22 studies meeting inclusion criteria aimed to find evidence that lower UTI, diagnosed by positive culture, is a cause of confusion. Most studies exploring this link were found to have serious methodologic flaws, including the definition of UTI (e.g., failure to distinguish between pyelonephritis and cystitis). The authors concluded that there is no compelling evidence that lower UTIs in non-catheterized, afebrile patients are an underlying cause of confusion/altered mental status. Clinicians should therefore remain vigilant in searching for other causes of confusion in older patients diagnosed with acute cystitis. All societies that make recommendations for treating UTIs endorse dipstick urinalysis plus positive symptoms to make the diagnosis of UTI, and all societies recommend the same first-line agents for uncomplicated UTIs: nitrofurantoin, TMP-SMX, and f­osfomycin. It is important to prescribe a targeted, adequate course of antibiotics, as a­ n indiscriminate and/or partial course of broad-spectrum antibiotics may lead to bacterial r­ esistance, persistent or ascending infection, and bacteremia, which can result in urosepsis.2 7. ICD-10 and CPT (E/M) Codes ICD-10 Code: N30.08 Acute cystitis without hematuria. When the etiology of patient symptoms is definitively established, the diagnosis code used should be as specific as possible. ICD-10 Code: R41.0 Disorientation (confusion). When the etiology of patient’s chief complaint/primary symptom is not determinable at the time of the visit, the diagnosis code used should be the symptom itself. ICD-10 Code: N95.2 Atrophic vaginitis. To be reimbursed by insurers for the vaginal wet mount, a corresponding diagnosis code must be included. Because the cause of her vaginitis was definitively diagnosed by wet mount at the time of the visit, the most specific code, “atrophic vaginitis,” rather than “vaginitis,” should be used. CPT (E/M) Code: Level 4—99215 (Established Patient) This is a primary care visit for the evaluation and management of an established patient with multiple medical problems. The patient has a recurrent complaint with a possibility for health complications if untreated or misdiagnosed. The level of documentation for history, including ROS and physical exam including vital signs, data, risk, and medical decision-making support a level 4 visit; however, the clinician spent and documented 50 minutes of face-to-face time with the patient, with greater than 50% of this time spent on counseling. 8. Patient Education Topics L

L

L

L

L L

Sepsis: Explained this diagnosis with associated signs/symptoms, none of which are present; reassured patient that her body temperature is within the normal range.4 Vaginal hygiene: Advised to avoid application of powder directly to vulva/vagina/urethra due to risk of drying out mucosa; prolonged use of topical steroids, which also thin and dry the mucosa and can cause steroid dermatitis, should be avoided. Vaginal moisturizers available OTC can help soothe irritation caused by dryness. Medication compliance: Reiterated importance of compliance with all prescribed medications and invited her to bring any questions and concerns about prescriptions and symptoms directly to us or her healthcare providers rather than searching the internet. “Natural” products: Explained that this label does not equate to "safe" or "effective" and that she should always ask us or her healthcare providers before starting any new product. Dangers of surfing the internet for medical information. How to recognize symptoms of UTI in older patients.

9. Medical or Legal Concerns Missing a diagnosis of UTI is common in older patients, who may present without typical symptoms. This can lead to urosepsis, which may be life threatening. Likewise, assuming the underlying cause of confusion is a UTI without considering other causes, even when the urinalysis and/or culture is positive, may result in preventable complications. 10. Interprofessional Collaboration L L L

L L

Receptionist registers the patient. MA takes brief history and vital signs and performs urinalysis. PCP performs complete history, physical examination, and microscopic examination of vaginal wet mount; PCP also formulates diagnosis and initiates treatment plan. Phlebotomist draws blood for labs. Urologist provides ongoing specialty care for chronic urinary retention.

Outcome

The patient’s daughter called the next day to report that the confusion was gone, and many months later, it has not returned. This, along with the normal blood tests, supports that the acute anxiety exacerbation stemming from her fear of sepsis was the most likely cause of her confusion. The patient finished the full course of antibiotics, which she tolerated moderately well by taking with food. Our reassurance, along with the recommendation that she take her diazepam every night at bedtime instead of PRN, was effective in abating her acute anxiety. Unfortunately, the patient continues to refuse a trial of first-line medications for her GAD, saying, “I’m allergic to everything. . . . I know what works for me”; she remains on nightly diazepam, repeatedly educated about risks, including falls and cognitive changes. She has taken up online games and puzzles as a replacement for internet surfing.

Insight

from the

PCP

This patient is well known to all members of our care team. Her multiple significant medical problems, including anxiety, fear of medication side effects, and spending significant time alone surfing the internet, have led to a pattern of mistrust of medications and poor compliance with care plans. Failure to improve and emotional distress led to a dysfunctional pattern of repeated phone calls to the office for various symptoms, demanding treatment over the phone. As the patient would repeatedly refuse an office visit, empiric treatments were sometimes used but generally failed, given the patient’s emotional limitations. I was pleasantly surprised when I successfully convinced her to schedule weekly check-in appointments, which provided the regular, personal counseling and reassurance needed to minimize her internet-fueled fears and anxieties. She kept all scheduled appointments and is now seen every 2 months for follow-up. She finally returned to her urologist as advised and started the prophylactic medication (methenamine hippurate) recommended 18 months prior. Its primary side effect, looser stools, allowed her to discontinue her daily laxative, and she has not had any recurrence of cystitis since. Her quality of life is significantly improved, as has her daughter’s, and the frequent frantic phone call habit has resolved.

References

1. ACOG Bulletin N. 91 American College of Obstetrics and Gynecologists: treatment of urinary tract infections in

nonpregnant women. Obstet Gynecol. 2008; 111: 785–794. doi:10.1097/aog.0b013e318169f6ef

2. Markowitz MA, Wood LN, Raz S, Miller LG, Haake DA, Kim JH. Lack of uniformity among United States

recommendations for diagnosis and management of acute, uncomplicated cystitis. Int Urogynecol J. July 2019; 30(7):1187–1194. doi:10.1007/s00192-018-3750-z 3. Mayne S, Bowden A, Sundvall P. et al. The scientific evidence for a potential link between confusion and urinary tract infection in the elderly is still confusing: a systematic literature review. BMC Geriatr. 2019;19:32. doi:10.1186/ s12877-019-1049-7 4. Obermeyer Z, Samra J, Mullainathan S. Individual differences in normal body temperature: longitudinal big data analysis of patient records. BMJ. 2017;359:j5468. doi:10.1136/bmj.j5468

LUMP IN THROAT, ADULT MALE

CASE

86

Chief CoMPlaint

“Lump in throat.”

historY

Present illness

of

A 35-year-old man arrives at his PCP’s office reporting the sensation of a lump in his throat for over 1 month and episodes of mild epigastric pain with occasional nausea. He has experienced bloating and belching. He has a dry cough and feels like he needs to clear his throat often. He has a sour taste in his mouth and states that he frequently gets painful canker sores. He also complains of a burning type of pain to the anterior chest. The burning pain starts in the upper chest area and works its way up to the neck and throat area. He experiences the symptoms daily, and they become worse when he lies down or after eating.

reVieW

of

sYsteMs

The patient’s ROS is positive for nausea, bloating, belching, cough, canker sores, chest pain, and repetitive throat clearing. His ROS is negative for sore throat, hoarseness, dysphagia, vomiting, abdominal pain, melena, weight loss, change in appetite, asthma, ear pain, tooth pain, or exertional type chest pain. He denies a cardiac history or hypertension. He does not smoke. He occasionally drinks alcohol and daily drinks caffeinated soda. He denies any SOB or wheezing.

releVant historY

The patient has a history of Paget Schroetter syndrome at age 25. He is a full-time high school teacher. He exercises regularly. He is sexually active in a long-term relationship with his live-in girlfriend. His family history includes both grandparents with diabetes and hypertension. His father, who is living, has a history of stage 4 throat cancer at age 55.

allergies

No known drug allergies; no known food allergies.

MeDiCations None.

PhYsiCal eXaMination

Vitals: T 37.2°C (98.8°F), P 82, R 20, BP 120/80, HT 193 cm (76 in.), WT 127 kg (280 lbs), BMI 34.08. General: Alert and cooperative. Sitting comfortably on exam table. Well nourished. No acute distress. Psychiatric: Mildly anxious.

346  Case 86  •  Lump in Throat, Adult Male

ENT/Mouth: Tympanic membranes clear; canals clear bilaterally. Nose with no nasal septal deviation; mucous membranes moist and pink. Throat with pink and moist oropharynx; no erythema, no tonsillar enlargement, no lesions, no tooth erosion. Mouth with no sores or lesions present. Neck: Supple with no lymphadenopathy; thyroid normal size. Chest: Symmetric with no palpated tenderness. Lungs: Clear to auscultation bilaterally; no rales, wheezes, or rhonchi. Heart: RRR; no murmurs or gallops. Abdomen: Non-distended and soft with normal active bowel sounds present in all quadrants; no pain on palpation of the abdomen.

Clinical Discussion Questions 1. What is the differential diagnosis?

2. What is the most likely diagnosis? Why?

3. Demonstrate your understanding about the pathophysiology in regard to the most likely diagnosis.

4. Should tests/imaging studies be ordered? Which ones? Why? Think about tests/imaging beyond the

primary care setting as well.

Case 86  •  Lump in Throat, Adult Male 347 5. What are the next appropriate steps in management?

6. What is the prevalence and treatment approach of the diagnosis and the risks associated with the

medications when used long term? Provide references(s) for your response.

7. What are the pertinent ICD-10 and CPT (E/M) codes for this visit? Provide a short rationale.

8. What is the appropriate patient education topic for this case?

9. If not managed appropriately, what is/are the medical/legal concern(s) that may arise?

10. Think about interprofessional collaboration for this case. Provide a list of specialties or other disciplines

and indicate what contribution these professionals might make to managing the patient.

348  Case 86  •  Lump in Throat, Adult Male

Bedside Manner Question

11. What would your communication style/approach be with this patient?

ANSWER KEY: LUMP IN THROAT, ADULT MALE

CASE

86

1. Differential Diagnosis L

L

L

L

L

L

Hiatal hernia is a condition where a portion of the stomach protrudes through the diaphragm into the thoracic region. Most hiatal hernias do not produce symptoms but are often found incidentally after radiology testing. However, esophageal reflux and its symptoms (heartburn, abdominal discomfort, throat irritation, belching, and regurgitation) may be associated with the presence of a hiatal hernia. Peptic ulcer disease is most often seen in patients using NSAIDs or with a history of Helicobacter pylori infection. This can be evaluated with an H. pylori breath test and endoscopy if treatment fails. This patient was not on NSAIDs and had no history of H. pylori infection, making this a less likely diagnosis. Malignancy is unlikely in this case because the patient is relatively young, and he has not had any unexplained weight loss. However, because of the patient’s sensation in the throat and a family history of throat cancer, this must be explored. Laryngopharyngeal reflux, also known as silent reflux, has atypical reflux-like symptoms. This patient does not have sore throat or hoarseness, which are two common symptoms of laryngopharyngeal reflux. This patient presents with chest pain, so another diagnosis is more likely. Gastroesophageal reflux disease has a high prevalence in the U.S. population. Heartburnlike symptoms are the most common complaint, and this patient has atypical chest pain and heartburn-like symptoms. He also has a chronic cough and some nausea. He has a globus sensation, a feeling of a lump in the throat, which is also a symptom. Coronary artery disease should always be ruled out first in patients who complain of chest pain. This is not the likely diagnosis in this case as the patient has no exertional chest pain, is relatively young, and has no personal or family history of cardiac disease.

2. Most Likely Diagnosis Gastroesophageal reflux disease. Based on the patient’s age and presenting symptoms, GERD is the most likely diagnosis. His mild epigastric pain, burning sensation in the chest (likely heartburn), sensation of the lump in the throat, bloating, and cough all support this to be the most likely diagnosis. 3. Pathophysiology In normal function, the LES provides a barrier to reflux of acid from the stomach back into the esophagus. Additionally, esophageal peristalsis waves toward the stomach when the sphincter is relaxed is meant to ensure that stomach acid does not reflux into the esophagus. Loss of sphincter tone, increasing frequency of sphincter relaxation, loss of esophageal peristalsis, increased stomach volume and pressure (i.e., with obesity), or increased acid production will contribute to reflux of acidic stomach contents into the esophagus. Acute reflux of acid into the esophagus will cause pain or erosion of the esophageal mucosa. Recurrent damage to the mucosa will result in inflammation and damage at the cellular level.1

Hiatal hernias are found in one-fourth of patients with non-erosive GERD, 75% of patients with severe erosive esophagitis and over 90% of patients with Barrett esophagus. They are caused by the movement of LES above the diaphragm, resulting in dysfunction of the gastroesophageal junction reflux barrier.2 4. Diagnostic Tests/Images L

L

L

L

Initial diagnostics are not warranted for patients with symptoms suggestive of uncomplicated reflux disease. If treatment response is as expected, no further imaging/testing is required.2 An EKG might be ordered in cases of “atypical” manifestations of GERD including noncardiac chest pain. This would be to rule out an underlying cardiac etiology if this symptom were present, including angina or other ischemic diseases.2 Testing for H. pylori is only indicated for patients with a history of active peptic ulcer disease, MALT lymphoma, or dyspepsia with local prevalence >10%. It is not indicated as a first-line assessment in patients with uncomplicated GERD.3 Subsequent follow-up evaluations may include an upper endoscopy for persistent symptoms despite treatment. This would be to document the type and extent of tissue damage secondary to GERD.2

5. Next Steps L L L

L

Prescribe a H2 blocker. Recommendations on LSM to reduce symptoms. Schedule a follow-up in 2 weeks to determine initial response to treatment and decide if referral to a GI specialist is necessary. Inform the patient of ED precautions and the need to seek immediate treatment in the event symptoms suddenly worsen or new symptoms develop.

6. Prevalence, Treatment Approach, and Risk Associated with Long-Term Medication L

L

L

GERD affects 18% to 28% of adults in the United States.4 It is a common chief complaint and one PCPs see frequently. Many patients perceive GERD as a life-limiting disease. Patients have difficulties with diet and lifestyle changes required to manage their disease and require ongoing support, resources, and understanding. Patients require monitoring to ensure that any complications or worsening of symptoms is caught quickly. LSM with education about the factors precipitating physiologic and pathologic reflux is the first-line treatment for GERD.5,6 The education includes dietary changes, alcohol and tobacco use, sleep position, and weight loss. Even though many GERD patients are advised to avoid certain foods, there is not much evidence to support this recommendation. However, weight loss, quitting smoking, elevating the head of the bed, and avoiding late evening meals can significantly reduce symptoms.7 After LSM, initiating acid suppression medications is the next step. These include antacids, histamine-receptor antagonists, and PPIs. Among them, PPIs are the preferred medication. Surgical therapy is the last step and usually reserved for patients who do not respond well to acid suppression medication, patients who prefer surgical approach, and patients who present with complications due to GERD.8 Evolving studies demonstrate some side effects when the patients are on medications for long-term management of GERD. The long-term use of PPIs may have the following risks: (a) malabsorption of calcium, magnesium, and vitamin B12, which can lead to bone fractures9,10; (b) increased risk of community-acquired pneumonia11 and enteric infections11,12; (c) interactions with cytochrome P450 isozyme 2C19, which can result in interference with metabolism of other coadministered drugs (clopidogrel)11; (d) increased risk of chronic kidney disease12–17; and (e) an association with microscopic colitis.18 PCPs should have a good understanding about the long-term use of PPIs and potential risks.

7. ICD-10 and CPT (E/M) Codes ICD-10 Code: K21.9 GERD without esophagitis. Considering all his presenting symptoms, such as mild epigastric pain, burning sensation in the chest (heartburn), sensation of the lump in the throat, bloating, and cough, this is an appropriate ICD-10 code to use during the initial visit. CPT (E/M) Code: Level 4—99214 (Established Patient) This patient required complex medical decision-making including prescribing a medication. It took approximately 25 minutes, which meets criteria for level 4. 8. Patient Education Topics L L L

Lifestyle and diet modifications Managing symptoms Avoiding aspirin and ibuprofen, which can cause further irritation

9. Medical or Legal Concerns L

L

L

Previously, PPIs were one of those most commonly prescribed medications worldwide and have been the mainstay of treatment for GERD. Emerging data are showing an association between long-term PPI use and bone fractures, Clostridium difficile infection, pneumonia, MI, and stroke. Potent acid suppression has now been shown to increase the reflexive production of gastrin. Multiple observational studies suggest that long-term use of PPIs is also associated with a higher risk of gastric cancer development.19 This is significant given the widespread use and OTC availability of PPIs. It is important to advise a patient of this and determine medication based on risks and benefits. All information provided regarding the risks, benefits, and side effects of medication should be documented in a patient’s medical record. A missed diagnosis or failure to explain the potential serious nature of the outcome if recommendations are not complied with could result in litigation against a PCP. Persistent, untreated esophageal reflux, from whatever underlying cause, will result in recurrent injury and over time strictures, pain, obstruction, perforation, or Barrett esophagus. Patients with Barrett esophagus have an 11-fold increased risk of esophageal carcinoma compared to patients without Barrett esophagus.20

10. Interprofessional Collaboration L L

L

L

Outcome

Receptionist registers the patient. MA rooms the patient and takes vital signs; reviews the current medication; takes a social history, including use of alcohol or smoking; and takes a family history. PCP performs a focused history and exam based on the chief complaint, formulates likely diagnosis, and develops treatment plan. GI specialist is consulted if initial treatment is ineffective.

At the follow-up appointment, the patient had achieved some relief; however, he continued to have reflux symptoms and was eventually sent to a gastroenterologist and then an o ­ tolaryngologist. Ambulatory pH monitoring was done and showed severe reflux disease. The otolaryngologist performed a laryngoscope to address the patient’s need to constantly clear his throat. The otolaryngologist did more tests and found that the patient also suffered from laryngopharyngeal reflux (note this was listed on the differential). The patient realized he needed to follow the lifestyle recommendations to get better. He decreased his alcohol and caffeine intake, and this made a big difference. He continued on medications but never completely resolved all his symptoms.

Insight

from the

PCP

GERD is a common complaint that can mimic other diseases. It is important to get a good history. Communication from provider to the patient is important as well as explaining the diagnosis in a way the patient can understand. This patient was at a higher risk as his father had severe GERD that resulted in throat cancer. His father’s history caused the patient to worry even more. He was worried he would develop throat cancer. It was a great opportunity to discuss with the patient the importance of following the treatment plan as well as the importance of regular follow-up and LSM. Today, he is doing better and has lost weight; he no longer drinks alcohol and continues to follow the dietary restrictions that have helped significantly. He is screened each year by me for symptoms related to his GERD diagnosis.

References

1. Hammer GD, McPhee SJ. Pathophysiology of Disease: An Introduction to Clinical Medicine. 8th ed. McGraw-Hill

Education; 2018.

2. Papadakis MA, McPhee SJ. Current Medical Diagnosis and Treatment. McGraw-Hill Education; 2019.2019. 3. Kamboj AK, Cotter TG, Oxentenko AS. Helicobacter pylori: the past, present, and future in management. Mayo

Clin Proc. 2017 Apr;92(4):599-604. doi: 10.1016/j.mayocp.2016.11.017. Epub 2017 Feb 13. PMID: 28209367.

4. Granderath FA, Kamolz T, Pointer R. Gastroesophageal Reflux Disease: Principles of Disease, Diagnosis, and

Treatment. Springer; 2006.

5. Meining A, Classen M. The role of diet and lifestyle measures in the pathogenesis and treatment of

gastroesophageal reflux disease. Am J Gastroenterol. 2000;95:2692–2697. doi:10.1111/j.1572-0241.2000.03175.x

6. Kaltenbach T, Crockett S, Gerson LB. Are lifestyle measures effective in patients with gastroesophageal reflux

disease? An evidence-based approach. Arch Intern Med. 2006;166:965–971. doi:10.1001/archinte.166.9.965

7. Ness-Jensen E, Hveem K, El-Serag H, et al. Lifestyle intervention in gastroesophageal reflux disease. Clin

Gastroenterol Hepatol. 2016;14:175–182.e1-3. doi:10.1016/j.cgh.2015.04.176

8. Heidelbaugh JJ, Nostrant TT, Kim C, et al. Management of gastroesophageal reflux disease. Am Fam Physician.

2003;68:1311–1318. doi:10.1016/j.osfp.2010.12.003

9. Yang YX, Lewis JD, Epstein S, et al. Long-term proton pump inhibitor therapy and risk of hip fracture. JAMA.

2006;296:2947–2953. doi:10.1001/jama.296.24.2947

10. Ito T, Jensen RT. Association of long-term proton pump inhibitor therapy with bone fractures and effects on 11. 12. 13. 14. 15. 16. 17. 18. 19. 20.

absorption of calcium, vitamin B12, iron, and magnesium. Curr Gastroenterol Rep. 2010;12:448–457. doi:10.1007/ s11894-010-0141-0 Strand DS, Kim D, Peura DA. 25 Years of proton pump inhibitors: a comprehensive review. Gut Liver. 2017;11:27– 37. doi:10.5009/gnl15502 Peng YC, Lin CL, Yeh HZ, et al. Association between the use of proton pump inhibitors and the risk of ESRD in renal diseases: a population-based, case-control study. Medicine. 2016;95:e3363. doi:10.1097/md.0000000000003363 Lazarus B, Chen Y, Wilson FP, et al. Proton pump inhibitor use and the risk of chronic kidney disease. JAMA Intern Med. 2016;176:238–246. doi:10.1001/jamainternmed.2015.7193 Arora P, Gupta A, Golzy M, et al. Proton pump inhibitors are associated with increased risk of development of chronic kidney disease. BMC Nephrol. 2016;17:112. doi:10.1186/s12882-016-0325-4 Xie Y, Bowe B, Li T, et al. Proton pump inhibitors and risk of incident CKD and progression to ESRD. J Am Soc Nephrol. 2016;27:3153–3163. doi:10.1681/asn.2015121377 Xie Y, Bowe B, Li T, et al. Long-term kidney outcomes among users of proton pump inhibitors without intervening acute kidney injury. Kidney Int. 2017;91:1482–1494. doi:10.1016/j.kint.2016.12.021 Klatte DCF, Gasparini A, Xu H, et al. Association between proton pump inhibitor use and risk of progression of chronic kidney disease. Gastroenterology. 2017;153:702–710. doi:10.1053/j.gastro.2017.05.046 Bonderup OK, Nielsen GL, Dall M, et al. Significant association between the use of different proton pump inhibitors and microscopic colitis: a nationwide Danish case-control study. Aliment Pharmacol Ther. 2018;48:618– 625. doi:10.1111/apt.14916 Cheung KS, Leung WK. Long-term use of proton-pump inhibitors and risk of gastric cancer: a review of the current evidence. Therap Adv Gastroenterol. 2019;12:1756284819834511. Published 2019 Mar 11. doi:10.1177/1756284819834511 Menezes MA, Herbella FAM. Pathophysiology of gastroesophageal reflux disease. World J Surg. 2017;41:1666–1671. doi:10.1007/s00268-017-3952-4

FEVER AND RASH, ADULT MALE

CASE

87

chiEf comPlaint

“Fever and rash.”

history

of

PrEsEnt illnEss

A 22-year-old man presents to the office for the first time with an influenza-like illness he has had for the past week. Symptoms developed gradually and consist of fever (measured at home up to 101.4°F), sore throat, fatigue, anorexia, myalgias, and a mild diffuse headache. Yesterday, he noted an erythematous rash on the arms, torso, and legs, which prompted his visit today. He denies any known sick contacts, unusual exposures, or recent travel.

rEViEW

of

systEms

The patient denies chills. He denies stiff neck or any focal neurologic complaints. He denies rhinorrhea, ear pain or discharge, sinus pressure or discharge, cough, SOB, chest pain, abdominal pain, nausea, vomiting, diarrhea, dysuria, hematuria, penile discharge, or arthralgia. A comprehensive ROS is otherwise unremarkable.

rElEVant history

The patient has no history of significant medical problems or STIs. He has not had any recent dental procedures. Since last year he has been on HIV PrEP. He is currently taking tenofovir/emtricitabine. He tested negative for HIV about 10 weeks ago as it is a recommended screening every three months while on PrEP. He received a flu shot this past fall. He received meningococcal vaccination prior to entering college. He otherwise believes he is up to date on all required immunizations. The patient is a graduate student in civil engineering. He drinks alcohol at parties on weekends and has no history of tobacco use or vaping. He does not use any illicit substances, including marijuana. He is sexually active with men; he broke up with his previous partner 7 to 8 months ago, and he does not have a current partner. His last sexual activity was approximately 2 months ago after meeting someone at a party. The patient’s family history is negative for chronic or recurrent infections; it has been reviewed and is otherwise noncontributory.

allErgiEs

No known drug allergies; no known food allergies.

mEDications

Tenofovir/emtricitabine 25 mg/200 mg QD.

350  Case 87  •  Fever and Rash, Adult Male

Physical Examination Vitals: T 37.7°C (99.8°F), P 86, R 12, BP 125/80, HT 175 cm (69 in.), WT 70 kg (154 lbs), BMI 22.7. General: Thin male in no acute distress. Psychiatric: Normal mood and affect. Skin, Hair, and Nails: Skin: There is a faint maculopapular rash on torso, back, and the upper and lower extremities including the palms and soles. No abnormal findings with hair or nails. Head: Normocephalic, atraumatic. Eyes: No scleral icterus. ENT/Mouth: Sinuses non-tender to palpation. Poor dentition without gingival inflammation or tenderness to palpation. No foul odor to the breath. Neck: Supple, FROM. No cervical lymphadenopathy. Chest: Non-tender to palpation. Lungs: Clear to auscultation bilaterally. Heart: RRR without murmur. Abdomen: Soft, non-tender, non-distended, active bowel sounds, no organomegaly, no masses felt. Genital/Rectal: No lesions noted. No penile discharge. Scrotum non-tender. Multiple small, non-tender inguinal lymph nodes are palpable bilaterally. Rectal exam deferred. Musculoskeletal: No inflammation or tenderness to palpation diffusely. Neurologic: A&O×3, grossly non-focal.

Clinical Discussion Questions 1. What is the differential diagnosis?

2. What is the most likely diagnosis? Why?

3. Demonstrate your understanding about the pathophysiology in regard to the most likely diagnosis.

Case 87  •  Fever and Rash, Adult Male 351 4. Should tests/imaging studies be ordered? Which ones? Why? Think about tests/imaging beyond the

primary care setting as well.

5. What are the next appropriate steps in management?

6. What is the level of understanding of this diagnosis among noninfectious disease providers? What is the

treatment recommendation for this diagnosis? List the references you consulted.

7. What are the pertinent ICD-10 and CPT (E/M) codes for this visit? Provide a short rationale.

8. What is the appropriate patient education for this case?

352

Case 87 • Fever and Rash, Adult Male

9. If not managed appropriately, what is/are the medical/legal concern(s) that may arise?

10. Think about interprofessional collaboration for this case. Provide a list of specialties or other disciplines

and indicate what contribution these professionals might make to managing the patient.

BEDsiDE mannEr QuEstions

11. What would your communication style/approach be with this patient?

12. If a patient is distressed by the diagnosis, what might offer support?

ANSWER KEY: FEVER AND RASH, ADULT MALE

CASE

87

1. Differential Diagnosis L

L

L

L

L

L

L

Acute Epstein-Barr virus is possible; however, most adults have been exposed to the EBV and have built up immunity and will not contract mononucleosis. No splenomegaly was appreciated during the abdominal exam. Collectively, this is an unlikely diagnosis. Acute cytomegalovirus is not likely. Healthy people who are infected with CMV usually develop minor symptoms or no symptoms at all. However, if patients have weak immune systems (i.e., patients with HIV), they can be symptomatic. When symptoms occur, they are similar to the symptoms of mononucleosis. From the relevant history, it appears his HIV status was negative as 10 weeks ago. This is an apparently healthy person without significant medical history. Therefore, this is unlikely the diagnosis. HIV could be high on the differential, except the patient has been taking PrEP (HIV prophylaxis) regularly, lowering this possibility. If the patient had only recently started tenofovir/emtricitabine for HIV prophylaxis, one might more highly consider a drug reaction as a leading diagnosis, but he has been taking this medication since last year, so this is less likely as well. Frequent HIV monitoring is necessary for patients who are on PrEP, and HIV screening should be considered. Secondary syphilis is a likely diagnosis in the case in light of the patient’s rash (particularly the distribution including his palms and soles of the feet) and his recent sexual activity. Also, multiple small, non-tender inguinal lymph nodes found during physical exam suggest this is very likely. Gonococcemia/gonococcal infection can occur even in the absence of discharge and is possible here in light of the patient’s recent sexual activity. However, he has no dysuria. His testicles are non-tender. The presenting rash here is not really consistent with gonorrhea. Therefore, this is unlikely. Influenza is not likely. The patient received an influenza vaccine prior to the onset of “flu season” but, of course, the vaccine is not completely protective against all influenza strains, particularly those not included in the annual vaccine. That being said, a rash such as the one observed in this case is not commonly observed in influenza infections. Subacute bacterial endocarditis is on the differential. Although he does not have great dentition, the fact that he had not had any recent dental procedures probably makes subacute bacterial endocarditis less likely as well. Keep in mind, however, that transient bacteremia occurs all the time, so his poor dentition would still place him at ongoing risk moving forward.

2. Most Likely Diagnosis Secondary syphilis. The mononucleosis syndrome accompanied by the classic widespread maculopapular rash on the palms of the hands and soles of the feet raises high suspicion for secondary syphilis as the diagnosis.

3. Pathophysiology Syphilis is caused by a bacterium (a spirochete) called Treponema pallidum. T. pallidum enters the body by directly penetrating mucous membranes, typically during intercourse (oral–genital, genital–genital, anal). It can also enter through micro-abrasions of the skin epithelium with contact of fluid that is full of bacteria. The painless ulcers of the primary disease are often undetectable. In the secondary phase, in which one becomes “spirochetemic,” T. pallidum can then migrate to essentially any tissue. Therefore, almost every organ can be affected, often resulting in the mononucleosis syndrome. The classic maculopapular rash associated with secondary syphilis involves the palms of the hands and soles of the feet, although this is not always the case.1 4. Diagnostic Tests/Images L L L L L L L

Syphilis ELISA: positive RPR: 1:32 FTA-ABS: positive HIV: negative Hepatitis A, B, C: negative Gonorrhea: negative Chlamydia: negative

Helpful Information About Syphilis Testing There are two types of serologic tests classified based on the type of antigen the antibodies are directed against. Treponemal tests (the FTA-ABS test, T. pallidum particle agglutination) detect antibody to T. pallidum proteins. Nontreponemal tests, such as the RPR and the VDRL test, detect antibodies directed against lipoidal antigens, damaged host cells, and possibly from treponemes. Both tests are used to confirm the infection and determine whether the disease is active.2 L In the past, when testing for clinical findings or for screening purposes, the nontreponemal tests (e.g., RPR or VDRL) were used. If a nontreponemal test was positive, a confirmatory treponemal test (e.g., FTA-ABS) was performed. However, the advent of automated and rapid treponemal testing has allowed the algorithm to be reversed. So, it is now accepted to order a treponemal test as the initial diagnostic tool. L Practically speaking, many labs currently perform a panel of tests in combination rather than only one. This includes three tests: a syphilis ELISA (a general screen, which is very sensitive and less expensive to perform than other syphilis tests), an FTA-ABS (treponemal test, specific), and an RPR (quantitative/titer). Therefore, it is this combination that is reported. L The RPR looks for nonspecific antibodies in the blood that may indicate infection with syphilis. The term reagin means that this test does not look for antibodies against the bacterium itself but rather for antibodies against substances released by cells when the bacterium damages them, particularly cardiolipin and lecithin. The results for this test can be qualitative (reactive or nonreactive) or quantitative (nonreactive or titers 1:1, 1:2, 1:4, 1:8, 1:16 . . .). The RPR is usually sensitive for syphilis, so it has been useful as a screening test, but it is not completely specific. Many things can lead to a similar cellular release of substances such as cardiolipin and lecithin. Therefore, a positive RPR does not always indicate an infection with syphilis. Many other things can result in a positive RPR, including other spirochetal infections (e.g., Lyme disease), many other infections (viral and bacterial), autoimmune conditions (e.g., SLE), and some medications (most classically chlorpromazine and procainamide).2 L It is important to remember that for patients who present early in the course of disease, serologic testing may be negative. If there is a high clinical suspicion for syphilis, presumptive treatment should be administered and then repeat serologic testing should be performed in 2 to 4 weeks. L Last, there are several tests that can be used to directly detect the organism, although these methods are not widely available. These include dark-field microscopy, PCR, and direct L

fluorescent Ab testing for T. pallidum. In some cases, these tests may allow the diagnosis of syphilis prior to a serologic response. However, most clinical centers do not have access to these methods and must rely on clinical manifestations and serologic testing.3 5. Next Steps L

L

L L

Serologic syphilis testing was performed. As coinfections are common, tests for HIV; hepatitis A, B, and C; gonorrhea; and chlamydia were done. Note these tests are ordered during the initial visit. But the patient needs a follow-up appointment to discuss results. Due to high clinical suspicion, the patient was treated with a single injection of benzathine penicillin G 2.4 million units intramuscularly at the time of the initial visit.4 Follow-up appointment was scheduled in 1 week. If test results are confirmed positive for any STI, the clinic is required by law to report the case to the public health department.

6. Understanding the Diagnosis and Treatment Recommendations L

L

L

The number of syphilis cases is increasing in the United States. Therefore, recognition and timely diagnosis by medical providers are vital to initiate treatments and prevent future transmission.5 A cross-sectional survey was performed in Rhode Island between 2016 and 2017 including 231 participants. Of those, 45% were medical students, 34% residents or fellows, 11% medicine-attending physicians, and 10% infectious disease–attending physicians. The survey assessed basic knowledge of syphilis. The study concluded that general syphilis knowledge among noninfectious-disease medical providers (medical students, residents, fellows, and medicine-attending physicians) was low compared to infectious disease– attending physicians.5 Better education and clinical training for PCPs are needed to promote early diagnosis, treatment, and prevention efforts. Benzathine penicillin G, a penicillin formulation with a long half-life, is the recommended treatment for all syphilis stages.6 Individuals with syphilis frequently have other unmet sexual health needs, including screening for other STIs including HIV,7 sexual health–related vaccinations,8 and PrEP.9

7. ICD-10 and CPT (E/M) Codes ICD-10 Code: B27.90 Infectious mononucleosis, unspecified without complication. ICD-10 Code: A51.39 Other secondary syphilis of the skin. ICD-10 Code: A51.49 Other secondary syphilitic conditions. During the initial visit, ICD-10 code B27.90 is the most appropriate given the mono-like symptoms. In addition, A51.39 and A51.49 may be added as the history and physical exam suggest secondary syphilis is the most likely diagnosis. However, it is important to add a note stating that the evaluation is still in progress. An alternative approach would be using only the B27.90 ICD-10 code and add a note that it is likely secondary syphilis and the labs are pending. Note: Although not all cases of the mononucleosis syndrome are infectious in etiology, the ICD-10 code for mononucleosis includes the term infectious. CPT (E/M) Code: Level 3—99203 (New Patient) This code is designated for an office or other outpatient visit for the evaluation and management of a new patient, which requires these three components: a detailed history, a detailed examination, and medical decision-making of low complexity.

8. Patient Education Topics L L L L L L L

Syphilis Mononucleosis syndrome Notifications of sexual partners Safer-sex counseling When to resume sexual activity Testing for other STIs HIV PrEP discussion, pending HIV testing results

9. Medical or Legal Concerns Syphilis has been increasing in incidence over the past decade.10 It is a notifiable disease and must be reported to the local DOH authorities.11 Most clinical practices have mechanisms and staff who handle this reporting, and in some cases even the lab services will forward results to the DOH. As a clinician, however, do not assume this happens. Ensure someone in the clinical facility is reporting positive confirmatory syphilis test results, and if not, the responsibility falls to the PCP. Each state has different mechanisms for this, often having an online system on which patient information can be confidentially entered. 10. Interprofessional Collaboration L L L L

Receptionist registers the patient. MA takes initial complaint and vital signs and files report with the public health department. PCP gathers history/physical and formulates differential diagnosis and treatment. Public health official performs epidemiologic studies/review.

Outcome

This patient’s symptoms resolved shortly after the administration of intramuscular penicillin. Syphilis EIA was positive, and the RPR was positive at a titer of 1:32. The remainder of STI testing, including an HIV test, was negative. Follow-up RPR testing at 3 months was 1:2; at 6 and 12 months, the RPR was negative. The patient contacted the sexual partner from the party he had attended a few months ago, whom he had dated briefly afterward, and recommended that he seek medical attention to be tested and treated. The patient continued taking HIV PrEP, and subsequent HIV and STI screenings remained negative.

Insight

from the

PCP

Patients presenting with mononucleosis syndrome are a diagnostic challenge. This patient presents with the mononucleosis syndrome, a constellation of signs and symptoms that may include any combination of fever, chills, fatigue, swollen lymph nodes, and cough. The differential diagnosis of the mononucleosis syndrome is quite broad and includes many etiologies, both infectious (viral, bacterial, fungal, protozoal) and noninfectious (autoimmune/rheumatologic diseases, drug reactions, malignancies, particularly lymphoma). While many, if not most, episodes of the mononucleosis syndrome are viral in etiology (including acute EBV infection) and there is no specific treatment, it is important to remember there are many other etiologies on the differential that do have available treatments. With this in mind, when evaluating a patient with an unexplained febrile illness or the mononucleosis syndrome, I always take the time to obtain a comprehensive history and physical exam to uncover symptoms, signs, and historical clues that could point to a specific etiology that can be evaluated and treated. In my initial interaction with this patient, that is how I explained the process is the symptoms he presents, the broad differential diagnosis, and how I ask the specific questions to narrow down the differential and formulate a most likely diagnosis. Of course, oftentimes the ultimate diagnosis is a virus for which there is no treatment or a specific etiology goes undetermined. However, what drives the investigation is to try to find something else we can treat.

In this instance, the new rash was instrumental in the diagnosis. This led to an in-depth discussion of the sexual history, which must be done in a straightforward and nonjudgmental manner. As many patients find some lines of questioning unusual or as a breach of privacy, I find that setting the stage early on in an encounter and making it clear that my goal is to figure out something that we can treat becomes crucial. This patient later told me that he never would have volunteered his recent sexual encounter, which he did not associate with his current illness, had I not asked. He appreciated how matter of fact I was with my questioning, particularly as it directly related to my likely diagnosis. This was instrumental to building trust and rapport.

References

1. Radolf JD, Tramont EC, Salazar JC. Chapter 237: Syphilis. In: Mandell, Douglas, and Bennett’s Principles 2. 3. 4. 5. 6. 7. 8. 9.

10. 11.

and Practice of Infectious Diseases. 9th ed. Elsevier; 2015:2865–2892. https://www.elsevier.com/books/ mandell-douglas-and-bennetts-principles-and-practice-of-infectious-diseases/bennett/978-0-323-48255-4 Centers for Disease Control and Prevention. Syphilis testing algorithms using treponemal tests for initial screening – four laboratories, New York City, 2005–2006. MMWR Morb Mortal Wkly Rep. 2008;57(32):872. doi:10.15585/ mmwr.mm6739a3 Sena AC, White BL, Sparling PF. Novel treponema pallidum serologic tests: a paradigm shift in syphilis screening for the 21st century. Clin Infect Dis. 2010;51:700. Richardson D, Fitzpatrick C, Finnerty F, et al. Symptomatic secondary syphilis: empirical antimicrobial treatment or await microbiology? Sex Health. October 31, 2019;16(6):598. doi:10.1071/sh19079 Bonnewell J, Magaziner S, Fava JL, et al. A survey of syphilis knowledge among medical providers and students in Rhode Island. SAGE Open Medicine. January 2020. doi:10.1177/2050312120902591 Workowski KA, Bolan GA. Centers for Disease Control and Prevention sexually transmitted diseases treatment guidelines. MMWR Morb Mortal Wkly Rep. 2015;64:1–137. Centers for Disease Control and Prevention. Screening recommendations and considerations referenced in treatment guidelines and original sources. https://www.cdc.gov/std/tg2015/screening-recommendations.htm Centers for Disease Control and Prevention. Recommended immunization schedule for adults aged 19 years or older by medical conditions and other indications. https://www.cdc.gov/vaccines/schedules/hcp/imz/adultconditions.html Centers for Disease Control and Prevention. Preexposure prophylaxis for the prevention of HIV infection in the United States – 2017. Update: a clinical practice guideline. https://www.cdc.gov/hiv/pdf/risk/prep/cdc-hiv-prepguidelines-2017.pdf Schmidt R, Carson PJ, Jansen RJ. Resurgence of syphilis in the United States: an assessment of contributing factors. Infect Dis Res Treat. 2019;12:1–9. Centers for Disease Control and Prevention. 2018 STI surveillance report. Content source: Division of STD Prevention, National Center for HIV/AIDS, Viral Hepatitis, STD, and TB Prevention, Centers for Disease Control and Prevention. https://www.cdc.gov/std/stats18/default.htm

EXTREME WEAKNESS, ADULT MALE

CASE

88

Chief Complaint

“Extreme weakness.”

History

Present Illness

of

A 22-year-old Vietnamese male was carried into his PCP’s office by two male family members. The family members told the nurse the patient is weak and unable to walk on his own. The patient had immigrated to the United States from Vietnam 2 years ago and has no previously diagnosed major medical problems. His weakness started yesterday, initially as muscle cramps, which gradually progressed to generalized muscle weakness. He denied any muscle pain and has had no associated symptoms. He denied any injury and any psychiatric problems. The patient mentioned that the weakness started in his lower extremities first and then gradually progressed to his upper extremities. He denied any breathing problems. The patient also stated that he was at a birthday party the day before his symptoms started and he ate a large meal containing rice.

Review

of

Systems

The patient’s ROS is positive for generalized muscle weakness, fatigue, and muscle cramps; he is unable to walk. His ROS is negative for any recent weight loss, fever, chills, nausea, vomiting, headaches or URI symptoms, SOB, chest pain, or abdominal pain. The review is also negative for any urinary symptoms or mood changes or depression.

Relevant History

The patient is Vietnamese and has only been in the United States a few years. He has occasional mild seasonal asthma. He smokes half a pack of cigarettes a day. Once a week, he has one or two beers. He denies any THC or illicit drug use. He lives with his older brother in an apartment. He works as a store clerk at a convenience store and has no unusual stress at work. He has no girlfriend at this time and has not been sexually active for a few months. His father had a heart attack at age 50; his mother is healthy. Both live in Vietnam. His older brother had been hospitalized for an unknown medical condition.

Allergies

No known drug or food allergies.

Medications None.

354  Case 88  •  Extreme Weakness, Adult Male

Physical Examination Vitals: T 98.2°F (36.8°C), BP 128/74, P 110, R 12, SPO2 98%, WT 72.57 kg (160 lbs), HT 172.7 cm (68 in.), BMI 24.3. General: Appeared anxious and weak but in no other distress. Nontoxic. Psychiatric: Normal mood and affect. Skin, Hair, and Nails: Clear with no rashes, lesions, or petechiae or purpura. No abnormal findings with hair or nails. ENT/Mouth: TM clear bilaterally, no discharge in the ears, oral mucosa moist, no pharyngeal erythema. Neck: Soft and supple with no nuchal rigidity or thyromegaly. Chest: Adequate and symmetric chest expansion. Lungs: Clear bilaterally with no wheezing, rales, or rhonchi. Heart: Mild tachycardia but regular with no murmurs or gallops. Abdomen: Soft. Non-tender. No palpable masses or organomegaly. Bowel sounds normal. Lymphatic: No palpable lymphadenopathy symmetrically. Musculoskeletal: Patient had diffuse muscle weakness at 2/5 of all his large muscle groups in all extremities. No muscle tenderness. He was unable to walk due to generalized weakness. Neurologic: A&O×3. Cranial nerves II to XII grossly normal. No detectable sensory deficits. DTRs 1+ bilaterally; decreased symmetrically.

Clinical Discussion Questions 1. What is the differential diagnosis?

2. What is the most likely diagnosis? Why?

3. Demonstrate your understanding about the pathophysiology in regard to the most likely diagnosis.

Case 88  •  Extreme Weakness, Adult Male 355 4. Should tests/imaging studies be ordered? Which ones? Why? Think about tests/imaging beyond the

primary care setting as well.

5. What are the next appropriate steps in management?

6. What are the triggers, associated conditions, and treatment approaches for the diagnosis? Provide

references for your responses.

7. What are the pertinent ICD-10 and CPT (E/M) codes for this visit? Provide a short rationale.

8. What is the appropriate patient education for this case?

356  Case 88  •  Extreme Weakness, Adult Male 9. If not managed appropriately, what is/are the medical/legal concern(s) that may arise?

10. Think about interprofessional collaboration for this case. Provide a list of specialties or other disciplines

and indicate what contribution these professionals might make to managing the patient.

Bedside Manner Question

11. What would your communication style/approach be with this patient?

ANSWER KEY: EXTREME WEAKNESS, ADULT MALE

CASE

88

1. Differential Diagnosis L

L

L

L

L

L

Hyper/hypokalemia is well known to cause generalized muscle weakness. These would be high on the differential given the patient’s extreme presentation. Primary hypothyroidism may cause periodic hypokalemic paralysis leading to generalized muscle weakness and could be the cause of his symptoms. Infection is unlikely here. The patient has no history of fevers or chills or other clinical signs or presentations to suggest an infectious process contributing to his muscle weakness. Guillain-Barré syndrome is also unlikely. The patient has had no neurologic complaints, such as initial tingling of fingers and toes, facial muscle weakness, prior infection with campylobacter, influenza, CMV, RBV or mononucleosis, mycoplasma pneumonia, to suggest Guillain Barré syndrome. In addition, the patient also has not had a typical pattern of ascending paralysis, which is commonly seen in Guillain-Barré syndrome. Botulism is also unlikely. The patient has not experienced weakness of the muscles that control the eyes, face, mouth, and throat; nausea; vomiting; or diarrhea with subsequent constipation and abdominal distention. Spinal cord injury is also unlikely because there is no report of a recent or remote trauma.

2. Most Likely Diagnosis Hypokalemia. A metabolic and/or endocrine disorder should be considered. Patients with Asian ethnicity have a higher genetic propensity to get TPP that is closely associated with HPP. Therefore, it would be reasonable to obtain a STAT CMP and a CBC as a starting point before going further into sophisticated testing. 3. Pathophysiology Hypokalemia has multiple possible pathophysiologic mechanisms. Generally, the etiology may be arrived at by considering the three broad causes of hypokalemia: 1. Inadequate intake of K+ 2. Increased excretion of K+ 3. A shift of K+ from extracellular space into the intracellular space HPP is one of the sub-etiologies of K+ shift from extracellular space into the intracellular space, causing a relative hypokalemia. This is a result of genetic susceptibility of these patients get hypokalemia secondary to TPP. Patients with Asian ethnicity have a higher genetic propensity to get TPP that is closely associated with HPP. TPP has elevated T3, which regulates K+ channels. Since this entity is a result of K+ shift from extracellular space to the intracellular space and does not reflect a true K+ deficit, total body K+ is not necessarily low. Therefore, K+ replacement must be approached very cautiously. Overcorrecting K+ level in HPP may cause hyperkalemia.1 4. Diagnostic Tests/Images Primary Care Setting L CBC (to screen for anemia and/or infection) L CMP: Na+, K+, Ca++, Mg++ levels, BG, creatinine, LFTs (to evaluate muscle weakness electrolytes, kidney function, and liver function)

Results CBC within normal limit L CMP shows K+: 2.6 (all other chemistry results within normal limit) L

Emergency Department EKG to evaluate any cardiac abnormality L Thyroid panel to evaluate thyroid and acute thyroid conditions L CBC to screen infection L

Results EKG: prominent U waves, long QT interval L TSH: 0.3 mIU/L (reference range 0.4–4 mIU/L) L T3: 220 ng/dL (reference range 80–180 ng/dL) L CBC: normal, T4 (serum thyroxine): 24 mcg/dL (reference range 4.6–12 mcg/dL) L Note: CBC was ordered at both primary care and ED. L

5. Next Steps With the finding of significantly low K+ levels and the extreme muscle weakness that prevented him from walking or standing, the patient was transferred by ambulance to the ED at the nearby hospital for further evaluation and management. 6. Triggers, Associated Conditions, and Treatment Approaches L

L

L

L

 PP may be triggered by a large meal rich in carbohydrates, following strenuous exercise (due H to release of endogenous insulin and relative influx of potassium into the intracellular space), high sodium meals, emotional stress, alcohol ingestion, drugs, and traumatic events.2–4 This patient had a large meal containing rice the day before developing symptoms. HPP should always be considered as a consequence of hyperthyroidism. Whenever a low serum potassium level is noticed, thyroid function tests should be included to rule out the existence of hyperthyroidism.5,6 Because the primary physiologic mechanism is a shift of K+ from extracellular compartment into the intracellular compartment, total body K+ is not depleted. Therefore, acute treatment (oral and intravenous potassium) must be managed cautiously and slowly to treat with K+ with close monitoring to avoid a rebound with hyperkalemia, which may be life threatening.7,8 Treatment of thyrotoxicosis in these patients is the definitive therapy, with the treatment of hypokalemia as the secondary treatment.9,10 In addition to medications (propranolol, other treatment for thyrotoxicosis and hypokalemia), patients with TPP and HPP should avoid high-carbohydrate meals and strenuous physical activity until the they become euthyroid. Euthyroid patients do not get TPP, which then avoids HPP.2

7. ICD-10 and CPT (E/M) Codes ICD-10 Code: M62.81 Muscle weakness. ICD-10 Code: G72.3 Periodic paralysis. ICD-10 Code: E87.6 Hypokalemia. This is the initial diagnosis at the primary care visit. The patient is having muscle weakness, periodic paralysis, and hypokalemia. Consequently, you include the previously mentioned ICD-10 coding.

CPT (E/M) Code: Level 4—99214 (Established Patient) This is a primary care visit for the evaluation and management of an established patient. According to the case, the patient has a new complaint with a possibility for major health complications if untreated or misdiagnosed. This criterion meets 99214, the appropriate level for this clinical scenario. 8. Patient Education Topics L L L L L

What is hypokalemia? Why the transfer to ED is necessary Genetic counseling for the patient and family Avoidance of large carbohydrate meals Management of hyperthyroid condition

9. Medical or Legal Concerns The most concerning legal issue is missing the diagnosis of hypokalemia. It’s possible that severe, potassium disorders can lead to life-threatening cardiac conduction disturbances and neuromuscular dysfunction.11 Consequently, PCPs need to recognize the seriousness of the condition and start the next step instantly. This patient’s generalized muscle weakness can lead to multiple differentials, making it a challenging case. However, his clinical presentation, relevant history, and physical examination suggest that something serious may be happening. At that time, PCPs have a few different ways to handle it: (a) sending the patient to an ED for further evaluation, (b) do the STAT lab (if available) and then take the next step based on the lab results, or (c) do the labs and then keep an eye on the lab results and have a follow-up with the patient on the next day. These decisions (a, b, or c) are based on the patient’s clinical presentation and the resources available (i.e., STAT lab) at the clinic setting. The most important lesson is to recognize, act fast, and continue to follow the patient. 10. Interprofessional Collaboration L

L L

L L L L

L L

Outcome

Receptionist registers the patient and notifies appropriate staff that patient has been carried into the office by family. MA takes a brief history and vital signs. PCP examines the patient and formulates likely diagnoses, explains the need for the family to take the patient to the ED, arranges transfer to the ED and consults with the on-call physician, provides reassurance to the family, and interfaces with the EMS staff to transfer the patient to the ED. EMS staff transfer patient to ED. ED triage nurse triages patient. ED nurse takes history and vitals. ED physician confirms diagnosis, admits patient, and arranges urgent consult with endocrinologist. Endocrinologist develops treatment plan and follows patient. Nursing staff provide care to the patient while in the hospital.

The patient was further evaluated in the ED. HPP is usually episodic and is present with thyrotoxicosis. It is common among Asian males. The actual mechanism causing this type of hypokalemic paralysis is not clear. However, it may be due to increased Na-K-ATPase activity, which is present in patients with thyrotoxicosis and paralysis.12

This patient needed cardiac monitoring and an EKG. K+ replacement was done slowly via intravenous and oral administration with close monitoring to avoid hyperkalemia and rebound hypokalemia.1 Because this patient also has concurrent hyperthyroid findings, propranolol was given as the first-line treatment to treat hyperthyroidism and to prevent rebound hyperkalemia.9 Then he was recommended follow-up with his PCP and an endocrinologist for other modalities of treatment of hyperthyroidism, such as methimazole.10

Insight

from the

PCP

Although a patient presenting with generalized muscle weakness has multiple possible diagnoses, a careful history, including a family history, and a complete physical examination will often provide sufficient clues to build a reasonable differential diagnosis. A few basic laboratory tests (done immediately as a STAT order, which may not be available at every primary care clinic) may provide a working diagnosis to move forward with evaluation and management of such a patient. This patient immigrated from Vietnam. His older brother had been hospitalized for an unknown medical condition. With no evidence to support an infectious, rheumatologic, neurologic, or a traumatic etiology, I suspected a metabolic cause, considering his ethnicity. Finding hypokalemia in these patients should also include a consideration of concurrent hyperthyroidism. His mild tachycardia and anxiety may support the latter. However, his generalized muscle paralysis may have also triggered anxiety and a tachycardic response. Because these conditions are treatable, we were able to reassure the patient and his family and allay their anxieties. The patient was treated successfully with oral and slow IV K+ and was also started on medications to counter his hyperthyroid activity. He was referred to an endocrinologist for further evaluation and management.

References

1. Vijayakumar A, Ashwath G, Thimmappa D. Thyrotoxic periodic paralysis: clinical challenges. J Thyroid Res.

2014;2014:1–6. doi:10.1155/2014/649502

2. Yu TS, Tseng CF, Chuang YY, et al. Potassium chloride supplementation alone may not improve hypokalemia 3. 4. 5. 6. 7. 8. 9. 10. 11. 12.

in thyrotoxic hypokalemic periodic paralysis. J Emerg Med. April 2007;32(3):263–265. doi:10.1016/j. jemermed.2006.06.009 Fontaine B, Lapie P, Plassart E, et al. Periodic paralysis and voltage-gated ion channels. Kidney Int. 1996;49:9. doi:10.1038/ki.1996.2 Vijayakumar A, Ashwath G, Thimmappa D. Thyrotoxic periodic paralysis: clinical challenges. J Thyroid Res. 2014;2014:649502. doi:10.1155/2014/649502 Neki NS. Hyperthyroid hypokalemic periodic paralysis. Pak J Med Sci. July–August 2016;32(4):1051–1052. doi:10.12669/pjms.324.11006 Correia M, Darocki M, Hirashima ET. Changing management guidelines in thyrotoxic hypokalemic periodic paralysis. J. Emerg Med. August 2018;55(2):252–256. doi:10.1016/j.jemermed.2018.04.063 Thornton MD. Lower-extremity weakness in a teenager due to thyrotoxic periodic paralysis. J Emerg Med. April 2017;52(4):e133–e137. doi:10.1016/j.jemermed.2016.11.006 Tachamo N, Lohani S, Nazir S, Juliano N. Paralysis that easily reverses: a case of thyrotoxic periodic paralysis. BMJ Case Rep. 2017 Jan 30;2017:bcr2016218951. doi:10.1136/bcr-2016-218951 Lin SH, Lin YF. Propranolol rapidly reverses paralysis, hypokalemia, and hypophosphatemia in thyrotoxic periodic paralysis. Am J Kidney Dis. March 2001;37(3):620–623. doi:10.1053/ajkd.2001.22090 Lee SL. Hyperthyroidism and thyrotoxicosis treatment & management. Medscape. March 15, 2018. https:// emedicine.medscape.com/article/121865-overview. Last visited Oct 5 2020. Viera AJ, Wouk N. Potassium disorders: hypokalemia and hyperkalemia. Am Fam Physician. 2015 September 15;92(6):487–495. Lederer E, Alsauskas ZY, Mackelaite L, et al. What is thyrotoxic periodic paralysis (TTPP)? Medscape. December 2018. https://www.medscape.com/answers/242008-160467/what-is-thyrotoxic-periodic-paralysis-ttpp. Last visited Oct 5 2020.

LEG PAIN, GERIATRIC MALE

CASE

89

CHIEF COMPLAINT “Leg pain.”

HISTORY

OF

PRESENT ILLNESS

A 74-year-old man, well established in this primary care clinic, presents complaining of leg pain that has been bothering him for the past few months. He states that the 4/10, crampy, calf pain in both legs occurs about 15 minutes into his twice-daily walks with his dog. The pain only goes away when he stops walking and sits for about 5 minutes. Once he starts walking again, the pain returns just a few minutes later. He also states that his legs feel weak. He reports pain relief upon elevating his legs while relaxing in his recliner but reports he sometimes cannot sleep at night as his legs start cramping when he lies down. He feels he cannot hold them still because the pain at times becomes an intense 6/10 to 7/10 until he falls asleep. He denies taking any medications for this pain but states standing in one place for an extended period, such as when fishing, also makes the pain return.

REVIEW

OF

SYSTEMS

The patient’s ROS is positive for leg cramps and weakness, varicose veins, and arthritis in hands and feet. His ROS is negative for fainting, blackouts, seizures, weakness, paralysis, tingling, tremors, or erectile dysfunction. He denies chest pain, palpitations, dyspnea at rest or upon exertion, orthopnea, paroxysmal nocturnal dyspnea, edema, and any recent trauma to his lower extremities.

RELEVANT HISTORY

The patient has hypercholesterolemia, hypertension, and bilateral cataracts with lens implants. The patient is a retired engineer, happily married to his wife of 54 years. He is a former tobacco abuser, 2 packs per day for 30 years; he quit at age 48. He enjoys two martinis with dinner each evening and denies recreational or illicit drug use ever. His family history is significant for lung cancer, acute MI, hyperlipidemia, and hypertension.

ALLERGIES

No known drug allergies; no known food allergies.

MEDICATIONS ● ●

Atorvastatin 40 mg PO QHS Amlodipine 5 mg PO QD

PHYSICAL EXAMINATION Vitals: T 36.9°C (98.4°F), P 78, R 14, BP 146/88, HT 178 cm (70 in.), WT 82.6 kg (182 lbs), BMI 26.1. General: Pleasant male of stated age sitting comfortably on the examination table, in no acute distress. Makes good eye contact, converses with ease, makes jokes. A&O. Skin, Hair, and Nails: Tight, thin, shiny, atrophied skin, slightly dusky red/ruborous color, overlying dorsum of mid-feet to include all toes, extending proximally and circumferentially to

358  Case 89  •  Leg Pain, Geriatric Male

bilateral ankles and knees. Bald, slightly cool to the touch bilaterally and symmetrically. No lesions or masses. Thin, short brittle nails bilateral feet. Head: Atraumatic, normocephalic. Neck: Trachea midline, no masses or lymphadenopathy. Lungs: Clear to auscultation bilaterally without wheezes or rales. Heart: RRR; no murmurs, rubs, or gallops. Peripheral Vascular: Carotid pulses 2+ bilaterally; no thrills or bruits. Distal upper extremity pulses 2+ and symmetric bilaterally, capillary refill 38.0°F), or (young or old modifier) uses a point system to assign likelihood and guide management of suspected strep pharyngitis.1 3. Pathophysiology L

Streptococcus is a gram-positive coccus, belonging to the Streptococcaceae family. The members of the genus Streptococcus cause health problems such as pharyngitis, pneumonia, neonatal meningitis, sepsis, endocarditis, bacteremia, and UTIs.2 Streptococci are further categorized based on the inherent hemolytic properties (alpha-hemolytic or beta-hemolytic). The beta-hemolytic group of Streptococci is further classified into several serotypes using the Lancefield grouping system based on the presence or absence of specific carbohydrate moieties on the bacterial cell wall. Of the different serotypes, Lancefield group A (group A streptococcus) and Lancefield group B (group B streptococcus) are important in medical settings based on their ability to cause life-threatening diseases. Lancefield groups A and B (also known as group A strep and group B strep) are also considered to be highly relevant in clinical and biomedical setups.2

L

Group A strep pharyngitis is caused by Streptococcus pyogenes, a gram-positive coccus. It exhibits beta-hemolysis (complete hemolysis) and belongs to group A in the Lancefield classification system for beta-hemolytic Streptococcus and thus is called group A streptococci.3 Group A strep pharyngitis has direct person-to-person transmission through saliva or nasal secretions. Spread of group A strep infections rarely occurs through food.3 Humans are the main reservoir for group A strep, and there is no evidence to indicate pets can transmit the bacteria to humans.3

4. Diagnostic Tests/Images The patient met the CENTOR criteria of exudate, anterior cervical adenopathy, fever, and absence of cough. Therefore, a rapid strep antigen test could have been deferred and management initiated based upon clinical diagnosis.1 However, the test was performed to properly confirm the diagnosis. Result Rapid antigen strep test: Positive. 5. Next Steps L

L L

L

L

The patient was started on amoxicillin for 10 days for strep pharyngitis. Penicillin V potassium was considered; however, amoxicillin was initiated primarily related to acceptance of the taste of the suspension. Ibuprofen and acetaminophen were prescribed for fever and pain control. The patient’s mother was advised to get a pediatric-formula electrolyte drink and sugar-free popsicles for hydration. The mother was advised to bring the child back in 1 week or sooner if the symptoms worsen or if there is no improvement. ED precautions were discussed.

6. Most Common Cause, Treatment, and Risks Associated With Misdiagnosis L

L

L

GAS is a common cause of bacterial pharyngitis in children aged 5 to 15. The illness usually peaks during winter and early spring and makes up 35% to 40% cases of pharyngitis in children.4 The mainstay of treatment of GAS is penicillin as a first-line agent. If a penicillin allergy is suspected or present, the first-generation cephalosporin or macrolide is the next best choice. The treatment should also be directed toward comfort and symptom control. An antipyretic such as acetaminophen should be prescribed; ibuprofen can also be added for tissue inflammation associated with GAS. Aspirin should not be prescribed to children because of Reye syndrome. Steroids have no role in treatment of GAS.5 Appropriate diagnosis and treatment of GAS are important because of the risk of developing rheumatic fever and acute glomerulonephritis.4 Untreated GAS has associated delayed complications due to cross-reactivity of the streptococcal antigen with a patient’s tissue. A major complication in approximately 3% of this patient population is rheumatic fever, which can result in cardiac complications.2 PSGN is another sequela of GAS and can occur after a patient has been treated. PSGN is an immune-mediated nephritis syndrome that occurs in 10% of patients following 1 to 3 weeks postinfection. Signs and symptoms of PSGN are facial and orbital edema, hypertension, hematuria, proteinuria, weakness, and lethargy.6

7. ICD-10 and CPT (E/M) Codes ICD-10 Code: J02.0 Streptococcal pharyngitis. This is the final diagnosis for this visit. The diagnosis of strep pharyngitis relies on physical exam and often on throat swab. In this case, the patient presented with typical physical exam signs such as fever, fussiness, beefy red tonsillar pharyngeal area with white exudate, as well as cervical adenopathy, and the rapid strep test was positive.

CPT (E/M) Code: Level 3—99213 (Established Patient) Pharyngitis is considered a simple visit for diagnosis and treatment for sore throat. Most of the time, a physical exam suffices to determine the proper diagnosis while other times a throat culture may be necessary to ensure appropriate use of antibiotics. 8. Patient Education Topics L L L L

Strep throat: what you need to know How to control fever at home Supportive care at home for strep throat Appropriate antibiotic use

9. Medical or Legal Concerns It is important that PCPs differentiate streptococcal pharyngitis from other types of pharyngitis. It is easy to treat, but if unrecognized or untreated a patient may develop rheumatic fever. Recognizing and initiating treatment for GAS is important. Treatment of the infected patient reduces the circulation of GAS in a household, day care, or school populations.5 Even if the GAS is treated, PSGN can still occur. Therefore, preventing GAS from transmitting to others is very important. Parents/family should be educated regarding the importance of isolation and infection control measures to prevent transmission to other household members, and this education should be documented. 10. Interprofessional Collaboration L L L L L

Receptionist registers the patient. MA takes vital signs and a brief history. Nurse takes throat swab for rapid strep test. PCP formulates diagnosis and treats the patient. Pharmacist dispenses medication.

Outcome

The patient was successfully treated with a 10-day course of amoxicillin for infection and acetaminophen and ibuprofen for fever and inflammation. The patient was at his baseline at the follow-up visit.

Insight

from the

PCP

This mother and child came in as a walk-in patient at their regular PCP office. Although I was not the PCP for this patient, I evaluated the patient because his pediatrician was on vacation. The first thing I noticed as I entered the room was that the mother was on the edge of her seat and seemed concerned and disheveled as if she had not slept at all. Then I looked at the child, who was hugging his mother closely and was fussy and unhappy. I reviewed the patient’s vitals and immunization status and did a thorough physical exam while going through several differentials in my head. Once I looked in his mouth, I knew exactly what the most likely diagnosis was. That unmistakable look of beefy, swollen tonsillopharyngeal area with white exudates highly indicated streptococcal pharyngitis and was confirmed with a rapid strep test. I took some time explaining the diagnosis and the treatment plan. I also educated the mother about the disease prognosis and how to control dehydration and fussiness, as well as emergency and return precautions. I took some time to answer all her questions and perceived a sense of relief on her part. The office staff made a 1-week follow-up appointment but instructed his mother to return sooner if her child’s condition worsened. The mother and child returned on day 6 of the treatment for follow-up. The boy was cheerful and interactive with me, and the mother was happy to report he was eating and sleeping. On physical exam, there was a little residual erythema around the tonsils but the rest of the exam was back to baseline. The mother was thankful for how I took time to communicate the diagnosis and treatment to calm her anxiety and worries. Again, good communication is the key to quality care.

References

1. Roggen I, van Berlaer G, Gordts F, CENTOR criteria in children in a pediatric emergency department: for what it is

worth. BMJ Open. 2013;3:e002712. doi:10.1136/bmjopen-2013-002712

2. Pattnaik S, Syed A, Siddhardha B. Pathogenesis, virulence factors, and antibiotic resistance of group B 3. 4. 5. 6.

streptococcus. In: Siddhardha B, Dyavaiah M, Syed A, eds. Model Organisms for Microbial Pathogenesis, Biofilm Formation and Antimicrobial Drug Discovery. Springer Publishing Company; 2020. Centers for Disease Control and Prevention. Group A streptococcal (GAS) disease: pharyngitis (strep throat). https://www.cdc.gov/groupastrep/diseases-hcp/strep-throat.html#figure1. Wald ER. Group A streptococcal tonsillopharyngitisin children and adolescents: clinical features and diagnosis. 2019. https://www.uptodate.com/contents/group-a-streptococcal-tonsillopharyngitis-in-children-and-adolescentsclinical-features-and-diagnosis Lustig LR, Schindler JS. Ear, nose and throat disorders. In: Papadakis MA, Mcphee SJ, Rabow MW, eds. Current Medical Diagnosis and Treatment. McGraw-Hill; 2019. http://accessmedicine.mhmedical.com/content. aspx?bookid=2449§ionid=194433886 National Center for Immunization and Respiratory Diseases, Division of Bacterial Diseases. Post streptococcal glomerulonephritis. https://www.cdc.gov/groupastrep/diseases-hcp/post-streptococcal.html

UNEXPLAINED WEIGHT GAIN, ADULT FEMALE

CASE

96

Chief CoMPlaint

“Unexplained weight gain.”

historY

of

Present illness

A 34-year-old Caucasian woman presents to her PCP with a 1-year history of weight gain despite efforts to eat healthy and increase physical activity. She is married and a mother of three children, 13-year-old twin boys and a 6-year-old girl. She states she did not pay attention to the gradual weight gain until about 4 months ago, when she began making efforts to lose weight to include walking and eating less fast food.

reVieW

of

sYsteMs

The patient’s ROS is positive for a 25-lb weight gain over the past year. Her last menstrual period was 2 weeks ago. Her ROS is negative for fatigue, dry skin, acne, fever, thinning hair, hypertension, swelling, weakness, reduced exercise tolerance, abdominal pain or distension, constipation, excessive hair growth, nipple discharge, anxiety, depression, substance abuse concerns, cold intolerance, difficulty sleeping, snoring, joint pain, polydipsia, or polyuria.

releVant historY

The patient’s relevant history includes vitamin D deficiency. She smokes half a pack of cigarettes per day. The patient admits she is less active than she used to be and consumes a diet high in carbohydrates. She denies alcohol or illicit drug use. She lives with her husband and describes it as stressful. Her family history is positive for type 2 diabetes mellitus, hypertension, hyperlipidemia, and early heart disease.

allergies

No known drug allergies; no known food allergies.

MeDiCations L L

OTC multivitamin daily. Vitamin D 50,000 IU weekly.

PhYsiCal eXaMination Vitals: T 36.7°C (98.1°F), P 70, R 18, BP 116/77, HT 170 cm (67 in.), WT 93 kg (205 lbs), BMI 32.1. General: Obese and well groomed in no acute distress. Psychiatric: Normal mood and affect. Skin, Hair, and Nails: Skin warm and dry with no excessive facial hair or rash. No abnormal findings with nails. Neck: Thyroid is normal size; consistency and is non-tender. Lungs: Clear to auscultation bilaterally.

386  Case 96  •  Unexplained Weight Gain, Adult Female

Heart: RRR, radial pulse 2+ bilaterally. Lymphatic: Nonpalpable cervical nodes Abdomen: Rounded, non-distended, soft, non-tender, normal bowel sounds. Musculoskeletal: No extremity edema. FROM both upper and lower extremities, bilateral. Neurologic: A&O×3 without tics or tremors.

Clinical Discussion Questions 1. What is the differential diagnosis?

2. What is the most likely diagnosis? Why?

3. Demonstrate your understanding about the pathophysiology in regard to the most likely diagnosis.

4. Should tests/imaging studies be ordered? Which ones? Why? Think about tests/imaging beyond the

primary care setting as well.

Case 96  •  Unexplained Weight Gain, Adult Female 387 5. What are the next appropriate steps in management?

6. What are the diagnostic criteria and risk factors for this diagnosis? Provide references for your response.

7. What are the pertinent ICD-10 and CPT (E/M) codes for this visit? Provide a short rationale.

8. What is the appropriate patient education for this case?

9. If not managed appropriately, what is/are the medical/legal concern(s) that may arise?

388  Case 96  •  Unexplained Weight Gain, Adult Female 10. Think about interprofessional collaboration for this case. Provide a list of specialties or other disciplines

and indicate what contribution these professionals might make to managing the patient.

Bedside Manner Questions

11. What would your communication style/approach be with this patient?

12. If the patient shows distress at what you communicate, how would you provide support?

ANSWER KEY: UNEXPLAINED WEIGHT GAIN, ADULT FEMALE

CASE

96

1. Differential Diagnosis L

L

L

L

L

Hypothyroidism should be considered with weight gain of unknown origin because symptoms are not always noticeable in the early stages. While the patient denies hypothyroidism symptoms such as fatigue, sensitivity to cold, constipation, dry skin, or thinning hair, this diagnosis would need to be considered until thyroid hormone studies confirm or exclude this diagnosis. Polycystic ovarian syndrome could be considered due to weight gain of unknown origin. However, this diagnosis can reasonably be excluded because the patient has regular periods and has no acne, excessive hair growth, or thinning hair, all characteristics of PCOS. Metabolic syndrome is high on the differential list because the patient is obese and has a history of inactivity with increased central adiposity. Depressive-anxiety disorders are a possible differential diagnosis here. Depressive-anxiety disorders may lead to weight gain through poor eating habits, physical inactivity, and noncompliance with lifestyle modifications, which could result in adiposity. The patient indicates her life is stressful. However, she has been trying to eat healthy and do exercise. During the physical exam, she demonstrates normal mood and affect, suggesting this is unlikely. Pregnancy is unlikely as she has been gaining weight for about 12 months; however, it should be ruled out with any woman of childbearing age with unexplained weight gain.

2. Most Likely Diagnosis Metabolic syndrome. Given the patient’s presentation and family history of type 2 diabetes mellitus, hypertension, hyperlipidemia, and early heart disease, metabolic syndrome should be considered the likely diagnosis. Further tests and lipid profile are needed. 3. Pathophysiology The pathophysiology of metabolic syndrome is complex and not fully understood yet does involve both genetic and acquired factors.1 Metabolic syndrome represents a cluster of clinical conditions, including central obesity, hypertension, insulin resistance, and dyslipidemia, and is commonly associated with cardiovascular disease, gout, kidney disease, and obstructive sleep apnea.2 It is the result of a prothrombic and proinflammatory state characterized by increased inflammatory cytokine activity.2 Adipose tissue is central to its pathophysiology as “adipose tissue is now considered a biologically active endocrine and paracrine organ.”2 “Visceral adiposity has been demonstrated to be a primary trigger for most of the pathways involved in metabolic syndrome, thus stressing the importance of a high caloric intake as a major causative factor.”1 4. Diagnostic Tests/Images L L L L L

Waist circumference to assess for metabolic syndrome Point-of-care urine qualitative pregnancy test to rule out pregnancy CMP to test for fasting blood sugar, electrolytes, kidney function, and liver enzymes Fasting lipid profile to assess cholesterol TSH to assess for thyroid dysfunction

Results Waist circumference 93 cm (36.5 in.) L Point-of-care urine qualitative pregnancy test negative L Fasting glucose 112 mg/dL L HDL 37 mg/dL, triglycerides 170 mg/dL L Total cholesterol, LDL, and TSH within normal limits L

5. Next Steps This patient was asked to return in 1 week to discuss her lab results. 6. Diagnostic Criteria and Risk Factors L

L

Metabolic syndrome is also referred to as syndrome X, the insulin resistance syndrome, the deadly quartet, or the obesity dyslipidemia syndrome.3 It has gained significant focus due to the exponential worldwide increase in obesity.1 The various names and the differing criteria from leading organizations over the years make comparing data from studies difficult. In an attempt to create consistent diagnostic criteria for metabolic syndrome, the American Heart Association, the World Heart Federation, the International Association for the Study of Obesity, the International Atherosclerosis Society, the International Diabetes Federation, and the National Heart, Lung, and Blood Institute have agreed on five criteria to diagnose metabolic syndrome4 M Waist circumference over 40 in. (men) or 35 in. (women) M Triglycerides ≥ 150 mg/dL, HDL < 40 mg/dL for men and < 50 mg/dL for women (or current treatment for low HDL) M Systolic BP ≥ 130, diastolic BP ≥ 85 (or current treatment for hypertension) M Fasting glucose of ≥ 100 mg/dL (or previously diagnosed diabetes) The presence of any three or more items is diagnostic for metabolic syndrome. This condition is a proinflammatory state that increases the risk of cardiovascular disease and type 2 DM.5 A large waist circumference is a major risk factor and can identify “up to 46% of individuals who will develop metabolic syndrome within 5 years.”6 Other risk factors for metabolic syndrome include age, race, postmenopausal status, smoking, low household income, high carbohydrate diet, no alcohol consumption, and physical inactivity.7

7. ICD-10 and CPT (E/M) Codes ICD-10 Code: E66.9 Obesity, Unspecified. This diagnosis was selected because it is known based on the patient’s BMI of 32.1. This diagnosis does not specify the reason for obesity, such as excess calories, as this information is unknown. The pertinent diagnosis of E88.81, metabolic syndrome, would be added at the follow-up appointment if supported by her lab work. This diagnosis could be added to the original visit if it is specified that the diagnosis is suspected. The diagnosis of metabolic syndrome requires support from lab work. CPT (E/M) Code: Level 4—99214 (Established Patient) This patient is established with this practice and meets four elements for history of present illness, with two to nine systems covered in ROS, one of three elements covered in the past family and social history, and nine or more elements covered in the physical examination, which qualifies for a CPT (E/M) code of level 4: 99214. 8. Patient Education Topics L L L L

Weight reduction Increased physical activity Healthy diet Smoking cessation

9. Medical or Legal Concerns Professional demeanor and terminology should be used in sensitive subjects such as a patient’s weight or the patient may file a complaint against the provider with the practice and/or the licensing board. Additionally, provider bias should not affect the evaluation, diagnosis, and treatment of this patient population. 10. Interprofessional Collaboration L

L

L

Receptionist registers the patient, verifies demographics and insurance, collects payment (if required), and schedules follow-up appointment. MA takes brief history, vital signs, and waist circumference; draws lab work; and processes it for testing. PCP obtains the history, performs the physical examination, and formulates the most likely diagnosis.

Outcome

Once this patient’s lab results returned, it was confirmed she met the diagnostic criteria for metabolic syndrome: waist circumference 93 cm (36.5 in.), triglycerides 170 mg/dL, HDL 37 mg/dL, and fasting glucose 112 mg/dL. Education topics were reinforced with the patient, and she was provided with an opportunity to ask questions. Through joint decision-making with the patient, it was decided that she would implement lifestyle modifications including increased physical activity and a diet low in carbohydrates, sugars, and saturated fats as well as smoking cessation. The patient declined to use smoking cessation tools such as varenicline at this time. The patient did agree to discuss the status of lifestyle modifications and obtain repeat fasting lab work in 3 months to determine efficacy. At the 3-month follow-up, the patient reduced smoking to 4 cigarettes per day and states she is continuing to strive for complete cessation. She states her diet improved with elimination of fast food and increased intake of fruits and vegetables daily. The patient strives to work out 4 days a week but has been consistently working out for at least 1 hour on 2 days per week. Her weight has dropped by 18 lbs. Her waist circumference was reduced by 7.6 cm (3 in.). Her fasting glucose reduced from 112 to 76 mg/dL. Her repeat fasting lipid profile also improved, with her HDL rising from 37 to 50 mg/dL. Her triglycerides improved from 170 to 141 mg/dL. Her total cholesterol and LDL remain within normal limits. The patient was encouraged by the improved lab values and states she is feeling better and has improved energy. She agreed to continue with her lifestyle modifications and to follow up in 6 months.

Insight

from the

PCP

Some basic rules when dealing with your patients: L

L

L

Smile at the patient while making good eye contact and introduce yourself. Explain your role and desire to help them. Sit down in a chair at the bedside instead of standing while speaking when discussing sensitive issues such as weight. Medical providers should never be degrading or inflammatory when discussing weight with patients, and they should focus on the negative health risks of obesity rather than the weight itself. If the patient begins to show distress, it is important not to take it personally and to remain calm. Try to show respect, practice active listening, maintain your focus on the patient, and try to understand the underlying cause of the distress. Reinforce your desire to help and ask the patient how they would like you to help them. Explain patient choices and allow the patient to make care decisions, when appropriate. Use language that the patient will understand, and ask the patient to inform you if you say something that is not understood. Close the discussion by asking the patient what questions they have instead of asking if they have any questions. Though unlikely, this patient’s weight gain could be the result of a rare and/or serious medical condition such as a tumor. So, keep that in mind and ensure there is continuous follow-up with patients with weight gain.

Metabolic syndrome is increasingly common as the rates of obesity rise in the United States. In clinical practice, I have found obesity is typically not addressed by medical providers until the patient’s condition progresses to hypertension, diabetes, or other obesity-related conditions. I encourage providers to discuss weight at the stage of being overweight to aid in the prevention of associated chronic conditions as early diagnosis is important to initiate lifestyle and risk-factor modification. It is important to provide patient education related to an appropriate weight specific to the patient, as well as health risks that are increased by being overweight or obese. Additionally, it is important to encourage patients to compare themselves to what is healthy for them and not to others in the community because a large portion of the population is overweight to obese.

References

1. Rochlani Y, Pothineni NV, Kovelamudi S, et al. Metabolic syndrome: pathophysiology, management, and

modulation by natural compounds. Ther Adv Cardiovasc Dis. 2017;11(8):215–225. doi:10.1177/1753944717711379

2. Mccracken E, Monaghan M, Sreenivasan S. Pathophysiology of the metabolic syndrome. Clin Dermatol.

2018;36(1):14–20. doi:10.1016/j.clindermatol.2017.09.004

3. Grundy S, Brewer H Jr, Cleeman J, et al. Definition of metabolic syndrome: report of the National Heart, Lung, 4.

5. 6. 7.

and Blood Institute/American Heart Association conference on scientific issues related to definition. Circulation. 2004;109:433. doi:10.1161/01.CIR.0000111245.75752.C6 Alberti KG, Eckel RH, Grundy SM, et al. Harmonizing the metabolic syndrome: a joint interim statement of the International Diabetes Federation Task Force on Epidemiology and Prevention; National Heart, Lung, and Blood Institute; American Heart Association; World Heart Federation; International Atherosclerosis Society; and International Association for the Study of Obesity. Circulation. 2009;120:1640. doi:10.1161/ CIRCULATIONAHA.109.192644 Hu F, Meigs J, Li T, et al. Inflammatory markers and risk of developing type 2 diabetes in women. Diabetes. 2004;53:693. doi:10.2337/diabetes.53.3.693 Sigurdsson AF. Metabolic syndrome and insulin resistance. 2015. https://www.docsopinion.com/2015/01/29/ metabolic-syndrome-insulin-resistance-syndrome/ Palaniappan L, Carnethon MR, Wang Y, et al. Predictors of the incident metabolic syndrome in adults: the Insulin Resistance Atherosclerosis Study. Diabetes Care. 2004;27:788. doi:10.2337/diacare.27.3.788

WORSENING, SHARP ABDOMINAL PAIN, ADULT FEMALE

CASE

97

Chief CoMPlaint

“Worsening, sharp abdominal pain.”

historY

of

Present illness

A 22-year-old woman presents as a new patient to the primary care clinic, complaining of abdominal pain for the past week and a half. The pain became worse while having sexual intercourse with her boyfriend today. She states she had intercourse around 2.5 hours prior to arrival. She describes the pain as sharp in nature, located in the suprapubic region, rated 8/10, and constant; but it does not radiate, although she also has lower back pain. Her last menstrual period was almost 3 weeks ago. Her cycles are noted to be regular and last approximately 3 days. She states she has never experienced this type of pain before. The pain is exacerbated by movement and sexual intercourse, and she denies any relieving factors. She states she is currently in a monogamous relationship with her boyfriend of 2 years, and they do not use any form of contraception.

reVieW

of

sYsteMs

The patient’s ROS is positive for suprapubic pain, low back pain, dyspareunia, nausea, fever, chills, frequency, and yellow-white vaginal discharge. The ROS is negative for vomiting, diarrhea, urgency, dysuria, anorexia, chest pain, SOB, headaches, dizziness, and abnormal vaginal bleeding.

releVant historY

The patient is a full-time graduate student who lives alone. She has never been pregnant, denies tobacco use, drinks a glass of wine socially approximately two to three times a month, and denies recreational drug use. She has a history of trichomonas diagnosed and treated at age 18.

allergies

No known drug allergies; no known food allergies.

MeDiCations

OTC daily multivitamin.

PhYsiCal eXaMination

Vitals: T 37.7°C (99.9°F), P 67, R 18, BP 127/86, SpO2 98%, HT 167.64 cm (66 in.), WT 90.72 kg (200 lbs), BMI 32.27. General: Ill-appearing obese female lying in the right lateral decubitus position, in moderate distress secondary to pain. Skin, Hair, and Nails: No visible rash; no abnormal findings on hair or nails. Psychiatric: Cooperative, slightly anxious, likely secondary to pain. Chest: Normal anteroposterior diameter, symmetric chest wall expansion, no chest wall tenderness to palpation. Lungs: Breath sounds clear to auscultation bilaterally, without wheezing, rales, or rhonchi.

390  Case 97  •  Worsening, Sharp Abdominal Pain, Adult Female

Heart: RRR, S1, S2, no murmurs, rubs, or gallops. Abdomen: Non-distended, normal active bowel sounds, soft, tenderness noted in the right lower quadrant, no guarding, no rebound, no masses appreciated; psoas, rovsing, and obturator signs all negative. Genital/Rectal: Normal external genitalia without rash, ulcerations, nodularities, or abnormal lesions. Speculum exam notes moist, pink, vaginal walls with mild erythema, frothy, thin, yellow-white discharge pooled in the posterior fornix noted; no lesions or ulcerations. Cervix is erythematous, with noted discharge. Cervical motion tenderness noted. Bimanual exam notes positive bilateral adnexal tenderness to palpation, with no palpable masses appreciated. Rectal exam notes no lesions or ulcerations, normal sphincter tone, not tender to palpation. No palpable inguinal lymphadenopathy. Genital/rectal exam was performed with a chaperone (MA) present in the room. Musculoskeletal: Negative bilateral costovertebral angle tenderness. Neurologic: A&O×3. Cranial nerves II to XII grossly intact.

Clinical Discussion Questions 1. What is the differential diagnosis?

2. What is the most likely diagnosis? Why?

3. Demonstrate your understanding about the pathophysiology in regard to the most likely diagnosis.

4. Should tests/imaging studies be ordered? Which ones? Why? Think about tests/imaging beyond the

primary care setting as well.

Case 97  •  Worsening, Sharp Abdominal Pain, Adult Female 391 5. What are the next appropriate steps in management?

6. What are the adverse outcomes and treatment recommendations for this diagnosis and its relationship to

trichomonas? List the name of your references.

7. What are the pertinent ICD-10 and CPT (E/M) codes for this visit? Provide a short rationale.

8. What are the appropriate patient education topics for this case?

9. If not managed appropriately, what is/are the medical/legal concern(s) that may arise?

392  Case 97  •  Worsening, Sharp Abdominal Pain, Adult Female 10. Think about interprofessional collaboration for this case. Provide a list of specialties or other disciplines

and indicate what contribution these professionals might make to managing the patient.

Bedside Manner Question

11. What would your communication style/approach be with this patient?

ANSWER KEY: WORSENING, SHARP ABDOMINAL PAIN, ADULT FEMALE

CASE

97

1. Differential Diagnosis L

L

L

L

L

Ruptured ovarian cyst/ovarian cyst is a common noninfectious cause of pelvic pain with a variable presentation. This often presents with severe unilateral lower abdominal and pelvic pain (especially with rupture) and associated nausea and vomiting. This is often misdiagnosed as acute appendicitis when a patient presents with acute right-sided abdominal pain with nausea and vomiting. In this patient’s case, the vaginal discharge and adnexal tenderness to palpation make this diagnosis unlikely. Ectopic pregnancy always needs to be ruled out in a young, sexually active female with lower abdominal pain and vaginal bleeding. Patients can have a variable presentation but often present with lower quadrant tender to palpation with or without associated vaginal bleeding and a palpable mass, if unruptured. The patient may have adnexal tenderness to palpation with an ectopic pregnancy; however, with her last menstrual period noted to be 3 weeks ago and normal, and a pelvic exam yielding abnormal discharge, this diagnosis is less likely. Appendicitis typically presents with periumbilical or epigastric pain that localizes to the right lower quadrant, with associated fever, chills, and anorexia. The physical examination of a patient with acute appendicitis would not necessarily yield an abnormal vaginal exam unless it was a comorbidity. It would, however, yield guarding and possible peritoneal signs, with or without a positive psoas, Rovsing, or obturator sign, which were negative for this patient. Pelvic inflammatory disease always needs to be ruled out in a sexually active female with associated fever, chills, abdominal pain, and dyspareunia and a history of a previous STI. With the patient’s vaginal exam noting abnormal discharge and positive adnexal tenderness to palpation, this would rank highest on a list of differential diagnoses. Endometriosis often presents with associated dyspareunia; however, the pain would typically be chronic in nature. A patient with endometriosis would have presented with dysmenorrhea and/or chronic painful abdominal episodes long before this age.

2. Most Likely Diagnosis PID. Lower abdominal or suprapubic pain in a sexually active female with associated vaginal discharge is highly suggestive of PID. This is a clinical diagnosis strongly supported with her positive adnexal tenderness to palpation. Cervical motion tenderness, also known as a positive chandelier sign, may be noted on exam as well. The risk of PID is even higher in a patient with a history of STIs. 3. Pathophysiology PID is a cluster of disorders, specific to women, characterized by salpingitis and oophoritis.1 PID results from a vaginal or cervical infection that spreads to the upper genital tract and is common in sexually active women under age 30.2,3 Typically, the pathophysiology of PID is an untreated STI. Neisseria gonorrhoeae and Chlamydia trachomatis are associated with a majority of cases.4 However, an STI is not the sole cause of PID. Translocation of gut bacteria has been reported as a rare cause of PID and tubo-ovarian abscess in women who are not sexually active.5,6 There is a demonstrated relationship between PID and BV, but it is not yet clearly understood.5 It is known that BV alters the cervical mucosa, facilitating an ascending infection from an STI.7

4. Diagnostic Tests/Images L L L

L L L

Urine hCG to rule out pregnancy UA to rule out UTIs Microscopic exam (vaginal wet prep) to screen for bacterial vaginosis, trichomonas, and fungal infection CBC to rule out infection and anemia Cervical swab or urine NAAT for C. trachomatis and N. gonorrhoeae HIV screening

Results All tests were normal/negative, except her urine NAAT test, which came back positive for chlamydia. Her test was negative for gonorrhea.

L

5. Next Steps L

L

PID is a clinical diagnosis. In an otherwise healthy young female who will adhere to and tolerate outpatient treatment, it is recommended that treatment cover the most likely organisms (C. trachomatis and N. gonorrhoeae). This patient was treated presumably with one dose of ceftriaxone 250 mg IM and given a prescription for doxycycline 100 mg BID for 14 days. (Note this patient has no known drug allergies. Alternative regimens are available for patients who cannot take either or both of these medications.) The patient was advised to return in 1 week for a follow-up and to review her lab results. She was advised to inform her sexual partner of her diagnosis so he can seek evaluation and treatment.

6. Adverse Outcomes, Trichomonas, and Preferred Treatment L

L

L

PID is a painful condition that has the possibility for adverse patient outcomes, including infertility. C. trachomatis is one of the key causes of PID. Most of the individuals with C. trachomatis are asymptomatic. This underscores the need to screen for C. trachomatis during routine annual exams.8 It appears there is a link between TV and PID/infertility. Prevalence of TV in patients with recurrent PID was investigated and observed TV was frequently isolated from the vagina of women with PID/infertility. The researchers recommended doing additional investigation into whether TV may play an etiological role in adverse reproductive and gynecological outcomes.9 The preferred treatment for PID is broad-spectrum antibiotics, generally ceftriaxone 250 mg and doxycycline 100 mg BID for 14 days. Metronidazole, used to treat BV and trichomoniasis, can also be considered depending on lab findings.10

7. ICD-10 and CPT (E/M) Codes ICD-10 Code: N73.9 Female PID, unspecified. ICD-10 Code: N76.0 Acute vaginitis. These are the initial diagnoses at the primary care visit. The patient’s symptoms are consistent with PID. Therefore, these ICD-10 codes were used. Alternatively, a PCP may use relevant symptoms and code accordingly and mention on the chart notes that presentation is consistent with PID. CPT (E/M) Code: Level 4—99204 (New Patient) This code is designated for an office or other outpatient visits for the evaluation and management of a new patient, which requires three components: a detailed history, a detailed examination (at least nine systems examined), and moderate medical decision-making complexity. 8. Patient Education Topics L L L

STI/HIV PID and complications that can arise if untreated Safe sex precautions

L L L

Methods of contraception Notification of sexual partner Additional counseling: medication compliance; refraining from sexual activity until medication is complete, symptoms have resolved, and partner(s) have been evaluated and treated

9. Medical or Legal Concerns The adverse consequences of a misdiagnosis or an untreated case of PID include scar tissue, infertility, chronic pain, ectopic pregnancy, and sepsis. A PCP would be particularly exposed and may be held liable for poor outcomes resulting from a missed diagnosis. PCPs also need to properly report and document all STIs in accordance with state law. EPT is recommended by the CDC in cases of diagnosed chlamydia or gonorrhea infection, which as of this writing is permitted in almost all U.S. states. Guidelines for EPT are available at www.cdc.gov/std/ept/default.htm. Additionally, the diagnosis of any STI indicates risk for coinfection with other STIs. All patients should be offered concurrent screening for HIV, syphilis, gonorrhea, chlamydia, and hepatitis B and C. Pap testing with HPV screening should be provided to women, and cytologic testing for HPV in men should be considered based upon history. Screening for herpes simplex virus is not routinely performed. Finally, a pregnancy test must be performed in all women with suspected PID, as the presence of pregnancy guides treatment regimens. 10. Interprofessional Collaboration L L

L

Receptionist registers the patient. MA reviews medical history and medication lists, gathers chief complaint and takes vital signs, chaperones the genital examination, and reports the case of chlamydia to the public health department. PCP performs physical examination, formulates a differential diagnosis, treats and counsels the patient, and identifies the need for additional evaluation or treatment measures.

Outcome

The patient’s HCG was negative. She was treated for suspected PID possibly caused by C. trachomatis, with additional coverage for potential coinfection with N. gonorrhoeae based on her initial clinical presentation. Her urine NAAT test came back positive for chlamydia. Her test was negative for gonorrhea and an HIV screen was negative. The antibiotics prescribed for PID also covered the chlamydia infection, so prescribing another antibiotic was not necessary. The patient was in a monogamous relationship with her boyfriend; from the discussion where she advised him of her diagnosis, she learned that he had had several unprotected sexual encounters with at least three other females over the past 6 to 9 months. Hearing this as her provider was alarming, especially after screening for HIV; she may have future risk of exposure. The patient shared that she and her boyfriend are no longer in a relationship and he sought treatment with his PCP. I inquired about their last sexual encounter, to ensure they did not have sex after her last visit to the clinic. She assured me that they have not been intimate since her treatments related to this condition. I educated her to do another HIV screening in about 2 months and encouraged her to practice safer sex precautions, using a method that will protect her from STIs, including HIV, and to protect her against unwanted pregnancy as well. I reminded her this outcome could have been significantly different had she been diagnosed with an incurable STI such as HIV.

Insight

from the

PCP

It is important to advise young patients, female or male, to practice safe sex precautions to avoid unwanted pregnancy, STIs, including HIV. Many younger patients believe they are not at a high risk of HIV because of the “type of person they are” or the way their partners “seem to be.” Patients need to understand that there is no “type” of person who is affected by STIs. You can be the cleanest, smartest, most intelligent, well-kept, and well-liked person in the world and still contract an STI. What puts you at high risk are the behaviors the patient or patient’s partner practices, not the way someone appears to be.

When having sensitive conversations such as this, patients must be comfortable to share intimate behaviors. Developing this rapport with your patients in a fast-paced primary care practice requires demonstrated compassion and respect. Clinicians must ensure they keep an open mind with all patients no matter the situation, actively listen without judgment, and counsel patients such that they see a PCP as invested in them as a person. I approach every patient as if I were evaluating a family member, and I would encourage all clinicians to do the same.

References

1. Workowski KA, Bolan GA. Sexually transmitted diseases treatment guidelines, 2015. MMWR Recomm Rep.

2015;64(3):1–137.

2. Sutton MY, Sternberg M, Zaidi A, et al. Trends in pelvic in flammatory disease hospital discharges and ambulatory

visits, United States, 1985–2001. Sex Trans Dis. 2005;32(12):778–784. doi:10.1097/01.olq.0000175375.60973.cb

3. Rohrbeck P. Pelvic inflammatory disease among female recruit trainees, active component, U.S. armed forces,

2002–2012. MSMR. 2013;20(9):15–18.

4. Westrom L. Incidence, prevalence, and trends of acute pelvic inflammatory disease and its consequences in

industrialized countries. Am J Obstet Gynecol. 1980;138(7 pt 2):880–892. doi:10.1016/0002-9378(80)91077-7

5. Cho HW, Koo YJ, Min KJ, et al. Pelvic inflammatory disease in virgin women with tubo-ovarian abscess: a single6. 7. 8. 9. 10.

center experience and literature review. J Pediatr Adolesc Gynecol. April 2017;30(2):203–208. doi:10.1016/j. jpag.2015.08.001 Goodwin K, Fleming N, Dumont T. Tubo-ovarian abscess in virginal adolescent females: a case report and review of the literature. J Pediatr Adolesc Gynecol. 2013;26(4):e99–e102. doi:10.1016/j.jpag.2013.02.004 Yarbrough VL, Winkle S, Herbst-Kralovetz MM. Antimicrobial peptides in the female reproductive tract: a critical component of the mucosal immune barrier with physiological and clinical implications. Hum Reprod Update. 2015;21(3):353–377. doi:10.1093/humupd/dmu065 Dickson K, Macauley K. Chlamydia screening for high risk populations (Doctor of Nursing Practice Final Manuscripts). 2019. https://digital.sandiego.edu/dnp/90 Wiringa AE, Ness RB, Darville T, et al. Trichomonas vaginalis, endometritis and sequelae among women with clinically suspected pelvic inflammatory disease. Sex Trans Infect. 2020;96:436–438. Eloisa L, Jim B, Shivika T, et al. Treatment practices of pelvic inflammatory disease at selected STD clinics: STD surveillance network, 2015–2017 [16R]. Obstet Gynecol. May 2019;133:195S. doi:10.1097/01. AOG.0000559177.95674.a5

LUMP IN BREAST, ADULT MALE

CASE

98

chiEf comPlaint

“Lump in breast.”

history

of

PrEsEnt illnEss

A 42-year-old, generally healthy man presents for evaluation of a mass on his right chest wall/ breast. He states he has a “hard nodule” below his right nipple that he noticed 2 days ago; it is not painful and has not changed. He denies trauma or injury to the area. He reports no galactorrhea or bleeding from the nipple or skin and no skin changes on the chest. He denies any masses anywhere else on his body. He denies any systemic symptoms such as weight changes, fevers, or night sweats. He has never had anything like this before; nothing makes it better or worse, and he has not tried any medication or treatment at this time.

rEViEW

of

systEms

A ROS is positive for breast mass and a history of moderate erectile dysfunction after a vascular injury to his penis several years ago, and he states his erectile dysfunction may have worsened recently. The ROS is negative for skin lesions; chills or sweats; fatigue, malaise, or weakness; polyuria; polydipsia; polyphagia; heat or cold intolerance; change in voice, hair, or skin; excessive sweating or weight change; easy bruising or bleeding; abdominal pain; early satiety; changes in appetite or bowel habits; dysuria; obstructive voiding symptoms; testicular pain; swelling or testicular mass; or urethral discharge.

rElEVant history

The patient’s medical history is significant for carcinoma of the thyroid, with a total thyroidectomy and radiation treatment (age 16); traumatic vascular injury to penis/urethra with subsequent scarring, resulting in erectile dysfunction (age 22); and vasectomy without complications (age 40). His social history includes alcohol use (4 drinks/week), no tobacco use, and regular exercise. He is a firefighter, married with one child (age 7). His family history is significant for bladder cancer in father (age 56), breast cancer in paternal grandmother (age 59), ovarian cancer in a paternal aunt (age 43), lung cancer in maternal grandmother (age 75), and bone cancer in his maternal grandfather (age 55).

allErgiEs

No known drug allergies; no known food allergies.

mEDications ● ●

Levothyroxine 175 mcg QD. Sildenafil 25 mg PRN.

394  Case 98  •  Lump in Breast, Adult Male

Physical Examination Vitals: T 37°C (98.6°F), P 54, R 16, BP 140/83; HT 178 cm (70 in.), WT 70 kg (155 lbs), BMI 22. General: Well-developed, well-nourished male in no apparent distress; appears fit; appears stated age. Neck: Faint thyroidectomy scar across the anterior neck; no palpable thyroid tissue or neck mass noted; no cervical adenopathy or tenderness bilaterally. Lungs: Clear to auscultation bilaterally without wheezes, rales, or rhonchi. Breast: Right-sided retro-areolar non-tender breast mass palpated, firm, smooth, and mobile, approximately 3 cm in diameter without skin changes or retraction of the nipple; no erythema or warmth; no galactorrhea; left breast with no abnormal findings; no axillary or supraclavicular adenopathy bilaterally. Heart: RRR without murmur, rubs, or gallops. Abdomen: Soft, flat, non-tender; positive bowel sounds; no masses, no hepatosplenomegaly. Genital/Rectal: Circumcised penis with midline urethra; no plaques, no urethral discharge; scrotum without swelling or tenderness; testes palpated without masses or tenderness bilaterally. Rectal exam deferred.

Clinical Discussion Questions 1. What is the differential diagnosis?

2. What is the most likely diagnosis? Why?

3. Demonstrate your understanding about the pathophysiology in regard to the most likely diagnosis.

4. Should tests/imaging studies be ordered? Which ones? Why? Think about tests/imaging beyond the

primary care setting as well.

Case 98  •  Lump in Breast, Adult Male 395 5. What are the next appropriate steps in management?

6. What are the causes of this diagnosis and the most helpful diagnostic imaging studies for the initial

workup? Why thorough follow-up is vital for this diagnosis? Provide references for your response.

7. What are the pertinent ICD-10 and CPT (E/M) codes for this visit? Provide a short rationale.

8. What is the appropriate patient education topic for this case?

9. If not managed appropriately, what is/are the medical/legal concern(s) that may arise?

10. Think about interprofessional collaboration for this case. Provide a list of specialties or other disciplines

and indicate what contribution these professionals might make to managing the patient.

396  Case 98  •  Lump in Breast, Adult Male

Bedside Manner Questions

11. What would your communication style/approach be with this patient?

12. If a patient is distressed by the diagnosis, what might offer support?

ANSWER KEY: LUMP IN BREAST, ADULT MALE

CASE

98

1. Differential Diagnosis ●









Breast hypertrophy (gynecomastia) is the most likely diagnosis based on the physical examination. This patient has a non-tender and unilateral mass of approximately 3 cm. While this is the most likely diagnosis currently, it requires further evaluation and close follow-up. L ipoma presents as a common, benign, subcutaneous fat tumor anywhere on the body, especially on the trunk and upper extremities, making this a possible diagnosis; the acute onset and history of cancer in this patient as well as the physical exam finding of a firm mass near the nipple makes this diagnosis less likely. Testicular cancer is also a less likely possibility. Germ cell tumors account for about 95% of testicular neoplasms, which are most common in young men (typically ages 15–35 years) and commonly associated with gynecomastia because of secretion of hCG by foci of choriocarcinoma or trophoblastic cells in germ cell tumors. This patient has a history of thyroid cancer, making a subsequent cancer more likely. The presentation of gynecomastia in a relatively young man with worsening erectile dysfunction makes testicular cancer a likely diagnosis; but the absence of a mass in the scrotum/testicle and the patient’s age of over 35 makes this less likely. Male hypogonadism (primary or secondary) may be associated with gynecomastia; this patient has a history of vascular insufficiency to the genitals, which may result in decreased testosterone level and an estrogen–androgen imbalance. However, this patient’s ROS is negative for fatigue, malaise, or weakness, making this diagnosis less likely. Though it is unlikely, ordering serum testosterone level during further workup is recommended. Breast carcinoma, although rare in men, does occur in up to 1% of all breast cancers diagnosed in the United States each year. This patient has a history of thyroid cancer, which makes a subsequent cancer more likely; in addition, his family history is positive for breast and ovarian cancer, which contributes as a risk factor. However, the presentation in a young man with a smooth, mobile breast mass makes this less likely.

2. Most Likely Diagnosis Hypertrophy of the right breast. This is the diagnosis for the initial patient encounter because it was found on physical examination. This patient has a non-tender and unilateral mass of 3 cm. While this is the most likely diagnosis currently, it requires further evaluation and close follow-up. There are different causes for hypertrophy of the breast, so additional investigation is recommended. 3. Pathophysiology ●

Physiologic gynecomastia most commonly occurs in neonates, pubertal, and older males.1 Male and female breast tissue responds equally to hormone stimulation. Estrogens stimulate ductal epithelial hyperplasia, ductal elongation and branching, periductal fibroblast proliferation, and increases in the vascularity of the glandular breast tissue. Gynecomastia can be caused by decreased androgen production, increased estrogen production, or an increase in estrogen precursors that convert to estrogen in the peripheral tissues.1







An imbalance between the stimulatory effects of estrogen and the inhibitory effects of androgens are the general pathophysiology of gynecomastia, regardless of the underlying condition. Factors that may affect estrogen–androgen balance include the production of steroids from the adrenals and testes, extraglandular conversion of androgen to estrogen, the circulating concentration and binding of SHBG, and the end organ ability to respond to the hormones.1 Medications can cause gynecomastia and account for about 20% of gynecomastia in males. Common medication causes include antiandrogens, protease inhibitors, and nucleoside reverse transcriptase inhibitors. Other medications may cause gynecomastia less commonly or rarely.2 A rare cause of gynecomastia is ectopic hCG production by some types of carcinoma, including gastric adenocarcinomas. In the case of elevated beta-hCG levels not associated with a germ cell tumor, it is most often associated with poorly differentiated carcinoma.3

4. Diagnostic Tests/Images Primary Care Testing ● Diagnostic bilateral mammogram and right breast ultrasound to rule out breast cancer ● CBC to screen for infection ● CMP to evaluate electrolytes, kidney functions, and liver functions ● TSH, total serum testosterone, serum prolactin, serum LH, quantitative serum beta-hCG, serum AFP to screen for endocrinology-associated conditions. ● Ultrasound of the scrotum to evaluate germ cell malignancy Results The diagnostic mammogram bilateral with right breast ultrasound found a unilateral right gynecomastia, a benign mammographic finding. Further evaluation on a clinical basis was advisable. Significant lab results included the following: ● Low LH (0.6; normal 1.8–8.6 IU/ml) ● Elevated quantitative beta-hCG (256; normal 0–6 IU/L) ● Elevated AFP (1,304.5; normal 0–8 ng/mL) Ultrasound of the scrotum and all other labs were negative. Urology Testing CT scans of the chest, abdomen, and pelvis to evaluate for extragonadal germ cell primary



Result The CT scan revealed several small lesions in his liver that were thought to be cysts.



Gastroenterology Testing PET/CT to evaluate liver lesions (this was done 3 weeks after CT scan) ● CT-guided biopsy of one of the liver lesions and the celiac lymph node ● Endoscopy with biopsy of an ulcerated lesion at the GE junction ●

Results PET/CT showed an increase in the size and number of liver lesions with significant uptake in the gastro-hepatic ligament as well as the liver lobules and gastroesophageal junction. ● CT-guided biopsy showed poorly differentiated adenocarcinoma, suggestive of an upper GI or pancreatic origin. ● Endoscopy determined gastric adenocarcinoma. ●

5. Next Steps ● ●



Follow-up visit in 1 to 2 weeks to review results of his diagnostic studies. At the first follow-up visit, the results of his initial diagnostic testing were discussed, including elevated serum tumor markers and a normal diagnostic mammogram. The PCP discussed the differential diagnosis again based on the current available results. Because germ cell malignancy is a possible diagnosis after initial labs, an ultrasound of the scrotum was ordered and a second follow-up appointment was made.

● ●

At the second follow-up visit, the PCP discussed the ultrasound result, which was negative. The patient was referred to urology for further evaluation.

6. Causes, Initial Workup Imaging, and Thorough Follow-Up Physiologic gynecomastia, medications, cirrhosis, hypogonadism, testicular cancer, Graves hyperthyroidism, chronic kidney disease, and other rare causes can cause gynecomastia.1,2 ● When there is a male with gynecomastia/palpable mass, mammography and ultrasound remain the leading modalities for diagnostic workup. It is important to keep in mind that presentation with palpable mass may be associated with malignancy.4 ● While gynecomastia may be a benign symptom, it can also be a symptom of a serious health condition. PCPs should not underestimate gynecomastia. Even physiologic gynecomastia, transient and benign by definition, may lead to serious psychologic consequences. Therefore, any patient with gynecomastia will need proper evaluation. A detailed medical history and thorough physical examination are important. In addition, PCPs should be cognizant that— although rare—gynecomastia might be the first symptom of an underlying serious disease.5 ●

7. ICD-10 and CPT (E/M) Codes ICD-10 Code: N62 Hypertrophy of breast. This diagnosis was used for the initial patient encounter because it was found on physical examination, but the cause for the breast mass was not clear at the initial visit. Additional diagnostic testing was ordered by the PCP to determine the underlying diagnosis; a more specific diagnosis can be used at subsequent visits when appropriate. CPT (E/M) Code: Level 4—99214 (Established Patient) This code is for an established patient with a new complaint that requires additional diagnostic evaluation and medical complexity. Documenting medical decision-making, chart review, and/or consultation with the collaborating physician (if applicable for PAs) will support the medical complexity of this case and allow the PCP/PA to bill at a higher level of reimbursement. 8. Patient Education Topics ● ● ●

● ●

What is gynecomastia? What causes gynecomastia at midlife and beyond? Gynecomastia differential diagnosis (imbalance between the hormones estrogen and testosterone, medical conditions, and certain drugs) Gynecomastia workup Diagnostic mammogram and breast cancer in men

9. Medical or Legal Concerns This patient was ultimately diagnosed with an aggressive, poorly differentiated adenocarcinoma (see the Outcome section for additional information) of the gastroesophageal junction with metastases to the liver. If his diagnostic evaluation or management had been delayed, uncoordinated, or poorly managed, he or his family may have pursued legal action as a result of a poor outcome. It is recognized by the medical community that patients are more likely to pursue legal action when there are poor outcomes, poor communication, or a delay in medical care.6 10. Interprofessional Collaboration ●





Receptionist registers the patient and checks him in to the office and schedules follow-up appointments. PCP conducts history and physical exam, formulates most likely diagnosis, orders diagnostic testing, and recommends follow-up. Technologist performs diagnostic mammogram with breast ultrasound and scrotum ultrasound.



● ●



Radiologist provides interpretation and consultation regarding results of breast and scrotum ultrasound and mammogram. Phlebotomist performs phlebotomy and lab provides results. PCP, who is a PA, discusses the case with collaborating physician regarding plan of care and provides consultation with specialists to coordinate additional diagnostic evaluation and plan of care. Specialists: The patient was ultimately referred to a urologist, gastroenterologist, and oncologist for diagnostic evaluation and treatment.

Outcome

As a result of the initial laboratory evaluation showing elevated serum tumor markers and a normal diagnostic mammogram, ultrasound of the scrotum was ordered for possible germ cell malignancy. His scrotum ultrasound was normal. Then the patient was referred to a urologist for further evaluation of a possible germ cell malignancy. The urologist ordered CT scans of the chest, abdomen, and pelvis to evaluate for extragonadal germ cell primary. The CT scan revealed several small lesions in his liver that were thought to be cysts. He was then referred to a gastroenterologist, who ordered a PET/CT scan for further evaluation. The PET/CT scan, done 3 weeks after the previous CT scan, showed an increase in the size and number of liver lesions with significant uptake in the gastrohepatic ligament as well as the liver lobules and gastroesophageal junction. Due to these concerning findings, he underwent CT-guided biopsy of one of the liver lesions and the celiac lymph node, which came back as poorly differentiated adenocarcinoma suggestive of an upper GI or pancreatic origin. He subsequently underwent endoscopy with biopsy of an ulcerated lesion at the GE junction, which was determined to be the primary site of his gastric adenocarcinoma. He was referred to an oncologist. He continued to feel well with mild soreness of his right nipple and acne, thought to be related to the increases in serum tumor markers. The patient underwent three rounds of chemotherapy. He lost significant weight, approximately 30 lbs, and became anemic, with a serum Hgb level down to 3.3 g/dL. He was given blood transfusions, with improvement in his blood counts and fatigue. His cancer was extremely aggressive and progressed despite chemotherapy, with further metastases to the lungs and then bones. He was given palliative care and put on hospice, and he died 11 months after his diagnosis.

Insight

from the

PCP

I am a PA, and my collaborating physician and I were the PCPs for this patient and his family for 3 years prior to his cancer diagnosis. He was well known to our practice and the community. I was the first provider to evaluate his complaint and ordered the initial studies that led to his diagnosis. We were involved in coordinating his care, including diagnostic evaluation and communication with specialists. It was a real team effort to determine the diagnosis, which took almost 6 weeks from his initial presentation because it was such an unusual case. His case was presented at the hospital tumor board, and we were invited to attend because of our involvement in his evaluation. This patient’s death has had a lasting effect on me. I think of him and his family often. He was one of the first patients I personally diagnosed and cared for who died. His diagnosis was a mystery we solved as a team, but the outcome was heartbreaking. It really struck me that being a PCP is incredibly important, not only to prevent disease but in the evaluation, communication, and collaboration of patient care. What we do every day matters. And even when there is a poor outcome, it is an honor to be part of the journey. Special thanks to my collaborating physician for mentoring me and inviting me to be a part of his team.

References

1. Costanzo PR, Pacenza N, Aszpis SM, et al. Clinical and etiological aspects of gynecomastia in adult males: a

multicenter study. BioMed Res Int. May 2018;2018:1–7.

2. Bowman JD, Kim H, Bustamante JJ. Drug-induced gynecomastia. Pharmacotherapy. December

2012;32(12):1123–1140.

3. Greco FA, Hainsworth JD. Introduction: unknown primary cancer. Semin Oncol. 2009;36:6. 4. Lawson P, Nissan N, Faermann R, et al. Trends in imaging workup of the male breast: experience from a single

center. Isr Med Assoc J. October 2019;21(10):666–670.

5. Sansone A, Romanelli F., Sansone, M, et al. Gynecomastia and hormones. Endocrine. 2017;55:37–44. https://doi-

org.laneproxy.stanford.edu/10.1007/s12020-016-0975-9

6. Ioan BG, Ciuhodaru T, Velnic AA, et al. The role of doctor-patient communication in preventing malpractice

complaints. Int J Commun Res. October–December 2017;7(4):303–307.

Additional Helpful Information

to the

Case

Causes of elevated serum tumor markers in male patients3 ●



Elevated serum tumor markers (serum beta hCG, AFP) can be elevated in germ cell tumors such as testicular cancer; testicular ultrasound in this patient was negative. Other serum tumor markers such as CEA, CA-125, CA19-9, and CA15-3 are often elevated in carcinoma of unknown primary site; this patient had mildly elevated CEA and normal CA19-9 at his initial oncology evaluation.

Adenocarcinoma of unknown primary3 ●



CUP accounts for 4% to 5% of all invasive cancers, and about 70% of CUPs are adenocarcinomas. The most common primary sites are lung, pancreas, hepatobiliary, and kidney. Many patients have widespread metastases and poor prognosis at the time of diagnosis, and average life expectancy is 4 to 6 months after diagnosis. Diagnostic evaluation of patients with CUP includes a thorough history, physical examination, laboratory testing including routine tests such as CBC, CMP, as well as serum tumor markers, and CT scan of the chest and abdomen. PET is often a useful diagnostic study to evaluate or detect the primary tumor and sites of metastases.

BACK PAIN AND POOR POSTURE, PEDIATRIC FEMALE

CASE

99

Chief CoMPlaint

“Back pain and poor posture.”

historY

of

Present illness

A 9-year-old girl is brought to the office by her parents to discuss her poor posture and lingering back pain. They constantly remind her to sit up straight. Their daughter states she tries to sit straight and is doing the best she can. She admits to lingering mid-back pain. The pain is a 4/10, is localized to mid-thoracic area bilaterally, does not migrate, and is described as a consistent ache, not stabbing or pulsatile. The patient takes ibuprofen 200 mg every 6 hours sometimes but usually tries to just ignore her pain. She is not involved in any sports but plays violin and piano. She is active with friends and has not noted any change or limitation in interactions with them. She denies pain in her shoulders or legs. No muscle weakness in her extremities has been noted by her or her parents. They state this has been going on for a while and think they noted the problem the most over the past 6 to 9 months.

reVieW

of

sYsteMs

The ROS was unremarkable. The girl has mid-thoracic bilateral 4/10 pain as described. The ROS was negative for cough, SOB, chest pain, fatigue, abdominal pain, vomiting, diarrhea, dysuria, fever, muscle weakness, or dizziness.

releVant historY

The patient is fully vaccinated and has had no recent travel; she has no history of prior surgeries or trauma and no chronic health concerns. Her parents recall she reached developmental milestones on time. She has not been to a pediatrician since her kindergarten checkup because her parents’ insurance has a high deductible. The child lives with her parents in a house built about 10 years ago. Her parents deny any significant family history of illness.

allergies

No known food or drug allergies.

MeDiCations

Ibuprofen 200 mg q6h PRN for pain.

PhYsiCal eXaMination Vitals: T 37°C (98.6°F); P 80; R 17; BP 116/72; WT 41.27 kg (91 lbs), 90th to 95th percentile; HT 147.32 cm (58 in.), 95th to 100th percentile; BMI 19. General: Alert and in no distress. Skin, Hair, and Nails: Warm, dry, with no rashes noted. Capillary refill brisk. No abnormal findings with hair or nails. Head: Normocephalic and atraumatic. Neck: Supple with no adenopathy.

398  Case 99  •  Back Pain and Poor Posture, Pediatric Female

Chest: The sternum is normal, with no pectus carinatum or excavatum noted. Lungs: Clear to auscultation throughout all fields. Aeration is symmetric. Heart: RRR, with no murmur, gallop, or rub. Abdomen: Soft, non-tender, non-distended with no masses noted. Bowel sounds equal throughout. Genital/ Rectal: Tanner stage 1. Rectal exam deferred. Musculoskeletal: Obvious shoulder height discrepancy with right shoulder elevated. Right scapula is mildly elevated compared to the left. A forward bend test shows noticeable prominence to the right thoracic and ribs in the mid-thoracic range. There is also prominence to the paraspinal muscles of the left lumbar spine. A scoliometer is utilized and shows a 9° angle of rotation over the mid-thoracic spine and 7° angle of rotation over the lumbar spine. Neurologic: Cranial nerves II to XII intact. DTR symmetric in the upper and lower extremities at 2/4, and muscle strength symmetric in the upper and lower extremities at 5/5.

Clinical Discussion Questions 1. What is the differential diagnosis?

2. What is the most likely diagnosis? Why?

3. Demonstrate your understanding about the pathophysiology in regard to the most likely diagnosis.

4. Should tests/imaging studies be ordered? Which ones? Why? Think about tests/imaging beyond the

primary care setting as well.

Case 99  •  Back Pain and Poor Posture, Pediatric Female 399 5. What are the next appropriate steps in management?

6. Discuss screening and treatment recommendations for the diagnosis. Additionally, what role do CM play

in the diagnosis? Include references to support your response.

7. What are the pertinent ICD-10 and CPT (E/M) codes for this visit? Provide a short rationale.

8. What is the appropriate patient education topic for this case?

9. If not managed appropriately, what is/are the medical/legal concern(s) that may arise?

10. Think about interprofessional collaboration for this case. Provide a list of specialties or other disciplines

and indicate what contribution these professionals might make to managing the patient.

400  Case 99  •  Back Pain and Poor Posture, Pediatric Female

Bedside Manner Questions

11. How would you communicate your likely diagnosis to the patient and her parents?

12. If the patient and family show distress at what you communicate, how would you provide support?

ANSWER KEY: BACK PAIN AND POOR POSTURE, PEDIATRIC FEMALE

CASE

99

1. Differential Diagnosis ●









Compensatory scoliosis is a suspect here. Some children are born with a leg length discrepancy that eludes diagnosis for some time. When left untreated, pelvic tilt, scoliotic curvature, and shoulder height discrepancies may result. To determine this, the length from the anterior superior iliac spine to the medial malleolus is measured. In this patient’s case, there was no discrepancy noted. Neuromuscular disease is on the differential. Children born with diseases of or injuries to the neurologic system may develop scoliosis. They may have weakness to the core muscle groups of the trunk, which leads to curvature. Muscular dystrophy, cerebral palsy, and spina bifida are examples. Spinal cord trauma would also fit here. This is an unlikely diagnosis here as it does not meet with the patient’s history. Genetic syndrome manifestation is unlikely. Children born with certain syndromes may be more likely to develop an early scoliosis. Most commonly, connective tissue disorders such as Marfan syndrome exhibit scoliosis. Down syndrome (trisomy 21) is another condition in which early scoliosis can be seen. However, the patient’s history does not support either one of those conditions. Juvenile idiopathic scoliosis is a likely diagnosis. A scoliosis survey is a relatively safe and painless evaluation method (x-ray) to evaluate the spine. JIS occurs between ages 4 and 9 years, and this patient falls within this age range. Adolescent idiopathic scoliosis is the most common type of scoliosis.1 About 1% to 3% of adolescents in the United States are affected with AIS.2 However, its diagnosis is made at the age of 10 or after with no underlying etiology identified. Because the patient is younger, this diagnosis is less likely.

2. Most Likely Diagnosis Juvenile idiopathic scoliosis. For a young patient, the most likely diagnosis is JIS or a congenital spine deformity. A scoliosis survey is needed. Because most cases of JIS occurs between ages 4 to 9 years, this is the initial diagnosis. 3. Pathophysiology ●



The pathogenesis of scoliosis is poorly understood. It is reasonable to believe that an existing deformity would produce an asymmetrical loading of the growing spine, which in turn would cause asymmetrical growth of the vertebrae. But how does it start? And why is it progressive in some but not in others? Biomechanical, neural, metabolic, and hormonal changes have been reported in idiopathic scoliosis, but it is difficult to say whether these are primary or secondary to the deformity.3 Hypotheses attempting to explain the occurrence of scoliosis in patients with CM include cerebellar tonsillar compression of the cervicomedullary junction and uneven expansion of a syrinx in the horizontal plane of the spinal cord. Early detection of scoliosis on routine spinal examination and close follow-up on curve stability and progression are essential initial steps in the management of scoliosis, especially in patients with CM, who may require a full spine MRI to screen for associated neuro-axial anomalies. Bracing and spinal fusion may be subsequently pursued in high-risk patients.4

4. Diagnostic Tests/Images Primary Care Testing A scoliosis survey of the thoracic and lumbar spine with anteroposterior and lateral views was ordered. Results The patient had an x-ray, revealing a mid-thoracic curvature with a Cobb angle greater than 20° and a compensatory lumbar curvature of 18°. Pediatric Orthopedic Setting MRI of the spine and brain. Results MRI showed a syringomyelia secondary to CM type 1. 5. Next Steps Initial Visit ● Ordered a scoliosis survey of the thoracic and lumbar spine with anteroposterior and lateral views. Follow-up Visit At follow-up visit with family, discussed x-ray results. ● Because of the abnormal findings, referred patient to pediatric orthopedics for further workup, including an MRI. ● PCP may need to order the MRI; the choice may be somewhat dependent on the patient’s insurance as well. Contacting the insurance company to discuss prior authorization may be required. ●

6. Screening, Treatment, and Chiari Malformations ●





Screening for scoliosis can be done through a visual observation of the symmetry, or asymmetry, of a patient’s shoulders, scapulae, and hips and a bend forward test. Diagnosis is confirmed through x-rays, which will reveal the degree of spinal curvature.5 The typical treatment is bracing and a sequence of graduated casts to slow progression of the disease. Early (before puberty) treatment is important to slow the curvature. Spinal fusion is done after puberty when required.6 Brace compliance is important during the treatment plan, and PCPs should keep the following factors in mind to increase compliance: Having a comfortable brace and a superior appearance brace will improve the compliance rate and psychologic acceptance. Also, starting the brace at a younger age, involving the patient in the treatment plan, considering the child's habits, and ensuring the family’s awareness of the treatment plan, can improve overall compliance of the brace.7 It appears there is a link between CM and scoliosis. In patients with CM, early onset of scoliosis is a possibility. Therefore, it is important to screen scoliosis among these patients and detect the problem as early as possible.8 Upon detection, close monitoring of the spinal curve stability and progression is a vital responsibility of PCPs.

7. ICD-10 and CPT (E/M) Codes ICD-10 Code: M41.114 JIS, thoracic region. ICD-10 Code: M54.6 Thoracic back pain. JIS is the presumed diagnosis at this time. Ordering an x-ray to confirm the diagnosis and further evaluation of the condition is necessary. Based on the patient’s age, onset, and physical exam findings, these are the appropriate ICD-10 codes during the initial visit. However, it is appropriate to add a note that M41.114 (JIS, thoracic region) is the working diagnosis at this time and the evaluation is in progress.

CPT (E/M) Code: Level 4—99214 (Established Patient) Visit included a detailed history and exam. This was a moderately complicated case. The visit needed more than the 25-minute requirement, considering that over 50% of the visit was spent counseling about the diagnosis, its potential causes, the diagnostic tests needed, and the specialty care recommendations. 8. Patient Education Topics ● ● ●

Scoliosis Importance of yearly well child checks Scoliosis: early screening and routine monitoring

9. Medical or Legal Concerns It is important to document that this patient had not been seen in some time, which explains why this condition was not found earlier. It is also important not to assume this is classic AIS. Telling a family that scoliosis in a child younger than age 10 years is normal and to just monitor the child could lead to significant neural axis complications. Also, other potential complications of untreated JIS (abnormalities of the thorax, impingement of lung development, chronic pain) may occur and lead to medico-legal concerns. 10. Interprofessional Collaboration ● ● ●

● ●

Receptionist registers the patient. MA takes a brief history and vitals. PCP recognizes the unusualness of the diagnosis, which requires further care, and formulates the diagnosis and plan. Radiologist interprets x-rays and MRI. Pediatric orthopedist and pediatric neurosurgeon completes workup and determines care plan.

Outcome

The patient had a mid-thoracic curvature with a Cobb angle greater than 20° and a compensatory lumbar curvature of 18°. She was sent to a pediatric orthopedic specialist, who ordered an MRI of the spine and brain. This revealed a syringomyelia secondary to CM type 1. Surgical correction of the CM by neurosurgery led to resolution of the syringomyelia. The patient required bracing after surgery and the curvature improved, though full resolution was not achieved. She has routine follow-up with orthopedics and neurosurgery but currently has not required further therapy.

Insight

from the

PCP

Two rules I always follow came in handy in this case: Be honest and know my limitations. The parents were extremely concerned about their 9-year-old daughter’s life and possible disability once I explained what I saw on the exam and what my concerns were. I needed to not rush this discussion. The parents were very hard on themselves for not bringing the girl in sooner and not scheduling regular routine appointments. This was a time to support them and focus on “the now.” Therefore, I showed my support to the family even if I disagreed with their past decision (not bringing the child sooner). We discussed what we needed to do now to make a definitive diagnosis and what we would do next depending on the results. I gave them a focus and a plan rather than let them dwell on what was not done or what could not be changed. I did not make promises or tell them all would be fine; I was honest, which is my first rule. The second rule is to know my limitations. This was my first JIS case. I needed to do some research to ensure I was not missing anything. I was honest with the family about that as well. I have always found that my patients’ families appreciate this when a situation arises and I don’t have a definitive answer. Don’t be afraid to say, “I am not sure, but I am going to do everything to find out.”

References

1. Kuznia AL, Hernandez AK, Lee U. Adolescent idiopathic scoliosis: common questions and answers. Am Fam

Physician. January 1, 2020;101(1):19–23.

2. Dunn J, Henrikson NB, Morrison CC, et al. Screening for Adolescent Idiopathic Scoliosis: A Systematic Evidence 3. 4. 5. 6. 7. 8.

Review for the U.S. Preventive Services Task Force: Evidence Synthesis No. 156. Agency for Healthcare Research and Quality; 2018. Grauers A, Einarsdottir E, Gerdhem P. Genetics and pathogenesis of idiopathic scoliosis. Scoliosis. 2016;11:45. doi:10.1186/s13013-016-0105-8 Noureldine MHA, Shimony N, Jallo GI, et al. Scoliosis in patients with Chiari malformation type I. Childs Nerv Syst. 2019;35:1853–1862. doi:10.1007/s00381-019-04309-7 Dunn J, Henrikson NB, Morrison CC, et al. Screening for Adolescent Idiopathic Scoliosis: A Systematic Evidence Review for the U.S. Preventive Services Task Force: Evidence Synthesis No. 156. Agency for Healthcare Research and Quality; 2017. Rosenberg JJ. Scoliosis. Pediatr Rev. 2011;32(9):397–398. doi:10.1542/pir.32-9-397 Rahimi S, Kiaghadi A, Fallahian N. Effective factors on brace compliance in idiopathic scoliosis: a literature review. Disabil Rehabil Assist Technol. 2019 Jun 28:1-7. doi: 10.1080/17483107.2019.1629117. Epub ahead of print. PMID: 31248292. Noureldine MHA, Shimony N, Jallo GI, et al. Scoliosis in patients with Chiari malformation type I. Childs Nerv Syst. 2019;35:1853–1862. doi:10.1007/s00381-019-04309-7

ITCHY BUMPS, PEDIATRIC FEMALE

CASE

100

Chief ComPlaint

“Itchy bumps.”

historY

of

Present illness

A mother brings her 9-year-old daughter to her PCP for evaluation of an itchy, bumpy rash that comes and goes but resolves within a few hours. This has been going on for 1 month. The last episode of rash was 1 day ago and lasted all day, keeping the child and parent up at night. The rash was accompanied by a swelling of the girl’s eyelids. The mother administered diphenhydramine at night, which helped a little. Today, the eye swelling has resolved and the rash is slightly better. The mother was able to take some pictures of the initial rash with her phone. The rash in the pictures shows raised bumps and patches, round or oval across the body, especially on the abdomen, thigh, and back. The patient remembers having a few itchy bumps here and there in the past but thought they were mosquito bites. The patient and mother cannot recall a correlation between rash and possible triggers such as food, contact, or other stressors such as heat, cold, or medications such as NSAIDs. The mother is extremely concerned as she has tried to find answers online and worried her child may have a cancer or leukemia. The mother is unsure of any food allergies.

reVieW

of

sYstems

The patient’s ROS is positive for diffuse rash, pruritus, and upper eyelid swelling. Her ROS is negative for unexplained fever, adenopathy, recent unintentional weight loss, fatigue, headache, joint pain or swelling, wheezing, flushing, palpitation, abdominal pain, skin ulceration, wheezing, SOB, and swelling of throat or tongue. She has no history of thyroid dysfunction, vasculitis, or autoimmune-related health problems.

releVant historY

The patient has history of seasonal allergies to pollen and takes loratadine 10 mg daily during spring and early summer. She is up to date on her immunizations. There is no significant contributory history from her family.

allergies

No known drug allergies; no known food allergies.

meDiCations

Loratadine 10 mg PO daily PRN for seasonal allergies.

PhYsiCal eXamination Vitals: T 37°C (98.6°F), P 80, R 18, BP 110/72, WT 36.29 kg (80 lbs), HT 142.24 cm (56 in.), BMI 18. General: A&O, playful, and interactive. Psychiatry: Normal affect, good eye contact, answers general questions appropriately.

402  Case 100  •  Itchy Bumps, Pediatric Female

Skin, Hair, and Nails: No lesions or scales in hairline noted. Diffuse raised erythematous and edematous papules and plaques ranging from 0.2 cm to 1 cm present on neck, abdomen, thigh, legs, arm, hands, feet, buttocks, and back. Plantar and palmer surfaces are spared. Lesions are blanchable. No ulceration or target lesion noted. No jaundice noted. Skin has a normal turgor. No abnormal findings with hair or nails. Eyes: Both eyes with clear sclera and conjunctiva; ophthalmoscope exam shows no sign of hemorrhage. ENT/Mouth: Both TM clear with cone of light visible; oropharyngeal mucosa without redness or exudate. No swelling of eyelids, lips, or tongue. Neck: Non-tender, no adenopathy noted. Lungs: No wheezing, rales, or rhonchi. Heart: RRR, no murmur noted. Peripheral Vascular: Good peripheral perfusion present.

Clinical Discussion Questions 1. What is the differential diagnosis?

2. What is the most likely diagnosis? Why?

3. Demonstrate your understanding about the pathophysiology in regard to the most likely diagnosis.

4. Should tests/imaging studies be ordered? Which ones? Why? Think about tests/imaging beyond the

primary care setting as well.

Case 100  •  Itchy Bumps, Pediatric Female 403 5. What are the next appropriate steps in management?

6. What is the diagnostic approach and prognosis for this diagnosis? Provide references for your response.

7. What are the pertinent ICD-10 and CPT (E/M) codes for this visit? Provide a short rationale.

8. What are the appropriate patient/parent education topics for this case?

9. If not managed appropriately, what is/are the medical/legal concern(s) that may arise?

404  Case 100  •  Itchy Bumps, Pediatric Female 10. Think about interprofessional collaboration for this case. Provide a list of specialties or other disciplines

and indicate what contribution these professionals might make to managing the patient.

Bedside Manner Question

11. What would your communication style/approach be with this parent/patient?

ANSWER KEY: ITCHY BUMPS, PEDIATRIC FEMALE

CASE

100

1. Differential Diagnosis L

L

L

L

L

L

Pityriasis rosea is an inflammatory skin condition that presents generally as a single lesion called a herald patch followed by multiple papulosquamous lesions that can be itchy. This rash lasts up to several weeks and is self-limiting. This is inconsistent with the patient’s rash, which covers much of her body. Urticaria has a subacute onset and is episodic in nature. The rash is pruritic and migratory in nature. This is high on the differential given this patient’s presentation and recent history. Angioedema in children is typically associated with urticaria, which may or may not present with this condition. The patient has swelling on the orbital area and lips and tongue. The patient does not present with this finding today but has a history of eyelid involvement. Therefore, this diagnosis must be considered. Contact dermatitis is a localized reaction of itching and burning sensation and may have swelling or formation of blister. This child has a generalized rash on the neck, abdomen, thigh, legs, arm, hands, feet, buttocks, and back, suggesting contact dermatitis is less likely. Scabies rash can look generalized, but it is usually papular with burrows under skin. Additionally, scabies rash often has moderate secondary excoriations. This is also an unlikely diagnosis based on the patient’s presentation. Insect bites can produce a rash that mimics urticaria, especially when multiple small bites are present and there may be small hives at the bite site. It is an unlikely diagnosis here as the mother and child report no insect bites or exposure and the rash has been coming and going for over a month.

2. Most Likely Diagnosis Urticaria with reported angioedema. This rash has a subacute and episodic onset, wheals, plaques, and erythema with pruritus and is migratory in nature. The most common underlying cause of urticaria in children is viral or bacterial infections, food allergies, parasites in the gut, Helicobacter pylori infection, or atopy. 3. Pathophysiology Urticaria occurs as a result of the effects of mediators released from mast cell granules, primarily histamine. H1 and H2 histamine receptors are involved in urticaria. If H1 receptors are involved, it will lead to itching, flaring, erythema, and whealing. If H2 receptors are involved, it will lead to erythema and whealing.1 Mast cell degranulation is the key mechanism to the pathophysiology of urticaria. Mast cells, surface receptors, and molecules initiate a signal to trigger degranulation. The stimuli that trigger degranulation can be exogenous or endogenous. Cross-linking of mast cell receptors bound to specific IgE by exogenous allergens, also known as a type I hypersensitivity reaction, may be pertinent to acute urticaria.2 4. Diagnostic Tests/Images Food and inhalation allergy panel serum tests were done to rule out allergies. Results This patient’s blood test revealed multiple food allergies including grains, dairy, fruits, and vegetables.

5. Next Steps Initial Visit L A mild case of urticaria can be easily treated in a PCP office. An H1 histamine blocker such as cetirizine 10 mg daily in the morning (changed from loratadine to cetirizine because of the insurance coverage) and 12.5 mg of diphenhydramine can be administered at bedtime to help children sleep (in additon to treating the allergic reaction). In a severe case of urticaria, 3 to 5 days of prednisone should be considered. L If a patient fails initial treatment, then a referral should be made to an allergy and immunology specialist. L It is important to consider a prescription epinephrine pen for subcutaneous injection, usually one pen for home and one pen for school in case of anaphylaxis. Patients with urticaria may develop anaphylaxis reactions in the future, so access to an epinephrine pen is important. L The parent should keep a journal to raise awareness about what triggers an episode. L A follow-up appointment was made to go through lab results and reevaluate the patient. Follow-up Visit All the allergies detected on serum test were discussed. L Because of the multiple allergies, the patient was referred to an allergist for further evaluation and management. L

6. Diagnostic Approach and Prognosis L

L

Urticaria is a common skin condition, but its cause is often difficult to determine. Evaluations should be carried out with a specific test selected based on a patient’s history. Each case is different, so extensive and expensive laboratory testing is not recommended as it does not change the prognosis or management. Understanding pathophysiology and causes helps clinicians improve diagnosis and management of patients with urticaria.3 Diagnosing urticaria is challenging as there are other similar conditions. Those include urticaria pigmentosa, urticarial vasculitis, atopic dermatitis, contact dermatitis, drug eruptions, erythema multiforme, Henoch–Schonlein purpura, scabies, and viral exanthema.4 During the diagnostic approach, PCPs should keep in mind that there are other similar conditions that may present like urticaria. Acute urticaria generally reaches self-remission within 3 weeks5 but roughly about 20% of cases with acute urticaria progress to develop chronic conditions.6 The duration of chronic urticaria typically lasts about 1 to 5 years but may last >5 years in 14% of cases.7

7. ICD-10 and CPT (E/M) Codes ICD-10 Code: L50.0: Allergic urticaria that may be related to food, medication, or insect bite. ICD-10 Code: L50.1: Idiopathic urticaria. ICD-10 Code: L50.9: Urticaria, unspecified. Any of these ICD-10 codes is appropriate during the initial visit. Select one code and briefly describe what the rationale is on the chart notes. For example, if L50.9 urticaria, unspecified is used, PCPs may state that the exact cause is unknown at this time and the evaluation is in progress. CPT (E/M) Code: Level 4—99214 (Established Patient) This is a visit for an established patient in the clinic. This visit required a thorough history and physical exam, an order of blood tests for food and respiratory allergy panel, and a follow-up visit for blood test results. 8. Patient Education Topics L L

Urticaria How to manage acute non-anaphylactic urticaria

L L L

How to use an epinephrine pen Anaphylaxis and seeking help at an ED How to avoid potential trigger for urticaria

9. Medical or Legal Concerns There can be significant legal implications especially if the cause of the urticaria is severe food or respiratory allergies. Lack of appropriate allergy testing, treatment, and follow-up can lead to daily debility and even death via bronchospasm/anaphylaxis. It is imperative that a PCP consider an epinephrine pen and provide emergency precautions to the patient and parents. 10. Interprofessional Collaboration L L L

L L L

Receptionist registers the patient. MA takes brief history, vitals, and chief complaint. PCP examines, diagnoses, and treats the patient and demonstrates how to use an epinephrine pen. Nurse assists with administering medication. Phlebotomist draws blood. Allergy specialist provides additional evaluation and care.

Outcome

This patient’s blood test revealed multiple allergies to grains, dairy, fruits, and vegetables. The patient was started on treatment cetirizine oral tablet and referred to an allergy specialist for consideration of immune desensitization therapy. I saw the patient back in the clinic 4 months later; she seemed happy; the desensitizing therapy was helping, and her mother was also pleased with the treatment plan provided by the allergy specialist.

Insight

from the

PCP

As the PCP and family health provider for this patient, I was well aware of her medical and social history. My suspicion of food allergy was accurate given her symptoms. I explained to the mother and patient the blood test results and printed a list of foods to avoid. I also provided a list of alternative food items to aid in the transition to a new diet. I gave her two epinephrine pens, one for school and one for home in case of anaphylaxis reaction. I also demonstrated how to use the epinephrine pen in case of emergency as well as the protocol for calling 911. After detailed explanation of the condition and treatment, I made sure that the mother and child were at ease and left satisfied with the care provided. Clear communication, coupled with providing resources needed for self-care, goes a long way to ensure patients and their families see you as someone who is truly concerned about their well-being and “on their side.”

References

1. Monroe EW, Cohen SH, Kalbfleisch J, et al. Combined H1 and H2 antihistamine therapy in chronic urticaria. Arch

Dermatol. 1981;117(7):404–407. doi:10.1001/archderm.1981.01650070032018

2. Abraham SN, St John AL. Mast cellorchestrated immunity to pathogens. Nat Rev Immunol. 2010;10(6):440–452.

doi:10.1038/nri2782

3. Boonpiyathad T, Mitthamsiri W, Pradubpongsa P, Sangasapaviliya A. Urticaria diagnosis. EMJ. 2018;3(1):98–105. 4. Schaefer P. Urticaria: evaluation and treatment. Am Fam Physician. 2011;83(9):1078–1084. 5. Aoki T, Kojima M, Horiko M, et al. Acute urticaria: history and natural course of 50 cases. J Dermatol.

1994;21(2):73–77. doi:10.1111/j.1346-8138.1994.tb01417.x

6. Kulthanan K, Chiawsirikajorn Y, Jiamton S, et al. Acute urticaria: etiologies, clinical course and quality of life. Asian

Pac J Allergy Immunol. 2008;26(1):1–9.

7. Toubi E, Kessel A, Avshovich N, et al. Clinical and laboratory parameters in predicting chronic urticaria duration: a

prospective study of 139 patients. Allergy. 2004;59(8):869–873. doi:10.1111/j.1398-9995.2004.00473.x

FREQUENT ILLNESS AND FATIGUE, ADULT FEMALE

CASE

101

Chief CoMPlaint

“Frequent illness and fatigue.”

historY

of

Present illness

A 34-year-old African American woman presents for a third time in 6 months complaining of upper respiratory symptoms. At this time, her symptoms, including cough, are resolving. She reports that she tends to get sick easily. She is less concerned about her current symptoms but more concerned about always getting sick. Three months ago, she was also diagnosed with community-acquired pneumonia. Over the past 3 months, she has noted marked fatigue and generalized achiness. Nothing seems to help with her symptoms. She has taken ibuprofen for her symptoms with fair relief. She does not eat a well-balanced diet and calls her diet “the typical American diet.”

reVieW

of

sYsteMs

The patient’s ROS is positive for fatigue, generalized myalgias, and a nonproductive resolving cough. She denies recent changes in weight or appetite, heart palpitation, SOB, pale skin (pallor), heat or cold intolerance, nausea, vomiting, diarrhea, constipation, polyuria, polydipsia, polyphagia, headache, dizziness, numbness, and tingling in upper or lower extremities. She also denies fever, chills, and chest pain. She denies anxiety and depression.

releVant historY

Her history is positive for being overweight. She was treated for pneumonia 3 months ago. Her immunizations are up to date. Her surgical history includes a Cesarean section in 2015. She has no hospitalizations outside of childbirth. She was tested for HIV 9 months ago and the result was negative. Her last physical exam was 9 months ago. She has no history of cancer, diabetes, hypertension, or thyroid problems. The patient is married and has two young children. Her marriage is monogamous, and she has been married for 10 years. Her husband had vasectomy a few years ago. She is a kindergarten teacher. She reports missing work at least twice a month due to illness and fatigue. She does not use alcohol, illicit drugs, or tobacco products. She is not physically active and does not spend much time outside. Her family history is non-contributory.

allergies

Penicillin (hives); no known food allergies.

MeDiCations None.

PhYsiCal eXaMination

Vitals: T 36.94°C (98.5°F), P 86, R 16, BP 134/72, WT 68 kg (150 lbs) HT 160 cm (63 in.), BMI 26.6. General: Well nourished, well developed, in no acute distress. Overweight. Psychiatric: Calm and cooperative.

406  Case 101  •  Frequent Illness and Fatigue, Adult Female

Skin, Hair, and Nails: Skin warm and without rash; no pallor of skin; no abnormal findings on hair and nails. Head: Normocephalic; no deformities noted. Eyes: PERRLA. ENT/Mouth: TM clear bilaterally; no pharyngeal erythema; no nasal deformity. Neck: Supple; no thyromegaly; FROM. Lungs: Clear to auscultation in all lung fields; no wheezing; no crackles. Heart: Regular rate and rhythm, no murmurs or gallops noted. Musculoskeletal: FROM in all extremities; no muscle tenderness with palpation; no edema or deformities noted. Neurologic: A&O×3; follows commands; cranial nerves II to XII grossly intact.

Clinical Discussion Questions 1. What is the differential diagnosis?

2. What is the most likely diagnosis? Why?

3. Demonstrate your understanding about the pathophysiology in regard to the most likely diagnosis.

4. Should tests/imaging studies be ordered? Which ones? Why? Think about tests/imaging beyond the primary care setting as well.

Case 101  •  Frequent Illness and Fatigue, Adult Female 407 5. What are the next appropriate steps in management?

6. What are the risk factors and treatment approach of the diagnosis? Provide references for your

responses.

7. What are the pertinent ICD-10 and CPT (E/M) codes for this visit? Provide a short rationale.

8. What is the appropriate patient education for this case?

9. If not managed appropriately, what is/are the medical/legal concern(s) that may arise?

10. Think about interprofessional collaboration for this case. Provide a list of specialties or other disciplines

and indicate what contribution these professionals might make to managing the patient.

408  Case 101  •  Frequent Illness and Fatigue, Adult Female

Bedside Manner Question

11. What would your communication style/approach be with this patient and her parents?

ANSWER KEY: FREQUENT ILLNESS AND FATIGUE, ADULT FEMALE

CASE

101

1. Differential Diagnosis L

L

L

L

L

L

L

Diabetes mellitus type 2 should be suspected when someone presents with fatigue and frequent illness. The patient denies polyuria, polydipsia, or polyphagia, suggesting she is probably not diabetic. However, it is recommended to rule out diabetes. Anemia is another possibility if someone presents with frequent illness and fatigue. Because this patient does not have headache or dizziness, severe anemia is not likely. However, it is reasonable to evaluate for mild to moderate anemia as a contributing factor. HIV is unlikely, as her risk factors are low: She is in a monogamous long-term relationship. She reports she was most recently tested 9 months ago, and the result was negative. However, this diagnosis should be considered in a patient with recurrent illness and persistent fatigue. Hypothyroidism is also an unlikely diagnosis for this patient. Although she has fatigue, she has no other symptoms, such as weight gain, constipation, hair or skin changes, or anxiety/ depression associated with hypothyroidism. Vitamin B12 deficiency is a possible diagnosis. With someone with fatigue, it is reasonable to rule out vitamin B12 deficiency. She does not have heart palpitations, SOB, pallor, constipation, diarrhea, loss of appetite, numbness, or tingling in extremities suggesting this is unlikely. Chronic fatigue syndrome is a possibility. However, she has no muscle tenderness with palpation. Also, chronic fatigue syndrome is seen less frequently compared to some of the other conditions listed here. Therefore, this is less likely. Vitamin D deficiency is high on the differential. The patient is not physically active and has a poor diet that may be deficient in this nutrient. Also, her race as an African American and not spending much time outside increase the risk for vitamin D deficiency.

2. Most Likely Diagnosis Vitamin D deficiency. The patient’s frequent illnesses, fatigue, myalgias, diet, and lack of outdoor physical activity all point to a vitamin D deficiency. In addition, her race as an African American makes her more susceptible to vitamin D deficiency. 3. Pathophysiology L

L

L

Vitamin D is a hormone mainly formed in human skin under the stimulation of UV radiation. Apart from its endocrine role in bone metabolism, vitamin D offers notable immunomodulatory properties. Vitamin D effects on the immune system include the enhancement of microbicidal ability of monocytes/macrophages and the down-modulation of inflammatory cytokines produced by T lymphocytes. Vitamin D deficiency may lead to other health problems.1 Vitamin D is produced when the skin is exposed to sunlight.2 UVB photons act on provitamin D3, a precursor in the cholesterol biosynthetic pathway, in the plasma membrane of epidermal cells to form pre-vitamin D3.3 Pre-vitamin D3 is rapidly transformed into vitamin D3 and transferred to the extracellular space, where it binds to vitamin D–binding protein.3 From there, it is transported to the liver, where it is hydroxylated into 25(OH)D.4 Serum concentration of vitamin 25(OH)D is the best indicator of its status.5 Malabsorption of vitamin D from dietary sources can result from disorders of the GI tract or metabolism, including Crohn disease, celiac disease, chronic active hepatitis, chronic kidney disease, chronic pancreatitis, cystic fibrosis, diabetes mellitus, gastric bypass, and primary biliary cirrhosis.3,6–11

4. Diagnostic Tests/Images L L L L L L

CBC to screen for anemia Lipid panel to screen cholesterol level along with other routine labs Vitamin B12 to screen for vitamin B12 deficiency Thyroid stimulating hormone to screen for thyroid disease CMP to evaluate electrolytes, renal functions, and liver functions 25-hydroxyvitamin D [25(OH)D] to screen for vitamin D deficiency

Results All results were within normal limits, with the exception of the vitamin D level, which was 10 ng/mL, indicating deficiency (normal range: 30 ng/mL or greater). 5. Next Steps L L

L L

L L

Possible diagnosis and patient education were discussed. The PCP-primary care provider informed the patient that lab work would need to be completed to get a clear diagnosis. Most of the suspected conditions for which the patient was evaluated can be managed outpatient by the PCP. Follow-up appointment was made to review the results of the lab work. All labs were normal, except the vitamin D level, which was 10 ng/mL, demonstrating vitamin D deficiency. These results were discussed with the patient during the follow-up visit. Vitamin D3 5,000 units daily for 8 weeks and more sunlight exposure were prescribed. The need to balance sunlight exposure for vitamin D versus risk of skin cancer was discussed.

6. Risk Factors and Treatment Approach L

L

L

L

L

L

The risk factors for vitamin D insufficiency include limited sunlight exposure and time spent outdoors, inadequate dietary vitamin D intake, winter season, and increased age.12 Vitamin D deficiency is a public health problem in many countries. People with darker skin require a greater dose of UV-B radiation from sunlight than people with paler skin to synthesize adequate amounts of vitamin D. Education on safe sunlight exposure and dietary vitamin D is warranted to promote vitamin D sufficiency in people with darker skin.13 Vitamin D deficiency is currently the most common nutritional deficiency worldwide. It is associated with skeletal effects, higher morbidity, and higher mortality. The solar UV-B photons and usual dietary intake may not be enough to achieve adequate serum concentrations of vitamin D. Therefore, people with vitamin D deficiency need nutritional supplements and fortified foods to fill the gap and meet the recommended dose of vitamin D.14 While vitamin D supplementation is an acceptable and easy short-term solution for vitamin D deficiency, food fortification such as staple foods may be the best and long-term solution to increase vitamin D consumption.14 Vitamin D can be obtained by sunlight exposure. The American Academy of Dermatology stated UV radiation to be a known skin carcinogen, so it may not be safe or efficient to obtain vitamin D via sunlight exposure or artificial sources. Consequently, many clinicians recommend appropriate vitamin D supplementation to achieve optimal plasma concentration.15 For adults found to be vitamin D deficient, 50,000 IU of vitamin D3 per week for 8 weeks to achieve a serum 25(OH)D level of 30 ng/mL is recommended.16,17 For patients who prefer a daily dosing regimen, 5,000 IU of vitamin D3 daily for 8 weeks may be used. If intestinal malabsorption is suspected, gastroenterology consultation should be considered.

7. ICD-10 and CPT (E/M) Codes ICD-10 Code: R53.82 Fatigue. ICD-10 Code: M79.10 Myalgia, unspecified site. These are diagnoses at the initial primary care visit. Although the patient was found to be deficient of vitamin D, these are the symptoms she presented with originally that led to the vitamin D

deficiency diagnosis. Once diagnosis is confirmed, the ICD-10 code E 55.9 for vitamin D deficiency can be used. CPT (E/M) Code: Level 4—99214 (Established Patient) This is a primary care visit for the evaluation and management of an established patient. According to the case, the patient has a new complaint (fatigue) for which lab work and a thorough exam history needed to be completed to finalize the diagnosis. Based on this criterion, the code for this visit is 99214. 8. Patient Education Topics L L L L

Proper administration of vitamin D Increased intake of food rich in vitamin D18 Recommendations to get sunlight safely during peak hours The role of vitamin D in the body

9. Medical or Legal Concerns Liability in this case is low. This is not a life-threatening condition. However, there are some serious underlying systemic diseases that can cause vitamin D deficiency (liver disease, kidney disease). For example, chronic kidney disease prevents the conversion of vitamin D to its active form, resulting in vitamin D deficiency. Therefore, PCPs need to keep that in mind. Also, follow-up appointments and monitoring are important to ensure that the vitamin D supplement takes care of the problem. After starting a vitamin D supplement, if the patient continues to have persistent fatigue, a further workup may be necessary. The risk of vitamin D toxicity is relatively low but does need to be considered. It is essential to follow up the serum vitamin D level and adjust the therapeutic dosing accordingly. 10. Interprofessional Collaboration L L L L

Receptionist registers the patient. MA takes a brief history and vital signs. Phlebotomist draws blood for the lab work. PCP formulates the most likely diagnosis.

Outcome

The patient took vitamin D3 5,000 IU daily consistently for 8 weeks as prescribed. She also tried to get more sunlight. Her level was rechecked in 2 months, and it was in a therapeutic range at 45. She was then reduced to 2,000 IU daily. She reports having more energy, and she has not been ill since she has been taking vitamin D on a regular basis. She will continue on a maintenance dose to ensure she does not become deficient again.

Insight

from the

PCP

Vitamin D deficiency is relatively common. This patient’s recent health decline was a good opportunity to order baseline labs and determine her health status. Diagnosing that she is vitamin D deficient and treating as appropriate will lead to a reduced risk for falls and osteoporosis in the future.19 Educating patients is imperative to helping them identify health risk factors. In this case, her race and inadequate sunlight exposure were risk factors for her vitamin D deficiency, which she did not know. This patient later informed me she told her three sisters about her vitamin D deficiency; they were checked, and all were deficient as well. Fatigue is one of those vague symptoms that lead to many differential diagnoses. Therefore, PCPs need to have a strategy to evaluate patients present with fatigue. Obviously, doing baseline labs including fatigue-specific tests (vitamin D, vitamin B12, etc.) is a good starting point. It is important that PCPs follow up the patients and monitor vitamin D levels and ask if the fatigue is improved. If the vitamin D level is normal but the fatigue continues, PCPs need to explore the condition further. When talking with a patient with vague symptoms (i.e., fatigue), use gentle and direct language; let them know the possible differential diagnoses linked with fatigue. it is important to have routine follow-ups. Let them know that vitamin D deficiency is easily manageable but that further

follow-up is necessary. Language that can help to facilitate dialogue during the initial encounter includes “Sorry that you are feeling this way; let’s start evaluating to find an answer” or “I know we don’t have a clear diagnosis for you at this time, but I will do my best to find an answer.” As a PCP, always make sure your patients feel satisfied with the current care and self-assured about their next steps.

References

1. Ishikawa LLW, Colavite PM, Fraga-Silva TFC, et al. Vitamin D deficiency and rheumatoid arthritis. Clin Rev

Allergy Immunol. June 2017;52(3):373–388. doi:10.1007/s12016-016-8577-0

2. Holick MF. Vitamin D: a millenium perspective. J Cell Biochem. 2003;88(2):296–307. doi:10.1002/jcb.10338 3. Holick MF. Vitamin D deficiency. N Engl J Med. 2007;357(3):266–281. doi:10.1056/NEJMra070553 4. Christakos S, Ajibade DV, Dhawan P, et al. Vitamin D: metabolism. Endocrinol Metab Clin North Am.

2010;39(2):243–253. doi:10.1016/j.ecl.2010.02.002

5. Vitamin D: fact sheet for health professionals. National Institutes of Health website. https://ods.od.nih.gov/

factsheets/VitaminD-HealthProfessional. Visited on Sep 27, 2020.

6. Bikle DD. Vitamin D insufficiency/deficiency in gastrointestinal disorders. J Bone Miner Res. 2007;22(suppl

2):V50–V54. doi:10.1359/jbmr.07s208

7. Dibble JB, Sheridan P, Losowsky MS. A survey of vitamin D deficiency in gastrointestinal and liver disorders. Q J

Med. 1984;53(209):119–134. doi:10.1093/oxfordjournals.qjmed.a067783

8. Johnson JM, Maher JW, DeMaria EJ, et al. The long-term effects of gastric bypass on vitamin D metabolism. Ann

Surg. 2006;243(5):701–704. doi:10.1097/01.sla.0000216773.47825.c1

9. Thomas MK, Lloyd-Jones DM, Thadhani RI, et al. Hypovitaminosis D in medical inpatients. N Engl J Med.

1998;338(12):777–783. doi:10.1056/NEJM199803193381201

10. Geier A. Shedding new light on vitamin D and fatty liver disease. J Hepatol. 2011;55(2):273–275. doi:10.1016/j.

jhep.2010.12.026

11. Kumar R. Hepatic and intestinal osteodystrophy and the hepatobiliary metabolism of vitamin D. Ann Intern Med.

1983;98(5 pt 1):662–663. doi:10.7326/0003-4819-98-5-662

12. Matyjaszek-Matuszek B, Lenart-Lipińska M, Woźniakowska E. Clinical implications of vitamin D deficiency. Prz

Menopauzalny. June 2015;14(2):75–81. doi:10.5114/pm.2015.52149

13. Horton-French K, Dunlop E, Lucas RM, et al. Prevalence and predictors of Vitamin D deficiency among 14. 15. 16. 17. 18. 19.

African immigrants living in Australia. Int J Environ Res Public Health. August 2019;16(16):2855. doi:10.3390/ ijerph16162855 Khayyatzadeh SS, Bagherniya M, Abdollahi Z, et al. What is the best solution to manage vitamin D deficiency? IUBMB Life. September 2019;71(9):1190–1191. doi:10.1002/iub.2038 Chang SW, Lee HC. Vitamin D and health: the missing vitamin in humans. Pediatr Neonatol. June 2019;60(3):237– 244. doi:10.1016/j.pedneo.2019.04.007 Pepper K, Judd S, Nanes, M, et al. Evaluation of vitamin D repletion regimens to correct vitamin D status in adults. Endocr Practice. 2009;15(2):95–103. doi:10.4158/EP.15.2.95 Bordelon P, Ghetu MV, Langan RC. Recognition and management of vitamin D deficiency. Am Fam Physician. 2009;80(8):841–846. https://health.gov/our-work/food-nutrition/2015-2020-dietary-guidelines/guidelines/appendix-12/. Visited on September 27, 2020. Chung M, Lee J, Terasawa T, et al. Vitamin D with or without calcium supplementation for prevention of cancer and fractures: an updated meta-analysis for the U.S. preventive services task force. Ann Intern Med. 2011;155(12):827–838. doi:10.7326/0003-4819-155-12-201112200-00005

LIST OF ABBREVIATIONS AAFP

American Academy of Family Practice

A1AT Alpha-1-antitrypsin AAO

Awake, alert, and oriented

AAP

American Academy of Pediatrics

Ab Antibody

AMI

Acute myocardial infarction

ANA

Anti-nuclear antibodies

ANC

Absolute neutrophil count

AOM

Acute otitis media

A&O

Alert and oriented

AP

Advanced placement

APC

Adenomatous polyposis coli

ARBs

Angiotensin receptor blockers

ART

Antiretroviral therapy

ASAP

As soon as possible

ASCVD

Atherosclerotic cardiovascular disease

ABG

Arterial blood gas

ABI

Ankle-brachial index

ACA

Affordable Care Act

ACE

Angiotensin-converting enzyme

ACIP

Advisory Committee on Immunization Practices

ACLE

Acute cutaneous lupus erythematosus

ASD

Autism spectrum disorder

ACLS

Advanced cardiac life support

ASO

Antistreptolysin O

ACS

Acute coronary syndrome

ASQ-3

ACTH

Adrenocorticotropic hormone

Ages and Stages Questionnaire, Third Edition

ADA

Americans with Disabilities Act

AST

Aspartate aminotransferase

ADHD

Attention deficit hyperactivity disorder

AUA

American Urological Association

AUD

Alcohol use disorder

ADLs

Activities of daily living

AUDIT-C

Alcohol Use Disorders Identification Test-Concise

AFP Alpha-fetoprotein AGEs

Advanced glycation end products

AHI

Apnea-hypopnea index

AIP

Acute intermittent porphyria

AIS

Adolescent idiopathic scoliosis

AKI

Acute kidney injury

ALP

Alkaline phosphatase

ALT

Alanine transaminase

AV Atrioventricula AVL

Augmented vector left

AVN

Avascular necrosis

BASE

Brief Abuse Screen of Elderly

BG

Blood glucose

BID

Twice daily

BL Bilateral BLE

Bilateral lower extremity

410  List of Abbreviations

BMI

Body mass index

CMAP

Compound muscle action potential

BMP

Basic Metabolic Panel

CMP

Comprehensive metabolic panel

BNP

Brain natriuretic peptide

CMV Cytomegalovirus

BP

Blood pressure

CN

Cranial nerves

BPH

Benign prostatic hypertrophy

CNS

Central nervous system

BRAT

Bananas, rice, applesauce, and toast

COMPLETES

BRCA

Breast cancer gene

Color, Other conditions, Mobility, Position, Lighting, Entire surface, Translucency, External auditory canal and auricle, Seal

BRUE

Brief resolved unexplained event

COPD

BS

Bowel sounds

Chronic obstructive pulmonary disease

BUN

Blood urea nitrogen

COVID-19

Coronavirus disease 2019

BV

Bacterial vaginosis

CAD

Coronary artery disease

CBC

Complete blood count

COX Cyclooxygenase CPAP

Continuous positive airway pressure

CPF

Chronic plantar fasciitis

CBD Cannabidiol

CPS

Child Protective Services

CCBs

Calcium channel blockers

CPT

Current Procedural Terminology

CCM

Chronic care model

CPT (E/M)

CCP

Cyclic citrullinated peptide

Current Procedural Terminology (Evaluation and management)

CD4

Cluster of differentiation 4

CRP

C-reactive protein

CD8

Cluster of differentiation 8

CSD

Cat scratch disease

CDC

Centers for Disease Control and Prevention

CSF

Cerebrospinal fluid

CTA

Clear to auscultation

CEA

Carcinoembryonic antigen

CTS

Carpal tunnel syndrome

CENTOR

Cough, Exudates, Nodes, Temperature, OR

CUP

Cancer of unknown primary

CFS

Cerebrospinal fluid

CFU

Colony-forming unit

CGM

Continuous glucose monitor

CHF

Congestive heart failure

CHIP

Children’s Health Insurance Program

CK

Creatine kinase

CKD

CV Cardiovascular CVA

Cerebrovascular accident

CVD

Coronary vascular disease

2D Two-dimensional DASH

Dietary approaches to stop hypertension

DCCT

Diabetes Control and Complications Trial

Chronic kidney disease

DDx

Differential diagnosis

CKMB

Creatine kinase myocardial band

DEXA

Dual-energy x-ray absorptiometry

CLIA

Clinical Laboratory Improvement Amendments

DGP

Deamidated gliadin peptide

CM

Chiari malformations

DJD

Degenerative joint disease

List of Abbreviations 411

DKA

Diabetic ketoacidosis

EPT

Expedited partner therapy

DM

Diabetes mellitus

ESLD

End-stage liver disease

DMT

Disease-modifying therapy

ESR

Erythrocyte sedimentation rate

DOH

Department of health

ESWT

Extracorporeal shock wave therapy

DSM-5

Diagnostic and Statistical Manual of Mental Disorders, Fifth Edition

FABER

Flexion, abduction, external rotation

DSME

Diabetes self-management, education

FBS

Fasting blood sugar

FDA

Food and Drug Administration

DSMS

Diabetes self-management, support

FEV1

Forced expiratory volume

DTaP

Diphtheria and tetanus toxoids and acellular pertussis

FM Fibromyalgia

DTR

Deep tendon reflexes

DVT

Deep vein thrombosis

EB

Elementary body

EBV

Epstein–Barr virus

ED

Emergency Department

EEG Electroencephalogram EF

Ejection fraction

EGD Esophagogastroduodenoscopy

FMLA

Family Medical Leave Act

FOBT

Fecal occult blood testing

FODMAP

Fermentable oligosaccharides, disaccharides, monosaccharides and polyols

FPG

Fasting blood glucose

FQHC

Federally qualified health center

FROM

Full range of motion

FSH

Follicle-stimulating hormone

FSS

Function Status Scale

FTA-ABS

Fluorescent treponemal antibody absorbed

EGFR

Epidermal growth factor

eGFR

Estimated glomerular filtration rate

EHR

Electronic health records

FVC

Forced vital capacity

EIA

Enzyme immunoassay

GP

Gravidity and parity

ELISA

Enzyme-linked immunoassay

GAD

General anxiety disorder

EM

Erythema multiforme

GAD

65 Glutamic acid decarboxylase

EmA

Endomysium antibodies

GAS

Group A streptococcus

EMG

Electromyelography

GBM

Glioblastoma multiforme

EMR

Electronic medical records

GC Gonococcal

EMS

Emergency medical services

GCA

Giant cell arteritis

EMT

Emergency medical technician

GCS

Glasgow coma scale

ENG Electronystagmography

GE Gastroesophageal

ENP

Emergency nurse practitioner

GERD

Gastroesophageal reflux disease

ENT

Ear, nose, throat

GFD

Gluten-free diet

EOM

Extraocular movement

GFR

Glomerular filtration rate

EOMI

Extraocular movements intact

GH

Growth hormone

EP Electrophysiology

GI Gastrointestinal

412  List of Abbreviations

IBS

Irritable bowel syndrome

IBS-C

Irritable bowel syndrome with predominant constipation

IBS-D

Irritable bowel syndrome with predominant diarrhea

ICD

International Classification of Diseases

IDSA

Infectious Diseases Society of America

IEP

Individualized education program

Hct Hematocrit

IFA

Immunofluorescent antibody

HCTZ Hydrochlorothiazide

IgA

Immunoglobulin A

HCV

Hepatitis C virus

IGF-1

Insulin-like growth factor 1

HD

Hypokinetic dysarthria

IGIM

Intramuscular immunoglobulin

HDL

High-density lipoprotein

IgM

Immunoglobulin M

HEENT

Head, Eye, Ear, Nose, and Throat

IL-15 Interleukin-15

HER

Human epidermal growth factor receptor 2

IM Intramuscular

HF

Heart failure

HFMD

Hand, foot, and mouth disease

GINA

Global Initiative for Asthma Management and Prevention

GNAS1

Guanine nucleotide binding protein, alpha stimulating

GU Genitourinary 2-h PG

2-hour plasma glucose

HAV

hepatitis A virus

HC

Head circumference

hCG

Human chorionic gonadotropin

Hgb Hemoglobin

INS

Idiopathic nephrotic syndrome

IOP

Intraocular pressure

ISD

Intrinsic sphincteric deficiency

ITP

Idiopathic thrombocytopenic purpura

Hgb A1C

Hemoglobin A1C

HLA

Human leukocyte antigen

IU

International unit

HNF

Hepatocyte nuclear factor

IUD

Intrauterine device

HPI

History of present illness

IV Intravenous

HPP

Hypokalemic periodic paralysis

JIA

Juvenile idiopathic arthritis

HPV

Human papillomavirus

JIS

Juvenile idiopathic scoliosis

HR

Heart rate

JRA

Juvenile rheumatoid arthritis

HRA

High-resolution anoscopy

JVD

Jugular Venous Distention

HS

Hidradenitis suppurativa

KRAS

Kirsten rat sarcoma

LABA/ICS

Long-acting beta-agonists and inhaled corticosteroids

HSP Henoch-Schonlein purpura HSV

Herpes simplex virus

HT Height HTN Hypertension

LAD Lymphadenopathy LADA

Latent autoimmune diabetes of adulthood

IACS

Intra-articular corticosteroid

LAMA

Long-acting muscarinic antagonist

IBC

Inflammatory breast cancer

LDH

Lactate dehydrogenase

IBD

Inflammatory bowel disease

LDL

Low-density lipoproteins

List of Abbreviations 413

LES

Lower esophageal sphincter

LFTs

Liver function tests

LH

Luteinizing hormone

LLC

Lower limb cellulitis

LLQ

Left lower quadrant

LMP

Last menstrual period

LP Lipoprotein

MRSA Methicillin-resistant Staphylococcus aureus MS

Multiple sclerosis

MSCRAMMs

Microbial surface components recognizing adhesive matrix molecules

MSI

Microsatellite instability

MSM

Men who have sex with men

Lp(a) Lipoprotein(a)

MSSA Methicillin-sensitive Staphylococcal Aureus

LPN

Licensed practical nurses

MVA

Motor vehicle accident

LROM

Limited range of motion

NAAT

Nucleic acid amplification test

LS

Lumbar spine

NABS

Normal Active Bowel Sounds

LSM

Lifestyle modifications

NAD

No acute distress

LSW

Licensed clinical social worker

NAFLD

Nonalcoholic fatty liver disease

LVN

Licensed vocational nurse

NASH

Nonalcoholic steatohepatitis

MA

Medical assistant

NCAT

Normocephalic, atraumatic

MALT

Mucosa-associated lymphoid tissue

NCS

Nerve conduction study

MAS

Macrophage activation syndrome

NF

Necrotizing fasciitis

MCH

Mean corpuscular hemoglobin

NGSP

MCHC

Corpuscular hemoglobin concentration

National Glycohemoglobin Standardization Program

NGU

Nongonococcal urethritis

MCLS

Medial collateral ligament strain

NICHQ

MCN

Minimal-change nephropathy

National Initiative for Children's Healthcare Quality

MCV

Corpuscular volume

NO

Nitric oxide

MD

Meningococcal disease

NOS

Not Otherwise Specified

MDI

Metered dose inhaler

NPO

Nil per os

MDS

Movement Disorder Society

NSAIDs

Nonsteroidal antiinflammatory drugs

MI

Myocardial infarction

NSTEMI

MIRM

Mycoplasma pneumoniae-induced rash and mucositis

Non-ST segment elevation myocardial infarction

NSVD

MKPS

Medial knee plica syndrome

Normal Spontaneous Vaginal Delivery

MMR

Measles, mumps, and rubella

NTND

Non-tender non-distended

MMRV

Measles, mumps, rubella, and varicella

OA Osteoarthritis

MMSE

Mini mental-state exam

MODY

Maturity-onset diabetes of youth

MRA

Magnetic resonance angiography

OAB

Overactive bladder

OAB-q

Overactive Bladder Questionnaire

OAB–V8

Overactive bladder-validated 8

OD

Oculus dexter

414  List of Abbreviations

ODD

Oppositional defiant disorder

PLT Platelets

OGTT

Oral glucose tolerance test

PMI

Point of maximal impulse

OM

Otitis media

PMNs

Polymorphonuclear neutrophils

OME

Otitis media with effusion

PNS

Primary nephrotic syndrome

PO

By mouth

OP Oropharynx OR

Operating room

POCBG

Point-of-care blood sugar

OS

Oculus sinister (left eye)

POCUS

Point-of-care ultrasound

OSA

Obstructive sleep apnea

PPIs

Proton-pump inhibitors

OT

Occupational Therapy

PR

Plaque rupture

PrEP

Pre-exposure prophylaxis

OTC Over-the-counter OU

O´culus uter´que (each eye)

PRN

As needed

PA

Physician assistant

PRP

Platelet-rich plasma

PACE

Program of All-Inclusive Care for the Elderly

PSA

Prostate-specific antigen

PAD

Peripheral arterial disease

PSGN

Post-streptococcal acute glomerulonephritis

PAG

Periaqueductal gray matter

PT

Physical therapist

PBP-2a

Penicillin-binding protein 2A

PTA

Peritonsillar abscess

PCOS

Polycystic ovarian syndrome

PTH

Parathyroid hormone

PCP

Primary care physician

PTNS

Peripheral tibial nerve stimulation

PCR

Polymerase chain reaction

PTT

Partial thromboplastin time

PCV13

Pneumococcal conjugate vaccine 13

PVCs

Premature ventricular contractions

PD

Parkinson’s disease

PE

Pulmonary embolism

PERRL

Pupil, equal, round, reactive (to), light

PERRLA

Pupil, equal, round, reactive (to), light, accommodation

RAAS Renin–angiotensin–aldosterone system

PF

Plantar flexion

RAI

Radioactive iodine therapy

PFS

Patellofemoral syndrome

RAMs

Rapid alternating movements

PHQ

Personal health questionnaire

RB

Reticulate body

PHQ-9

Personal Health Questionnaire-9

RBC

Red blood cell

PI Phosphatidylinositol

RBS

Random blood sugar

PICU

Pediatric ICU

RDW

Red cell distribution width

PID

Pelvic inflammatory disease

RF

Rheumatoid factor

PKC

Protein kinase C

RICE

PKD

Polycystic kidney disease

Rest, ice, compression, and elevation

RIDT

Rapid Influenza Diagnostic Test

QD Daily QHS

Every night at bedtime

QID

Four times daily

QTC

QT interval corrected

List of Abbreviations 415

RLQ

Right lower quadrant

SSS

Symptom Severity Scale

RMSF

Rocky Mountain spotted fever

SSTI

Skin and soft tissue infection

ROM

Range of motion

STAT Immediately

ROS

Review of systems

STDs

Sexually transmitted diseases

RPR

Rapid plasma reagin

STEMI

RR

Regular rate

ST-segment elevation myocardial infarction

RRMS

Relapsing remitting multiple sclerosis

STI

Sexually transmitted infection

T1DM

Type 1 diabetes mellitus

RRR

Regular rate and rhythm

T2DM

Type 2 diabetes mellitus

RSR

Regular sinus rhythm

TAH/BSO

RSV

Respiratory Syncytial Virus

Total Abdominal Hysterectomy with Bilateral Salpingo-Oophorectomy

RT-PCR

Reverse transcription polymerase chain reaction

RUQ

Right upper quadrant

RZV

Recombinant zoster vaccine

SA Sinoatrial SB

Sickle beta

SC

Sickle cell

SCC

Staphylococcal chromosome cassette

SCD

TB Tuberculosis TBI

Traumatic brain injury

Tdap

Tetanus, diphtheria, acellular pertussis

TEACCH

Treatment and Education of Autistic and related Communication Handicapped

TG Transglutaminase THC Tetrahydrocannabinol TIBC

Total iron binding capacity

Sickle cell disease

TID

Three times a day

SCQ

Social Communication Questionnaire

TM

Tympanic membrane

SG

Specific gravity

TMN

Thin membrane nephropathy

SHBG

Sex hormone binding globulin

SIRS

Systemic inflammatory response syndrome

SJIA

Systemic juvenile idiopathic arthritis

SJS

Steven-Johnson Syndrome

SLE

Systemic lupus erythematosus

SLR

Straight leg raise

SOAP

Subjective, objective, assessment, and plan

SOB

Shortness of breath

SRCs

Sports-related concussions

SSRIs

Selective serotonin reuptake inhibitors

TMP–SMX Trimethoprim–sulfamethoxazole TPO

Thyroid peroxidase

TPP

Thyrotoxic periodic paralysis

TRT

Testosterone replacement therapy

TSH

Thyroid-stimulating hormone

TSH-R

Thyroid-stimulating hormone

TSI

Thyroid-stimulating immunoglobulin

TTG

Tissue transglutaminase

TTP

Thrombotic thrombocytopenic purpura

TV

Trichomonas vaginalis

UA

Urine analysis

UAhCG

Urine analysis human chorionic gonadotropin

416  List of Abbreviations

UC

Ulcerative colitis

UVB

Ultraviolet B

UE

Upper extremity

VDRL

URI

Upper respiratory infection

Venereal Disease Research Laboratory

WBC

White blood cell

WHO

World Health Organization

WNL

Within normal limit

US Ultrasound USPSTF

United States Preventive Services Task Force

UTI

Urinary tract infection

UV Ultraviolet

WT Weight

INDEX A

abdominal pain clinical discussion questions, 10–12, 118–120, 300–302, 390–392 medication for, 9, 117, 389 patient history, 9, 117, 299, 389 patient presentation, 9, 117, 299, 389 physical examination, 10, 118, 299–300, 389–390 review of systems, 9, 117, 299, 389 acetaminophen, 77, 81, 101, 113, 126, 134, 137, 145, 153, 267, 287, 319 adult well visit clinical discussion questions, 246–248 patient history, 245 patient presentation, 245 physical examination, 245–246 review of systems, 245 albuterol inhaler, 9, 89, 94, 149, 157, 197, 241, 263, 307 alendronate, 145 allopurinol, 340 alprazolam, 101 amlodipine, 69, 259, 279, 331, 357 amoxicillin, 81, 134 anal bumps, painless clinical discussion questions medication for, 206 patient history, 205 patient presentation, 205 physical examination, 206 review of systems, 205 ankle and calf pain clinical discussion questions, 2–4 medication for, 1 patient history, 1 patient presentation, 1 physical examination, 1–2 review of systems, 1 armpit lump, painful clinical discussion questions, 284–286 medication for, 283 patient history, 283 patient presentation, 283 physical examination, 284 review of systems, 283 aspirin, 70 aspirin, 26, 161 atorvastatin, 22, 26, 101, 121, 141, 189, 263, 291, 357

B

back pain and poor posture clinical discussion questions, 398–400 medication for, 397 patient history, 397 patient presentation, 397 physical examination, 397–398 review of systems, 397 beclomethasone dipropionate, 263

benzoyl peroxide, 9 biceps tendinitis, 201. see also shoulder pain birth control pills, 286 blood in stool clinical discussion questions, 373–376 medication for patient history, 373 patient presentation, 373 physical examination, 373 review of systems, 373 blurred vision clinical discussion questions, 38–39, 142–144 HEENT symptoms, 141 medication for, 141 patient history, 37, 141 patient presentation, 37, 141 physical examination, 37–38, 142 review of systems, 37, 141 breast lump clinical discussion questions, 394–396 medication for, 393 patient history, 393 patient presentation, 393 physical examination, 394 review of systems, 393 breast redness and pain clinical discussion questions, 362–364 patient history, 361 patient presentation, 361 physical examination, 362 review of systems, 361 bruising on legs clinical discussion questions, 130–132 patient history, 129 patient presentation, 129 physical examination, 129–130 review of systems, 129 buttocks pain clinical discussion questions, 296–298 medication for, 295 patient history, 295 patient presentation, 295 physical examination, 296 review of systems, 295

C

calcium, 117, 319 calcium citrate, 145 carbidopa-levodopa, 340 chest pain clinical discussion questions, 182–183, 230–232 medication for, 181, 229 patient history, 181, 229 patient presentation, 181, 229 physical examination, 181–182, 230 review of systems, 181, 229

418  Index child trouble following directions clinical discussion questions, 42–43 patient history, 41 patient presentation, 41 physical examination, 41–42 review of systems, 41 chlorthalidone, 331 citalopram, 69 clonazepam, 94 confused state clinical discussion questions, 341–343 medication for, 340 patient history, 339–340 patient presentation, 339 physical examination, 340 review of systems, 339 constipation clinical discussion questions, 94–96, 194–196 medication for, 94, 194 patient history, 93, 193 patient presentation, 93, 193 physical examination, 94, 194 review of systems, 93, 193 coronary artery disease, 145 cough, persistent bad clinical discussion questions, 242–244 medication for, 241 patient history, 241 patient presentation, 241 physical examination, 242 review of systems, 241 coughing and gagging clinical discussion questions, 324–326 patient history, 323 patient presentation, 323 physical examination, 323–324 review of systems, 323 coughing and wheezing clinical discussion questions, 198–200 medication for, 197 patient history, 197 patient presentation, 197 physical examination, 197–198 review of systems, 197 crusty, irritated left eye clinical discussion questions, 46–48 patient history, 45 patient presentation, 45 physical examination, 45–46 review of systems, 45

D

daytime fatigue clinical discussion questions, 260–262 medication for, 259 patient history, 259 patient presentation, 259 physical examination, 259–260 review of systems, 259 desonide, 94 diabetes clinical discussion questions, 292–294 medication for, 291 patient history, 291 patient presentation, 291 physical examination, 292 review of systems, 291 diarrhea in adolescent female clinical discussion questions, 126–128 medication for, 126

patient history, 125 physical examination, 126 review of systems, 125 stool studies, 125 symptoms, 125 diazepam, 340 Dietary Approaches to Stop Hypertension diet, 141 difficulty relating to others clinical discussion questions, 58–60 medication for, 57 patient history, 57 patient presentation, 57 physical examination, 58 review of systems, 57 diltiazem, 161 dizziness and hearing difficulty clinical discussion questions, 110–112 medication for, 109 patient history, 109 patient presentation, 109 physical examination, 109–110 review of systems, 109 double and blurred vision clinical discussion questions, 165–168 patient history patient presentation, 165 physical examination, 166 review of systems, 165

E

elbow injury clinical discussion questions, 66–68 patient history, 65 patient presentation, 65 physical examination, 65–66 review of systems, 65 enalapril, 185, 295 erection difficulties clinical discussion questions, 378–380 patient history, 377 patient presentation, 377 physical examination, 377–378 review of systems, 377 esomeprazole, 241 eye injury clinical discussion questions, 328–330 medication for, 327 patient history, 327 patient presentation, 327 physical examination, 327–328 review of systems, 327

F

facial hair, excessive clinical discussion questions, 218–220 medication for, 218 patient history, 217 patient presentation, 217 physical examination, 218 review of systems, 217 facial numbness clinical discussion questions, 62–64 medication for, 61 patient history, 61 patient presentation, 61 physical examination, 61–62 review of systems, 61 famotidine, 362

Index 419 fatigue and illness clinical discussion questions, 406–408 patient history, 405 patient presentation, 405 physical examination, 405–406 review of systems, 405 fatigue and joint aches clinical discussion questions, 268–270 medication for patient history, 267 patient presentation, 267 physical examination, 267–268 review of systems, 267 fatigue and weakness clinical discussion questions, 6–8 medication for, 5 patient history, 5 patient presentation, 5 physical examination, 6 review of systems, 5 fatigue with general malaise clinical discussion questions, 102–104 medication for, 101 patient history, 101 patient presentation, 101 physical examination, 102 review of systems, 101 fever and lump on neck clinical discussion questions, 82–84 medication for, 81 patient history, 81 patient presentation, 81 physical examination, 82 review of systems, 81 fever with back pain clinical discussion questions, 146–148 medication for, 145 patient history, 145 patient presentation physical examination, 146 review of systems, 145 fever with bad cough clinical discussion questions, 154–156 medication for, 153 patient history, 153 patient presentation, 153 physical examination, 153–154 review of systems, 153 fever with body aches clinical discussion questions, 30–32, 222–223 medication for, 29, 221 patient history, 29, 221 patient presentation, 29, 221 physical examination, 29–30, 221 review of systems, 29, 221 fever with cough and runny nose clinical discussion questions, 288–289 medication for, 287 patient history patient presentation, 287 physical examination, 287–288 review of systems, 287 fever with headache clinical discussion questions, 158–160 medication for, 157 patient history, 157 patient presentation, 157 physical examination, 158 review of systems, 157

fever with rashes clinical discussion questions, 34–36, 350–352 medication for, 349 patient history, 33, 349 patient presentation, 33, 349 physical examination, 33–34, 350 review of systems, 33, 349 fever with sore throat and chills clinical discussion questions, 150–152 medication for, 149 patient history, 149 patient presentation, 149 physical examination, 149–150 review of systems, 149 fish oil, 26 fluoxetine, 218 fluticasone propionate nasal spray, 241 fluticasone propionate/salmeterol, 241 foot pain and swelling clinical discussion questions, 186–188 medication for, 185 patient history, 185 patient presentation, 185 physical examination, 185–186 review of systems, 185

G

gait (unsteady) and nausea clinical discussion questions, 214–216 patient history, 213 patient presentation, 213 physical examination, 214 review of systems, 213 gastroesophageal reflux disease (GERD), 105 glargine insulin, 26 glipizide, 295 guaifenesin cough syrup, 263

H

hand pain at night clinical discussion questions, 86–88 medication for, 85 patient history, 85 patient presentation, 85 physical examination, 86 review of systems, 85 headache clinical discussion questions, 114–116 medication for, 113 patient history patient presentation physical examination, 114 review of systems, 113 heart palpitations clinical discussion questions, 162–164 medication for, 161 patient history, 161 patient presentation, 161 physical examination, 162 review of systems, 161 heel pain, bilateral clinical discussion questions, 312–314 medication for, 311 patient history, 311 patient presentation, 311 physical examination, 311–312 review of systems, 311 hip pain clinical discussion questions, 14–16 medication for, 13

420  Index hip pain (cont.) patient history, 13 patient presentation, 13 physical examination, 14 review of systems, 13 hydrochlorothiazide (HCTZ), 61, 77, 101, 201, 279 hydrochlorothiazide/lisinopril, 141, 145, 189, 221, 255 hypertension, 145

I

ibuprofen, 1, 5, 13, 29, 177, 181, 201, 233, 263, 286–287, 311, 315, 331, 397 interprofessional collaboration, 6 irregular menstrual cycle clinical discussion questions, 174–176 patient history, 173 patient presentation, 173 physical examination, 173–174 review of systems, 173 irritable and crying toddler clinical discussion questions, 304–306 patient history, 303 patient presentation, 303 physical examination, 303–304 review of systems, 303 itchy bumps clinical discussion questions, 402–404 medication for, 401 patient history, 401 patient presentation, 401 physical examination, 401–402 review of systems, 401

K

knee pain clinical discussion questions, 178–180, 234–235 medication for, 177, 233 patient history, 177, 233 patient presentation, 177, 233 physical examination, 177–178, 233–234 review of systems, 177, 233

L

Lantus, 69 leg pain clinical discussion questions, 358–360 medication for, 357 patient history, 357 patient presentation, 357 physical examination, 357–358 review of systems, 357 leg pain, bilateral clinical discussion questions, 336–338 patient history, 335 patient presentation, 335 physical examination, 335–336 review of systems, 335 leg redness and pain clinical discussion questions, 370–372 medication for, 369 patient history, 369 patient presentation, 369 physical examination, 369–370 review of systems, 369 levothyroxine, 94, 189, 340, 374, 393 libido, fatigue and decreased clinical discussion questions, 210–212 medication for, 209

patient history, 209 patient presentation, 209 physical examination, 209–210 review of systems, 209 lightheadedness clinical discussion questions, 280–282 medication for patient history, 279 patient presentation, 279 physical examination, 279–280 review of systems, 279 Linagliptin, 70 lisinopril, 22, 26, 85, 101, 161, 209, 233, 263, 369 lithium, 94 loperamide, 125 loratadine, 340, 401 lump in throat clinical discussion questions, 346–348 patient history, 345 patient presentation, 345 physical examination, 345–346 review of systems, 345

M

magnesium, 319 meclizine, 109 melatonin, 275 metformin, 26, 85, 141, 185, 291, 295 methadone, 49 metoprolol succinate, 161, 340 metoprolol tartrate, 295 mood changes clinical discussion questions, 78–80 medication for, 77 patient history, 77 patient presentation, 77 physical examination, 78 review of systems, 77 morphine sulfate, 94 muscle and joint pain clinical discussion questions, 190–192 medication for, 189 patient history, 189 patient presentation, 189 physical examination, 190 review of systems, 189

N

naproxen sodium, 307 nausea/vomiting/headache, episodes in adolescent male clinical discussion questions, 106–108 patient history, 105 patient presentation physical examination, 105–106 review of systems, 105 nipple pain clinical discussion questions, 170–172 patient history, 169 patient presentation, 169 physical examination, 169–170 review of systems, 169 numbness in hands and legs clinical discussion questions, 22–24 medication for, 22 patient history, 21 patient presentation, 21 physical examination, 22 review of systems, 21

Index 421

O

omeprazole, 94 osteoporosis, 145 oxycodone hydrochloride, 69

P

pantoprazole, 117 penicillin, 73, 93, 201 pimple on leg clinical discussion questions, 226–227 medication for, 225 patient history, 225 patient presentation, 225 physical examination, 225 review of systems, 225 platelet count, low clinical discussion questions, 251–253 patient history, 249 patient presentation, 249 physical examination, 249–250 review of systems, 249 polyethylene glycol, 340 pravastatin, 295

R

rash, painful clinical discussion questions, 26–28, 138–140 medication for, 26, 137 patient history, 25, 137 patient presentation, 25, 137 physical examination, 26, 137–138 review of systems, 25, 137 refusal to eat and drink clinical discussion questions, 382–384 patient history, 381 patient presentation, 381 physical examination, 381–382 review of systems, 381 rosuvastatin, 331 rosuvastatin calcium, 69

S

sexually transmitted infections (STIs), 205 shortness of breath clinical discussion questions, 264–266 medication for, 263 patient history, 263 patient presentation, 263 physical examination, 263–264 review of systems, 263 shoulder pain clinical discussion questions, 202–205, 272–274 medication for patient history, 271 patient presentation, 201, 271 physical examination, 201–202, 271–272 review of systems, 201, 271 sildenafil, 393 simvastatin, 61 skin dryness and scaly patches clinical discussion questions, 316–317 medication for, 315 patient history, 315 patient presentation, 315 physical examination, 315–316 review of systems, 315 skin lesions after sun exposure clinical discussion questions, 256–257

medication for patient history, 255 patient presentation, 255 physical examination, 255 review of systems, 255 sotalol, 69 speech difficulties, geriatric male clinical discussion questions, 74–76 medical history, 73 medication for, 73 patient history, 73 physical examination, 74 review of systems, 73 stomach pain with bloating clinical discussion questions, 366–368 patient history, 365 patient presentation, 365 physical examination, 366 review of systems, 365 sulfamethoxazole-trimethoprim, 137 swelling around eyes and swollen ankles clinical discussion questions, 308–310 medication for, 307 patient history, 307 patient presentation, 307 physical examination, 308 review of systems, 307 swollen knee clinical discussion questions, 134–136 medication for, 134 patient history, 133 patient presentation, 133 physical examination, 134 review of systems, 133 symptoms, 133

T

tenofovir/emtricitabine, 349 toenails, thick and yellow clinical discussion questions, 70–72 medication for patient history physical examination, 70 review of systems, 69 treatment, 69 toe pain clinical discussion questions, 332–334 medication for, 331 patient history, 331 patient presentation, 331 physical examination, 332 review of systems, 331

U

urination, burning sensation while clinical discussion questions, 90–91, 238–239 medication for, 89 patient history, 89, 237 patient presentation, 89, 237 physical examination, 89–90, 237–238 review of systems, 89, 237 urination, frequent clinical discussion questions, 50–52 medication for, 49 patient history, 49 patient presentation, 49 physical examination, 50 review of systems, 49

422  Index urination, painful clinical discussion questions, 122–124 medication for, 121 patient history, 121 patient presentation, 121 physical examination, 121–122 review of systems, 121 urine leakage clinical discussion questions, 320–322 medication for, 319 patient history, 319 patient presentation, 319 physical examination, 320 review of systems, 319

V

vaginal bleeding clinical discussion questions, 18–19 patient history, 17 patient presentation, 17 physical examination, 17–18 review of systems, 17 vitamin B12, 362 vitamin D, 117, 145, 319, 340, 385 vomiting clinical discussion questions, 54–55, 98–100

patient history, 53, 97 patient presentation, 53, 97 physical examination, 53–54, 98 review of systems, 53, 97

W

weakness, extreme clinical discussion questions, 354–356 patient history, 353 patient presentation, 353 physical examination, 354 review of systems, 353 weight gain, unexplained clinical discussion questions, 386–388 medication for, 385 patient history, 385 patient presentation, 385 physical examination, 385–386 review of systems, 385 weight loss, unexplained clinical discussion questions, 276–278 patient history patient presentation, 275 physical examination, 275–276 review of systems, 275